MAT E9, Maternity Exam 8, Maternity Exam 7, Maternity Exam 6, Maternity Exam Ch. 13,14,15, Maternity Chapters 1-12

अब Quizwiz के साथ अपने होमवर्क और परीक्षाओं को एस करें!

When giving an APGAR score to a newborn, the nurse is assessing which of the following?

Appearance, pulse, respirations, grimace, activity Score 0-2 Total score: 0-3: severe distress 4-6: moderate distress 7-10: WNL

1) Which third-trimester client would the nurse suspect might be having difficulty with psychological adjustments to her pregnancy? A) A woman who says, "Either a boy or a girl will be fine with me" B) A woman who puts her feet up and listens to some music for 15 minutes when she is feeling too stressed C) A woman who was a smoker but who has quit at least for the duration of her pregnancy D) A woman who has not investigated the kind of clothing or feeding methods the baby will need

swer: D Explanation: A) Acceptance of gender is indicative of healthy adaptation to pregnancy. B) Using stress reduction techniques are indicative of healthy adaptation to pregnancy. C) Quitting smoking is indicative of healthy adaptation to pregnancy. D) By the third trimester, the client should be planning and preparing for the baby (for example, living arrangements, clothing, feeding methods).

1. The nurse suspects that a client has developed a perineal hematoma. What assessment findings would lead the nurse to this conclusion? A) Facial petechiae B) Large, soft hemorrhoids C) Tense tissues with severe pain D) Elevated temperature

wer: C Explanation: A) Facial petechiae do not indicate perineal hematoma. B) Large, soft hemorrhoids are not indicative of perineal hematoma. C) Tenseness of tissues that overlie the hematoma is characteristic of perineal hematomas. D) An elevated temperature is not a symptom of a perineal hematoma.

Answer: C Explanation: C) In a tracheoesophageal fistula, the lower esophageal segment connects to the lower trachea, with the upper esophageal segment ending blindly.

1) The nurse is preparing teaching material for the parents of a newborn with tracheoesophageal fistula. Where on the diagram should the nurse identify the location of this disorder? A) A B) B C) C D) D

Answer: C Explanation: C) Episodic accelerations are not associated with contractions and tend to be more peaked and abrupt. They are often associated with fetal movement, stimulation, or an environmental stimulus. Episodic accelerations are reassuring F H R patterns, whether or not they are accompanied by fetal movement. Early decelerations are a visually apparent, usually symmetrical, gradual decrease and return of the F H R associated with a uterine contraction. Periodic accelerations are associated with uterine contractions. When they occur on a repetitive basis, they may be smooth in configuration, multiphasic, and may precede variable decelerations. In variable decelerations with overshoot, the timing of the decelerations is variable, and most have a sharp decline. Rebound accelerations (overshoot) occur after most of the decelerations.

1) A laboring patient being monitored has the following rhythm strip: How should the nurse interpret this finding? A) Early decelerations B) Periodic accelerations C) Episodic accelerations D) Variable decelerations with overshoot

Answer: C Explanation: C) For mastitis, erythema and swelling are present in the upper outer quadrant of the breast. Axillary lymph nodes are often enlarged and tender. Cystitis is a urinary tract infection. Pelvic cellulitis (parametritis) is an infection that has ascended to involve the connective tissue of the broad ligament or, in more severe forms, the connective tissue of all the pelvic structures. Postpartum endometritis (metritis) is an inflammation of the endometrium portion of the uterine lining.

1. The nurse selects the following diagram to take when making a home visit to a postpartum patient. For which health problem is the nurse preparing educational material? A) Cystitis B) Metritis C) Mastitis D) Parametritis

1) At birth, a newborn weighs 8 pounds 4 ounces. When discussing the infant's weight over the next week, what is the maximum amount of weight the mother should expect that the infant will lose? (Calculate to the first decimal point.)

13.2 ounces Explanation: The maximum amount of weight loss over the first 5 to 10 days of extrauterine life is 10%. If the infant weighs 8 pounds 4 ounces, this weight in ounces is 132 ounces. Multiply this weight by 10% to find that the mother can expect the infant to lose 13.2 ounces. When converted to pounds and ounces, the infant should not weight less than 7 pounds 5 ounces.

1) Which of the following is the most prevalent medical complication of pregnant adolescents? A) Constipation B) Preeclampsia-eclampsia C) Heartburn D) Rapid enlargement and sensitivity of breasts

A) Answer: B Explanation: A) Constipation is a discomfort of pregnancy, but not a medical complication. B) Preeclampsia-eclampsia is the most prevalent medical complication of pregnant adolescents and is typically characterized by high blood pressure, proteinuria, and edema. C) Heartburn is a discomfort of pregnancy, but not a medical complication. Rapid enlargement and sensitivity of breasts are discomforts of pregnancy, but not medical complications

1) While attending a community fair the nurse is surprised to learn the number of women who are homemakers, teachers, and nurses. What impact should the nurse recognize that this has on these individuals' income? A) Potential for longer lifespan B) Overall lower income than men C) Reduced risk for chronic illnesses D) Increased risk for health problems

A) Answer: B Explanation: Historical trends that have contributed to an existing wage gap include being limited to occupations such as child care, teaching, and nursing, which by virtue of being female-dominated professions pay lower salaries. The professions of homemakers, teachers, and nurses will not impact lifespan and chronic illness, or increase the risk for health problems

A pregnant client who test positive for Group B streptococcus (GBS) between 35 and 37 week gestation require which of the following nursing intervention during labor? A. Administration of intravenous antibiotic prophylaxis B. No intervention is required this is normal vaginal flora C. Monitor the client for fever every 2 hrs D. Prepare the client for a c-section

A. Administration of intravenous antibiotic prophylaxis

In preparing the client in labor for vacuum extraction, it is important to explain that the infant might initially have which appearance after delivery? A. Edema of the caput B. Red mark of the face C. Edema of the face D. Swelling of the eyes

A. Edema of the caput

A nurse is caring for a client who is at 36 weeks of gestation and who has a suspected placenta previa. Which of the following findings support this diagnosis? A. Painless red vagina bleeding B. Increasing abdominal pain with a non-relaxed uterus C. Abdominal pain with scant red vaginal bleeding D. Intermittent abdominal pain following passage of bloody mucus

A. Painless red vagina bleeding

After delivering a 4355g (9lb, 10oz) baby, client who is gravida 5, para 5 is admitted to the postpartum unit. What should be the priority nursing intervention? A. palpate the fundus because she is risk for uterine atony B. Offer fluids since multiparas generally dehydrate faster during labor C. Perform passive range of motion on extremities because she is at risk for thromboembolism D. Assess the client's diet because she is at risk for anemia

A. palpate the fundus because she is risk for uterine atony

1) The nurse determines that a patient in the clinic has a learning disability. What did the nurse assess to make this clinical determination? Note: Credit will be given only if all correct choices and no incorrect choices are selected. Select all that apply. 1. Dyslexia 2. Dysgraphia 3. Hearing loss 4. Osteoarthritis 5. Bilateral cataracts

Answer: 1, 2 Explanation: Learning disabilities can inhibit educational attainment and employment and include dyslexia and dysgraphia. Hearing loss and bilateral cataracts are sensory changes that would not necessarily impact learning. Osteoarthritis is not a learning disability.

The nurse explains to a labor client that the relaxation period between contractions are important for which of the following reasons? A. avoids uterine rupture B. Allows fetal oxygenation C. Permits fetal assessment D. Prevents uterine ischemia

B. Allows fetal oxygenation

A labor and delivery nurse is working in a triage unit when four pregnant clients present to the unit. Which of the following client requires priority invention? A. A client at 37 weeks gestation who came with mild to moderate vaginal bleeding, pulse 98 and BP 120/80 B. A client at 6 weeks gestation who complains of abdominal cramping and pinkish to brownish spotting C. A client at 32 weeks gestation with bright red bleeding, pulse 110, and BP of 90/50 D. A client at 36 weeks gestation with dark red bleeding, pulse 90, BP 130/90, presence of fetal movement

C. A client at 32 weeks gestation with bright red bleeding, pulse 110, and BP of 90/50

A nurse caring for a laboring client observes late decelerations on the electronic fetal monitor. The priority nursing action is A. continue monitoring as this is expected during labor B. Call the PCP as this require emergency C-section C. Assist the client to her left side D. Check for ruptured membranes

C. Assist the client to her left side

A client at her first trimester of pregnancy complains of urinary frequency. Which of the following nurse responses is correct about urinary frequency? A. You probably have a UTI B. Your uterus is growing putting pressure on your bladder. C. Your bladder increases in size during pregnancy. D. The hormone for pregnancy irritates your bladder

D. The hormone for pregnancy irritates your bladder

A nurse is counseling a client about the use of contraceptive methods. The states that she is interested in using COCs. The nurse knows that a history of which of the following contraindication for use of combined oral contraceptives? A. ovarian cysts B. Dysmenorrhea C. Fatigue D. Thromboembolism

D. Thromboembolism

Answer: C Explanation: C) The greater, or anterior, fontanelle (bregma) is diamond shaped, measures 2 to 3 c m, and is situated at the junction of the sagittal, coronal, and frontal sutures. It permits growth of the brain by remaining unossified for as long as 18 months. Choice 1 is the posterior fontanelle. Choice 2 is the sagittal suture. Choice 4 is the frontal suture.

The nurse is reviewing educational material on newborn care with a patient in the 3rd trimester of pregnancy. Which area on the following diagram should the nurse point out as being the anterior fontanelle? A) A B) B C) C D) D

15) A client delivered 30 minutes ago. Which postpartal assessment finding would require close nursing attention? A) A soaked perineal pad since the last 15-minute check B) An edematous perineum C) The client experiencing tremors D) A fundus located at the umbilicus

answer: A Explanation: A) If the perineal pad becomes soaked in a 15-minute period, or if blood pools under the buttocks, continuous observation is necessary. As long as the woman remains in bed during the first hour, bleeding should not exceed saturation of one pad. B) An edematous perineum is a normal postpartal finding. C) Tremors are a normal postpartal finding. D) A fundus located at the umbilicus is a normal postpartal finding.

1) During a postpartum assessment, the nurse suspects the patient has a full bladder. Where is the fundus located for the nurse to make this clinical determination?

displaced and deviated to the right

Answer: D Explanation: D) The knee-chest position is used to relieve cord compression during a cord prolapse emergency. The supine, side-lying, or lithotomy positions are not beneficial to the fetus in cord prolapse.

1) During an intrapartum vaginal examination the following is assessed. In which position should the patient be placed at this time? A) Supine B) Side-lying C) Lithotomy D) Knee-chest

Answer: B Explanation: B) About 6 to 12 hours after birth, the fundus is at the level of (or one fingerbreadth below) the umbilicus. The height of the fundus then decreases about one fingerbreadth (approximately 1 c m) each day.

1) The home care nurse is assessing the level of a mother's fundus who delivered 3 days ago. Where should the fundus be located? A) A B) B C) C D) D

Answer: B Explanation: B) This diagram is used to instruct a new mother on the process of wrapping the baby. This diagram would not be appropriate to use when teaching feeding, bathing, or changing the diaper.

1) The nurse is preparing teaching material for a new mother. For which activity should the nurse provide the following diagram? A) Preparing to feed B) Wrapping the baby C) Preparing for a bath D) Changing the diaper

Answer: C Explanation: C) The middle third of the vastus lateralis muscle is the preferred site for intramuscular injection in the newborn. The middle third of the rectus femoris is an alternative site, but its proximity to major vessels and the sciatic nerve necessitates caution in using this site for injection.

1) The nurse is preparing to provide a newborn with a prescribed intramuscular injection. Which area should the nurse use for this injection? A) A B) B C) C D) D

Answer: B Explanation: B) This is the stepping reflex. When held upright with one foot touching a flat surface, the newborn puts one foot in front of the other and "walks." This reflex is more pronounced at birth and is lost in 4 to 8 weeks. This is not abnormal behavior. It does not mean that the infant will begin walking at an early age. This is not an indication that the infant needs to be fed.

1) While conducting a home visit, the nurse observes the father of a newborn hold the infant so that the following is observed. What should the nurse explain to the parents about this behavior? A) This is abnormal and should be reported. B) This will disappear between 4 and 8 weeks of age. C) It means the child will begin walking at an early age. D) The infant should be given more formula when this occurs.

Answer: C Explanation: C) This is a technique used to palpate the fundus to determine if it needs to be massaged to reduce bleeding and enhance healing after delivery. This technique is not used to assess for bladder distention, estimate the weight of the uterus, or to evaluate for remaining placenta contents.

28) The nurse is observed performing the following with a patient: What information will this assessment technique provide to the nurse? A) Assesses for bladder distention B) Estimates the weight of the uterus C) Determines the height of the fundus D) Evaluates the remaining placenta contents

Answer: C Explanation: For the football hold position, place the baby on the side of the breast that will be used for feeding. The baby should be in a feet first position with the baby's bottom resting on the pillow near mother's elbow. Turn baby slightly on her side so that she faces the breast. The mother should clutch the baby's body close to her own. Baby's body should feel securely tucked in under the mother's arm. Choice 1: For the modified cradle position, place baby on mother's lap and turn baby's entire body toward mother (baby is in side-lying position). Position the baby's body so that the baby's nose lines up to the nipple. Maintain baby's body in a horizontal alignment. Choice 2: For the cradle position place the baby on the mother's lap and turn baby's entire body toward the mother (baby is in side-lying position). Position the baby's body so that the baby's nose lines up to the nipple. Maintain baby's body in a horizontal alignment. Choice 4: For the side-lying position, place baby in side-lying position next to mother's body. Baby's body should face mother's body. Baby's nose should line up to mother's nipple. Place a roll behind baby's back, if desired.

1) A new mother asks the nurse about the football hold to use when breastfeeding. Which picture should the nurse use to explain this position to the new mother?

1) A nursing instructor is demonstrating how to perform a heel stick on a newborn. To obtain an accurate capillary hematocrit reading, what does the nursing instructor tell the student do? A) Rub the heel vigorously with an isopropyl alcohol swab prior to obtaining blood. B) Use a previous puncture site. C) Cool the heel prior to obtaining blood. Use a sterile needle and aspirate

1) A nursing instructor is demonstrating how to perform a heel stick on a newborn. To obtain an accurate capillary hematocrit reading, what does the nursing instructor tell the student do? A) Rub the heel vigorously with an isopropyl alcohol swab prior to obtaining blood. B) Use a previous puncture site. C) Cool the heel prior to obtaining blood. D) Use a sterile needle and aspirate. Answer: A Explanation: A) The site should be cleaned by rubbing vigorously with a 70% isopropyl alcohol swab. The friction produces local heat, which aids vasodilation. B) A microlancet is used to make the puncture in an unpunctured site. C) The heel should not be cooled. D) A microlancet, not a needle, is used to make the puncture.

Answer: 2, 3 Explanation: The epidural space lies between the dura mater and the ligamentum flavum, extending from the base of the skull to the end of the sacral canal.

1) A patient in labor is being prepared for an epidural. Where should the nurse indicate the location of the epidural space on the diagram below? 1. A 2. B 3. C 4. D

Answer: A Explanation: A) This is a picture of local infiltration anesthesia which is used to repair an episiotomy. This anesthesia is not used to deliver the placenta, insert a urinary catheter, or conduct an intrapartum vaginal examination.

1) A patient is having the following anesthesia provided. What is the purpose of this type of anesthesia? A) Repair of an episiotomy B) Delivery of the placenta C) Insertion of a urinary catheter D) Intrapartum vaginal examination

Answer: C Explanation: C) A lower uterine segment vertical incision (Sellheim) is preferred for multiple gestations. The lower uterine segment transverse incision (Kerr) is the most commonly used and easiest to repair, has less blood loss, and is less likely to rupture with future pregnancies.

1) A patient is scheduled to have the following type of incision for a cesarean birth. What advantage should the nurse explain to the patient that this type of incision has? A) Easier to repair B) Less blood loss C) Expedites delivery of multiple fetuses D) Less likely to rupture with future pregnancies

Answer: A Explanation: A) Many practitioners choose to deliver any nonvertex presentation via cesarean birth. In 20% of cases, twin A is nonvertex and requires a cesarean birth. Approximately 40% of twins present in a vertex/vertex presentation. More than 80% of twins in vertex/vertex presentation are born vaginally. Since the patient will need a cesarean birth, spinal block and continuous lumbar epidural will not be indicated.

1) A patient pregnant with twins late in the 3rd trimester has an ultrasound that shows the position of the fetuses as follows. What should the nurse expect will be planned for this patient? A) Cesarean birth B) Vaginal delivery C) Spinal block during labor D) Continuous lumbar epidural

Answer: C Explanation: C) This diagram demonstrates complete separation with massive vaginal bleeding. In central separation the placenta separates centrally, and the blood is trapped between the placenta and the uterine wall. Entrapment of the blood results in concealed bleeding. In marginal separation blood passes between the fetal membranes and the uterine wall and escapes vaginally. Separation begins at the periphery of the placenta; this marginal sinus rupture may or may not become more severe. Anticipated separation is not a type of placental separation.

1) A pregnant patient is diagnosed with premature separation of the placenta. The nurse provides the patient with the following diagram. What amount of placenta separation is this patient experiencing? A) Central B) Marginal C) Complete D) Anticipated

Answer: B Explanation: B) Cerclage is a surgical procedure in which a stitch is placed in the cervix to prevent a spontaneous abortion or premature birth. After placement, the string is tightened and secured anteriorly. This procedure is not used to correct marginal placenta previa, reduce the risk of developing central abruptio placentae, or help with fetal rotation for delivery.

1) A pregnant patient is scheduled to have the procedure depicted in the diagram performed. What should the nurse explain is the purpose for this procedure? A) Correct marginal placenta previa B) Prevent preterm cervical dilatation and pregnancy loss C) Reduce the risk of developing central abruptio placentae D) Assist the fetus to rotate into the appropriate position for delivery

Answer: B Explanation: B) In a pneumothorax, rupture of the alveoli sacs allows air to leak through the pleura, forming collections of air outside the lung. Air shows on X-ray as a dark area over the lung.

1) An infant with a pneumothorax has the following chest x-ray result. Which area should the nurse identify as being the pneumothorax? A) A B) B C) C D) D

Answer: A Explanation: A) On vaginal examination of a breech presentation the nurse may feel the anal sphincter. The tissue of the fetal buttocks feels soft. In a shoulder presentation the nurse may feel the acromion process as the fetal presenting part. In the occiput face presentation the nurse may palpate the nose. In the occiput brow presentation the nurse may palpate the forehead.

1) During an intrapartum vaginal examination the following is palpated. In which type of presentation is this fetus? A) Breech B) Shoulder C) Occiput face D) Occiput brow

Answer: C Explanation: When the posterior fontanelle is in the upper right quadrant of the maternal pelvis, the fetus is in the right occiput anterior (ROA) position. Choice 1: When the fetus is in left occiput anterior (LOA) position, the posterior fontanelles are in the upper left quadrant of the maternal pelvis. Choice 2: When the fetus is in the left occiput posterior (LOP) position, the posterior fontanelle is in the lower left quadrant of the maternal pelvis. Choice 4: When the fetus is in the right occiput posterior (ROP) position, the posterior fontanelle is in the lower right quadrant of the maternal pelvis

1) During an intrapartum vaginal examination the nurse palpates the following (see image). Based upon this information, which diagram best describes the fetal descent and station?

1) During newborn resuscitation, how does the nurse evaluate the effectiveness of bag-and-mask ventilations? A) The rise and fall of the chest B) Sudden wakefulness C) Urinary output D) Adequate thermoregulation

1) During newborn resuscitation, how does the nurse evaluate the effectiveness of bag-and-mask ventilations? A) The rise and fall of the chest B) Sudden wakefulness C) Urinary output D) Adequate thermoregulation Answer: A Explanation: A) With proper resuscitation, chest movement is observed for proper ventilation. Pressure should be adequate to move the chest wall. B) Sudden wakefulness is not associated with effectiveness of bag-and-mask ventilations. C) Urinary output is not associated with effectiveness of bag-and-mask ventilations. Adequate thermoregulation is not associated with effectiveness of bag-and-mask ventilations

Answer: B Explanation: B) During uterine contractions, traction is applied to the forceps in a downward and outward direction to follow the birth canal. Upward, midline, or towards the left or right do not follow the direction of the birth canal.

1) Forceps are being used to deliver the fetus of a laboring patient, as identified in the diagram. In which direction should the healthcare provider use the forceps to guide the fetus for delivery? A) Upward and outward B) Downward and outward C) Midline and towards the left D) Midline and toward the right

1) The community nurse is planning care for a family that experienced the loss of twins at 20 weeks. Which of these steps should be part of the nurse's care of this family? A) Base care on the reactions of previous clients who experienced stillbirth. B) Express the belief that the family will be able to get through this experience. C) Encourage the couple to keep their feelings to themselves. Honor the birth by reminding the couple that their babies are happy in heaven

1) The community nurse is planning care for a family that experienced the loss of twins at 20 weeks. Which of these steps should be part of the nurse's care of this family? A) Base care on the reactions of previous clients who experienced stillbirth. B) Express the belief that the family will be able to get through this experience. C) Encourage the couple to keep their feelings to themselves. Honor the birth by reminding the couple that their babies are happy in heaven

Answer: D Explanation: D) The head has completed internal rotation. The cervix is fully dilated. In Choice 1, the fetus is at station 0 with the head engaged at the spine and a significant amount of cervix that has not yet dilated. In Choice 2, the fetus is engaged at station +2 as the cervix is thinner and more of the fetal head can be palpated. In Choice 3. the fetus is engaged at station +4 with the posterior fontanelle towards the maternal left pelvis.

1) The fetus of a laboring patient is in the following position: What should the nurse expect when conducting the intrapartum vaginal examination?

Answer: C Explanation: C) The heart tones for the right occiput anterior position would be in the maternal lower right quadrant next to the symphysis pubis. Choice 1 is the location to auscultate sounds if the fetus is in the left occiput posterior position. Choice 2 is the location to auscultate sounds if the fetus is in the left occiput anterior position. Choice 4 is the location to auscultate sounds if the fetus is in the right occiput posterior position.

1) The fetus of a pregnant patient is in the right occiput anterior (R O A) position. Where should the nurse auscultate the fetal heart sounds? A) A B) B C) C D) D

Answer: B Explanation: B) The mediolateral episiotomy begins in the midline of the posterior fourchette (to avoid incision into the Bartholin gland) and extends at a 45-degree angle downward to the right or left (the direction depending on the handedness of the clinician).

1) The healthcare provider of a laboring patient is considering a mediolateral episiotomy. On the following diagram, identify where this incision should be made. A) A B) B C) C D) D

Answer: 1, 3, 4, 5 Explanation: The infant has a cleft lip. The nurse should feed with a special nipple and bottle, burp frequently, clean the cleft with sterile water to prevent crusting on cleft before repair, and support parental coping. Assessing patency of nares would be appropriate for choanal atresia.

1) The nurse is caring for a newborn with the following anomaly. What actions should the nurse take when caring for this infant? Note: Credit will be given only if all correct choices and no incorrect choices are selected. Select all that apply. 1. Burp frequently 2. Assess patency of nares 3. Assist with parental coping 4. Clean the area with sterile water 5. Feed with a special nipple and bottle

Answer: C Explanation: C) When measuring gavage tube length, measure the distance from the tip of the ear to the nose to the midpoint between the xiphoid process and the umbilicus, and mark the point with a small piece of paper tape to ensure there is enough tubing to enter the stomach.

1) The nurse is observed conducting the following measurement. For what will this measurement be used? A) Determine fetal length B) Gauge stomach contents C) Placement of gavage tube D) Estimate chest circumference

Answer: A Explanation: A) The diagram is of a partial placenta previa. The internal o s is partially covered by the placenta. In total placenta previa the internal o s is covered completely by the placenta. In marginal placenta previa the edge of the placenta is at the margin of the internal o s. In low-lying placenta previa the placenta is implanted in the lower segment but does not reach the o s, although it is in close proximity of it.

1) The nurse is preparing teaching material for a pregnant patient with the following type of placenta previa. What information should the nurse provide the patient about this health problem? A) Internal o s is partially covered by the placenta B) Internal o s is covered completely by the placenta C) Edge of the placenta is at the margin of the internal o s D) Placenta is implanted in the lower segment but does not reach the o s

Answer: C Explanation: C) In a diaphragmatic hernia, a portion of the intestines enters the thoracic cavity through an abnormal opening in the diaphragm. This most commonly occurs on the left side. An omphalocele is the herniation of abdominal contents into the base of the umbilical cord. Gastroschisis is a full-thickness defect in the abdominal wall that allows viscera outside the body to the right of an intact umbilical cord. Prune belly syndrome is the congenital absence of one or more layers of abdominal muscles.

1) The nurse is preparing teaching material for the parents of a newborn. For which health problem should the nurse select the following diagram to be used for teaching? A) Gastroschisis B) Omphalocele C) Diaphragmatic hernia D) Prune belly syndrome

Answer: C Explanation: C) A lumbar epidural block is placed within the epidural space and affects a larger area of nerves to include the uterus, cervix, vagina, and perineum. In this diagram the dark area demonstrates the peridural (epidural) space and nerves affected, and the gray tube represents a continuous plastic catheter. A lumbar sympathetic (spinal) block relieves uterine pain only. A pudendal block relieves perineal pain only. Local infiltration affects the perineum only.

1) The nurse is preparing to instruct a patient in the third trimester of pregnancy on various methods to control pain during labor, and selects the following diagram. For which type of pain control is the nurse planning to instruct the client? A) Lumbar spinal block B) Pudendal block C) Lumbar epidural block D) Local infiltration

Answer: 1, 5 Explanation: The infant's lateral heel is the site of choice because it precludes damaging the posterior tibial nerve and artery, plantar artery, and the important longitudinally oriented fat pad of the heel, which in later years could impede walking

1) The nurse is preparing to measure a newborn's blood glucose level. Which areas on the heel should be used to obtain a blood sample for this test? 1. A 2. B 3. C 4. D 5. E

Answer: D Explanation: D) This picture diagrams the position that the vertebrae need to be in for insertion of the epidural. Preparation of the skin prior to the insertion of the catheter would not be completed with a needle and syringe. The length of the needle to be inserted is not usually a part of patient teaching about the procedure. The vertebrae are not rotated forward in this picture.

1) The nurse is reviewing the procedure for a continuous lumbar epidural block with a patient in the 3rd trimester of pregnancy. When reviewing this information what does the following diagram demonstrate to the patient? A) Preparation of the skin prior to the insertion of the catheter B) Status of the vertebra when the patient is in the correct position C) The length of the needle to be inserted in the subarachnoid space D) Vertebrae rotated forward when the patient is side-lying without a pillow

1) The nurse is working with a family who experienced the stillbirth of a son 2 months ago. Which statement by the mother would be expected? A) "I seem to keep crying for no reason." B) "The death of my son hasn't changed my life." C) "I have not visited my son's gravesite." D) "I feel happy all the time."

1) The nurse is working with a family who experienced the stillbirth of a son 2 months ago. Which statement by the mother would be expected? A) "I seem to keep crying for no reason." B) "The death of my son hasn't changed my life." C) "I have not visited my son's gravesite." D) "I feel happy all the time." Answer: A Explanation: A) Mourning may be manifested by certain behaviors and rituals, such as weeping, which help the person experience, accept, and adjust to the loss. B) A fetal loss is devastating to parents, and results in significant life changes. C) Mourning may be manifested by certain behaviors and rituals, such as visiting a gravesite, which can help the person experience, accept, and adjust to the loss. Happiness is not an expected part of mourning

Answer: B Explanation: B) This is a Plastibell, which is used for circumcision. This device is not used to assess hearing, conduct a neurological exam, or to assess for an umbilical hernia.

1) The nurse obtains the following device to be used on a newborn. For which procedure is the nurse preparing this infant? A) Hearing test B) Circumcision C) Neurological exam D) Assess for umbilical hernia

Answer: D Explanation: This infant is receiving phototherapy. The phototherapy light is positioned over the incubator. Bilateral eye patches are always used during phototherapy to protect the baby's eyes. Eye coverings are not needed for hypoglycemia, hyperglycemia, or hypotension

1) The nurse sees the following patient in the neonatal intensive care unit and realizes that additional care will need to be provided for which health problem? A) Hypotension B) Hypoglycemia C) Hyperglycemia D)Hyperbilirubinemia

1) The pregnant client at 41 weeks is scheduled for labor induction. She asks the nurse whether induction is really necessary. What response by the nurse is best? A) "Babies can develop postmaturity syndrome, which refers to a number of complications that can occur after 42 weeks of pregnancy." B) "When infants are born 2 or more weeks after their due date, they have meconium in the amniotic fluid." C) "Sometimes the placenta ages excessively, and we want to take care of that problem before it happens." D) "The doctor wants to be proactive in preventing any problems with your baby if he gets any bigger."

Answer: A Explanation: A) The term postmaturity applies to the infant who is born after 42 completed weeks of gestation and demonstrates characteristics of postmaturity syndrome. B) Although this statement is partially true, meconium-stained amniotic fluid is not always present or the only complication of postmaturity syndrome. C) Although this statement is true, it is too vague. It is better to be specific and call postmaturity syndrome by its name. D) Although this is true, the answer is incomplete. The risk of postmaturity syndrome is also an issue.

1) The nurse assesses the newborn's ears to be parallel to the outer and inner canthus of the eye. The nurse documents this finding to be which of the following? A) A normal position B) A possible chromosomal abnormality C) Facial paralysis D) Prematurity

Answer: A Explanation: A) The top of the ear (pinna) is parallel to the outer and inner canthus of the eye in the normal newborn. B) Low-set ears could indicate a chromosomal abnormality. C) This ear position is not indicative of facial paralysis. D) This ear position is not indicative of prematurity.

1) Which term will the nurse use when teaching a client information regarding the entire female external genitalia? A) Vulva B) Clitoris C) Mons pubis D) Perineal body

Answer: A Explanation: A) The vulva is the term the nurse will use when documenting information about the entire female external genitalia. B) The clitoris is a structure included in the female external genitalia. This term is not used when referring to the entire female external genitalia. C) The mons pubis is a structure included in the female external genitalia. This term is not used when referring to the entire female external genitalia. The perineal body is a structure included in the female external genitalia. This term is not used when referring to the entire female external genitalia

11) The nurse is preparing for a postpartum home visit. The client has been home for a week, is breastfeeding, and experienced a third-degree perineal tear after vaginal delivery. The nurse should assess the client for which of the following? A) Dietary intake of fiber and fluids B) Dietary intake of folic acid and prenatal vitamins C) Return of hemoglobin and hematocrit levels to baseline D) Return of protein and albumin to predelivery levels

Answer: A Explanation: A) This mother needs to avoid the risk of constipation. She might be hesitant to have a bowel movement due to anticipated pain from the perineal tear, and constipation will decrease the healing of the laceration. B) Dietary intake of prenatal vitamins is important while breastfeeding, but folic acid is more important prior to conception and in the first weeks of pregnancy to prevent neural tube defects. C) It will take several months for the laboratory levels to return to normal. It will take several months for the laboratory levels to return to normal

1) At birth, an infant weighed 6 pounds 12 ounces. Three days later, he weighs 5 pounds 2 ounces. What conclusion should the nurse draw regarding this newborn's weight? A) This weight loss is excessive. B) This weight loss is within normal limits. C) This weight gain is excessive. D) This weight gain is within normal limits.

Answer: A Explanation: A) This newborn has lost more than 10% of the birth weight; this weight loss is excessive. Following birth, caloric intake is often insufficient for weight gain until the newborn is 5 to 10 days old. During this time there may be a weight loss of 5% to 10% in term newborns. B) This weight loss is greater than the expected 5% to 10%. C) This is not a weight gain. D) This is not a weight gain.

1) The nurse is preparing new parents for discharge with their newborn. The father asks the nurse why the baby's head is so pointed and puffy-looking. What is the best response by the nurse? A) "His head is molded from fitting through the birth canal. It will become more round." B) "We refer to that as 'cone head,' which is a temporary condition that goes away." C) "It might mean that your baby sustained brain damage during birth, and could have delays." D) "I think he looks just like you. Your head is much the same shape as your baby's."

Answer: A Explanation: A) This statement is accurate and directly answers the father's question. B) Although nursing staff might refer to a molded head as "cone head" and the shape is temporary, it is better to be more specific in explaining why the head is shaped as it is. In addition, this answer does not answer the "why" question posed by the father. C) A molded head shape does not indicate brain damage. Molding is normal and transient. Although this might be true, it is better to give a factual answer that does not imply that the father's head is abnormally shaped. This answer could be perceived as insulting

6) The laboring client presses the call light and reports that her water has just broken. What would the nurse's first action be? A) Check fetal heart tones. B) Encourage the mother to go for a walk. C) Change bed linens. D) Call the physician.

Answer: A Explanation: A) When the membranes rupture, the nurse notes the color and odor of the amniotic fluid and the time of rupture and immediately auscultates the F H R. B) If there has been a rupture of membranes, the laboring client should not be allowed to walk. C) The bed linens can be changed after assessing the heart rate. D) The physician does not need to be called after rupture of the membranes unless there is a change in the status of the fetus or client.

1) The nurse is preparing to administer postpartum neuraxial morphine to a client who is morbidly obese. For which side effect related to morbid obesity should the nurse monitor? A) Respiratory depression B) Confusion C) Constipation D) Hypotension

Answer: A Explanation: A) Women who are morbidly obese are at increased risk for respiratory depression, which is managed by administration of naloxone (Narcan), mask ventilation, and endotracheal intubation with mechanical ventilation, if necessary. B) The client is not at increased risk of confusion, as this is not affected by postpartum neuraxial morphine administration in the context of morbid obesity. C) The client is not at increased risk of constipation, as this is not affected by postpartum neuraxial morphine administration in the context of morbid obesity. D) The client is not at increased risk of hypotension, as this is not affected by postpartum neuraxial morphine administration in the context of morbid obesity.

1) A patient with female genital mutilation is being prepared for a gynecologic examination. What cultural implications should the nurse keep in mind when assisting with this examination? A) Maintain a nonjudgmental attitude B) The procedure was performed by choice C) The patient is at increased risk for genital infection D) The procedure was performed to increase sexual satisfaction

Answer: A Explanation: A) Women who immigrate to other countries may feel a sense of shame or embarrassment once the procedure has been performed. The nurse needs to maintain a nonjudgmental attitude to support this patient's cultural needs. Most female genital mutilations are not performed by choice. They can increase the patient's risk for genital infection however this would not support the patient's cultural needs. Female genital mutilation is performed to reduce sexual satisfaction.

5) The nurse evaluates the diet of a pregnant client and finds that it is low in zinc. The nurse knows that zinc intake should increase during pregnancy to promote protein metabolism. Which food should the nurse suggest in order to increase intake of zinc? A) Shellfish B) Bananas C) Yogurt D) Cabbage

Answer: A Explanation: A) Zinc is found in greatest concentration in meats, shellfish, and poultry. Other good sources include whole grains and legumes. B) Bananas are high in other nutrients, but do not have significant levels of zinc. C) Yogurt is high in other nutrients, but does not have significant levels of zinc. D) Cabbage is high in other nutrients, but does not have significant levels of zinc.

1) The nurse is cross-training maternal-child health unit nurses to provide home-based care for parents after discharge. Which statements indicate that additional teaching is required? Note: Credit will be given only if all correct choices and no incorrect choices are selected. Select all that apply. A) "The behavioral assessment should be done as soon after birth as possible." B) "The behavioral assessment can be performed without input from parents." C) "The behavioral assessment might be incomplete in a 1-hour home visit." D) "The behavioral assessment includes orientation and motor activity." E) "The behavioral assessment can detect neurological impairments."

Answer: A, B Explanation: A) Because the first few days after birth are a period of behavioral disorganization, the complete assessment should be done on the third day after birth. B) Parental input is required. It provides a way for the healthcare provider, in conjunction with the parents (primary caregivers), to identify and understand the individual newborn's states, temperament, capabilities, and individual behavior patterns. C) A full behavioral assessment includes the nurse observing the newborn's sleep-wake patterns, which is not likely to take place in a 1-hour home visit. D) Orientation to visual and auditory clues and motor activity are portions of the behavioral assessment. E) The behavioral assessment can detect neurological impairments.

During a wellness visit, a 50-year-old female experiencing menopause says that she jogs three times a week and feels like her symptoms are becoming worse. What should the nurse recommend to help with the discomfort of menopause? Note: Credit will be given only if all correct choices and no incorrect choices are selected. Select all that apply. A) Yoga B) Tai chi C) Meditation D) Weight lifting E) Kegel exercises

Answer: A, B, C Explanation: A) A variety of therapeutic modalities have been proposed as treatment or prevention measures for the discomforts and ailments of the perimenopausal and postmenopausal years, including mind-body practices such as yoga, tai chi, and meditation. Weight lifting helps maintain bone mass caused by the reduction in estrogen. Kegel exercises help maintain vaginal muscle tone and increase blood circulation to the perineal area. B) A variety of therapeutic modalities have been proposed as treatment or prevention measures for the discomforts and ailments of the perimenopausal and postmenopausal years, including mind-body practices such as yoga, tai chi, and meditation. Weight lifting helps maintain bone mass caused by the reduction in estrogen. Kegel exercises help maintain vaginal muscle tone and increase blood circulation to the perineal area. C) A variety of therapeutic modalities have been proposed as treatment or prevention measures for the discomforts and ailments of the perimenopausal and postmenopausal years, including mind-body practices such as yoga, tai chi, and meditation. Weight lifting helps maintain bone mass caused by the reduction in estrogen. Kegel exercises help maintain vaginal muscle tone and increase blood circulation to the perineal area. D) A variety of therapeutic modalities have been proposed as treatment or prevention measures for the discomforts and ailments of the perimenopausal and postmenopausal years, including mind-body practices such as yoga, tai chi, and meditation. Weight lifting helps maintain bone mass caused by the reduction in estrogen. Kegel exercises help maintain vaginal muscle tone and increase blood circulation to the perineal area. A variety of therapeutic modalities have been proposed as treatment or prevention measures for the discomforts and ailments of the perimenopausal and postmenopausal years, including mind-body practices such as yoga, tai chi, and meditation. Weight lifting helps maintain bone mass caused by the reduction in estrogen. Kegel exercises help maintain vaginal muscle tone and increase blood circulation to the perineal area

1) During the assessment phase of a family, the community nurse recognizes that culture influences childrearing and childbearing in which of the following ways? Note: Credit will be given only if all correct choices and no incorrect choices are selected. Select all that apply. A) Beliefs about the importance of children B) Beliefs and attitudes about pregnancy C) Norms regarding infant feeding D) Acculturation is important in rearing children E) Time orientation to the future is very important

Answer: A, B, C Explanation: A) Culture influences beliefs about the importance of children. B) Culture influences attitudes about pregnancy and the right vs. the obligation of women to bear children. C) Culture influences infant feeding norms and practices. D) Acculturation is not important in rearing children. E) Time orientation is a cultural difference and can emphasize the past, present, or future. It does not influence childbearing and childrearing.

What issues should the nurse consider when counseling a client on contraceptive methods? Note: Credit will be given only if all correct choices and no incorrect choices are selected. Select all that apply. A) Cultural perspectives on menstruation and pregnancy B) Effectiveness of the method C) Future childbearing plans D) Whether the client is a vegetarian E) Age at menarche

Answer: A, B, C Explanation: A) Decisions about contraception should be made voluntarily with full knowledge of advantages, disadvantages, effectiveness, side effects, contraindications, and long-term effects. Many outside factors influence this choice, including cultural practices, religious beliefs, personality, cost, effectiveness, availability, misinformation, practicality of method, and self-esteem. B) Decisions about contraception should be made voluntarily with full knowledge of advantages, disadvantages, effectiveness, side effects, contraindications, and long-term effects. Many outside factors influence this choice, including cultural practices, religious beliefs, personality, cost, effectiveness, availability, misinformation, practicality of method, and self-esteem. C) Decisions about contraception should be made voluntarily with full knowledge of advantages, disadvantages, effectiveness, side effects, contraindications, and long-term effects. Many outside factors influence this choice, including cultural practices, religious beliefs, personality, cost, effectiveness, availability, misinformation, practicality of method, and self-esteem. D) Vegetarianism has no impact on contraceptive method use. E) Age at menarche has no impact on contraceptive method use.

1) The nurse is performing an assessment on a client admitted to the birthing unit. Which assessment finding(s) contraindicate(s) an epidural block? Note: Credit will be given only if all correct choices and no incorrect choices are selected. Select all that apply. A) Maternal refusal B) Local infection of the skin on the lower back C) Coagulation disorder D) Long-term N S A I D use Previous back surgery

Answer: A, B, C Explanation: A) Maternal refusal is an absolute contraindication to an epidural block. B) Local infection is an absolute contraindication to an epidural block. C) Coagulation disorders are an absolute contraindication to an epidural block. D) Long-term N S A I D use may be cause for concern that needs to be evaluated on an individual basis. It is not considered a contraindication for an epidural block. E) Previous back surgery may be cause for concern that needs to be evaluated on an individual basis. It is not considered a contraindication for an epidural block.

1) Which serum markers are assessed when conducting a quadruple screen? Note: Credit will be given only if all correct choices and no incorrect choices are selected. Select all that apply. A) Alpha-fetoprotein (A F P) B) Human chorionic gonadotropin (h C G) C) Unconjugated estriol (U E) D) Inhibin-A E) Glycated hemoglobin

Answer: A, B, C, D Explanation: A) A quadruple screen assesses for the serum marker of A F P. B) A quadruple screen assesses for the serum marker of h C G. C) A quadruple screen assesses for the serum marker of U E. D) A quadruple screen assesses the serum marker of inhibin-A. E) A quadruple screen does not assess for glycated hemoglobin.

1) Risk factors for tachysystole include which of the following? Note: Credit will be given only if all correct choices and no incorrect choices are selected. Select all that apply. A) Cocaine use B) Placental abruption C) Low-dose oxytocin titration regimens D) Uterine rupture E) Smoking

Answer: A, B, D Explanation: A) Cocaine use is a risk factor for tachysystole. B) Placental abruption is a risk factor for tachysystole. C) High-dose oxytocin titration regimens are a risk factor for tachysystole. D) Uterine rupture is a risk factor for tachysystole. E) Smoking is not risk factor for tachysystole.

1) Antibiotics have been ordered for a newborn with an infection. Which interventions would the nurse prepare to implement? Note: Credit will be given only if all correct choices and no incorrect choices are selected. Select all that apply. A) Obtain skin cultures. B) Restrict parental visits. C) Evaluate bilirubin levels. D) Administer oxygen as ordered. Observe for signs of hypoglycemia

Answer: A, C, D, E Explanation: A) The nurse will assist in obtaining skin cultures. Skin cultures are taken of any lesions or drainage from lesions or reddened areas. B) Restricting parental visits has not been shown to have any effect on the rate of infection and may be harmful to the newborn's psychologic development. C) The nurse will observe for hyperbilirubinemia, anemia, and hemorrhagic symptoms. D) The nurse will administer oxygen as ordered. E) ) The nurse will observe for signs of hypoglycemia.

1) A new mother is concerned because the anterior fontanelle swells when the newborn cries. Explaining normal findings concerning the fontanelles, the nurse states which of the following? Note: Credit will be given only if all correct choices and no incorrect choices are selected. Select all that apply. A) The fontanelles can swell with crying. B) The fontanelles might be depressed. C) The fontanelles can pulsate with the heartbeat. D) The fontanelles might bulge. E) The fontanelles can swell when stool is passed.

Answer: A, C, E Explanation: A) Newborn fontanelles can swell when the newborn cries. B) Depressed fontanelles indicate dehydration. C) Newborn fontanelles can pulsate with the heartbeat. D) Bulging fontanelles signify increased intracranial pressure. E) Newborn fontanelles can swell when the newborn passes a stool.

1) The nurse is caring for the newborn of a drug-addicted mother. Which assessment findings would be typical for this newborn? Note: Credit will be given only if all correct choices and no incorrect choices are selected. Select all that apply. A) Hyperirritability B) Decreased muscle tone C) Exaggerated reflexes D) Low pitched cry E) Transient tachypnea

Answer: A, C, E Explanation: A) Newborns born to drug-addicted mothers exhibit hyperirritability. B) Newborns born to drug-addicted mothers show increased, not decreased, muscle tone. C) Newborns born to drug-addicted mothers exhibit exaggerated reflexes. D) Newborns born to drug-addicted mothers exhibit a high-pitched, not a low-pitched, cry. E) Newborns born to drug-addicted mothers exhibit transient tachypnea.

1) The nurse is admitting a client who was diagnosed with hydramnios. The client asks why she has developed this condition. The nurse should explain that hydramnios is sometimes associated with which of the following? Note: Credit will be given only if all correct choices and no incorrect choices are selected. Select all that apply. A) Rh sensitization B) Postmaturity syndrome C) Renal malformation or dysfunction D) Maternal diabetes E) Large-for-gestational-age infants

Answer: A, D Explanation: A) Hydramnios is associated with R h sensitization. B) Postmaturity is associated with oligohydramnios. C) Renal malformation or dysfunction is associated with oligohydramnios. D) Hydramnios is associated with maternal diabetes. E) Large-for-gestational-age infants are not associated with hydramnios.

1. The nurse is teaching a prenatal class about false labor. The nurse should teach clients that false labor will most likely include which of the following? Note: Credit will be given only if all correct choices and no incorrect choices are selected. Select all that apply. A) Contractions that do not intensify while walking B) An increase in the intensity and frequency of contractions C) Progressive cervical effacement and dilation D) Pain in the abdomen that does not radiate E) Contractions are at regular intervals

Answer: A, D Explanation: A) True labor contractions intensify while walking. B) True labor results in increased intensity and frequency of contractions. C) True labor results in progressive dilation. D) The discomfort of true labor contractions usually starts in the back and radiates around to the abdomen. E) With false labor, contractions are irregular.

1) Many newborns exposed to H I V/A I D S show signs and symptoms of disease within days of birth that include which of the following? Note: Credit will be given only if all correct choices and no incorrect choices are selected. Select all that apply. A) Swollen glands B) Hard stools C) Smaller than average spleen and liver D) Rhinorrhea E) Interstitial pneumonia

Answer: A, D, E Explanation: A) Signs that may be seen in the early infancy period include swollen glands. B) Signs that may be seen in the early infancy period include recurrent gastrointestinal (G I) problems that include diarrhea. C) Signs that may be seen in the early infancy period include enlarged spleen and liver. D) Signs that may be seen in the early infancy period include rhinorrhea. Signs that may be seen in the early infancy period include interstitial pneumonia

1) The nurse is preparing an educational in-service presentation about jaundice in the newborn. What content should the nurse include in this presentation? Note: Credit will be given only if all correct choices and no incorrect choices are selected. Select all that apply. A) Physiologic jaundice occurs after 24 hours of age. B) Pathologic jaundice occurs after 24 hours of age. C) Phototherapy increases serum bilirubin levels. D) The need for phototherapy depends on the bilirubin level and age of the infant. Kernicterus causes irreversible neurological damage

Answer: A, E Explanation: A) Physiologic or neonatal jaundice is a normal process that occurs during transition from intrauterine to extrauterine life, and appears after 24 hours of life. B) Diagnosis of pathologic jaundice is given to newborns who exhibit jaundice within the first 24 hours of life. C) Phototherapy decreases serum bilirubin levels. D) The decision to start phototherapy is based on two factors: gestational age and age in hours. E) Kernicterus refers to the deposition of unconjugated bilirubin in the basal ganglia of the brain and to permanent neurologic sequelae of untreated hyperbilirubinemia.

24) The pregnant teen who was prescribed prenatal vitamins at her initial prenatal visit states that she does not like to take them. How should the nurse respond? Note: Credit will be given only if all correct choices and no incorrect choices are selected. Select all that apply. A) "Folic acid has been found to be essential for minimizing the risk of neural tube defects." B) "You do not have to take these supplements if you think you are healthy enough." C) "These medications do the same thing. I will call your doctor to cancel one of your medications." D) "You can trust your doctor to know what you need." E) "You need the supplements because your dietary intake may not be adequate for fetal development."

Answer: A, E Explanation: A) The C D C estimates that most neural tube defects could be prevented if women followed folic acid supplementation recommendations before they became pregnant. B) One role of the nurse is educator, and this client needs additional information on why she needs the supplements. C) One role of the nurse is educator, and this client needs additional information on why she needs the supplements. D) Therapeutic communication requires addressing the client's concern. One role of the nurse is educator, and this client needs additional information on why she needs the supplements. E) One role of the nurse is educator, and this client needs additional information on why she needs the supplements. This response answers the client's concerns.

1) The nurse assesses a sleeping 1-hour-old, 39-weeks'-gestation newborn. The assessment data that would be of greatest concern would be which of the following? A) Temperature 97.9°F B) Respirations 68 breaths/minute C) Stable vital signs 45 minutes ago D) Heart rate 156 beats/min

Answer: B Explanation: A) 97.9°F is within the normal temperature range of 97.5-99°F. B) The normal respiratory rate is 30-60 breaths/min; 68 breaths/min could represent a less-than-ideal transition. C) The vital signs for a healthy term newborn should be monitored at least every 30 minutes until the newborn's condition has remained stable for 2 hours. D) This heart rate is within the normal range of 110-160 beats/min.

1. A postpartum woman is at increased risk for developing urinary tract problems because of which of the following? A) Decreased bladder capacity B) Inhibited neural control of the bladder following the use of anesthetic agents C) Increased bladder sensitivity D) Abnormal postpartum diuresis

Answer: B Explanation: A) A postpartum woman is at increased risk for developing urinary tract problems because of increased bladder capacity. B) A postpartum woman is at increased risk for developing urinary tract problems because of inhibited neural control of the bladder following the use of anesthetic agents. C) A postpartum woman is at increased risk for developing urinary tract problems because of decreased bladder sensitivity from stretching or trauma. D) A postpartum woman is at increased risk for developing urinary tract problems because of normal postpartum diuresis.

The nurse is seeing clients in the women's clinic. Which client should be treated with ceftriaxone I M and doxycycline orally? A) A pregnant client with gonorrhea and a yeast infection B) A nonpregnant client with gonorrhea and chlamydia C) A pregnant client with syphilis D) A nonpregnant client with chlamydia and trichomoniasis

Answer: B Explanation: A) A pregnant client would not be treated with doxycycline. B) The combined treatment of ceftriaxone I M and doxycycline orally provides dual treatment for gonorrhea and chlamydia, which frequently occur together. C) The combined treatment of ceftriaxone I M and doxycycline orally is not the correct treatment for syphilis, and a pregnant client would not be treated with doxycycline. D) The combined treatment of ceftriaxone I M and doxycycline orally is not the correct treatment for trichomoniasis.

1) A 16-year-old is making her first prenatal visit to the clinic in her fourth month of pregnancy. What is the nurse's first responsibility? A) Contact the social worker. B) Develop a trusting relationship. C) Schedule the client for prenatal classes. D) Teach the client about proper nutrition.

Answer: B Explanation: A) A social worker might be able to provide assistance, but contacting one is not the first priority. B) Developing a trusting relationship with the pregnant adolescent is essential. Honesty and respect for the individual and a caring attitude promote the client's self-esteem. C) Childbirth education is important, however, scheduling classes is not the first goal when dealing with a pregnant teen. D) Although nutrition is an important physiologic need, it is not the first priority in dealing with the pregnant adolescent.

1) The client at 39 weeks' gestation is undergoing a cesarean birth due to breech presentation. General anesthesia is being used. Which situation requires immediate intervention? A) The baby's hands and feet are blue at 1 minute after birth. B) The fetal heart rate is 70 prior to making the skin incision. C) Clear fluid is obtained from the baby's oropharynx. D) The neonate cries prior to delivery of the body.

Answer: B Explanation: A) Acrocyanosis is an expected finding at 1 minute of age. B) Fetal bradycardia occurs when the fetal heart rate falls below 110 beats/minute during a 10-minute period of continuous monitoring. When fetal bradycardia is accompanied by decreased variability, it is considered ominous and could be a sign of fetal compromise. C) Clear fluid from the baby's oropharynx is an expected finding. D) A primary danger of general anesthesia is fetal depression. Crying after delivery of just the head indicates that no neonatal depression has occurred.

1. To prevent the spread of infection, the nurse teaches the postpartum client to do which of the following? A) Address pain early B) Change peri-pads frequently C) Avoid overhydration D) Report symptoms of uterine cramping

Answer: B Explanation: A) Addressing pain early would not be a preventive action for infection. B) Changing peri-pads frequently decreases skin contact with a moist medium that favors bacteria growth. C) Avoiding overhydration actually would increase the risk for infection by not providing adequate fluids to flush the kidneys and bladder. D) Reporting symptoms of uterine cramping would not be a preventive action for infection.

1) The nurse suspects clubfoot in the newborn and assesses for the condition by doing which of the following? A) Adducting the foot and listening for a click. B) Moving the foot to midline and determining resistance. C) Extending the foot and observing for pain. D) Stimulating the sole of the foot.

Answer: B Explanation: A) Adducting the foot and listening for a click is not a typical assessment. B) Clubfoot is suspected when the foot does not turn to a midline position or align readily. C) Extending the foot and observing for pain does not confirm or rule out clubfoot. D) Stimulating the sole of the foot elicits the plantar grasp reflex, and is not an appropriate assessment for clubfoot.

1) The nurse is training a nurse new to the labor and delivery unit. They are caring for a laboring client who will have a forceps delivery. Which action or assessment finding requires intervention? A) Regional anesthesia is administered via pudendal block. B) The client is instructed to push between contractions. C) Fetal heart tones are consistently between 110 and 115. D) The client's bladder is emptied using a straight catheter.

Answer: B Explanation: A) Adequate anesthesia must be given for the type of forceps procedure anticipated. Low forceps may be done with a pudendal block; however, midforceps or a rotation of more than 45 degrees requires an epidural, spinal-epidural, or general anesthesia. B) During the contraction, as the forceps are applied, the woman should avoid pushing. C) Fetal heart tones between 110 and 115 are normal. No intervention is needed. D) The maternal bladder should be emptied.

1) The nurse expects an initial weight loss for the average postpartum client to be which of the following? A) 5 to 8 pounds B) 10 to 12 pounds C) 12 to 15 pounds D) 15 to 20 pounds

Answer: B Explanation: A) An initial weight loss of 10 to 12 l b s. occurs as a result of the birth of infant, placenta, and amniotic fluid. B) An initial weight loss of 10 to 12 l b s. occurs as a result of the birth of infant, placenta, and amniotic fluid. C) An initial weight loss of 10 to 12 l b s. occurs as a result of the birth of infant, placenta, and amniotic fluid. D) An initial weight loss of 10 to 12 l b s. occurs as a result of the birth of infant, placenta, and amniotic fluid.

1) Narcotic analgesia is administered to a laboring client at 10:00 a.m. The infant is delivered at 12:30 p.m. What would the nurse anticipate that the narcotic analgesia could do? A) Be used in place of preoperative sedation B) Result in neonatal respiratory depression C) Prevent the need for anesthesia with an episiotomy D) Enhance uterine contractions

Answer: B Explanation: A) Analgesics do not take the place of preoperative sedation. B) Analgesia given too late is of no value to the woman and may cause neonatal respiratory depression. C) Local anesthetic is needed for an episiotomy. D) Analgesics do not enhance uterine contractions.

1) The nurse recognizes that what are the most common disabilities in women? A) Asthma and headaches B) Arthritis or rheumatism C) Adverse kidney and nervous system functioning D) Cardiovascular diseases

Answer: B Explanation: A) Asthma and headaches are health conditions associated with air pollution. B) The most common disabilities in women are arthritis or rheumatism and the incidence of disability increases with age. C) Adverse kidney and nervous system functioning are effects of lead exposure. Cardiovascular diseases are health conditions associated with air pollution

1) A new mother who is breastfeeding tells the nurse that her infant is spitting up frequently, has very loose stools and copious gas, and feeds for only short periods of time. The nurse suspects a feeding intolerance and, after questioning the mother about her diet, suggests that she do which of the following? A) Stop breastfeeding and switch to formula. B) Eliminate dairy products from her diet. C) Supplement breastfeeding with a soy-based formula. Offer the baby water between feedings

Answer: B Explanation: A) At this point, there is no reason to stop breastfeeding. B) Breastfeeding babies may not be allergic to the mother's milk but rather to the cow's milk protein (an antigen) in the mother's milk. By eliminating the culprit (e.g., the bovine protein) from the mother's diet and therefore from the breast milk, the mother can continue to breastfeed, providing optimal nutrition and immune factors to her infant. C) Soy-protein-based formula is not the first choice for term infants unless they have special nutritional needs. D) Increased water can cause hyponatremia and, in excessive amounts, can cause seizures.

1) A client was admitted to the labor area at 5 c m with ruptured membranes about 14 hours ago. What assessment data would be most beneficial for the nurse to collect? A) Blood pressure B) Temperature C) Pulse D) Respiration

Answer: B Explanation: A) Blood pressure can assist in the diagnosis of infection, but is not the primary vital sign. B) Rupture of membranes places the mother at risk for infection. The temperature is the primary and often the first indication of a problem. C) Pulse can assist in the diagnosis of infection, but is not the primary vital sign. Respirations can assist in the diagnosis of infection, but are not the primary sign

12) Which of the following is important for the development of the central nervous system of the fetus? A) Calcium and phosphorus B) Essential fatty acids C) Iron D) Vitamin D

Answer: B Explanation: A) Calcium and phosphorus are involved in the mineralization of fetal bones and teeth, energy and cell production, and acid-base buffering. B) Essential fatty acids are important for the development of the central nervous system of the fetus. Of particular interest are the omega-3 fatty acids and their derivatives. C) Iron requirements increase during pregnancy because of the growth of the fetus and placenta and the expansion of maternal blood volume. D) Vitamin D is known for its role in the absorption and utilization of calcium and phosphorus in skeletal development.

1) In succenturiate placenta, one or more accessory lobes of fetal villi have developed on the placenta, with vascular connections of fetal origin. What is the gravest maternal danger? A) Cord prolapse B) Postpartum hemorrhage C) Paroxysmal hypertension D) Brachial plexus injury

Answer: B Explanation: A) Cord prolapse is not considered a danger of succenturiate placenta. B) The gravest maternal danger is postpartum hemorrhage if this minor lobe is severed from the placenta and remains in the uterus. C) Paroxysmal hypertension is a symptom of autonomic dysreflexia. D) Brachial plexus injury is an injury due to improper or excessive traction applied to the fetal head during birth.

1. Which of the following would be considered a clinical sign of hemorrhage? A) Increased blood pressure B) Increasing pulse C) Increased urinary output D) Hunger

Answer: B Explanation: A) Decreased blood pressure would be considered a clinical sign of hemorrhage. B) Increasing pulse, widening pulse pressure would be considered a clinical sign of hemorrhage. C) Decreased urinary output would be considered a clinical sign of hemorrhage. D) Thirst, not hunger, would be considered a clinical sign of hemorrhage.

1) The nurse is assessing a client who has severe preeclampsia. What assessment finding should be reported to the physician? A) Excretion of less than 300 m g of protein in a 24-hour period B) Platelet count of less than 150,000/m m3 C) Urine output of 50 m L per hour D) 12 respirations

Answer: B Explanation: A) Excretion of more than 300 m g of protein in a 24-hour period is considered abnormal. B) HELLP syndrome (hemolysis, elevated liver enzymes, and low platelet count) complicates 10% to 20% of severe preeclampsia cases and develops prior to 37 weeks' gestation 50% of the time. Vascular damage is associated with vasospasm, and platelets aggregate at sites of damage, resulting in low platelet count (less than 150,000/m m3). C) Urine output of a least 30m L/hour is considered normal. D) Respirations of 12 are normal.

1) The community nurse is meeting a new mother for the first time. The client delivered her first child 5 days ago after a 12-hour labor. Neither the mother nor the infant had any complications during the birth or postpartum period. Which statement by the client would indicate to the nurse that the client is experiencing postpartum blues? A) "I am so happy and blessed to have my new baby." B) "One minute I'm laughing and the next I'm crying." C) "My husband is helping out by changing the baby at night." D) "Breastfeeding is going quite well now that the engorgement is gone."

Answer: B Explanation: A) Feeling happy is not a symptom of postpartum blues. B) The postpartum blues consist of a transient period of depression that occurs during the first few days of puerperium. Symptoms may include mood swings, anger, weepiness, anorexia, difficulty sleeping, and a feeling of letdown. C) The husband's assistance does not relate to postpartum blues. D) Successful lactation is not a symptom of postpartum blues.

1) A client is at 12 weeks' gestation with her first baby. She has cardiac disease, class Ⅲ. She states that she had been taking sodium warfarin (Coumadin), but her physician changed her to heparin. She asks the nurse why this was done. What should the nurse's response be? A) "Heparin is used when coagulation problems are resolved." B) "Heparin is safer because it does not cross the placenta." C) "They are the same drug, but heparin is less expensive." D) "Coumadin interferes with iron absorption in the intestines."

Answer: B Explanation: A) Heparin is used when coagulation problems develop. B) Heparin is safest for the client to take because it does not cross the placental barrier. C) Heparin does not cost less than Coumadin. D) Coumadin does not interfere with iron absorption in the intestines.

1) The nurse should anticipate the labor pattern for a fetal occiput posterior position to be which of the following? A) Shorter than average during the latent phase B) Prolonged as regards the overall length of labor C) Rapid during transition D) Precipitous

Answer: B Explanation: A) Overall labor is often prolonged, not shorter. B) Occiput posterior (O P) position of the fetus is the most common fetal malposition and occurs when the head remains in the direct O P position throughout labor. This can prolong the overall length of labor. C) Overall labor is often prolonged, not more rapid. D) Overall labor is often prolonged, not precipitous.

1) An H I V-positive mother delivered 2 days ago. The infant will be placed in foster care. The nurse is planning discharge teaching for the foster parents on how to care for the newborn at home. Which instructions should the nurse include? A) Do not add food supplements to the baby's diet. B) Place soiled diapers in a sealed plastic bag. C) Wash soiled linens in cool water with bleach. D) Shield the baby's eyes from bright lights.

Answer: B Explanation: A) Small, frequent feedings are recommended, as well as food supplementation as necessary to support weight gain. B) The nurse should instruct the parents about proper hand-washing techniques, proper disposal of soiled diapers, and the importance of wearing gloves when diapering. C) Soiled linens should be washed in hot, sudsy water with bleach. D) Shielding the baby's eyes from bright lights would be recommended for a preterm infant, not an infant with H I V.

1) The special care nursery nurse is working with parents of a 3-day-old infant who was born with myelomeningocele and has developed an infection. Which statement from the mother is unexpected? A) "If I had taken better care of myself, this wouldn't have happened." B) "I've been sleeping very well since I had the baby." C) "This is probably the doctor's fault." D) "If I hadn't seen our baby's birth, I wouldn't believe she is ours."

Answer: B Explanation: A) Some parents may feel guilty about their baby's condition and think they have caused the problem. B) A sick infant is a source of great anxiety for parents. This response is from the mother would be unexpected. C) Parents express grief as shock and disbelief, denial of reality, anger toward self and others, guilt, blame, and concern for the future. Parents express grief as shock and disbelief, denial of reality, anger toward self and others, guilt, blame, and concern for the future

1) A patient with a rectocele is experiencing progressive pain and constipation. What should the nurse expect to be indicated for this patient? A) Enemas B) Surgery C) Laxatives D) Antibiotics

Answer: B Explanation: A) Surgery is often indicated for a rectocele. Enemas, laxatives, and antibiotics may help treat the symptoms but will not cure the problem.

1) A woman has been admitted for an external version. She has completed an ultrasound exam and is attached to the fetal monitor. Prior to the procedure, why will terbutaline be administered? A) To provide analgesia B) To relax the uterus C) To induce labor D) To prevent hemorrhage

Answer: B Explanation: A) Terbutaline has no analgesic effect. B) Terbutaline is administered to achieve uterine relaxation. C) Terbutaline does not induce labor. D) Terbutaline does not prevent hemorrhage.

1) What type of forceps are designed to be used with a breech presentation? A) Midforceps B) Piper C) Low D) High

Answer: B Explanation: A) The criterion for midforceps application is that the fetal head must be engaged, but the leading edge of the fetal skull is above a plus 2 (+2) station. B) Piper forceps are designed to be used with a breech presentation. They are applied after the birth of the body, when the fetal head is still in the birth canal and assistance is needed. C) The criterion for low forceps application is that the leading edge of the fetal skull must be at a station of plus 2 (+2) or below, but not on the pelvic floor. D) High forceps are not indicated in current obstetric practice.

1) Which of the following may be the main presenting symptom of iron deficiency anemia? A) Frequent urination B) Fatigue C) Nausea D) Headaches

Answer: B Explanation: A) The main presenting symptom of iron deficiency anemia would not be frequent urination. B) The main presenting symptom of iron deficiency anemia may be fatigue. C) The main presenting symptom of iron deficiency anemia would not be nausea. D) The main presenting symptom of iron deficiency anemia would not be headaches.

1) A client at 10 weeks' gestation has developed cholecystitis. If surgery is required, what is the safest time during pregnancy? A) Immediately, before the fetus gets any bigger B) Early in the second trimester C) As close to term as possible D) The risks are too high to do it anytime in pregnancy

Answer: B Explanation: A) The risk of miscarriage is greater in the first trimester. B) The early second trimester is the best time to operate because there is less risk of spontaneous abortion or early labor, and the uterus is not so large as to impinge on the abdominal field. C) A fetus close to term is so large that it might interfere with the abdominal field. D) There is always a risk, but it is greater in the first and third trimesters.

1) The nurse is caring for a client at 35 weeks' gestation who has been critically injured in a shooting. Which statement by the paramedics bringing the woman to the hospital would cause the greatest concern? A) "Blood pressure 110/68, pulse 90." B) "Entrance wound present below the umbilicus." C) "Client is positioned in a left lateral tilt." D) "Clear fluid is leaking from the vagina."

Answer: B Explanation: A) These are normal vital signs, indicating a hemodynamically stable client. B) Penetrating trauma includes gunshot wounds and stab wounds. The mother generally fares better than the fetus if the penetrating trauma involves the abdomen, as the enlarged uterus is likely to protect the mother's bowel from injury. C) Positioning the client in a lateral tilt position prevents vena cava syndrome. D) Clear fluid from the vagina could be amniotic fluid from spontaneous rupture of the membranes. Although this is not a normal finding at 35 weeks, this fetus is near term, and would likely survive birth at this time.

1) At birth, an infant weighed 8 pounds 4 ounces. Three days later, the newborn is being discharged. The parents note that the baby now weighs 7 pounds 15 ounces. The nurse explains that the change in the newborn's weight is which of the following? A) Excessive B) Within normal limits C) Less than expected D) Unusual

Answer: B Explanation: A) This is not an excessive weight loss. B) This newborn's weight loss is within normal limits. A weight loss of up to 10% for term newborns is considered within normal limits during the first week of life. C) This is not a less-than-expected amount of weight loss. D) This weight loss is not unusual.

1) The nurse provides the following diagram to a patient in the 3rd trimester of pregnancy. For which diagnostic test is the nurse preparing this patient? A) Ultrasound B) Amniocentesis C) Biophysical profile D) Contraction stress test

Answer: B Explanation: B) For an amniocentesis the patient is usually scanned by ultrasound to determine the placental site and to locate a pocket of fluid. As the needle is inserted, three levels of resistance are felt when the needle penetrates the skin, fascia, and uterine wall. When the needle is placed within the amniotic cavity, amniotic fluid is withdrawn. A needle is not injected into the amniotic sac for an ultrasound, biophysical profile, or contraction stress test.

1) Which of the following would be considered normal newborn urinalysis values? Note: Credit will be given only if all correct choices and no incorrect choices are selected. Select all that apply. A) Color bright yellow B) Bacteria 0 C) Red blood cells (R B C) 0 D) White blood cells (W B C) more than 4-5/h p f E) Protein less than 5-10 m g/d L

Answer: B, C, E Explanation: A) Urine color should be pale yellow. B) Bacteria value should be 0. C) Red blood cells (R B C) should be 0. D) White blood cells (W B C) should be less than 2-3/h p f. E) Protein less than 5-10 m g/d L would be considered normal.

1) What are the three functions of cervical mucosa? Note: Credit will be given only if all correct choices and no incorrect choices are selected. Select all that apply. A) Form the relatively fixed axis of the birth passage B) Provide lubrication for the vaginal canal C) Provide nourishment and protective maternal antibodies to infants D) Provide an alkaline environment to shelter deposited sperm from the acidic vaginal secretions E) Act as a bacteriostatic agent

Answer: B, D, E Explanation: A) The female boney pelvis forms the relatively fixed axis of the birth passage. B) The cervical mucosa provides lubrication for the vaginal canal. C) The breasts provide nourishment and protective maternal antibodies to infants. D) The cervical mucosa provides an alkaline environment to shelter deposited sperm from the acidic vaginal secretions. E) The cervical mucosa acts as a bacteriostatic agent.

1) During a pelvic examination, a patient is diagnosed with a Bartholin gland cyst. For which treatment should the nurse prepare this patient? Note: Credit will be given only if all correct choices and no incorrect choices are selected. Select all that apply. A) Pelvic ultrasound B) Antibiotic therapy C) Exploratory laparotomy D) Incision and drainage of the cyst E) Culture and sensitivity of the discharge

Answer: B, D, E Explanation: A) Treatment of a Bartholin gland cyst involves antibiotic therapy, incision and drainage of the abscess, and culture and sensitivity of the discharge. A pelvic ultrasound and exploratory laparotomy are not indicated for this health problem.

1) A 58-year-old father and a 45-year-old mother gave birth to a baby boy 2 days ago. The nurse assesses a single palmar crease and low-set ears on the newborn. The nurse plans to counsel the couple about which chromosomal abnormality? A) Trisomy 13 B) Trisomy 18 C) Trisomy 21 D) Trisomy 26

Answer: C Explanation: A) A single palmar crease and low-set ears are not characteristics of trisomy 13. B) A single palmar crease and low-set ears are not characteristics of trisomy 18. C) A single palmar crease and low-set ears are characteristics of trisomy 21 (Down syndrome). D) Trisomy 26 is not a chromosomal abnormality.

1) A woman is experiencing mittelschmerz and increased vaginal discharge. Her temperature has increased by 0.6°C (1.0°F) over the past 36 hours. This most likely indicates what? A) Menstruation is about to begin. B) Ovulation will occur soon. C) Ovulation has occurred. D) She is pregnant, and will not menstruate.

Answer: C Explanation: A) A temperature increase does not occur when menstruation is about to begin. B) A temperature increase does not occur before ovulation has occurred. C) In some women, ovulation is accompanied by mid-cycle pain, known as mittelschmerz. This pain may be caused by a thick tunica albuginea or by a local peritoneal reaction to the expelling of the follicular contents. Body temperature increases about 0.3°C to 0.6°C (0.5°F to 1°F) 24 to 48 hours after the time of ovulation. D) Pregnancy can be detected through the presence of human chorionic gonadotropin hormone.

1) The nurse is planning a community educational presentation for people living below the poverty level. The nurse knows that which of the following is the largest population in this socioeconomic category? A) Adults in communal living situations B) Young married couples under the age of 20 C) Single women with children D) Single adults

Answer: C Explanation: A) Adults living together are not usually below the poverty level. B) Young married couples are not the most likely to encounter poverty. C) Of households headed by single mothers, 40% live in poverty. D) Single adults are not the most likely to encounter poverty.

1) The nurse is assessing a client's risk for death during the postpartum period. Which finding would cause the nurse to increase surveillance for this client? A) Maternal age of 25 B) History of bipolar disorder C) Increased body mass index D) Decreased blood pressure

Answer: C Explanation: A) Advanced maternal age is a risk factor for maternal death during the postpartum period. Advanced maternal age is defined as greater than 35 years of age. B) A history of bipolar disorder increases the risk of postpartum depression and psychosis, not maternal demise. C) An increased body mass index is often a risk factor for maternal demise during the postpartum period. An increased, not decreased, blood pressure is a risk factor for maternal demise during the postpartum period

1) A new grandfather is marveling over his 12-hour-old newborn grandson. Which statement indicates that the grandfather needs additional education? A) "I can't believe he can already digest fats, carbohydrates, and proteins." B) "It is amazing that his whole digestive tract can move things along at birth." C) "Incredibly, his stomach capacity was already a cupful when he was born." D) "He will lose some weight but then miraculously regain it by about 10 days."

Answer: C Explanation: A) At birth, neonates can digest fats, simple carbohydrates, and proteins. B) The stomach empties intermittently, starting within a few minutes of the beginning of a feeding and ending between 2 and 4 hours after feeding. C) The newborn's stomach has a capacity of 50 to 60 m L. D) Following birth, caloric intake is often insufficient for weight gain until the newborn is 5 to 10 days old. During this time there may be a weight loss of 5% to 10% in term newborns.

1) What would be a normal cervical dilatation rate in a first-time mother ("primip")? A) 1.5 c m per hour B) Less than 1 c m cervical dilatation per hour C) 1 c m per hour D) Less than 0.5 c m per hour

Answer: C Explanation: A) Dilatation in a "multip" is about 1.5 c m per hour. B) Less than 1 c m cervical dilatation per hour is prolonged labor. C) Cervical dilatation in a first-time mother is just over 1 c m per hour. D) Dystocia is a rate of cervical dilatation of less than 0.5 c m per hour.

1) The nurse is teaching a group of women about menopause at a community clinic. The nurse tells them that the best indicator of menopause is which of the following symptoms? A) No menses for 8 consecutive months B) Hot flashes and night sweats C) F S H levels rise and ovarian follicles cease to produce estrogen D) Diagnosed with osteoporosis 4 months ago

Answer: C Explanation: A) Eight consecutive months of amenorrhea are enough to qualify as menopause. B) Although hot flashes and night sweats are common in menopause, they are not the most reliable indicator of menopause. C) Examining F S H and estrogen levels is a very accurate indication of menopause. D) Menopause is not the only cause of osteoporosis; therefore, the diagnosis of osteoporosis 4 months ago is not an indicator of menopause.

1) What would the nurse include as part of a routine physical assessment for a second-trimester primiparous patient whose prenatal care began in the first trimester and is ongoing? A) Pap smear B) Hepatitis B screening (H Bs A g) C) Fundal height measurement D) Complete blood count

Answer: C Explanation: A) Pap smear is usually done at the initial prenatal appointment. B) Hepatitis B screening is done at the initial prenatal appointment. C) At each prenatal visit, the blood pressure, pulse, and weight are assessed, and the size of the fundus is measured. Fundal height should be increasing with each prenatal visit. D) Complete blood count is done at the initial prenatal appointment.

1. The charge nurse is assessing several postpartum clients. Which client has the greatest risk for postpartum hemorrhage? A) The client who was overdue and delivered vaginally B) The client who delivered by scheduled cesarean delivery C) The client who had oxytocin augmentation of labor D) The client who delivered vaginally at 36 weeks

Answer: C Explanation: A) The client who was overdue and delivered vaginally has a lower risk for postpartum hemorrhage than would another client. B) The client who delivered by scheduled cesarean delivery has a lower risk for postpartum hemorrhage than would another client. C) Uterine atony is a cause of postpartal hemorrhage. A contributing factor to uterine atony is oxytocin augmentation of labor. D) The client who delivered vaginally at 36 weeks has a lower risk for postpartum hemorrhage than would another client.

1) The nurse in the community should use a family assessment tool to obtain what type of information? A) How long the family has lived at its current address B) What other health insurance the family has had in the past C) How the family meets its nutritional needs and obtains food D) What eye color the family desires in its unborn child

Answer: C Explanation: A) The length of time at a residence is not included in the family assessment tool. B) Past health insurance coverage is not included in the family assessment tool. C) A family assessment is a collection of data about the family's type and structure, current level of functioning, support system, sociocultural background, environment, and needs. D) Desired eye color of a child is not included in the family assessment tool.

1) A fetus has a brow cephalic presentation. Which head shape should the nurse expect when the infant is delivered?

Answer: C Explanation: C) In a brow presentation the head is molded forward. Choice 1 is an occiput anterior presentation. Choice 2 is an occiput posterior presentation. Choice 4 is face presentation.

1) The nurse is conducting a health maintenance assessment for a female client. Which neurologic data would cause the nurse to further assess for intimate partner abuse? A) Anxiety B) Depression C) Weight gain D) Tension headaches

Answer: D Explanation: A) Anxiety is a psychiatric, not neurologic, assessment finding that would cause the nurse to further assess for intimate partner abuse. B) Depression is a psychiatric, not neurologic, assessment finding that would cause the nurse to further assess for intimate partner abuse. C) Weight gain is a constitutional, not neurologic, assessment finding that would cause the nurse to further assess for intimate partner abuse. D) Tension headaches are a neurologic assessment finding that would cause the nurse to further assess for intimate partner abuse.

1) The nurse is presenting a class to newly pregnant families. What form of trauma will the nurse describe as the leading cause of fetal and maternal death? A) Falls B) Domestic violence C) Gun accidents D) Motor vehicle accidents

Answer: D Explanation: A) Falls are not the leading cause of fetal and maternal death. B) Domestic violence is not the leading cause of fetal and maternal death. C) Gun accidents are not common in pregnancy. D) Trauma from motor vehicle accidents is the leading cause of fetal and maternal death.

1) After delivery, it is determined that there is a placenta accreta. Which intervention should the nurse anticipate? A) 2 L oxygen by mask B) Intravenous antibiotics C) Intravenous oxytocin D) Hysterectomy

Answer: D Explanation: A) Use of oxygen will not assist in the separation of the placenta. B) Use of intravenous antibiotics will not assist in the separation of the placenta. C) Use of intravenous oxytocin will not assist in the separation of the placenta. D) The primary complication of placenta accreta is maternal hemorrhage and failure of the placenta to separate following birth of the infant. An abdominal hysterectomy may be the necessary treatment, depending on the amount and depth of involvement.

The nurse suspects that a patient is experiencing bacterial vaginosis. What finding caused the nurse to make this clinical determination? A) Dysuria B) Vaginal itching C) Thick white vaginal discharge D) Fishy odor to vaginal discharge

Answer: D Explanation: D) The person with bacterial vaginosis may have a thin watery discharge with a fishy odor. Dysuria, vaginal itching, and thick white vaginal discharge are manifestations of vulvovaginal candidiasis.

Answer: B Explanation: B) If the placenta separates from the outer margins inward, it will roll up and present sideways with the maternal surface delivering first. This is known as the Duncan mechanism of placental delivery and is commonly called dirty Duncan because the placental surface is rough. If the placenta separates from the inside to the outer margins, it is expelled with the fetal (shiny) side presenting. This is known as the Schultze mechanism of placental delivery or, more commonly, shiny Schultze. This diagram does not demonstrate complete or incomplete mechanisms of placental delivery.

1. The nurse is assisting in the delivery of a pregnant patient's placenta post-delivery. The placenta appears as follows. What term should the nurse use to document this placental delivery? A) Schultz mechanism of delivery B) Duncan mechanism of delivery C) Complete mechanism of delivery D) Incomplete mechanism of delivery

Answer: D Explanation: D) Bimanual massage compresses the body of the uterus from below while the abdominal hand massages the fundus from above. Uterine tamponade is the insertion of a balloon into the uterus to apply pressure to the uterine walls. The B-Lynch compression procedure is conducted through a laparotomy. Manual removal of the placenta may be done immediately after the placenta is delivered.

1. The nurse provides a postpartum patient with the following diagram. For which procedure is the nurse preparing this patient? A) Uterine tamponade B) Manual removal of the placenta C) B-Lynch compression procedure D) Manual compression of the uterus

6) The client is being admitted to the birthing unit. As the nurse begins the assessment, the client's partner asks why the fetus's heart rate will be monitored. After the nurse explains, which statement by the partner indicates a need for further teaching? A) "The fetus's heart rate will vary between 110 and 160." B) "The heart rate is monitored to see whether the fetus is tolerating labor." C) "By listening to the heart, we can tell the gender of the fetus." D) "After listening to the heart rate, you will contact the midwife."

A) Answer: C Explanation: A) A normal fetal heart rate is 110-160. B) The fetal heart rate (F H R) is auscultated every 30 minutes. It should remain between 110 and 160 beats per minute (beats/m i n) without the presence of decelerations. C) Fetal heart rate is not a predictor of gender. D) Once the admission is complete, the nurse will contact the client's provider with the assessment findings.

7) In early-pregnancy class, the nurse emphasizes the importance of 8-10 glasses of fluid per day. How many of these should be water? A) 1 to 2 B) 2 to 4 C) 4 to 6 D) 3 to 5

A) Answer: C Explanation: A) One to two glasses of water is not an adequate intake. B) Two to four glasses of water is not an adequate intake. C) A pregnant woman should consume at least 8 to 12 (8 oz) glasses of fluid each day, of which 4 to 6 glasses should be water. D) Three to five glasses of water is not an adequate intake.

1) A female is offered a position as a manager with a major city hotel that offers flexibility with childcare and family issues. What should this individual also investigate financially before accepting this position? A) Number of sick days B) Expectations to work holidays C) Number of overtime hours expected to work every month D) Wages that are commensurate with those of male employee managers

A) Answer: D Explanation: One reason for wage discrepancy is that women accept lower salaries in exchange for provisions such as flexibility for childcare responsibilities and family-related issues. This person needs to investigate the wages she will receive before accepting this position. The number of sick days, expectations to work holidays, and overtime hours would have a higher impact on work-life balance than on finances

19) The kosher diet followed by many Jewish people forbids the eating of what foods? A) Pig products and shellfish B) Dairy products C) All animal products D) Dairy products and eggs

A) The kosher diet followed by many Jewish people forbids the eating of pig products and shellfish. Certain cuts of meat from sheep and cattle are allowed, as are fish with fins and scales. In addition, many Jews believe that meat and dairy products should not be mixed or eaten at the same meal. B) Lacto-vegetarian diets include dairy products but no eggs. C) Vegans are strict vegetarians who will not eat any food from animal sources. Lacto-ovo-vegetarians include milk, dairy products, and eggs in their diet, but no meat

1) An older patient is demonstrating a new onset of confusion and forgetfulness. While reviewing the patient's medical records, the nurse suspects these new manifestations are drug-induced. Which medications did the nurse identify as causing changes in cognitive functioning in this patient? Note: Credit will be given only if all correct choices and no incorrect choices are selected. Select all that apply. 1. Antihistamine for allergies 2. Antidepressant for nerve pain 3. Antibiotic for bronchial infection 4. Anticoagulant for atrial fibrillation 5. Antihypertensive for high blood pressure

Answer: 1, 2 Explanation: Certain drugs can cause cognitive impairment, especially when diminished kidney and liver function is present. Drugs that alter the central nervous system include antihistamines and antidepressants. These drugs may cause forgetfulness, confusion, disorientation, and inability to concentrate. Antibiotics, anticoagulants, and antihypertensives are not identified as causing cognitive impairment

1) A patient requiring back surgery wants to take family/medical leave to recover but is not sure if she is eligible. What should the nurse review as eligibility requirements for this coverage? Note: Credit will be given only if all correct choices and no incorrect choices are selected. Select all that apply. 1. Work more than 25 hours per week 2. Have been in the job for at least 1 year 3. Be expected to return to work within 4 weeks 4. Be willing to accept a lower-paying position upon return 5. Have provisions to self-pay for health insurance while off from work

Answer: 1, 2 Explanation: Limitations to the Family Medical Leave Act (F M L A) include that employees must work at least 25 hours per week to be eligible and must have been in their position for at least 1 year. The maximum amount of leave allowed for F M L A is 12 weeks. The Act provides job security for the person to return to their former position, or one that is considered comparable. Health insurance benefits are covered while on leave.

A female patient with amenorrhea is suspected to have pituitary dysfunction. For which health problems should the nurse explain that the patient will most likely be evaluated? Note: Credit will be given only if all correct choices and no incorrect choices are selected. Select all that apply. 1. Cancer 2. Adenoma 3. Head trauma 4. Turner syndrome 5. Polycystic ovarian syndrome

Answer: 1, 2, 3 Explanation: In pituitary dysfunction, cancer and head trauma can cause hypopituitarism. A pituitary adenoma can cause changes in the hormones that the pituitary gland manufactures. Turner syndrome is a genetic disorder that is linked to chronic anovulation or ovarian failure. Polycystic ovarian syndrome is a cause of chronic anovulation.

1) The community nurse is conducting health assessments at the homeless shelter and notes that the majority of clients are female. What should the nurse identify as reasons for the percentage of women who are homeless? Note: Credit will be given only if all correct choices and no incorrect choices are selected. Select all that apply. 1. Unemployment 2. Substance abuse 3. Recent prison release 4. Lack of family support 5. Inadequate child support

Answer: 1, 2, 3, 4 Explanation: Factors that increase the risk of female homelessness include unemployment, substance abuse, recent prison release and lack of family support. Inadequate child support is not identified as a factor that increases the risk of female homelessness.

1) The nurse is assessing a pregnant client in the second trimester of pregnancy during a scheduled prenatal visit. Which questions are appropriate during the assessment process? Note: Credit will be given only if all correct choices and no incorrect choices are selected. Select all that apply. 1. "Do you feel bloated?" 2. "Do you have hemorrhoids?" 3. "Are you experiencing heartburn?" 4. "Are you experiencing constipation?" 5. "Are you experiencing nausea and vomiting?"

Answer: 1, 2, 3, 4 Explanation: Gastrointestinal symptoms that often occur during the second trimester of pregnancy include feeling bloated, the development of hemorrhoids, heartburn, and constipation. Nausea and vomiting are more common during the first trimester of pregnancy.

1) A pregnant patient's healthcare provider is using the Hadlock method to determine gestational age and growth of the fetus. Which measurements will be used to make this fetal determination? Note: Credit will be given only if all correct choices and no incorrect choices are selected. Select all that apply. 1. Femur length 2. Biparietal diameter 3. Head circumference 4. Abdominal circumference 5. Crown to rump measurement

Answer: 1, 2, 3, 4 Explanation: The Hadlock method uses an average of measures of the biparietal diameter, head circumference, abdominal circumference, and femur length to estimate gestational age. The crown to rump measurement is used to determine gestational age of an embryo between 6 and12 weeks.

1) Which items will the nurse include in a memory box for a couple who have experienced a stillbirth? Note: Credit will be given only if all correct choices and no incorrect choices are selected. Select all that apply. 1. Pictures 2. I D bracelet 3. Memory card 4. Bereavement clothing Grief and loss pamphlets

Answer: 1, 2, 3, 4 Explanation: When preparing a memory box for a couple who lost their newborn to stillbirth, the nurse will include pictures of the baby, I D bracelets, a memory card, and bereavement clothing. Grief and loss pamphlets should be provided to the couple, but not included in the memory box.

1) It is confirmed that a 14-week fetus has died. The patient has an unfavorable cervix. Which approaches to deliver the fetus should be reviewed with the patient? Note: Credit will be given only if all correct choices and no incorrect choices are selected. Select all that apply. 1. Oxytocin 2. Misoprostol 3. Prostaglandins 4. Laminaria tent 5. Cesarean section

Answer: 1, 2, 3, 4 Explanation: Women with an unfavorable cervix may be given vaginal prostaglandin agents, misoprostol, or laminaria tents. Women whose gestations are less than 16 gestational weeks may have a laminaria tent inserted into the cervix before a dilatation and extraction procedure. Women less than 28 weeks' gestation are typically given prostaglandin E2 vaginal suppositories (10-20 mg q 4-6 h), misoprostol 400 mcg vaginally or orally every 4 to 6 hours, and/or oxytocin until spontaneous labor occurs. Cesarean section is not a method identified to deliver the dead fetus

1) During a routine prenatal visit the nurse suspects that a patient in the 14th week of gestation is affected by environmental pollution. What assessment findings caused the nurse to come to this conclusion? Note: Credit will be given only if all correct choices and no incorrect choices are selected. Select all that apply. 1. Headache 2. Scratchy throat 3. Chest wheezing 4. Abdominal pain 5. Itchy burning eyes

Answer: 1, 2, 3, 5 Explanation: Health conditions associated with air pollution include headache, throat irritation, asthma, and eye irritation. Abdominal pain is not a health condition associated with air pollution

1) The community nurse is preparing to visit the home of an adolescent who is 18 weeks pregnant. Which health problems should the nurse focus on when assessing this patient? Note: Credit will be given only if all correct choices and no incorrect choices are selected. Select all that apply. 1. Preeclampsia 2. Preterm birth 3. Low-birth-weight 4. Gestational diabetes 5. Iron deficiency anemia

Answer: 1, 2, 3, 5 Explanation: Risks for pregnant adolescents include preterm births, low-birth-weight (L B W) infants, preeclampsia-eclampsia and its sequelae, iron deficiency anemia, and cephalopelvic disproportion (C P D). Gestational diabetes is not identified as a health risk for pregnant adolescents.

1) The nurse is preparing material to present to a group of patients in the second trimester of their pregnancies. Which topics should the nurse include in this presentation? Note: Credit will be given only if all correct choices and no incorrect choices are selected. Select all that apply. 1. Clothing 2. Infant feeding 3. Fetal movement 4. Exercise and rest 5. Skin and breast care

Answer: 1, 2, 3, 5 Explanation: Topics appropriate to teach patients in the second trimester of pregnancy include clothing, infant feeding, fetal movement, and skin and breast care. Exercise and rest are topics to be reviewed during the first and third trimesters

1) The nurse is preparing teaching material for a new mother. What should the nurse include when instructing on areas to include when observing the infant? Note: Credit will be given only if all correct choices and no incorrect choices are selected. Select all that apply. 1. Touch 2. Vision 3. Hearing 4. Diaper care 5. General appearance

Answer: 1, 2, 3, 5 Explanation: When teaching on observing the baby, the nurse should include general appearance and the five senses - vision, hearing, touch, smell, and taste. Diaper care is included when teaching about the skin

31) A patient asks if seafood is permitted during pregnancy. Which foods should the nurse encourage the patient to consume during this time? Note: Credit will be given only if all correct choices and no incorrect choices are selected. Select all that apply. 1. Shrimp 2. Catfish 3. Salmon 4. Swordfish 5. Canned light tuna

Answer: 1, 2, 3, 5 Explanation: Women who are pregnant or who may become pregnant, breastfeeding mothers, and young children should not eat swordfish, shark, tilefish, or king mackerel because these fish contain high levels of methyl mercury. Commonly eaten fish that are low in mercury include canned light tuna, shrimp, salmon, catfish, and pollack. Albacore (white) tuna has more mercury than canned light tuna; therefore, only 6 oz. / week of albacore tuna is recommended.

While reviewing data, the nurse determines that a patient is at risk for pelvic inflammatory disease. Which information did the nurse use to make this clinical determination? Note: Credit will be given only if all correct choices and no incorrect choices are selected. Select all that apply. 1. Age 23 2. Douches weekly 3. Smokes cigarettes 1 ppd 4. I U D inserted 2 months ago 5. Received H P V vaccination

Answer: 1, 2, 4 Explanation: Pelvic inflammatory disease (P I D) occurs most often in sexually active women under age 25. Other risk factors include regular douching and recent insertion of an intrauterine device. Smoking and receiving the H P V vaccination are not risk factors for the development of P I D.

The nurse is concerned that a patient is at risk for developing vulvovaginal candidiasis (V V C). What assessment information caused the nurse to have this concern? Select all that apply. 1. 16 weeks pregnant 2. +3 glucose in the urine 3. Elevated blood pressure 4. Type 2 diabetes mellitus 5. Edematous lower extremities

Answer: 1, 2, 4 Explanation: Predisposing factors to vulvovaginal candidiasis (VVC) infections include pregnancy, glycosuria, and diabetes mellitus. Elevated blood pressure and edematous lower extremities is more likely to be associated with preeclampsia in the pregnant patient

1) A patient in the first trimester of pregnancy is experiencing ptyalism. What should the nurse suggest to help this patient? Note: Credit will be given only if all correct choices and no incorrect choices are selected. Select all that apply. 1. Use chewing gum 2. Suck on hard candy 3. Snack on soda crackers 4. Use an astringent mouthwash 5. Brush the teeth with baking soda

Answer: 1, 2, 4 Explanation: Ptyalism is a rare discomfort of pregnancy in which excessive, often bitter, saliva is produced. Its cause has not been established. Effective treatments are limited, however using astringent mouthwashes, chewing gum, or sucking on hard candy may minimize the problem. Many women also choose to carry tissues or a small towel to spit into when necessary. Soda crackers and brushing the teeth with baking soda are not identified as helpful with ptyalism.

1) A patient in the first trimester of pregnancy recently emigrated from a foreign country and is diagnosed with rubella. What can occur to the developing fetus because of this infection? Note: Credit will be given only if all correct choices and no incorrect choices are selected. Select all that apply. 1. Congenital cataracts 2. Sensorineural deafness 3. Ophthalmia neonatorum 4. Congenital heart defects 5. Intrauterine growth restriction

Answer: 1, 2, 4 Explanation: The most common clinical signs of rubella syndrome are congenital cataracts, sensorineural deafness, and congenital heart defects. Ophthalmia neonatorum is associated with maternal gonorrhea at the time of birth. Intrauterine growth restriction is not associated with rubella.

1) The nurse is reviewing a list of families scheduled for community health visits. To visit these families according to the family life cycle each is in, in which order from first to last should the nurse visit these families? 1. Family with a 12-month-old child 2. Family whose oldest child is in the 5th grade 3. Family whose oldest child is attending college 4. Family whose youngest child just got a driver's license 5. Family whose youngest child got married last weekend 6. Family whose male partner retired from full-time employment

Answer: 1, 2, 4, 3, 5, 6 Explanation: According to the family life cycle stages, the family with a 12-month-old child would be seen first. The family with the oldest child in 5th grade would be seen second. Third, the family with the oldest child in college would be seen. The fourth family seen should be the one with the child who just received a driver's license. The fifth family would be the one whose youngest child was married the previous weekend. And the last family would be the one with a new retiree.

1) A pregnant patient is concerned about the development of several urinary tract infections (U T Is) over the last few months of her pregnancy. What should the nurse explain as reasons for the development of these infections in this patient? Note: Credit will be given only if all correct choices and no incorrect choices are selected. Select all that apply. 1. Decrease in bladder tone 2. Hyperemic bladder mucosa 3. Urethral stricture and loss of micturition reflex 4. Ureters elongate and are displaced by the uterus 5. Distal ureters hypertrophy leading to ureteral stenosis

Answer: 1, 2, 4, 5 Explanation: A number of structural and functional changes occur during pregnancy that predispose pregnant women to urinary tract infections. Ureters elongate and are laterally displaced by the gravid uterus. Progesterone, which relaxes smooth muscles, can facilitate hypertrophy of the distal ureters with resulting ureteral stenosis and dilation, especially in the second half of pregnancy. Though the bladder has an increased capacity in pregnancy, it also has a decreased tone because progesterone relaxes the smooth muscle. Estrogen causes the bladder mucosa to become hyperemic and more susceptible to trauma and infection. Pregnancy does not cause urethral strictures and loss of the micturition reflex.

1) The fetus of a pregnant Latino American patient has died. When assisting the family with the perinatal loss, what should the nurse expect? Note: Credit will be given only if all correct choices and no incorrect choices are selected. Select all that apply. 1. The fetus will be baptized 2. Pictures will be taken of the fetus 3. Cremation will occur within a day 4. The fetus will have an open casket 5. The mother will be dressed in black

Answer: 1, 2, 4, 5 Explanation: For a perinatal loss in many Latino American cultures, the fetus will be baptized. Pictures and other mementos are important, as they are utilized to honor the infant in the home. Open casket ceremonies are preferred. The mother may wear traditional dark funeral clothing for an undefined mourning period. Burial is preferred over cremation

1) While visiting the home of a single patient who is raising school-age children, the nurse becomes concerned that the quality of care for the children after school is less than adequate. What did the nurse observe that led to this conclusion? Note: Credit will be given only if all correct choices and no incorrect choices are selected. Select all that apply. 1. Children fighting in the front yard 2. Youngest child failing spelling and arithmetic 3. Middle child received an A on a writing assignment 4. Youngest child received a black eye from a child in school 5. Oldest child riding the bicycle in the street without a helmet

Answer: 1, 2, 4, 5 Explanation: Low-quality childcare is associated with increased peer arguments, lower cognitive and language scores, bullying, and risky or impulsive behavior. Higher academic scores are associated with mothers who have college educations.

1) While conducting a health interview, the nurse suspects that a middle-aged female client has undiagnosed learning disabilities. What did the nurse observe to make this clinical determination? Note: Credit will be given only if all correct choices and no incorrect choices are selected. Select all that apply. 1. Difficulty reading instructions 2. Illegible signature on treatment forms 3. Difficulty tying shoes when eyeglasses are not being worn 4. Inability to hear normal conversation through the right ear 5. Inability to select correct paper money to pay the insurance co-pay

Answer: 1, 2, 5 Explanation: Learning disabilities can inhibit educational attainment and employment. Learning disabilities include dyslexia, which can hinder reading, writing, and spelling; dysgraphia, which manifests with poor handwriting; and dyscalculia, which includes difficulty with processing math. The inability to tie shoes without wearing eyeglasses and not hearing through the right ear are physical issues that do not necessarily impact learning.

1) During a routine prenatal visit, the nurse suspects that the patient is experiencing undiagnosed depression. What findings did the nurse use to make this clinical determination? Note: Credit will be given only if all correct choices and no incorrect choices are selected. Select all that apply. 1. Insomnia 2. Headaches 3. Fear of labor 4. Ritualistic behavior 5. Lack of interest in the pregnancy

Answer: 1, 2, 5 Explanation: Manifestations of depression in the pregnant patient include withdrawn behavior, physical fatigue due to insomnia, crying spells, sadness, hopelessness, feelings of guilt, lack of interest in the baby, thoughts of suicide, and headaches. Fear of labor is associated with a panic disorder. Ritualistic behavior is associated with obsessive-compulsive disorder (O C D).

1) A new mother is planning to bottle feed her infant and wants helps with suppressing lactation. What should the nurse suggest to help this new mother? Note: Credit will be given only if all correct choices and no incorrect choices are selected. Select all that apply. 1. Wear a 24-hour support bra 2. Apply cabbage leaves to the breast tissue 3. Apply warm compresses every 4 to 6 hours 4. Massage lotion on the breasts 3 times a day 5. Avoid all nipple stimulation for 7 to 10 days

Answer: 1, 2, 5 Explanation: Nurses should advise the non-breastfeeding mother to avoid any stimulation of her breasts and nipples by her baby, herself, breast pumps, or her sexual partner until the sensation of fullness has passed (usually in 7 to 10 days). Such stimulation will increase milk production and delay the suppression process. Heat is avoided for the same reason. The wearing of a 24-hour support bra and the use of cabbage leaves and/or cold compresses should be helpful during this period of time.

1) A pregnant patient in the early stages of labor asks for assistance to sit in the whirlpool tub. What are the advantages of using this intervention for the laboring patient? Note: Credit will be given only if all correct choices and no incorrect choices are selected. Select all that apply. 1. Increases relaxation 2. Increases pain threshold 3. Reduces postural hypotension 4. Promotes maternal-infant bonding 5. Reduces the need for pain medication

Answer: 1, 2, 5 Explanation: The benefits of using a whirlpool tub during labor include increased relaxation, increased pain threshold, and reduced need for pain medication. The whirlpool tub will not reduce postural hypotension. Breastfeeding promotes maternal-infant bonding.

1) The nurse learns that a patient who is 8 weeks pregnant continues to smoke 10 cigarettes a day. In which order should the nurse provide a 5 to 15 minute intervention about smoking with this patient? 1. Ask about tobacco use 2. Advise to quit smoking 3. Assist in attempt to quit 4. Arrange for follow-up care 5. Assess willingness to quit

Answer: 1, 2, 5, 3, 4 Explanation: A C O G suggests that a 5- to 15-minute intervention with women who smoke fewer than 20 cigarettes a day is most effective. This program and other programs encourage healthcare providers to use the five As: ask about tobacco use; advise to quit smoking; assess willingness to quit; assist in attempt to quit; and arrange for follow-up care.

1) The nurse is planning care for a client who is the victim of rape. Which psychosocial nursing diagnoses does the nurse include in the client's plan of care? Note: Credit will be given only if all correct choices and no incorrect choices are selected. Select all that apply. 1. Fear 2. Fatigue 3. Powerlessness 4. Risk for infection 5. Readiness for enhanced knowledge

Answer: 1, 3 Explanation: When planning the psychosocial care for a client who is the victim of rape, the nurse would include the nursing diagnoses of fear and powerlessness in the plan of care. Fatigue, risk for infection, and readiness for enhanced knowledge are not diagnoses that the nurse would include in the plan of care for a client who is the victim of rape.

1) The nurse is concerned that a clinic patient is at risk for experiencing poverty. Which information did the nurse use to make this clinical determination? Note: Credit will be given only if all correct choices and no incorrect choices are selected. Select all that apply. 1. Of African-American descent 2. Attends church on Sunday mornings 3. Completed only up to 10th grade 4. Raising 2 children under the age of 10 alone 5. Works as a clothing assistant in a retail store

Answer: 1, 3, 4 Explanation: Factors that contribute to the feminization of poverty include being a member of an ethnic minority, not having sufficient education, and raising children as a single mother. Attending church and working in a retail store do not contribute to the feminization of poverty.

1) A postpartum patient who received epidural morphine prior to a cesarean birth is concerned about a severe headache that has persisted for several days. What should the nurse suggest to this patient? Note: Credit will be given only if all correct choices and no incorrect choices are selected. Select all that apply. 1. Ingest fluids with caffeine 2. Engage in moderate exercise 3. Increase the intake of all fluids 4. Lie in bed in a quiet dark room 5. Restrict the intake of warm fluids

Answer: 1, 3, 4 Explanation: For a spinal headache the nurse should instruct the patient to engage in bed rest in a quiet dark room. Caffeine and hydration are also helpful. Moderate exercise could exacerbate the headache. There is no reason to restrict the intake of warm fluids.

1) At the end of a routine examination, a 68-year-old female asks the nurse what she should observe to determine if she should obtain custody of her two preschool-age grandchildren. What factors should the nurse tell this patient to look for? Note: Credit will be given only if all correct choices and no incorrect choices are selected. Select all that apply. 1. Parental divorce 2. Long work hours 3. Drug or alcohol abuse 4. Intimate partner violence 5. Use of public transportation

Answer: 1, 3, 4 Explanation: The most common reasons why grandparents obtain guardianship of grandchildren include parental divorce, drug or alcohol abuse, and intimate partner violence. Long work hours and use of public transportation are not reasons for grandparents to obtain guardianship of grandchildren.

1) The nurse is caring for a client diagnosed with cystitis. When teaching the client about self-care techniques, which foods or beverages will the nurse advise the client to avoid? Note: Credit will be given only if all correct choices and no incorrect choices are selected. Select all that apply. 1. Caffeine 2. Dairy products 3. Alcohol 4. Carbonated beverages 5. Acidic fruit juices

Answer: 1, 3, 4 Explanation: The nurse should advise the client to avoid foods or beverages that are bladder irritants, such as caffeine, alcohol, or carbonated beverages. Dairy products and acidic fruit juices are not considered bladder irritants and would not be included when advising the client to avoid bladder irritants.

A patient is being assessed for recurrent vulvovaginal candidiasis (V V C) infections. What should the nurse instruct this patient to do to help reduce the incidence of infection? Note: Credit will be given only if all correct choices and no incorrect choices are selected. Select all that apply. 1. Avoid douching 2. Use vaginal sprays 3. Wear cotton underwear 4. Avoid tight-fitting clothing 5. Apply cornstarch to the vulva

Answer: 1, 3, 4 Explanation: Ways to prevent the recurrence of V V C include avoiding douching, wearing cotton underwear and avoiding tight-fitting clothing. Vaginal sprays should be avoided since they can irritate the vulva. Applying cornstarch to the vulva could encourage itching.

1) During a routine physical examination a female patient asks the nurse what can be done to prevent the development of breast cancer. What should the nurse review with the patient? Note: Credit will be given only if all correct choices and no incorrect choices are selected. Select all that apply. 1. Exercise regularly 2. Discuss starting tamoxifen 3. Reduce the intake of red meat 4. Maintain a normal body weight 5. Reduce the intake of dietary fat

Answer: 1, 3, 4, 5 Explanation: Actions to reduce modifiable risks for breast cancer include exercising regularly, reducing the intake of red meat, avoiding obesity, and reducing dietary fat. Women at high risk of breast cancer may choose to begin chemoprevention using tamoxifen

1) The nurse is monitoring a postpartum patient receiving methylergonovine maleate (Methergine). Which assessment findings should the nurse identify as being expected adverse effects of this medication? Note: Credit will be given only if all correct choices and no incorrect choices are selected. Select all that apply. 1. Nausea 2. Leg pain 3. Headache 4. Hypertension 5. Uterine cramping

Answer: 1, 3, 4, 5 Explanation: Common adverse effects of methylergonovine maleate (Methergine) include nausea, headache, hypertension, and uterine cramping. Leg pain is not an identified adverse effect of this medication.

1) A college student is distraught after being diagnosed with pediculosis pubis. What should the nurse instruct this student to do to help prevent future infections? Note: Credit will be given only if all correct choices and no incorrect choices are selected. Select all that apply. 1. Both partners need to be treated 2. Take the prescribed medication for 10 days 3. Avoid all sexual contact until treated and cured 4. Wash bed linens in hot water and dry in a dryer for 20 minutes 5. Testing for other sexually transmitted infections is recommended

Answer: 1, 3, 4, 5 Explanation: For pediculosis pubis, both partners need to be tested. All sexual contact should be avoided until treated and cured. Bed linens, towels, clothing, and other objects should be machine washed in hot water (at least 103°F) and dried in a hot dryer for 20 minutes. Both partners must be treated and tested for other S T Is. The medication for pediculosis pubis is topical, and repeated if nits are still present.

1) A married couple of Ashkenazi Jewish descent is pregnant with their first child. For which genetic health problems should the nurse provide teaching in anticipation of further testing? Note: Credit will be given only if all correct choices and no incorrect choices are selected. Select all that apply. 1. Cystic fibrosis 2. Beta-thalassemia 3. Canavan disease 4. Tay-Sachs disease 5. Familial dysautonomia

Answer: 1, 3, 4, 5 Explanation: Genetic screening for individuals of Ashkenazi Jewish descent includes cystic fibrosis, Canavan disease, Tay-Sachs disease, and familial dysautonomia. Beta-thalassemia is a genetic disorder seen in individuals of Greek or Italian descent.

1) The nurse is reviewing data collected during a health history and physical assessment and suspects that the patient could be experiencing polycystic ovarian syndrome (P C O S). What information did the nurse use to make this clinical determination? Note: Credit will be given only if all correct choices and no incorrect choices are selected. Select all that apply. 1. Body mass index 31 2. Hair loss and warm moist skin 3. Periods occur every 3 to 4 months 4. Fasting capillary blood glucose 123 m g/d L 5. Inability to become pregnant after 2 years of unprotected intercourse

Answer: 1, 3, 4, 5 Explanation: The most common clinical signs and symptoms of PCOS include obesity since half of women with PCOS are clinically obese. Irregular menses is the hallmark of PCOS. Hyperinsulinemia is associated with PCOS. The majority of women with PCOS struggle with some degree of infertility. Hair loss and warm moist skin are not manifestations of PCOS

1) The nurse is preparing teaching materials for female clients who wish to perform breast self-examination. In which order should the nurse ensure that the teaching materials present the process of inspection? 1. Compare the breasts 2. Study the skin surface 3. Analyze for symmetry 4. Study the shape and direction 5. Look at color, thickening, edema, and venous patterns

Answer: 1, 3, 4, 5, 2 Explanation: When inspecting the breasts, the breasts should be first compared, then analyzed for symmetry. Next the shape and direction should be studied, followed by looking at color, thickening, edema, and venous pattern. Lastly, the skin surface should be studied.

The nurse is preparing instructions for a patient newly diagnosed with genital herpes. What should the nurse encourage to promote healing of the lesions? Note: Credit will be given only if all correct choices and no incorrect choices are selected. Select all that apply. 1. Take sitz baths 2. Use vaginal sprays 3. Wear cotton underwear 4. Douche after intercourse 5. Wear loose fitting clothing

Answer: 1, 3, 5 Explanation: Actions to promote healing of genital herpes lesions include taking sitz baths, wearing cotton underwear, and wearing loose fitting clothing. Vaginal sprays and douching after intercourse will not help heal genital herpes lesions.

1. A patient who is hemorrhaging after a vaginal delivery is being considered for a uterine tamponade. What should the nurse instruct the patient about this process? Note: Credit will be given only if all correct choices and no incorrect choices are selected. Select all that apply. 1. A balloon is inserted into the uterus 2. The balloon is kept in place for 12 hours 3. The balloon is inflated with 300 to 500 m L of saline 4. After removal, the uterus is packed with sterile gauze 5. The tube has an open tip to permit bleeding to be visualized

Answer: 1, 3, 5 Explanation: If utero-tonic agents are unsuccessful at correcting uterine atony, the physician may use uterine tamponade. One technique of uterine tamponade is the use of the Bakri balloon, which is inserted into the uterine cavity and inflated with 300-500 m L of isotonic saline to provide pressure against the uterine walls. The tube has an open tip, which permits any continuous bleeding from the uterus to be visualized. If bleeding is controlled, the tamponade is removed after 24 hours and not 12 hours. Packing the uterus with sterile gauze is no longer favored as a method of tamponade.

1) The nurse is a caring for a patient who is recovering from the loss of her 24-week fetus. Which observations should the nurse identify as being the patient's behavioral responses to this loss? Note: Credit will be given only if all correct choices and no incorrect choices are selected. Select all that apply. 1. Crying 2. Disorientation 3. Time confusion 4. Fear of being alone Absent-mindedness

Answer: 1, 4, 5 Explanation: Behavioral responses to loss include fear of being alone, absent-minded behavior, and crying. Cognitive responses to loss include disorientation and time confusion

1) The nurse is preparing teaching material for a patient with multiple sclerosis who is in the third trimester of pregnancy. What information should the nurse emphasize with this patient? Note: Credit will be given only if all correct choices and no incorrect choices are selected. Select all that apply. 1. Plan for frequent rest periods 2. Breastfeeding should be avoided 3. Plan to experience heightened pain during labor 4. Determine the need for childcare help after delivery 5. Relapses may increase during the first 3 months after delivery

Answer: 1, 4, 5 Explanation: For the pregnant patient with multiple sclerosis, the nurse should emphasize that the relapse rate is reduced during the second and third trimester but increased during the 3 months following birth. Exclusive breastfeeding for the first 2 months postpartum may be independently associated with decreased post-pregnancy relapse rate. Rest is important; help with childcare should be planned. Uterine contraction strength is not diminished, but because sensation is frequently lessened, labor may be almost painless.

1) A patient with gestational diabetes (G D M) is instructed to measure capillary blood glucose levels 4 times a day. If the patient eats meals at 8 a.m., 1 p.m., and 7 p.m., at which times should the nurse reinforce glucose testing to occur? Note: Credit will be given only if all correct choices and no incorrect choices are selected. Select all that apply. 1. 10 a.m. 2. 12 noon 3. 4 p.m. 4. 3 p.m. 5. 9 p.m.

Answer: 1, 4, 5 Explanation: Women with G D M typically measure their blood glucose four times a day - fasting and 1 to 2 hours after meals.

1) The nurse is preparing to provide a newborn with an injection of vitamin K. In which order should the nurse complete the following steps? 1. Cleanse skin with alcohol and allow to dry 2. Aspirate and then inject the medication slowly 3. Insert a 25-gauge 5/8 inch needle at a 90 degree angle 4. Remove the needle and massage with an alcohol swab 5. Bunch skin over mid-anterior lateral aspect of the thigh

Answer: 1, 5, 3, 2, 4 Explanation: Procedure for vitamin K injection. Cleanse area thoroughly with alcohol swab and allow skin to dry. Bunch the tissue of the mid-anterior lateral aspect of the thigh (vastus lateralis muscle) and quickly insert a 25-gauge 5/8-inch needle at a 90-degree angle to the thigh. Aspirate, and then slowly inject the solution to distribute the medication evenly and minimize the baby's discomfort. Remove the needle and massage the site with an alcohol swab.

1. A postpartum patient weighing 165 l b is prescribed a subcutaneous injection of Enoxaparin 1 m g/k g twice daily. The medication available is 50 m g/m L. How many m L of medication should the nurse provide for each injection? (Calculate to the nearest tenth decimal point.)

Answer: 1.5 m L Explanation: First determine the patient's weight in k g by dividing 165 l b by 2.2 or 165/2.2 = 75 k g. Next determine the prescribed dose by multiplying the amount of 1 m g by k g of body weight or 1 m g × 75 k g = 75 m g. Next calculate the amount of medication to provide by using the equation Dosage Required/Dosage Available × m L; 75 m g/50 m g × 1 m L = 75/50 × 1 = 1.5 m L. The nurse should provide 1.5 m L for each injection.

1) During a home visit the mother of a 2 week old newborn is concerned that the baby always seems to be "wet" and wonders if this is normal. The newborn weighs 4 k g. How many m L of fluid should the nurse explain that the infant makes each day? (Round to the nearest whole number.)

Answer: 100 m L Explanation: The newborn voids 5 to 25 times every 24 hours, with a volume of 25 m L/k g/day. If the newborn weighs 4 k g then the amount of urine produced every day is 25 m L × 4 = 100 m L.

1) A patient in preterm labor is prescribed magnesium sulfate 6 grams intravenous infusion now, followed by 5 grams per hour. The pharmacy prepares an infusion of 500 m L lactated Ringer's solution with 100 grams of magnesium sulfate. If the patient receives the loading dose and 3 hours of the medication, how many total m L of the infusion did the patient receive?

Answer: 105 m L Explanation: The solution is 100 grams/500 m L or 1 gram in every 5 m L of solution. If the loading dose is 6 grams, then the patient received 6 grams × 5 m L =30 m L. For each hourly dose of 5 grams, the patient received 5 grams × 5 m L = 25 m L. Since the patient received the dose of 5 grams for 3 hours, then the patient received 25 m L × 3 = 75 m L. With the loading dose of 30 m L plus the three hours of infusion equaling 75 m L, the patient received 105 m L of the infusion.

1) A pregnant patient weighing 165 lb is diagnosed with preeclampsia and is prescribed to ingest 1.5 g/kg/day of protein. How many grams of protein should the nurse instruct the patient to ingest?

Answer: 112.5 grams Explanation: First convert the patient's weight in lb to k g by dividing the weight by 2.2 or 165/2.2 = 75 k g. Next multiply the weight in k g by 1.5 grams of protein or 75 x 1.5 = 112.5 grams. The nurse should instruct the patient to ingest 112.5 grams of protein each day.

1) A patient is labor is prescribed to receive nalbuphine 10 m g/70 k g intravenously now. The patient weighs 198 l b s. How many m g of medication should the nurse provide this patient? (Round to the nearest tenth decimal point.)

Answer: 12.9 m g Explanation: First calculate the patient's weight in k g by dividing the weight in lb by 2.2 or 198/2.2 = 90 k g. Then set up the equation 10 m g/70 k g = x/90 k g. When cross-multiplying, the equation will be 10/70 = x/90; 70x = 900. Then solve for x by dividing 900/70 = 12.85 m g. When rounding to the nearing tenth decimal point the dose should be 12.9 m g.

28) A patient who is 12 weeks pregnant is counseled to increase her protein intake by an additional 40 grams per day. If each gram of protein is 4 calories, how many additional calories per day should this patient ingest to reach the recommended 300 calories more per day during the pregnancy?

Answer: 140 calories Explanation: The additional protein provides 160 calories (40 grams × 4 = 160). If the total increase in daily calories is to be 300 calories, then subtract 160 calories for the extra protein from 300 or 300 — 160 = 140 calories.

28) Before becoming pregnant, a patient had a B M I of 28.5 and weighed 150 l b s. What should be the minimum weight of this patient upon delivery?

Answer: 165 l b s. Explanation: For a patient who is overweight, the total weight gain during pregnancy should be between 15 and 25 l b s. The minimum weight of this patient upon delivery should be 165 l b s.

The nurse is ensuring that a patient has provided informed consent before agreeing to an amniocentesis. In which order should the nurse validate that informed consent was provided by the patient? 1. Information provides risk and benefits 2. Information provided clearly and concisely 3. Information included treatment alternatives 4. Information explaining the right to refuse treatment 5. Information reviews consequences if no treatment provided

Answer: 2, 1, 3, 5, 4 Explanation: Several elements must be addressed to ensure that the patient has given informed consent. The information must be clearly and concisely presented in a manner understandable to the patient and must include risks and benefits, the probability of success, and significant treatment alternatives. The patient also needs to be told the consequences of receiving no treatment or procedure. Finally, the patient must be told of the right to refuse a specific treatment or procedure. Each patient should be told that refusing the specified treatment or procedure does not result in the withdrawal of all support or care.

1) During an interview the nurse learns that a patient's sister was recently diagnosed with endometrial cancer. What should the nurse review to reduce the patient's risk for developing the same disease process? Note: Credit will be given only if all correct choices and no incorrect choices are selected. Select all that apply. 1. Smoking cessation 2. Maintain a normal body mass index 3. Consider birth control without estrogen 4. Limit the intake of alcohol to one drink per day 5. Increase exercise to 30 minutes most days of the week

Answer: 2, 3 Explanation: Risk factors for endometrial cancer include obesity and long-term use of unopposed estrogen. Smoking, alcohol, and limited exercise are not risk factors for the development of endometrial cancer

1) The nurse is providing care for a female client who is the victim of sexual assault. Which sexually transmitted infections (S T Is) does the nurse anticipate medication prescriptions to prevent? Note: Credit will be given only if all correct choices and no incorrect choices are selected. Select all that apply. 1. Syphilis 2. Gonorrhea 3. Chlamydia 4. Bacterial vaginosis 5. Herpes simplex virus

Answer: 2, 3, 4 Explanation: A client who is the victim of sexual assault is at the greatest risk for contracting gonorrhea, chlamydia, and bacterial vaginosis. The nurse would anticipate medication prescriptions for these S T Is. While the client is also at risk for syphilis and herpes simplex virus, these S T Is are not as common; therefore, the nurse would not anticipate medication prescriptions for these S T Is.

1) patient seeks medical attention after being exposed to blood during a gang fight several weeks ago. For which types of hepatitis should the nurse anticipate that this patient will be tested? Note: Credit will be given only if all correct choices and no incorrect choices are selected. Select all that apply. 1. A 2. B 3. C 4. D 5. E

Answer: 2, 3, 4 Explanation: Hepatitis B, C, and D are transmitted through blood, body fluids, and blood products. Hepatitis A and E are transmitted through the oral-fecal route. Hepatitis A is also transmitted through contaminated food and water.

1) A patient experiencing menopause asks what complementary and alternative therapy can be taken to reduce the symptoms. After reviewing the patient's health history, for which problems should the nurse encourage the patient to avoid taking phytoestrogens? Note: Credit will be given only if all correct choices and no incorrect choices are selected. Select all that apply. 1. Allergy to soy 2. Currently taking tamoxifen 3. Treated for breast cancer 5 years ago 4. Surgery for uterine fibroids in her 20s 5. Experiences insomnia several times a week

Answer: 2, 3, 4 Explanation: Women who have had or are at risk for diseases that are affected by hormones, such as breast cancer or uterine fibroids, and women who are taking medications that increase estrogen levels in the body such as tamoxifen need to be especially careful about using phytoestrogens. An allergy to soy and experiencing insomnia are not reasons for the patient to avoid taking phytoestrogens.

1) A pregnant adolescent is scheduled for her first prenatal visit. For which sexually transmitted infections (S T Is) should the nurse anticipate testing will occur? Note: Credit will be given only if all correct choices and no incorrect choices are selected. Select all that apply. 1. H P V 2. Candida 3. Gonorrhea 4. Chlamydia 5. Trichomonas

Answer: 2, 3, 4, 5 Explanation: Adolescents have an increased incidence of STIs. The initial prenatal examination should include gonococcal and chlamydial cultures and wet prep for Candida, Trichomonas, and Gardnerella. HPV is not identified as a sexually transmitted infection to be tested during the initial prenatal examination

1) A female comes into the emergency department seeking treatment for possible rape. The patient recalls having a cocktail with friends at a local club but woke up in an alley three blocks away from the business. For which date rape drugs should the nurse prepare to have this patient tested? Note: Credit will be given only if all correct choices and no incorrect choices are selected. Select all that apply. 1. Atropine 2. Ketamine 3. Scopolamine 4. Flunitrazepam 5. Gamma hydroxybutyrate

Answer: 2, 3, 4, 5 Explanation: Flunitrazepam (Rohypnol), a potent sedative-hypnotic has received considerable attention as the "date rape drug of choice" since the late 1990s. Typically, Rohypnol, which dissolves easily and is odorless, is slipped into the drink of an unsuspecting woman. Gamma hydroxybutyrate (G H B), ketamine, and scopolamine have also been identified as date rape drugs that are used to incapacitate a woman. Atropine is not identified as being a date rape drug.

1) The nurse is preparing an educational program for pregnant adolescents in the community. What goals should the nurse identify as appropriate for this teaching? Note: Credit will be given only if all correct choices and no incorrect choices are selected. Select all that apply. 1. Managing finances 2. Increasing self-esteem 3. Preparing for labor and birth 4. Developing adaptive coping skills 5. Providing information about resources

Answer: 2, 3, 4, 5 Explanation: Goals for prenatal classes include preparing the participants for labor and birth, increasing self-esteem, developing adaptive coping skills, and providing information about available community resources. Managing finances is not an identified goal for adolescent prenatal classes.

1) A female patient experiencing menopause is concerned that periodic lapses of memory are symptoms of Alzheimer disease. What should the nurse review with the patient to reduce the risk of developing Alzheimer disease (A D)? Note: Credit will be given only if all correct choices and no incorrect choices are selected. Select all that apply. 1. Increase rest 2. Stop smoking 3. Exercise regularly 4. Eat a healthy diet 5. Maintain mental activity

Answer: 2, 3, 4, 5 Explanation: Lifestyle practices may help prevent A D include smoking cessation, regular exercise of at least 30 minutes 5 days a week, eating a healthy diet, and remaining mentally active. Increased rest does not help prevent A D.

A patient in her late 40s asks the nurse what she should expect when entering menopause. In which order should the nurse identify changes that the patient will experience during menopause? 1. Amenorrhea 2. Anovulation 3. Reduced fertility 4. Changes in menstrual flow 5. Menstrual cycle irregularities

Answer: 2, 3, 4, 5, 1 Explanation: Beginning 2 to 8 years before menopause, women experience episodes of anovulation, reduced fertility, decreased or increased menstrual flow, menstrual cycle irregularities, and then, ultimately, amenorrhea.

The nurse suspects that a newly admitted patient is experiencing manifestations of hepatitis A. What assessment findings did the nurse use to make this clinical determination? Note: Credit will be given only if all correct choices and no incorrect choices are selected. Select all that apply. 1. Rash 2. Fever 3. Jaundice 4. Joint pain 5. Gray-colored stool

Answer: 2, 3, 5 Explanation: Hepatitis A is characterized by symptoms of fever, jaundice, and gray-colored bowel movements. A rash and arthritis are associated with hepatitis B, C, and D.

1) The nurse is preparing to administer a sitz bath to a postpartum patient. In which order should the nurse perform the steps of this procedure? 1. Open the clamp on the tubing 2. Anchor the infusion bag to the sitz bath basin, with the tube facing upward 3. Fill the drainage bag with warm or cool water up to the top line as indicated on the bag 4. Close the clamp on the tubing, dry perineum with a clean towel, and apply new peri-pad 5. Secure the drainage bag from a hook over the toilet or from the handle used to flush the toilet

Answer: 2, 3, 5, 1, 4 Explanation: When providing a sitz bath the nurse should: insert the large infusion bag or tube into the back of the sitz bath basin, anchoring it to the bottom of the basin with the small opening at the end of the tubing facing upward, toward the ceiling; fill the drainage bag with warm or cool water up to the top line as indicated on the bag; secure the drainage bag from a hook over the toilet or from the handle used to flush the toilet if it is a few feet higher than the toilet; open the clamp on the tubing; once the sitz bath is complete, instruct the woman to close the clamp on the tubing, dry perineum with a clean towel, and apply new peri-pad.

1) A premature newborn is unable to suck at the breast. The nurse plans care for the mother, who is going to hand-express milk. Arrange the steps for milk expression in the correct order. 1. Roll the thumb and fingers simultaneously forward. 2. Position the thumb at 12:00 and the forefinger and middle finger at 6:00 around the areola. 3. Repeat the sequence multiple times to completely drain the breasts. 4. Stretch the areola back toward the chest wall without lifting the fingers off the breast.

Answer: 2, 4, 1, 3 Explanation: Positioning the thumb at 12:00 and the forefinger and middle finger at 6:00 around the areola is the beginning position for manually expressing milk. Next the mother should stretch the areola back as far as possible. Then roll the thumb and fingers forward to stimulate the breast to empty. The sequence is repeated as necessary until the desired amount of milk has been expressed.

1) The nurse is preparing to instruct the parents of a newborn on the care of the umbilical cord. In which order should the nurse provide these instructions? 1. Check the cord for color 2. Wash hands with soap and water 3. Fold diaper below umbilical cord 4. Clean cord and base of cord with cotton swab 5. Check the cord for odor or oozing of green material

Answer: 2, 4, 1, 5, 3 Explanation: Wash hands with clean water and soap before and after care. Clean cord and skin around base with a cotton swab or cotton ball. Clean 2 to 3 times a day, or with each diaper change. Cord should look dark and dry up before falling off. Check cord each day for any odor, oozing of greenish yellow material, or reddened areas around the cord. Fold diapers below umbilical cord to air-dry the cord.

1) The nurse in the community clinic is preparing educational materials to be used for teaching patients with sexually transmitted infections. What information should the nurse include regarding the medications metronidazole or tinidazole? Note: Credit will be given only if all correct choices and no incorrect choices are selected. Select all that apply. 1. Take this medication until symptoms disappear 2. Abstain from all alcohol while taking these medications 3. Stop taking oral contraceptives while taking these medications 4. Abstain from all alcohol for 72 hours after completing tinidazole 5. Abstain from all alcohol for 24 hours after completing metronidazole

Answer: 2, 4, 5 Explanation: Alcohol should be avoided when taking either metronidazole or tinidazole. When combined with alcohol, both metronidazole and tinidazole can produce effects similar to that of alcohol and Antabuse-abdominal pain, flushing, and tremors. The C D C (2010b) recommends abstaining from alcohol for 24 hours after completing metronidazole and 72 hours after completing tinidazole. If the woman is taking oral contraceptives, a backup nonhormonal contraceptive method is recommended during treatment with metronidazole. The patient should be encouraged to complete the full course of prescribed medications.

30) The nurse is instructing a pregnant patient on the importance of increasing her dietary intake of vitamin E. Which foods should the nurse recommend to meet this dietary need? Note: Credit will be given only if all correct choices and no incorrect choices are selected. Select all that apply. 1. Milk 2. Eggs 3. Liver 4. Green salads 5. Whole grain bread

Answer: 2, 4, 5 Explanation: Vitamin E is widely distributed in foodstuffs, especially vegetable fats and oils, whole grains, greens, and eggs. Milk and liver are good sources of vitamin A.

1) The nurse is preparing an educational session for parents of adolescents. In which order should the nurse explain the psychosocial development for this population? 1. Thinks abstractly 2. Lack impulse control 3. Formal operational thought 4. Egocentric and a concrete thinker 5. Experiments with drugs and alcohol

Answer: 2, 4, 5, 1, 3 Explanation: The early adolescent (age 14 and under) lacks impulse control, is very egocentric, and is a concrete thinker. The middle adolescent (15 to 17 years) experiments with drugs, alcohol, and sex. The late adolescent (18 to 19 years) thinks abstractly and is capable of formal operational thought.

1) The nurse is providing care to a female client in the acute phase of recovery following a sexual assault. Which nursing actions are appropriate? Note: Credit will be given only if all correct choices and no incorrect choices are selected. Select all that apply. 1. Clarifying feelings 2. Creating a safe environment 3. Supporting advocacy efforts 4. Establishing a trusting relationship 5. Providing care for significant others

Answer: 2, 5 Explanation: During the acute phase of recovery following a sexual assault, the appropriate nursing actions include creating a safe environment and providing care for significant others. Clarifying feelings is an appropriate nursing action during the reorganizational phase of recovery following a sexual assault. Supporting advocacy efforts is an appropriate nursing action during the integration and recovery phases of recovery following a sexual assault. Establishing a trusting relationship is an appropriate nursing action during the outward adjustment phase following a sexual assault.

A female patient comes into the clinic because of concerns about a sore that was present on her labia but spontaneously healed. During the interview the patient asks what could occur if the infection is syphilis. In what order should the nurse explain the course of this sexually transmitted infection? 1. Latent period with no lesions 2. Development of a chancre sore 3. Skin eruptions and sore throat occur 4. Tertiary stage with various symptoms 5. Development of a fever, weight loss, and malaise

Answer: 2, 5, 3, 1, 4 Explanation: Syphilis is divided into early and late stages. During the early stage (primary), a chancre appears at the site where the T. pallidum organism entered the body. Symptoms include slight fever, loss of weight, and malaise. The chancre persists for about 4 weeks and then disappears. In 6 weeks to 6 months, secondary symptoms appear. Skin eruptions called condylomata lata, which resemble wart-like plaques and are highly infectious, may appear on the vulva. Other secondary symptoms are acute arthritis, enlargement of the liver and spleen, nontender enlarged lymph nodes, iritis, and a chronic sore throat with hoarseness. A latent phase with no lesions may be followed by a tertiary stage.

1) The nurse is preparing to conduct a nonstress test with a pregnant patient. In which order should the nurse complete the steps of the procedure? 1. Obtain baseline measurement for 20 minutes 2. Place the patient in the semi-Fowler's position 3. Place the pressure transducer over the uterine fundus 4. Give the patient a handheld marker to indicate when fetal movement is felt 5. Place the ultrasound transducer from the external fetal monitor over the F H R

Answer: 2, 5, 3, 1, 4 Explanation: When conducting a nonstress test the nurse should first place the patient in the semi-Fowler's position. Then the ultrasound transducer form the external fetal monitor should be placed over the fetal heart rate. The pressure transducer is then placed over the uterine fundus. Baseline measurement should be obtained for 20 minutes. Finally, the nurse should provide the patient with a marker to indicate when fetal movement is felt.

1) The nurse is preparing to bathe and dress a fetus that has died. In which order should the nurse provide this postmortem care? 1. Pat dry 2. Apply gloves 3. Diaper and dress the fetus 4. Place in second basin of water to rinse 5. Place infant in lukewarm basin of water 6. Bathe according to skin integrity condition

Answer: 2, 5, 6, 4, 1, 3 Explanation: When bathing and dressing a fetus as part of postmortem care, the nurse should apply gloves and then place the fetus in a lukewarm basin of water. Cleanse the skin according to the fetus's skin integrity. Place fetus in a second basin of lukewarm water to rinse. Gently pat dry. Diaper and dress fetus in clothing provided.

32) A breastfeeding mother is instructed to increase her daily caloric intake an additional 500 calories each day. If her daily intake of protein is 65 grams at 4 calories per gram, how many calories will this patient need to ingest to reach the recommended daily intake?

Answer: 240 calories Explanation: The amount of calories ingested with the protein is 65 grams × 4 calories or 260 calories. To reach the recommended daily caloric increase of 500 calories, subtract 260 from 500 or 500 — 260 = 240 calories.

1) A patient in the second trimester of pregnancy with gestational diabetes is counseled to ingest 35 calories/k g of ideal body weight. The patient's ideal weight is 154 l b s. How many calories should the patient ingest every day?

Answer: 2450 calories Explanation: First determine the patient's ideal body weight in k g by dividing 154 lbs. by 2.2 k g or 154/2.2 = 70 k g. Then multiply the weight in k g by the number of calories per k g to consume or 70 k g × 35 calories = 2450. The patient's daily caloric intake should be 2450 calories.

A patient experiencing postpartum hemorrhage is prescribed to receive 741 m L of a crystalloid solution. How many m L of blood did this patient lose from the hemorrhage? (Calculate to the nearest whole number.)

Answer: 247 m L Explanation: Lost intravascular volume is replaced initially with rapid administration of warmed crystalloid solutions, in a 3 m L solution per 1 m L of estimated blood lost ratio. To determine blood loss, divide the volume of fluid by 3 or 741/3 = 247 m L.

1) At birth a newborn weighed 7 pounds 10 ounces. If the average weight gain is 7 ounces every week for the first 6 months, what weight should the nurse expect when assessing an infant that is 20 weeks old? (Calculate the weight in ounces.)

Answer: 260 ounces Explanation: Multiply 7 ounces × 20 = 140 ounces. Divide the amount in ounces by 16 or 140/16 = 8.75. Convert 8.75 to pounds and ounces by using the equation 75/100 = x/16; 1200 = 100x; x= 12 ounces. Convert the birth weight to ounces: 7 × 16 = 112 + 10 = 122. Convert the weight gained over 20 weeks: 8 × 16 = 128 + 12 = 140. Add the two weights: 122 + 140 = 260. The baby should weigh 260 ounces by 20 weeks.

A patient using the calendar rhythm method of birth control asks for assistance to calculate her most fertile period. She states that her shortest cycle is 22 days and her longest cycle is 40 days. Using this information, which day should the nurse identify as being the end of the patient's fertile period?

Answer: 29 Explanation: The calendar rhythm method (C R M) is based on the assumption that ovulation tends to occur 14 days (plus or minus 2 days) before the start of the next menstrual period. The fertile phase is calculated from 18 days before the end of the shortest recorded cycle through 11 days from the end of the longest recorded cycle. For this situation the where the cycle lasts from 22 to 40 days, the fertile phase would be calculated as day 4 (22-18) to day 29 (40-11). The last day of the fertile phase would be day 29.

1) The nurse is preparing an educational seminar for a group of middle-aged healthy women on health screening recommendations. What information should the nurse include during this educational session? Note: Credit will be given only if all correct choices and no incorrect choices are selected. Select all that apply. 1. Get a Pap test every 3 years 2. Schedule mammograms every 5 years 3. Get testing for H I V before the age of 60 4. Have a screening for colorectal cancer 5. Have blood pressure measured every year if 140/90

Answer: 3, 4 Explanation: A Pap test every 3 years is appropriate for women between the ages of 18 and 39. Mammograms should be obtained every 2 years through age 74. If the blood pressure measurement is 140/90 or higher, treatment should be discussed with the doctor or nurse. There is no age limit for H I V testing. Patient should be tested for H I V at least once if age 65 and have never been tested. Starting at age 50, patients should be screened for colorectal cancer.

1) An adolescent in the third trimester of pregnancy comes into the clinic for a routine examination. What should the nurse assess to determine if the patient is working through this trimester's developmental tasks? Note: Credit will be given only if all correct choices and no incorrect choices are selected. Select all that apply. 1. Amount of weight gained 2. Adequacy of nutritional intake 3. Supplies purchased for the baby 4. Presence of heartburn or constipation 5. Place in the home identified for the baby

Answer: 3, 4, 5 Explanation: During the third trimester, the nurse should assess whether the adolescent is preparing for the baby by buying supplies and preparing a place in the home. Assess for discomforts of pregnancy, such as heartburn and constipation. Adolescent may be uncomfortable mentioning these and other problems. Weight gain and nutritional intake are developmental tasks associated with the second trimester of pregnancy.

The nurse is assisting with the collection of evidence for a female client who is the victim of sexual assault. Which actions by the nurse are appropriate? Note: Credit will be given only if all correct choices and no incorrect choices are selected. Select all that apply. 1. Drawing blood to test for gonorrhea 2. Placing each piece of clothing in a plastic bag 3. Pulling hair from the head and pubic region as evidence 4. Collecting a urine sample if drug-facilitated rape is suspected 5. Obtaining informed consent prior to photographing the injured areas

Answer: 3, 4, 5 Explanation: When collecting evidence for a female client who is the victim of sexual assault, the nurse will assist in pulling hair from the head and pubic region as evidence, collect a urine sample if drug-facilitated rape is suspected, and obtain informed consent prior to photographing the areas of injury. The nurse would draw blood to test for syphilis, not gonorrhea. The nurse would place each piece of the client's clothing into a paper bag, which is sealed and labeled. A plastic bag is not appropriate.

1) The nurse is preparing to gavage-feed a preterm infant. Put the steps in the order in which the nurse should provide this feeding. 1. Check p H of the gastric aspirate 2. Elevate the syringe 6-8 inches above the infant's head 3. Measure from the tip of the nose to the earlobe to the xiphoid process 4. Clear the tubing with 2-3 m L of air 5. Lubricate the tube by dipping it into sterile water

Answer: 3, 5, 1, 2, 4 Explanation: Measurement occurs before inserting the tube into the infant. Lubricating the tube helps with passage into the infant. After passage, the p H of gastric contents is determined. The syringe is elevated above the infant's head for the feeding. At the end of the feeding the tube is cleared with 2-3 m L of air.

1) The nurse is instructing a new mother on the type of stool to expect a breast-fed newborn to produce. In which order should the nurse explain the stool changes?

Answer: 3, 5, 2, 1, 4 Explanation: Choice 3: On day 1, the infant should produce at least one wet diaper and one meconium stool by 24 hours of age. Choice 5: On day 2, the infant should produce at least two wet diapers and two early transitional stools in a 24-hour period by 48 hours of age. Choice 2: On day 3, the infant should produce at least three wet diapers and three transitional stools in a 24-hour period by 72 hours of age. Choice 1: On day 4, the infant should produce at least four wet diapers and three to four yellow-green transitional stools or yellow milk stools in a 24-hour period by 96 hours of age. Choice 4: On day 5, the infant should produce at least five wet diapers and three to four yellow milk stools per day; the stools are typically explosive and have a curdy or seedy appearance.

The nurse is reviewing the Quality and Safety Education for Nurses (Q S E N) competencies while preparing an in-service program to address safety in the neonatal intensive care unit. In which order should the nurse present these competencies? 1. Safety 2. Informatics 3. Patient-centered care 4. Quality improvement 5. Evidence-based practice 6. Teamwork and collaboration

Answer: 3, 6, 5, 4, 1, 2 Explanation: The Quality and Safety Education for Nurses (Q S E N) project is designed "to meet the challenge of preparing future nurses who will have the knowledge, skills and attitudes (K S A s) necessary to continuously improve the quality and safety of the healthcare systems within which they work. The project focuses on competencies in six areas: 1. Patient-centered care, 2. Teamwork and collaboration; 3. Evidence-based practice; 4. Quality improvement; 5. Safety; and 6. Informatics.

1) A pregnant patient's first day of her last menstrual period was 6/14. What would be this patient's estimated date of birth?

Answer: 3/21 Explanation: To determine estimated date of birth, subtract 3 months and add 7 days from the first day of the patient's last menstrual period. For this patient, this would be 6 - 3 or 3 and 14 + 7 or 21. The estimated date of birth would be 3/21.

1) At birth a newborn's head circumference is 13 inches. What should the nurse expect the chest circumference to be in c m? (Round to the nearest whole number.)

Answer: 31 c m Explanation: The circumference of the newborn's head is approximately 2 c m greater than the circumference of the newborn's chest at birth. First determine the infant's head circumference in c m by multiplying 13 inches by 2.54 c m or 13 × 2.54 = 33.02 or 33 c m. If the chest circumference is 2 c m smaller than the head circumference, the nurse should expect the infant's chest to measure 31 c m.

1) Put the following components specific to a postpartum examination in the proper sequential order: 1. L-lochia 2. E-emotional 3. H-Homans'/hemorrhoids 4. B-breasts 5. E-episiotomy/lacerations

Answer: 4, 1, 5, 3, 2 Explanation: If following B U B B L E H E (B: breast, U: uterus, B: bladder, B: bowel, L: lochia, E: episiotomy/laceration, H: Homans'/hemorrhoids, and E: emotion) the breasts should be assessed first followed by lochia, episiotomy, Homan's/hemorrhoids, and ending with emotion.

1) The nurse is reviewing the process of pumping the breasts with a new mother. In which order should the nurse provide this information? 1. Fill glass or bottles 3/4 full 2. Massage the breasts and relax 3. Sit up straight or lean forward 4. Wash hands with soap and water 5. Pump each breast for 10 to 20 minutes

Answer: 4, 2, 3, 5, 1 Explanation: Wash hands well with soap and water before preparing to pump. Take a few minutes to massage the breasts and relax. Sit up straight or lean slightly forward. Pump each breast for 10 to 20 minutes. Pump the expressed milk preferably into glass or plastic bottles. Do not fill milk storage containers more than 3/4 full, because milk expands during freezing.

1) The nurse is participating in the collection of evidence from a victim of rape. In which order should the evidence be collected from this victim? 1. Oral swabs are obtained 2. Blood samples are drawn for syphilis 3. Hair samples and fingernail scrapings taken 4. Clothing is removed and bagged for evidence 5. Swabs of body stains and secretions are taken

Answer: 4, 5, 1, 3, 2 Explanation: When collecting evidence from a rape victim, the victim's clothing is removed and placed in a paper bag. Swabs of body stains and secretions are taken. Then oral swabs are obtained. Hair samples and fingernail scrapings are taken. Blood samples are then drawn to evaluate for syphilis.

1) The nurse is preparing to meet with a female patient to review the most appropriate contraceptive method. In which order should the nurse complete the steps of this process? 1. Emphasize actions if pregnancy occurs 2. Instruct on the use of the selected method 3. Review side effects and warning symptoms 4. Assess for medical contraindications to specific methods 5. Learn about lifestyle, attitudes, religious beliefs and plans for children

Answer: 4, 5, 2, 3, 1 Explanation: In addition to completing a history and assessing for any medical contraindications to specific methods, spend time with a woman learning about her lifestyle, personal attitudes about particular contraceptive methods, religious and cultural beliefs, personal biases, and plans for future childbearing. Once the woman chooses a method, help her learn to use it effectively. Review any possible side effects and warning symptoms related to the method chosen and counsel the woman about what action to take if she suspects she is pregnant.

1) A newborn weighs 7 l b s. 10 ounces at birth. What is the maximum number of calories that the nurse should instruct the mother that the baby needs to consume each day? (Calculate to the nearest whole number.)

Answer: 400 calories Explanation: Caloric intake for a newborn is 45.5 to 52.5 k cal/l b/day or 100 to 115 k cal/k g/day. The nurse needs to convert the baby's weight in ounces to pounds by dividing 10 ounces/16 ounces or 10/16 or 0.625. Since the baby weighs 7.625 l b s., the nurse should multiply 52.5 × 7.625 = 400.3125 calories. When rounded to a whole number the infant needs a maximum of 400 calories per day.

1) A patient in the 10th week of pregnancy is diagnosed with gestational diabetes. The dietician determines that the patient's daily caloric intake should be 2200 calories and instructs the patient to ingest 40% of calories from carbohydrates, 20% of calories from protein, and 40% of calories from fats. How many calories should the patient ingest of protein each day?

Answer: 440 calories Explanation: To determine the number of calories of protein that should be ingested each day the daily caloric total is multiplied by 20% or 2200 × 20% = 440 calories.

1) The mother of a ne wly circumcised infant is concerned about caring for the infant at home. What should the nurse instruct the mother about the infant's care? Place the following actions in the order that should be instructed to the mother. 1. Pat dry 2. Rinse area with warm water 3. Fasten diaper snuggly over the penis 4. Apply small amount of petroleum jelly 5. Squeeze water over the circumcision site

Answer: 5, 2, 1, 4, 3 Explanation: Squeeze water over circumcision site once a day. Rinse area off with warm water and pat dry. Apply small amount of petroleum jelly. Fasten diaper over penis snugly enough so that it does not move and rub the tender glans.

The manager of a maternal-child care area is preparing information to share with nursing staff regarding the leading causes of infant death in the United States. In which order, from most to least frequent, should the manager provide this information? 1. S I D S 2. Low birth weight 3. Unintentional injuries 4. Maternal complications 5. Congenital malformation

Answer: 5, 2, 1, 4, 3 Explanation: The five leading causes of deaths of infants in the United States, from highest to lowest in frequency, are congenital malformations, low birth weight, S I D S, maternal complications, and unintentional injuries.

1) The nurse is helping a victim of domestic violence create a safety plan. In which order should the nurse recommend that the steps of the plan be completed? 1. Decide where to go regardless of the day or time 2. Establish a code word that is shared with family and friends 3. Have money, identification, and bank account information prepared 4. Determine a planned escape route with emergency telephone numbers 5. Pack a change of clothes, toilet articles, and keys stored away from the home

Answer: 5, 3, 1, 2, 4 Explanation: The patient should pack a change of clothes including toilet articles and an extra set of car and house keys stored away from her house with a friend or neighbor; have money, identification papers, and bank account information prepared; have a plan for where she will go, regardless of the day or time; establish a code word for danger that is shared with family and friends; and have a planned escape route and emergency telephone numbers

1) A female patient is anxious about having a pelvic examination. To help reduce the patient's fears in which order should the nurse explain that the examination will be performed? 1. The speculum is inserted 2. The speculum is removed 3. The perineum is inspected 4. The rectal examination is performed 5. The healthcare provider applies gloves 6. The bimanual examination is performed

Answer: 5, 3, 1, 2, 6, 4 Explanation: For a pelvic examination, the examiner dons gloves for the procedure. Let the woman know that the examiner begins with an inspection of the external genitalia. The speculum is then inserted to allow visualization of the cervix and vaginal walls and to obtain specimens for testing. After the speculum is withdrawn the examiner performs a bimanual examination of the internal organs using the fingers of one hand inserted in the woman's vagina while the other hand presses over the woman's uterus and ovaries. The final step of the procedure is generally a rectal examination.

A patient is concerned about contracting herpes genitalis from a sexual partner and asks the nurse what to expect if the infection is present. In which order should the nurse explain the infection to the patient? 1. Emotional trigger occurs 2. Lesions spontaneously appear 3. Take oral acyclovir as prescribed 4. Virus enters a dormant phase with no lesions 5. Development of single or multiple blister-like vesicles

Answer: 5, 3, 4, 1, 2 Explanation: The primary episode (first outbreak) of herpes genitalis is characterized by the development of single or multiple blister-like vesicles. Primary episodes usually last the longest and are the most severe. The recommended treatment of the first clinical episode of genital herpes is oral acyclovir, valacyclovir, or famciclovir. After the lesions heal, the virus enters a dormant phase, residing in the nerve ganglia of the affected area. Recurrences are usually less severe than the initial episode and seem to be triggered by emotional stress, menstruation, ovulation, pregnancy, and frequent or vigorous intercourse. Recurrence of the lesions is less severe.

1) During an educational session the nurse learns that legislation was written to support breastfeeding mothers. In which order did the titles of this legislation occur? 1. Establish standards for safe and effective breast pumps 2. Include breastfeeding equipment as medical care for taxes 3. Require businesses with 50 or more employees to give lactating women breaks 4. Give tax incentives to businesses that establish a private place for breastfeeding 5. Protect lactating women from being fired or discriminated against in the workplace

Answer: 5, 4, 1, 2, 3 Explanation: The Breastfeeding Promotion Act of 2009 was enacted into law on March 23, 2010 and acts to protect breastfeeding in the workplace through five provisions. These include the following: Title Ⅰ: Amending the Civil Rights Act of 1964 to protect lactating women from being fired or discriminated against in the workplace. Title Ⅱ: Giving tax incentives to businesses that establish a private space in the workplace for their employees to breastfeed or express their milk. Employers can also receive tax credits for supplying breastfeeding equipment and providing lactation consultation services for their employees. Title Ⅲ: Establishing set standards for breast pumps to ensure that they are safe and effective. Title Ⅳ: Expanding the Internal Revenue Code definition of "medical care" to include breastfeeding equipment and lactation services as tax-deductible for families. Title Ⅴ: Requiring employers with 50 or more employees to provide lactating employees break time and a private area to express their milk.

1) The nurse is demonstrating to a patient the proper steps for breastfeeding a newborn. Put these steps in the logical order that would assist the patient in placing the newborn to her breast. 1. Tickle the newborn's lips with the nipple. 2. Allow the newborn to latch on to the nipple. 3. The newborn opens her mouth wide. 4. Have the newborn face the mother tummy to tummy. 5. Position the newborn so the nose is at the level of the nipple.

Answer: 5, 4, 1, 3, 2 Explanation: Positioning the baby's nose at nipple level enhances latching on. The baby needs to face the mother. Tickling the newborn's lips provides stimulation, and the baby can smell the milk. If hungry, the newborn opens her mouth. The mother holds the baby to the breast for feeding.

1) The nurse is identifying a plan to help a rape victim work through the phases of recovery. In which order should the nurse perform the following actions to help this victim? 1. Clarify the victim's feelings 2. Establish a trusting relationship 3. Acknowledge the victim's success 4. Provide advocacy as requested by the victim 5. Allow the victim to grieve and express feelings

Answer: 5, 4, 2, 1, 3 Explanation: The order in which nursing actions should be provided to a victim during the phases of rape recovery include allowing the victim to grieve and express feelings during the acute phase; provide advocacy as identified by the victim during the outward adjustment phase; establish a trusting relationship and clarify the victim's feelings during the reorganizational phase; and acknowledge the victim's success during the integration and recovery phase

1) A neonate weighing 5.5 l b is prescribed to receive intravenous cefotaxime (Claforan) 50 m g/k g/dose every 6 hours for treatment of sepsis. How many m g of the medication should the nurse anticipate the patient will need for 24 hours? (Calculate to the nearest whole number.)

Answer: 500 m g Explanation: First convert the patient's weight in lb to kg by dividing by 2.2, or 5.5/2.2 = 2.5 kg. Then calculate the amount of medication needed for each dose by multiplying 50 mg × 2.5 kg = 125 mg. Then multiply the amount of medication for each dose by 4 (one dose every 6 hours) or 125 mg × 4 = 500 mg. The patient will receive 500 mg of the medication in a 24-hour period.

1) The nurse is preparing to care for a patient who has just learned that her fetus has died. In which order should the nurse adhere to the RESPONDING approach to grieving families? 1. Genuine caring 2. Open communication 3. Nonjudgmental attitude 4. Decision making assistance 5. Spiritual and cultural accommodation Recognition and validation of the loss

Answer: 6, 5, 2, 4, 3, 1 Explanation: The nurse should follow the essential elements of responding effectively to the needs of grieving families by following the steps in the acronym RESPONDING: recognition and validation of the loss; spiritual and cultural accommodation; open communication; decision making assistance; nonjudgmental attitude; and genuine caring

1) At birth a newborn measured 20 inches. What length should the nurse instruct the mother to expect the baby to be at 4 months? (Calculate the anticipated length in c m and round to the nearest whole number.)

Answer: 61 c m Explanation: The conversion 2.54 c m = 1 inch will be used. If the initial length was 20 inches, convert this to c m by multiplying 20 × 2.54 = 50.8 c m. If the average growth is approximately one inch per month for the first 6 months, then multiply 2.54 × 4 = 10.16 c m and add this amount to the birth length of 50.8 c m or 10.16 c m + 50.8 c m = 60.96. With rounding, the mother can expect the infant to be 61 c m in length.

1) The nurse is instructing a new mother on the amount and frequency of bottle-feeding for her newborn. The mother plans to use formula that is available in 6-ounce cans. If the infant ingests 25 m L for each of 8 feedings per day, how many cans of formula should the mother have available for a week? (Round to the nearest whole number.)

Answer: 8 cans Explanation: One ounce is equal to 30 m L. For a 6-ounce can, the amount of formula is 30 × 6 or 180 m L. For one day, the infant will ingest 25 m L × 8 feedings or 200 m L. For 7 days the infant will ingest 200 m L × 7 = 1400 m L. To determine the number of cans of formula needed divide the weekly total of 1400 m L by 180 m L or 1400/180 = 7.78 or 8 cans of formula is needed.

1) At 3 weeks a newborn weighs 8 l b s. 1 ounce. What percent of this body weight should the nurse explain to the mother as being water? (Calculate the average weight in ounces to the first decimal point.)

Answer: 93.5 ounces Explanation: Approximately 70% to 75% of the newborn's body weight is water. First determine the infant's weight in ounces by multiplying 8 × 16 ounces = 128 ounces + 1 ounce = 129 ounces. Then multiply the weight in ounces by 70% and then by 75%: 129 ounces × 70% = 90.3 ounces; 129 ounces × 75% = 96.75. Then add the values of 70% and 75% and divide by 2 to determine the average: 90.3 + 96.75 = 187.05/2 = 93.525 ounces. To determine this weight in pounds divide by 16, or 93.525/16 = 5.84 of the infant's weight is water. To determine the weight in pounds and ounces: .84/100 = x/16; 1344 = 100x; 1344/100 = 13.44 ounces. The percentage of the infant's weight that is water is 5 pounds, 13.44 ounces.

1) The nurse is making an initial visit to a postpartum family's home. The mother states that she is having difficulty with breastfeeding. Which resource should the nurse tell the family about? A) The lactation consultant at the hospital B) Free immunizations through the county public health department clinics C) Sources of free formula at a local food pantry D) A support group for mothers who are experiencing postpartum depression

Answer: A Explanation: A) When the client specifies a problem with breastfeeding, the best resource for the nurse to inform the family about is the lactation consultant. B) Free immunizations do not help with breastfeeding. C) Providing a mother with formula undermines her breastfeeding efforts and sends a message that she will not be successful with breastfeeding. D) There is no evidence that the client is experiencing postpartum depression.

1) The client's Pap smear result is A S C-U S. Which statement is the best way for the nurse to explain this A S C-U S result? A) "Abnormal cells of an unknown cause." B) "Cancer has invaded the upper cervix." C) "High-grade squamous intraepithelial lesion (H S I L), which includes C I N." D) "The focus of the Pap smear is the detection of high-risk pregnancy." E) "The cervical cells are abnormal and the reason why is severe dysplasia and carcinoma in situ."

Answer: A Explanation: A) A S C-U S stands for abnormal squamous cells of undetermined significance. The nurse should tell the client that these are abnormal cells of an unknown cause. Preferred management is H P V testing; if positive, refer for colposcopy; if negative, repeat H P V co-testing in 3 years. B) A S C-U S does not indicate cancer. C) C I N refers to a lesion that may progress to invasive carcinoma (cancer). A S C-U S does not indicate cancer. D) The focus of the Pap smear is the detection of high-grade cervical disease, especially cervical intraepithelial neoplasia (C I N). ASC-US stands for abnormal squamous cells of undetermined significance. The cervical cells are abnormal, but the reason why is unknown

A nurse who tells family members the sex of a newborn baby without first consulting the parents would have committed which of the following? A) A breach of privacy B) Negligence C) Malpractice D) A breach of ethics

Answer: A Explanation: A) A breach of privacy would have been committed in this situation, because informing other family members of the child's sex without the parents' consent violates the parents' right to privacy. The right to privacy is the right of a person to keep his person and property free from public scrutiny (or even from other family members). B) Negligence is a punishable legal offense, and is more serious. C) Malpractice is a punishable legal offense, and is more serious. D) No breach of ethics has been committed in this situation.

1) The nurse assesses four newborns. Which of the following assessment findings would place a newborn at risk for developing physiologic jaundice? A) Cephalohematoma B) Mongolian spots C) Telangiectatic nevi D) Molding

Answer: A Explanation: A) A cephalohematoma is a collection of blood resulting from ruptured blood vessels between the surface of a cranial bone and the periosteal membrane. They may be associated with physiologic jaundice, because there are extra red blood cells being destroyed within the cephalohematoma. B) Mongolian spots are macular areas of bluish-black pigmentation on the dorsal area of the buttocks. C) Telangiectatic nevi are pale pink or red spots found on the eyelids, nose, lower occipital bone, or nape of the neck. D) Molding is caused by overriding of the cranial bones.

1) The nurse is caring for four newborns who have recently been admitted to the newborn nursery. Which labor event puts the newborn at risk for an alteration of health? A) The infant's mother has group B streptococcal (G B S) disease. B) The infant's mother had an I V of lactated Ringer's solution. C) The infant's mother had a labor that lasted 12 hours. The infant's mother had a cesarean birth with her last child

Answer: A Explanation: A) A common cause of neonatal distress is early-onset group B streptococcal (G B S) disease. Infected mothers transmit G B S infection to their infants during labor and birth. All infants of mothers identified as at risk should be assessed and observed for signs and symptoms of sepsis. B) An I V of lactated Ringer's solution will not affect the newborn's blood sugar. C) A 12-hour labor is normal. D) Having had a cesarean with her last child poses risk factors for the mother during labor, but does not affect this newborn.

1) The postpartum home care nurse has performed home visits to four breastfeeding mothers. Which mother is experiencing an expected outcome? A) Breasts are engorged; placing fresh cabbage leaves inside her bra B) Sore and cracked nipples; using hydrogel dressings to facilitate healing C) Breast engorgement; accompanied by erythema D) Concerns about milk supply; supplementing with formula

Answer: A Explanation: A) A compress of fresh green cabbage leaves helps reduce engorgement. B) The use of hydrogel dressings cannot be recommended at this time due to concerns about infection. C) For breast engorgement accompanied by erythema, the nurse should instruct the mother to keep the breast empty by frequent feeding, rest when possible with breasts elevated, take prescribed pain relief medication, and drink adequate fluids. D) Because milk production follows the principle of supply and demand, if breasts are not pumped, the milk supply will decrease. Supplementing with formula will decrease milk supply.

1) The home health nurse is visiting a client at 18 weeks who is pregnant with twins. Which nursing action is most important? A) Teach the client about foods that are good sources of protein. B) Assess the client's blood pressure in her upper right arm. C) Determine whether the pregnancy is the result of infertility treatment. D) Collect a cervicovaginal fetal fibronectin (fF N) specimen.

Answer: A Explanation: A) A daily intake of 4000 k c a l (minimum) and 135 g protein is recommended for a woman with normal-weight twins. B) Blood pressure can be assessed in either arm. C) The cause of the multifetal pregnancy does not impact nursing care. D) Preterm labor is not diagnosed until 20 weeks. This client is only at 18 weeks. Fetal fibronectin (f F N) testing is not indicated at this time.

1) The prenatal clinic nurse is explaining test results to a client who has had an assessment for fetal well-being. Which statement indicates that the client understands the test results? A) "The normal Doppler velocimetry wave result indicates my placenta is getting enough blood to the baby." B) "The reactive nonstress test means that my baby is not growing because of a lack of oxygen." C) "Because my contraction stress test was positive, we know that my baby will tolerate labor well." D) "My biophysical profile score of 6 points to everything being normal and healthy for my baby."

Answer: A Explanation: A) A decrease in fetal cardiac output or an increase in resistance of placental vessels will reduce umbilical artery blood flow. Doppler velocimetry is best used when intrauterine growth restriction is diagnosed; a normal result indicates that the baby is getting an adequate blood supply. B) The nonstress test involves using an external electronic fetal monitor to obtain a tracing of the fetal heart rate (F H R) and observation of acceleration of the F H R with fetal movement. C) A contraction stress test (C S T) provides a method for observing the response of the F H R to the stress of uterine contractions. The desired result is a negative test. D) A score of 6 is abnormal, and indicates that further assessment is needed.

1) A woman is admitted to the birth setting in early labor. She is 3 c m dilated, -2 station, with intact membranes and F H R of 150 beats/min. Her membranes rupture spontaneously, and the F H R drops to 90 beats/min with variable decelerations. What would the initial response from the nurse be? A) Perform a vaginal exam. B) Notify the physician. C) Place the client in a left lateral position. D) Administer oxygen at 2 L per nasal cannula.

Answer: A Explanation: A) A drop in fetal heart rate accompanied by variable decelerations is consistent with a prolapsed cord. The nurse would assess for prolapsed cord via vaginal examination. B) The vaginal exam should be done before the physician is notified. C) Repositioning the client will not relieve the decreased heart rate if the cord is compromised. D) Administering oxygen will not relieve the decreased heart rate if the cord is compromised.

1) The nurse auscultates the F H R and determines a rate of 112 beats/min. Which action is appropriate? A) Inform the maternal client that the rate is normal. B) Reassess the F H R in 5 minutes because the rate is low. C) Report the F H R to the doctor immediately. D) Turn the maternal client on her side and administer oxygen.

Answer: A Explanation: A) A fetal heart rate of 112 beats/min. falls within the normal range of 110-160 beats/min., so there is no need to inform the doctor. B) There is no need to reassess later. C) There is no need to inform the doctor. There is no need to reposition the client

1) A 27-year-old married woman is 16 weeks pregnant and has an abnormally low maternal serum alpha-fetoprotein test. Which statement indicates that the couple understands the implications of this test result? A) "We have decided to have an abortion if this baby has Down syndrome." B) "If we hadn't had this test, we wouldn't have to worry about this baby." C) "I'll eat plenty of dark green leafy vegetables until I have the ultrasound." D) "The ultrasound should be normal because I'm under the age of 35."

Answer: A Explanation: A) A low maternal serum alpha-fetoprotein test can indicate trisomy 18 or trisomy 21 (Down syndrome). Many couples abort a fetus that has a genetic abnormality that significantly affects quality of life or has multiple medical problems. Down syndrome is more likely to occur in the fetuses of women over the age of 35 at delivery, but is not limited to this age group. B) A low maternal serum alpha-fetoprotein test can indicate trisomy 18 or trisomy 21 (Down syndrome). C) The condition begins in very early fetal life. Dark green leafy vegetables contain folic acid. Low folic acid levels in the first trimester can lead to neural tube defects, which would cause a high maternal serum alpha-fetoprotein screen. D) Down syndrome is more likely to occur in women over the age of 35 at delivery, but is not limited to this age group.

1) While caring for a client admitted to the birthing unit, the nurse suspects that the client may be experiencing a uterine rupture. Which assessment finding should the nurse expect to appear first? A) Nonreassuring fetal heart rate B) Constant abdominal pain C) Loss of fetal station D) Cessation of contractions

Answer: A Explanation: A) A nonreassuring fetal heart rate is commonly the earliest warning sign of a possible uterine rupture. B) Constant abdominal pain is a finding that may be present, but is not commonly the earliest sign of uterine rupture. C) Loss of fetal station is a finding that may be present, but is not commonly the earliest sign of uterine rupture. D) Cessation of contractions is a finding that may be present, but is not commonly the earliest sign of uterine rupture.

1) A cesarean section is ordered for a pregnant client. Because the client is to receive general anesthesia, what is the primary danger with which the nurse is concerned? A) Fetal depression B) Vomiting C) Maternal depression D) Uterine relaxation

Answer: A Explanation: A) A primary danger of general anesthesia is fetal depression. The depression in the fetus is directly proportional to the depth and duration of the anesthesia. B) Vomiting is not a primary concern. C) Maternal depression is not a primary danger of general anesthesia. D) Uterine relaxation has nothing to do with general anesthesia.

1) A primary herpes simplex infection in the first trimester can increase the risk of which of the following? A) Spontaneous abortion B) Preterm labor C) Intrauterine growth restriction D) Neonatal infection

Answer: A Explanation: A) A primary herpes simplex infection can increase the risk of spontaneous abortion when infection occurs in the first trimester. B) Preterm labor (PTL) is a greater risk if the primary infection occurs late in the second trimester or early in the third trimester. C) Intrauterine growth restriction is a greater risk if the primary infection occurs late in the second trimester or early in the third trimester. Neonatal infection is a greater risk if the primary infection occurs late in the second trimester or early in the third trimester

1) The student nurse notices that a newborn weighs less today compared with the newborn's birth weight three days ago. The nursing instructor explains that newborns lose weight following birth due to which of the following? A) A shift of intracellular water to extracellular spaces. B) Loss of meconium stool. C) A shift of extracellular water to intracellular spaces. D) The sleep-wake cycle.

Answer: A Explanation: A) A shift of intracellular water to extracellular space and insensible water loss account for the 5% to 10% weight loss. B) Loss of meconium stool does not effect this amount of weight loss. C) A shift of intracellular water to extracellular space and insensible water loss account for the 5% to 10% weight loss. D) The sleep-wake cycle does not effect this amount of weight loss.

1. How would the nurse best analyze the results from a client's sonogram that shows the fetal shoulder as the presenting part? A) Breech, transverse B) Breech, longitudinal C) Breech, frank D) Vertex, transverse

Answer: A Explanation: A) A shoulder presentation is one type of breech presentation, and is also called a transverse lie. B) A shoulder presentation is not a longitudinal lie. C) In a frank breech, the buttocks are the presenting part. D) A shoulder presentation is not vertex.

1) The nurse is performing discharge teaching for a newly delivered first-time mother and her infant on the 2nd postpartum day. Which statement by the mother indicates that teaching has been successful? A) "Taking baths will help my perineum feel less sore each day." B) "If I develop heavy bleeding, I should take my temperature." C) "My bowel movements should resume in a week." D) "I will go back to the doctor in 4 days for my R h o G A M shot."

Answer: A Explanation: A) A sitz bath or tub bath promotes healing and provides relief from perineal discomfort during the initial weeks following birth. B) If heavy bleeding begins, the client should call her healthcare provider immediately, not take her temperature. Postpartum hemorrhage can be life-threatening. C) Bowel movements should resume in 2 to 3 days after birth. A week is too long a time frame, and indicates constipation. D) When R h o G A M is needed, it is given within 72 hours of birth, while still at the hospital.

1) A variety of drugs are used either alone or in combination to provide relief of postpartum pain. Which of the following would be an option for pain relief? A) Nonsteroidal anti-inflammatory agents B) Proquad C) Methergine D) Intravenous oxytocin

Answer: A Explanation: A) A variety of drugs are used alone or in combination to provide relief of postpartum pain. An option would include nonsteroidal anti-inflammatory agents such as ibuprofen and ketorolac. B) Proquad is a measles, mumps, rubella, and varicella live virus vaccine. C) Methergine is prescribed to promote uterine contractions. D) Intravenous oxytocin (Pitocin) remains the first-line drug for excessive bleeding related to postpartum uterine atony.

1) An older person contacts the emergency medical service at 11 P M to report that she has been left sitting in her wheelchair all day after her caregiver left in the morning to buy groceries. What type of elder abuse is this person experiencing? A) Abandonment B) Physical abuse C) Financial abuse D) Psychological abuse

Answer: A Explanation: A) Abandonment is the desertion of an elder by any person responsible for the care and custody of that elder, under circumstances in which a reasonable person would continue to provide care. There is no evidence that the older person is experiencing physical, financial, or psychological abuse.

1) Abdominal hysterectomy is generally recommended for which condition? A) Severe endometriosis B) Removal of the ovaries C) Suspected or confirmed cancer removal D) Abnormal uterine bleeding

Answer: A Explanation: A) Abdominal hysterectomy is recommended for severe endometriosis. B) Removal of the uterus through an abdominal incision is called a total abdominal hysterectomy (T A H), and removal of both fallopian tubes and ovaries is called a bilateral salpingo-oophorectomy (B S O); when both procedures are done at the same time it is termed a T A H-B S O. C) Total abdominal hysterectomy (T A H) is preferred when cancer is suspected or confirmed because it permits easier exploration of the abdomen and pelvis to determine the degree and extent of involvement. D) Vaginal hysterectomy is generally done for pelvic relaxation, abnormal uterine bleeding, or small fibroids.

1) The nurse is caring for a postpartal client of Hmong descent who immigrated to the United States 5 years ago. The client asks for the regular hospital menu because American food tastes best. The nurse assesses this response to be related to which of the following cultural concepts? A) Acculturation B) Ethnocentrism C) Enculturation D) Stereotyping

Answer: A Explanation: A) Acculturation (assimilation) is the correct assessment because the client adapted to a new cultural norm in terms of food choices. B) Ethnocentrism refers to a social identity that is associated with shared behaviors and patterns. C) Enculturation occurs when culture is learned and passed on from generation to generation, and often happens when a group is isolated. D) Stereotyping is the assumption that all members of a group have the same characteristics.

1) What condition is due to poor peripheral circulation? A) Acrocyanosis B) Mottling C) Harlequin sign D) Jaundice

Answer: A Explanation: A) Acrocyanosis is a bluish discoloration of the hands and feet that may be present in the first 24 hours after birth and is due to poor peripheral circulation, which results in vasomotor instability and capillary stasis, especially when the baby is exposed to cold. B) Mottling is a lacy pattern of dilated blood vessels under the skin and occurs as a result of general circulation fluctuations. C) Harlequin sign (clown) color change is a deep color that develops over one side of the newborn's body while the other side remains pale, so that the skin resembles a clown's suit. D) Jaundice is a yellowish discoloration of skin and mucous membranes first detectable on the face and the mucous membranes of the mouth.

The client reports relief from headaches when she rubs the temples on each side of her head. The nurse understands that this is a form of which of the following? A) Acupressure B) Acupuncture C) Reflexology D) Hydrotherapy

Answer: A Explanation: A) Acupressure uses pressure from the fingers and thumbs to stimulate pressure points to relieve symptoms. B) Acupuncture uses 6-12 very fine stainless steel needles to stimulate specific points, depending on the client's medical assessment and condition. C) Reflexology is a form of massage that involves the application of pressure to designated points or reflexes on the client's feet, hands, or ears using the thumb and fingers. Hydrotherapy is therapy that makes use of hot or cold moisture in any form

16) The nurse is planning an educational session for pregnant vegans. What information should the nurse include? A) Eating beans and rice provides complete protein needs. B) Soy is not a good source of protein for vegans. C) Rice contains a high level of vitamin B12. D) Vegan diets are excessively high in iron.

Answer: A Explanation: A) Adequate dietary protein can be obtained by consuming a varied diet with adequate caloric intake and plant-based proteins. Consuming an assortment of plant proteins throughout the day such as beans and rice, peanut butter on whole-grain bread, and whole-grain cereal with soy milk ensures that the expectant mother obtains all essential amino acids. B) Good sources of plant proteins include beans, soy products, lentils, nuts, and nut butters. C) Vitamin B12 is the cobalt-containing vitamin found only in animal sources. D) Supplementation may be recommended for vegans who have difficulty meeting the recommended amounts of iron through food sources.

1) The nurse is caring for a client in labor who has a history of physical dependence on narcotics. Which consideration should the nurse take with regard to the administration of naloxone (Narcan)? A) Inducing withdrawal symptoms B) Prolonging respiratory depression C) Exacerbating pruritis D) Increasing the risk for fetal depression

Answer: A Explanation: A) Administering naloxone (Narcan) to a client who is physically dependent on narcotics may induce withdrawal symptoms, which will adversely affect her and her baby. B) Naloxone (Narcan) is used to correct respiratory depression, and does not prolong it. C) Naloxone is an opiate antagonist, and is not expected to exacerbate the side effects of opioids such as pruritis. D) Fetal depression is not expected to occur with the use of naloxone (Narcan).

1) The 12-year-old client reports that menarche occurred 5 months ago. She has had bleeding every day this month, and is very worried. The nurse should explain that the most common cause of this bleeding is which of the following? A) Dysfunctional uterine bleeding (D U B) B) Diabetes mellitus (D M) C) Pregnancy D) Von Willebrand's disease

Answer: A Explanation: A) Adolescents often experience D U B during the first 2 years following menarche due to hypothalamic immaturity after menarche. B) Uterine bleeding is not a symptom of D M. C) Uterine bleeding is not a symptom of pregnancy. Although von Willebrand's disease can cause irregular uterine bleeding, it is quite rare

1) The nurse is preparing a class on breastfeeding for pregnant women in their first trimester. The women are from a variety of cultural backgrounds, and all speak English well. Which statement should the nurse include in this presentation? A) "Although some cultures believe colostrum is not good for the baby, it provides protection from infections and helps the digestive system to function." B) "Some women are uncomfortable with exposing their breasts to nurse their infant, but it really isn't a big deal. You will get used to it." C) "No religion prescribes a feeding method, so you all can choose whatever method makes the most sense to you." D) "In most cultures, it is culturally acceptable to speak about intimate matters in front of their families."

Answer: A Explanation: A) Although it is true that some cultures believe colostrum to be unhealthy, colostrum helps to protect the infant from disease and illness. B) It is not therapeutic to downplay a woman's concern by stating, "It's no big deal"; this type of statement should be avoided. C) Muslim women generally breastfeed until their children are 2 years of age. This is encouraged in the Koran. D) For Muslim women, it is culturally unacceptable for them to speak about intimate matters in front of their families. The nurse should be aware that it would be inappropriate to ask the new mother's husband or her children to be her interpreter.

1) The nurse is preparing a client for amniocentesis. Which statement would indicate that the client clearly understands the risks of amniocentesis? A) "I might go into labor early." B) "It could produce a congenital defect in my baby." C) "Actually, there are no real risks to this procedure." D) "The test could stunt my baby's growth."

Answer: A Explanation: A) Amniocentesis has the potential to cause spontaneous abortion. B) Congenital defects are the result of heredity or medications. C) Amniocentesis has potential complications such as infection, bleeding and spontaneous abortion. Growth retardation most commonly is associated with heredity or poor nutrition

1) The pregnant client and her partner are both 40 years old. The nurse is explaining the options of chorionic villus sampling (C V S) and amniocentesis for genetic testing. The nurse should correct the client if she makes which statement? A) "Amniocentesis results are available sooner than C V S results are." B) "C V S carries a higher risk of limb abnormalities." C) "Amniocentesis cannot detect a neural tube defect." D) "C V S is performed through my belly or my cervix."

Answer: A Explanation: A) Amniocentesis results take longer to process than do C V S results. B) Limb anomalies are associated with C V S, but not with amniocentesis. C) Neural tube defects are not genetic in nature; therefore, they are not detected by either amniocentesis or C V S. D) C V S can be performed through either a transabdominal or transvaginal approach.

1) The nurse is teaching the parents of an infant with an inborn error of metabolism how to care for the infant at home. What information does teaching include? A) Specially prepared formulas B) Cataract problems C) Low glucose concentrations D) Administration of thyroid medication

Answer: A Explanation: A) An afflicted P K U infant can be treated by a special diet that limits ingestion of phenylalanine. Special formulas low in phenylalanine, such as Phenyl-Free 1 and Phenex-1, are available. B) Cataracts are associated with infants who have galactosemia. C) Low glucose concentrations are not an indication an inborn error of metabolism. D) Thyroid medication is given to infants with congenital hypothyroidism.

13) A pregnant client who was of normal pre-pregnancy weight is now 30 weeks pregnant. She asks the nurse what appropriate weight gain for her should be. What is the nurse's best response? A) "25-35 pounds" B) "30-40 pounds" C) "17-18 pounds" D) "Less than 15 pounds"

Answer: A Explanation: A) An appropriate weight gain for a woman of normal weight before pregnancy would be 25-35 pounds. B) This is not the correct range for woman of normal weight before pregnancy. C) This is not the correct range for woman of normal weight before pregnancy. D) A woman of normal weight before pregnancy should gain more than 15 pounds by 30 weeks.

1) The nurse is caring for a client experiencing a uterine rupture. Which outcome demonstrates that the plan of care has been effective for the client? A) The mother remains hemodynamically stable throughout emergency cesarean birth. B) The mother has additional knowledge regarding the problems, implications, and treatment plans. C) The F H R remains in normal range with supportive measures. D) The family is able to cope successfully with fetal or neonatal anomalies, if they exist.

Answer: A Explanation: A) An emergency cesarean birth is warranted in the case of a client experiencing a uterine rupture. Hemodynamic stability is a major goal of interventions performed for a client with a uterine rupture. B) Knowledge deficit is not a priority nursing diagnosis for a client experiencing a uterine rupture. C) In the case of a uterine rupture, fetal heart rate anomalies are often already present. D) Uterine rupture is not indicative of fetal or neonatal anomalies.

1) The nurse has completed a community presentation about the changes of pregnancy, and knows that the lesson was successful when a community member states that which of the following is one probable or objective change of pregnancy? A) "Enlargement of the uterus" B) "Hearing the baby's heart rate" C) "Increased urinary frequency" D) "Nausea and vomiting"

Answer: A Explanation: A) An examiner can perceive the objective (probable) changes that occur in pregnancy. Enlargement of the uterus is a probable change. B) Hearing the fetal heart rate is a diagnostic, or positive, change of pregnancy. C) Increased urinary frequency is a subjective, or presumptive, change of pregnancy. D) Nausea and vomiting are subjective, or presumptive, changes of pregnancy.

22) Which statement is best to include when teaching a pregnant adolescent about her nutritional needs in pregnancy? A) "It is important to eat iron-rich foods like meat every day." B) "Calcium and milk aren't needed until the third trimester." C) "Folic acid intake is the key to having a healthy baby." D) "You just need to pay attention to what you eat now."

Answer: A Explanation: A) An inadequate iron intake is a major concern with the adolescent diet. Iron needs are high for the pregnant teen because of the requirement for iron by the enlarging maternal muscle mass and blood volume. Giving specific examples is helpful when giving nutritional information. B) Calcium is needed throughout pregnancy, and should be consumed daily. C) Although folic acid is important during pregnancy to prevent neural tube defects, and for lactation, there is no single nutritional element responsible for having a healthy baby. D) This response is too vague to be helpful. Adolescents will need specific information to improve nutrition during pregnancy.

A female patient schedules an appointment for a gynecologic examination. Which finding should indicate to the nurse that the patient is experiencing a vaginal infection? A) Foul odor from used tampons B) Scant menstrual flow at the end of the cycle C) Abdominal bloating a few days prior to menstruation D) Saturating a tampon every 2 hours during menstruation

Answer: A Explanation: A) An unusual odor when using tampons could indicate an infection. Scant menstrual flow at the end of the cycle is an expected finding. Abdominal bloating a few days prior to menstruation could be associated with premenstrual syndrome. Saturating a tampon every 2 hours during menstruation could indicate abnormal bleeding which should be evaluated. B) An unusual odor when using tampons could indicate an infection. Scant menstrual flow at the end of the cycle is an expected finding. Abdominal bloating a few days prior to menstruation could be associated with premenstrual syndrome. Saturating a tampon every 2 hours during menstruation could indicate abnormal bleeding which should be evaluated. C) An unusual odor when using tampons could indicate an infection. Scant menstrual flow at the end of the cycle is an expected finding. Abdominal bloating a few days prior to menstruation could be associated with premenstrual syndrome. Saturating a tampon every 2 hours during menstruation could indicate abnormal bleeding which should be evaluated. D) An unusual odor when using tampons could indicate an infection. Scant menstrual flow at the end of the cycle is an expected finding. Abdominal bloating a few days prior to menstruation could be associated with premenstrual syndrome. Saturating a tampon every 2 hours during menstruation could indicate abnormal bleeding which should be evaluated.

1) The postpartum unit nurse is caring for a client who delivered a term stillborn infant yesterday. The mother is heard screaming at the nutrition services worker, "This food is horrible! You people are incompetent and can't cook a simple edible meal!" The nurse understands this as which of the following? A) An indication that the mother is in the anger phase of grief. B) An abnormal response to the loss of the child. C) Reactive stress management techniques in use. Denial of the death of the child she delivered yesterday

Answer: A Explanation: A) Anger, resulting from feelings of loss, loneliness, and, perhaps, guilt, is a common reaction. Anger may be projected at significant others and/or healthcare team members. B) Anger is a common and normal response to the grief of perinatal loss. C) Reactive stress management techniques do not exist. Denial of the loss would manifest as believing that she still has a live fetus in utero

1) The nurse is providing care to a client who is the victim of sexual assault. Which assessment finding does the nurse anticipate during the disorganization phase of rape trauma syndrome? A) Anxiety B) Insomnia C) Dyspepsia D) Depression

Answer: A Explanation: A) Anxiety is an expected clinical manifestation that occurs during the disorganization phase of rape trauma syndrome. B) Insomnia is an expected clinical manifestation that often occurs during the reorganization phase of rape trauma syndrome. C) Dyspepsia is an expected clinical manifestation that often occurs during the reorganization phase of rape trauma syndrome. D) Depression is an expected clinical manifestation that often occurs during the reorganization phase of rape trauma syndrome.

1) The nurse is scheduling a client for an external cephalic version (E C V). Which finding in the client's chart requires immediate intervention? A) Previous birth by cesarean B) Frank breech ballotable C) 37 weeks, complete breech D) Failed E C V last week

Answer: A Explanation: A) Any previous uterine scar is a contraindication to E C V. Prior scarring of the uterus may increase the risk of uterine tearing or uterine rupture. B) There is no contraindication to E C V for this client. C) E C V is not attempted until 36 or 37 weeks. There is no contraindication for E C V for this client. Although this client is less likely to have a successful ECV this week if it was unsuccessful last week, there is no contraindication to attempting the procedure

1) A nursing instructor is demonstrating an assessment on a newborn using the Ballard gestational assessment tool. The nurse explains that which of the following tests should be performed after the first hour of birth, when the newborn has had time to recover from the stress of birth? A) Arm recoil B) Square window sign C) Scarf sign D) Popliteal angle

Answer: A Explanation: A) Arm recoil is slower in healthy but fatigued newborns after birth; therefore, arm recoil is best elicited after the first hour of birth, when the baby has had time to recover from the stress of birth. B) The square window sign does not have to be assessed after the first hour of birth. C) The scarf sign does not have to be assessed after the first hour of birth. D) The popliteal angle does not have to be assessed after the first hour of birth.

1. The nurse understands that the classic symptom of endometritis in a postpartum client is which of the following? A) Purulent, foul-smelling lochia B) Decreased blood pressure C) Flank pain D) Breast is hot and swollen

Answer: A Explanation: A) Assessment findings consistent with endometritis are foul-smelling lochia, fever, uterine tenderness on palpation, lower abdominal pain, tachycardia, and chills. B) Decreased blood pressure is a sign of hemorrhage. C) Flank pain is a symptom of a urinary tract infection. D) The breast being hot and swollen is a symptom of engorgement.

1) The nurse has received end-of-shift reports in the high-risk maternity unit. Which client should the nurse see first? A) The client at 26 weeks' gestation with placenta previa experiencing blood on toilet tissue after a bowel movement B) The client at 30 weeks' gestation with placenta previa whose fetal monitor strip shows late decelerations C) The client at 35 weeks' gestation with grade I abruptio placentae in labor who has a strong urge to push D) The client at 37 weeks' gestation with pregnancy-induced hypertension whose membranes ruptured spontaneously

Answer: A Explanation: A) Assessment of the woman with placenta previa must be ongoing to prevent or treat complications that are potentially lethal to the mother and fetus. Painless, bright red vaginal bleeding is the best diagnostic sign of placenta previa. This client is the highest priority. B) Late decelerations are an abnormal finding, but put only the fetus at risk. This client is not the highest priority. C) Grade I abruptio placentae creates slight vaginal bleeding. The urge to push indicates that delivery is near. This client is not the highest priority. D) Although pregnancy-induced hypertension puts a woman at risk for developing abruptio placentae, there is no indication that this client is experiencing this complication. This client is not the highest priority.

1) The pediatric clinic nurse is reviewing lab results with a 2-month-old infant's mother. The infant's hemoglobin has decreased since birth. Which statement by the mother indicates the need for additional teaching? A) "My baby isn't getting enough iron from my breast milk." B) "Babies undergo physiologic anemia of infancy." C) "This results from dilution because of the increased plasma volume." D) "Delaying the cord clamping did not cause this to happen."

Answer: A Explanation: A) At 2 months of age, infants increase their plasma volume, which results in physiologic anemia. This condition is not related to iron in the breast milk. B) This initial decline in hemoglobin creates a phenomenon known as physiologic anemia of the newborn. C) Hemoglobin values fall, mainly from a decrease in red cell mass rather than from the dilutional effect of increasing plasma volume. Early or delayed cord clamping does not affect hemoglobin levels at this age

1) A family has the following inherited disorder. What information should the nurse review with this family? A) There are various degrees of presentation B) The disorder will become milder in subsequent generations C) This is seen in consanguineous mating D) There is no male-to-male transmission

Answer: A Explanation: A) Autosomal dominant inherited disorders have varying degrees of presentation. This is an important factor when counseling families concerning autosomal dominant disorders. Although a parent may have a mild form of the disease, the child may have a more severe form. There is no evidence to support autosomal dominant inherited disorders will become milder in subsequent generations. Autosomal recessive inherited disorders are seen more when there is a history of consanguineous mating. No male-to-male transmission is associated with X-linked recessive inheritance disorders.

A premenopausal female received a recommendation by her healthcare provider to have a bone mineral density (B M D) test done. What should the nurse identify as being the reason for the test at this time in the patient's life? A) History of an eating disorder B) Takes N S A I Ds for osteoarthritis C) Lives with a spouse who smokes cigarettes D) Surgery for carpal tunnel syndrome last year

Answer: A Explanation: A) B M D testing may be indicated for premenopausal women with certain medical conditions such as eating disorders. B M D testing is not indicated when taking N S A I Ds for osteoarthritis, living with a spouse who smokes, or after having carpal tunnel surgery. B) B M D testing may be indicated for premenopausal women with certain medical conditions such as eating disorders. B M D testing is not indicated when taking N S A I Ds for osteoarthritis, living with a spouse who smokes, or after having carpal tunnel surgery. C) B M D testing may be indicated for premenopausal women with certain medical conditions such as eating disorders. B M D testing is not indicated when taking N S A I Ds for osteoarthritis, living with a spouse who smokes, or after having carpal tunnel surgery. D) B M D testing may be indicated for premenopausal women with certain medical conditions such as eating disorders. B M D testing is not indicated when taking N S A I Ds for osteoarthritis, living with a spouse who smokes, or after having carpal tunnel surgery.

1) Babies should sleep in what position every time they are put down for sleep? A) On their backs B) On their stomachs C) On their left sides D) On their right sides

Answer: A Explanation: A) Babies should sleep on their backs every time they are put down for sleep. B) Babies should sleep on their backs every time they are put down for sleep, not their stomachs. C) Babies should sleep on their backs every time they are put down for sleep; all caregivers should be informed that side positioning is unsafe. D) Babies should sleep on their backs every time they are put down for sleep; all caregivers should be informed that side positioning is unsafe.

1) A woman has a hydatidiform mole (molar pregnancy) evacuated, and is prepared for discharge. The nurse should make certain that the client understands that what is essential? A) That she not become pregnant until after the follow-up program is completed B) That she receive R h o G A M with her next pregnancy and birth C) That she has her blood pressure checked weekly for the next 30 days D) That she seek genetic counseling with her partner before the next pregnancy

Answer: A Explanation: A) Because of the risk of choriocarcinoma, the woman treated for hydatidiform mole should receive extensive follow-up therapy. Follow-up care includes a baseline chest X-ray to detect lung metastasis and a physical examination including a pelvic examination. The woman should avoid pregnancy during this time because the elevated h C G levels associated with pregnancy would cause confusion as to whether cancer had developed. B) There is no indication for the administration of R h o G A M. C) There is no indication of blood pressure problems or preeclampsia. D) This is not a genetic defect that genetic counseling could/would resolve.

1) The nurse knows that a baby born to a mother who had oligohydramnios could show signs of which of the following? A) Respiratory difficulty B) Hypertension C) Heart murmur D) Decreased temperature

Answer: A Explanation: A) Because there is less fluid available for the fetus to use during fetal breathing movements, pulmonary hypoplasia may develop. B) Hypertension has no relation to oligohydramnios. C) Heart murmur has no relation to oligohydramnios. D) Decreased temperature has no relation to oligohydramnios.

1) The nurse wishes to demonstrate to a new family their infant's individuality. Which assessment tool would be most appropriate for the nurse to use? A) Brazelton Neonatal Behavioral Assessment Scale B) New Ballard Score C) Dubowitz gestational age scale D) Ortolani maneuver

Answer: A Explanation: A) Brazelton Neonatal Behavioral Assessment Scale is an assessment tool that identifies the newborn's repertoire of behavioral responses to the environment and documents the newborn's neurologic adequacy and capabilities. B) Ballard developed the estimation of gestational age by maturity rating. C) The Dubowitz assessment tool assesses physical characteristics and neurological or neuromuscular development. D) The Ortolani maneuver is an assessment technique to evaluate for hip dislocation or hip instability.

1) The nurse is explaining to a new prenatal client that the certified nurse-midwife will perform clinical pelvimetry as a part of the pelvic exam. The nurse knows that teaching has been successful when the client makes which statement about the reason for the exam? A) "It will help us know how big a baby I can deliver vaginally." B) "Doing this exam is a part of prenatal care at this clinic." C) "My sister had both of her babies by cesarean." D) "I am pregnant with my first child."

Answer: A Explanation: A) By performing a series of assessments and measurements, the examiner assesses the pelvis vaginally to determine whether the size and shape are adequate for a vaginal birth; this procedure is called clinical pelvimetry. B) Although this is a true statement, the estimation of the pelvis size is a better indication of the client's understanding. C) Stating that the client's sister had her babies by cesarean would not indicate that the client understood the teaching. D) Clinical pelvimetry is done with the first pregnancy, but the client's stating that this is her first child does not indicate that the client understood the teaching.

1) Which of the following is a localized, easily identifiable soft area of the infant's scalp, generally resulting from a long and difficult labor or vacuum extraction? A) Caput succedaneum B) Cephalohematoma C) Molding D) Depressed fontanelles

Answer: A Explanation: A) Caput succedaneum is a localized, easily identifiable soft area of the scalp, generally resulting from a long and difficult labor or vacuum extraction. B) Cephalohematoma is a collection of blood resulting from ruptured blood vessels between the surface of a cranial bone and the periosteal membrane. C) The head may appear asymmetric in the newborn who had a vertex presentation. This asymmetry (molding) is caused by the overriding of the cranial bones during labor and birth. D) A depressed fontanelle indicates dehydration.

1) On assessment, a laboring client is noted to have cardiovascular and respiratory collapse and is unresponsive. What should the nurse suspect? A) An amniotic fluid embolus B) Placental abruption C) Placenta accreta D) Retained placenta

Answer: A Explanation: A) Cardiovascular and respiratory collapse are symptoms of an amniotic fluid embolus and cor pulmonale. B) Placental abruption does not have any of these symptoms. C) Placenta accreta does not have any of these symptoms. D) Retention of the placenta beyond 30 minutes after birth is termed retained placenta and does not have any of these symptoms.

1. A nurse needs to evaluate the progress of a woman's labor. The nurse obtains the following data: cervical dilation 6 c m; contractions mild in intensity, occurring every 5 minutes, with a duration of 30-40 seconds. Which clue in this data does not fit the pattern suggested by the rest of the clues? A) Cervical dilation 6 c m B) Mild contraction intensity C) Contraction frequency every 5 minutes D) Contraction duration 30-40 seconds

Answer: A Explanation: A) Cervical dilation of 6 c m indicates the active phase of labor. During this phase the cervix dilates from about 4 to 7 c m and contractions and pain intensify. B) Mild contractions are consistent with most of the scenario. C) This contraction frequency is consistent with most of the scenario. D) This contraction duration is consistent with most of the scenario.

1. The postpartum client who delivered 2 days ago has developed endometritis. Which entry would the nurse expect to find in this client's chart? A) "Cesarean birth after extended labor with ruptured membranes." B) "Unassisted childbirth and afterbirth." C) "External fetal monitoring used throughout labor." D) "The client has history of pregnancy-induced hypertension."

Answer: A Explanation: A) Cesarean birth is the single most significant risk factor for postpartum endometritis, along with prolonged premature rupture of the amniotic membranes (P P R O M). B) Instrument-assisted childbirth (vacuum or forceps) and manual removal of the placenta are risk factors for postpartum endometritis. C) Use of fetal scalp electrode or intrauterine pressure catheter for internal monitoring during labor are risk factors for postpartum endometritis. D) Pregnancy-induced hypertension is not a risk factor for postpartum endometritis.

1. A pregnant patient's fetus is in the left-occiput-transverse position. Which diagram should the nurse use to explain this position to the patient?

Answer: A Explanation: A) Choice 1 is the L O T or left-occiput-transverse position. Choice 2 is the L O P or left-occiput-posterior position. Choice 3 is the R O T or right-occiput-transverse position. Choice 4 is the L O A or left-occiput-anterior position.

1) A pregnant patient is completing an ultrasound to grade the integrity of the placenta. Which diagram should the nurse provide to the patient if the findings are grade 0?

Answer: A Explanation: A) Choice 1 shows the ultrasonic appearance of a grade 0 placenta. Choice 2 shows the ultrasonic appearance of a grade I placenta. Choice 3 shows the ultrasonic appearance of a grade Ⅱ placenta. Choice 4 shows the ultrasonic appearance of a grade Ⅲ placenta.

1) During a prenatal exam, a client describes several psychosomatic symptoms and has several vague complaints. What could these behaviors indicate? A) Abuse B) Mental illness C) Depression D) Nothing, they are normal

Answer: A Explanation: A) Chronic psychosomatic symptoms and vague complaints can be indicators of abuse. B) Chronic psychosomatic symptoms and vague complaints are not indicators of mental illness. C) Chronic psychosomatic symptoms and vague complaints are not indicators of depression. D) Chronic psychosomatic symptoms and vague complaints should not be discounted as normal.

1) The nurse is providing care to a pregnant client who experienced a fetal demise in utero. The client opted to wait more than 24 hours prior to scheduling the induction procedure. Which complication is the client at increased risk for based on this data? A) Anxiety B) Hemorrhage C) Hypertension D) Mania

Answer: A Explanation: A) Clients who opt to wait more than 24 hours to schedule the induction procedure after a fetal demise are at an increased risk for anxiety. B) Hemorrhage is not a complication that the client is at increased risk for when waiting more than 24 hours to schedule the induction procedure. C) Hypertension is not a complication that the client is at increased risk for when waiting more than 24 hours to schedule the induction procedure. D) Mania is not a complication that the client is at increased risk for when waiting more than 24 hours to schedule the induction procedure.

1) Which of the following is a benefit of delayed umbilical cord clamping for the preterm infant? A) Fewer infants require blood transfusion for anemia B) Fewer infants require blood transfusion for high blood pressure C) Increase in the incidence of intraventricular hemorrhage D) Increase in incidence of infant breastfeeding

Answer: A Explanation: A) Clinical trials in preterm infants found that delaying umbilical cord clamping was associated with fewer infants who required blood transfusion for anemia. B) Clinical trials in preterm infants found that delaying umbilical cord clamping was associated with fewer infants who required blood transfusion for low blood pressure. C) Delayed umbilical cord clamping shows a significant reduction in the incidence of intraventricular hemorrhage. D) Delayed umbilical cord clamping does not impact the incidence of breastfeeding.

1) Which nonspecific immune mechanism helps the ability of antibodies and phagocytic cells to clear pathogens from an organism? A) Complement B) Coagulation C) Inflammatory response D) Phagocytosis

Answer: A Explanation: A) Complement helps or "complements" the ability of antibodies and phagocytic cells to clear pathogens from an organism. B) Coagulation is the process by which blood forms a clot. C) Inflammatory response is the complex biologic response of vascular tissues to harmful stimuli such as pathogens, damaged cells or irritants. D) Phagocytosis is a major mechanism to remove pathogens and cell debris.

1) A laboring client's obstetrician has suggested amniotomy as a method for creating stronger contractions and facilitating birth. The client asks, "What are the advantages of doing this?" What should the nurse cite in response? A) Contractions elicited are similar to those of spontaneous labor. B) Amniotomy decreases the chances of a prolapsed cord. C) Amniotomy reduces the pain of labor and makes it easier to manage. D) The client will not need an episiotomy.

Answer: A Explanation: A) Contractions after amniotomy are similar to those of spontaneous labor. B) A disadvantage of amniotomy is the increased chance of prolapsed cord, especially if the fetal presenting part is not well applied against the cervix. C) A disadvantage of amniotomy is that it can increase pain and make labor more difficult to manage. D) There is no correlation between amniotomy and episiotomy.

1) A newborn delivered at term is being discharged. The parents ask the nurse how to keep their baby warm. The nurse knows additional teaching is necessary if a parent states which of the following? A) "A quick cool bath will help wake up my son for feedings." B) "I can check my son's temperature under his arm." C) "My baby should be dressed warmly, with a hat." D) "Cuddling my son will help to keep him warm."

Answer: A Explanation: A) Cool baths will chill a newborn, and should not be given. Bathing under warm water is ideal. B) The axilla is the preferred site for checking a newborn's temperature. C) Adequate clothing is needed to keep an infant warm. A snug cap placed on the infant's head reduces heat loss further. D) Encourage the mother to snuggle with the newborn under blankets to keep him or her warm.

1) The client at 37 weeks' gestation calls the clinic nurse to report that neither she nor her partner has felt fetal movement for the past 48 hours. The nurse anticipates that the physician will order which test to assess fetal viability? A) Ultrasound B) Serum progesterone levels C) Computed tomography (C T) scan Contraction stress test

Answer: A Explanation: A) Diagnosis of intrauterine fetal death (I U F D) is confirmed by visualization of the fetal heart with absence of heart action on ultrasound. B) Serum estriol levels drop during fetal demise, but progesterone levels are not measured. C) A computed tomography (C T) scan is not indicated. D) Contraction stress tests are used to determine whether a fetus will tolerate the stresses of labor.

5) The nurse is presenting a preconception counseling class. The nurse instructs the participants that niacin intake should increase during pregnancy to promote metabolic coenzyme activity. The nurse will know that teaching has been effective if a client suggests which food as a source of niacin? A) Fish B) Apples C) Broccoli D) Milk

Answer: A Explanation: A) Dietary sources of niacin include meats, fish, and whole grains. B) Apples are a source of other vitamins; however, they do not contain significant niacin. C) Broccoli is a source of other vitamins; however, it does not contain significant niacin. D) Milk is a source of other vitamins; however, it does not contain significant niacin.

1) Which is a known characteristic of domestic violence batterers? A) Feeling inferior to others B) Working in a low-paying job C) Having a low socioeconomic status D) Being diagnosed with posttraumatic stress disorder

Answer: A Explanation: A) Domestic violence batterers often have feelings of insecurity, inferiority, powerlessness, and helplessness that conflict with their assumptions of male supremacy. B) Batterers come from all occupations, not just from low-paying jobs. C) Batterers come from all socioeconomic strata. A diagnosis of posttraumatic stress disorder is not a known characteristic of domestic violence batterers

1) The nurse is working with a client who has experienced a fetal death in utero at 20 weeks. The client asks what her baby will look like when it is delivered. Which statement by the nurse is best? A) "Your baby will be covered in fine hair called lanugo." B) "Your child will have arm and leg buds, not fully formed limbs." C) "A white, cheesy substance called vernix caseosa will be on the skin." D) "The genitals of the baby will be ambiguous."

Answer: A Explanation: A) Downy fine hair called lanugo covers the body of a 20-week-old fetus. B) Limb buds have developed by 35 days post-fertilization. C) Vernix caseosa forms at about 24 weeks. D) Male and female external genitals appear similar until end of ninth week. At 16 weeks, sex determination is possible.

1) Before drying off the newborn after birth, which assessment finding should the nurse document to ensure an accurate gestational rating on the Ballard gestational assessment tool? A) Amount and area of vernix coverage B) Creases on the sole C) Size of the areola D) Body surface temperature

Answer: A Explanation: A) Drying the baby after birth will disturb the vernix and potentially alter the gestational age criterion. The nurse should document the amount and areas of vernix coverage before drying the newborn. B) Creases on the sole are not affected by drying the newborn. C) The size of the areola is not affected by drying the newborn. D) Body surface temperature is not part of the Ballard gestational assessment tool.

1) The charge nurse is looking at the charts of laboring clients. Which client is in greatest need of further intervention? A) Woman at 7 c m, fetal heart tones auscultated every 90 minutes B) Woman at 10 c m and pushing, external fetal monitor applied C) Woman with meconium-stained fluid, internal fetal scalp electrode in use D) Woman in preterm labor, external monitor in place

Answer: A Explanation: A) During active labor, the fetal heart tones should be auscultated every 30 minutes; every 90 minutes is not frequent enough. B) External monitoring can be done instead of auscultation of the fetal heart tones during labor. C) Meconium-stained amniotic fluid is not an expected finding. Internal fetal monitoring with the internal fetal scalp electrode is often utilized when meconium-stained amniotic fluid is present. D) External monitoring during preterm labor will assess both contractions and fetal status.

1) The nurse is assessing clients after delivery. For which client is early discharge at 24 hours after delivery appropriate? A) Woman and baby who have had two successful breastfeedings B) Woman who is bottle-feeding her infant and has not voided since delivery C) Twins delivered at 35 weeks, bottle-feeding D) Cesarean birth performed for fetal distress

Answer: A Explanation: A) Early discharge may be advantageous if mother and baby are doing well, help is available for the mother at home, and the family and physician/C N M agree that both clients are healthy and ready for discharge. Feeding successfully is one of the physiologic needs of the infant and both mother and infant appear to be doing well. B) Early discharge may be advantageous if mother and baby are doing well. Voiding is a physiologic need of the mother and has not yet been accomplished. C) Preterm infants are not appropriate for early discharge. D) Infants who experienced distress in labor are not appropriate for early discharge.

1) A postpartum client has inflamed hemorrhoids. Which nursing intervention would be appropriate? A) Encourage sitz baths. B) Position the client in the supine position. C) Avoid stool softeners. D) Decrease fluid intake.

Answer: A Explanation: A) Encouraging sitz baths is the correct approach because moist heat decreases inflammation and provides for comfort. B) Position the client in a side-lying position. C) Avoiding stool softeners would put the client at risk for constipation and increase the likelihood of inflammation. D) Decreasing fluid intake would put the client at risk for constipation and would be contraindicated for reducing inflammation.

1) During a postpartum home visit, which step should the nurse take to establish a caring relationship? A) Ask family members how they want to be addressed. B) Do a portion of what the nurse agrees to do for the family, to avoid overwhelming them. C) Speak directly to the father when asking questions. D) Present information to the family instead of asking questions.

Answer: A Explanation: A) Establishing a caring relationship starts with introducing yourself to the family and by calling the family members by their surnames until you have been invited to use the given or a less formal name. B) Trust will be built only if the nurse follows through and does whatever is promised. C) It is important to allow the mother to be the spokesperson. D) It is important to ask questions instead of talking at the family. Be prepared for the visit, honestly answer questions, provide information, and be truthful. If you do not know the answer to a question, tell the patient you will find the information and report back.

1) The nurse is working with male teens whose partners are pregnant. What statement by the father-to-be requires that the nurse intervene? A) He will be the only other person who will be present for the birth, although his girlfriend wants her mother to be with her. B) He was very sexually active at an earlier age and he has had more sexual partners than his girlfriend. C) The pregnancy does not seem real to him, and he is not sure what he should do to plan for the future. D) He does not want to be married.

Answer: A Explanation: A) Even if the adolescent father has been included in the healthcare of the client throughout the pregnancy, it is not unusual for her to want her mother as her primary support person during labor and birth. Overriding his girlfriend's expressed desire could be an indication that their relationship is abusive. B) This may be true, but is not a reason for the nurse to intervene. C) The adolescent who attempts to assume his responsibility as a father faces many of the same psychologic and sociologic risks as the adolescent mother. It is normal for a teen to be unsure about the future. D) Although not married, many adolescent couples are involved in meaningful relationships. This is not a reason for the nurse to intervene.

1) A 38-year-old client in her second trimester states a desire to begin an exercise program to decrease her fatigue. What is the most appropriate nursing response? A) "Fatigue should resolve in the second trimester, but walking daily might help." B) "Avoid a strenuous exercise regimen at your age. Drink coffee to combat fatigue." C) "Avoid an exercise regimen due to your pregnancy. Try to nap daily." D) "Fatigue will increase as pregnancy progresses, but running daily might help."

Answer: A Explanation: A) Even mild to moderate exercise is beneficial during pregnancy. Regular exercise-at least 30 minutes of moderate exercise daily or at least most days of the week-is preferred. B) The age of 38 is not too old to begin an exercise routine. Nurses should advise women of common sources of caffeine, including coffee, tea, colas, and chocolate and suggest they limit their caffeine intake to about 300 m g/day. C) Mild to moderate exercise is beneficial during pregnancy. Scheduling activities to allow for napping is helpful. D) Jogging or running is acceptable for women already conditioned to these activities, as long as they avoid exercising at maximum effort and overheating.

1) The nurse is teaching a prenatal class about feeding methods. A father-to-be asks the nurse which method, breast or formula, leads to the fastest infant growth and weight gain. Which response by the nurse is best? A) "In the first 3 to 4 months breastfed babies tend to gain weight faster." B) "In the first 3 to 4 months there is no difference in weight gain." C) "In the first 3 to 4 months bottle-fed babies grow faster." D) "In the first 3 to 4 months growth isn't as important as your comfort with the method."

Answer: A Explanation: A) Exclusively breastfed infants have the same or slightly higher weight gain than their formula-fed and combination-fed peers in the first 3 to 4 months. B) Once feedings are established, growth rates for breastfed and formula-fed infants vary. C) Bottle-fed babies do not gain weight faster. D) Although comfort with the feeding method is important, the question is specifically about growth and weight gain; it is not therapeutic to change the topic and not answer the question.

The nurse obtains a health history from four clients. To which client should she give priority for teaching about cervical cancer prevention? A) Age 30, treated for P I D B) Age 25, monogamous C) Age 20, pregnant D) Age 27, uses a diaphragm

Answer: A Explanation: A) Exposure to sexually transmitted infections increases the risk of abnormal cell changes and cervical cancer. B) Practicing monogamy does not increase the risk of cervical cancer. C) Pregnancy does not increase the risk of cervical cancer. D) Use of a diaphragm does not increase the risk of cervical cancer.

1) A 3-month-old baby who was born at 25 weeks has been exposed to prolonged oxygen therapy. The nurse explains to the parents that due to oxygen therapy, their infant is at a greater risk for which of the following? A) Visual impairment B) Hyperthermia C) Central cyanosis D) Sensitive gag reflex

Answer: A Explanation: A) Extremely premature newborns are particularly susceptible to injury of the delicate capillaries of the retina, causing characteristic retinal changes known as retinopathy of prematurity (R O P). Judicious use of supplemental oxygen therapy in the premature infant has become the norm. B) Hypothermia is more common in premature infants. C) Central cyanosis can be caused by decreased oxygen. D) An absent or decreased gag reflex is more common in premature infants.

1) In planning care for a new family immediately after birth, which procedure would the nurse most likely withhold for 1 hour to allow time for the family to bond with the newborn? A) Eye prophylaxis medication B) Drying the newborn C) Vital signs D) Vitamin K injection

Answer: A Explanation: A) Eye prophylaxis medication instillation may be delayed up to 1 hour after birth to allow eye contact during parent-newborn bonding. B) Drying the newborn after birth is an essential nursing intervention, and should not be withheld. C) Taking vital signs is an essential nursing intervention, and should not be withheld. D) Vitamin K usually is given within 1 hour following birth, but does not interfere with eye contact and bonding between parent and newborn.

1) The nurse is supervising care by a new graduate nurse who is working with a couple who have experienced a stillbirth. Which statement made by the new nurse indicates that further instruction is necessary? A) "I should stay out of their room as much as possible." B) "The parents might express their grief differently from each other." C) "My role is to help the family communicate and cope." D) "Hopelessness might be expressed by this family."

Answer: A Explanation: A) Families experiencing perinatal loss need support. The nurse should stay with the couple so they do not feel alone and isolated; however, cues that the couple wants to be alone should be assessed continuously. B) Partners often express grief very differently. C) Families need to communicate to cope effectively. Hopelessness is commonly experienced by families after perinatal loss

1) A client who delivered 2 hours ago tells the nurse that she is exhausted and feels guilty because her friends told her how euphoric they felt after giving birth. How should the nurse respond? A) "Everyone is different, and both responses are normal." B) "Most mothers do feel euphoria; I don't know why you don't." C) "It's good for me to know that because it might indicate a problem." D) "Let me bring your baby to the nursery so that you can rest."

Answer: A Explanation: A) Following birth, some women feel exhausted and in need of rest. Other women are euphoric and full of psychic energy, ready to retell their experience of birth repeatedly. B) The nurse should not imply that a mother's emotional response is not expected. C) Both euphoria and exhaustion are normal feelings after birth. Fatigue after birth is not indicative of a problem. D) The client might want to be with her newborn, and the nurse should not encourage unnecessary separation of mother and child.

1) A nurse is checking the postpartum orders. The doctor has prescribed bed rest for 6-12 hours. The nurse knows this is an appropriate order if the client had which type of anesthesia? A) Spinal B) Pudendal C) General D) Epidural

Answer: A Explanation: A) Following the birth, the woman may be kept flat. Although the effectiveness of the supine position to avoid headache following a spinal is controversial, the physician's orders may include lying flat for 6-12 hours. B) It is not necessary to keep the postpartum client in bed for 6-12 hours after receiving a pudendal. C) The decision to keep the postpartum client in bed after receiving a general anesthesia depends on the client. D) It is not usually necessary to keep the postpartum client in bed for 6-12 hours after receiving an epidural.

15) The prenatal clinic nurse is caring for a 15-year-old client who is at 8 weeks' gestation. The client asks the nurse why she is supposed to gain so much weight. What is the best response by the nurse? A) "Gaining 25-35 pounds is recommended for healthy fetal growth." B) "It's what your certified nurse-midwife recommended for you." C) "Inadequate weight gain delays lactation after delivery." D) "Weight gain is important to ensure that you get enough vitamins."

Answer: A Explanation: A) For an appropriate-weight woman, 25-35 pounds of weight gain is recommended for optimal fetal growth and development. B) Although this statement might be true, the client has asked a "why" question that should be directly answered. C) Inadequate weight gain can lead to decreased fetal growth and development. D) Vitamin intake is related to the types of food consumed, not to caloric intake. Because this client is 15, her diet may not be optimal from a nutritional standpoint.

1) The nurse is caring for a laboring client with thrombocytopenia. During labor, it is determined that the client requires a cesarean delivery. The nurse is preparing the client for surgery, and should instruct the client that the recommended method of anesthesia is which of the following? A) General anesthesia B) Epidural anesthesia C) Spinal anesthesia D) Regional anesthesia

Answer: A Explanation: A) General anesthesia will be recommended. Women with thrombocytopenia should avoid regional blocks. B) Women with thrombocytopenia should avoid regional blocks. C) Women with thrombocytopenia should avoid regional blocks. D) Women with thrombocytopenia should avoid regional blocks.

1) The nurse is teaching a class to the community on mind-based therapies. A class participant gives an example of a friend with leukemia who was taught by her complementary therapist to concentrate on making antibodies that will fight and kill the cancer cells in the bloodstream. How would the nurse identify this technique? A) Guided imagery B) Qigong C) Biofeedback D) Homeopathy

Answer: A Explanation: A) Guided imagery is a state of intense, focused concentration used to create compelling mental images and is useful in imagining a desired effect. B) Qigong involves the use of breathing, meditation, self-massage, and movement. C) Biofeedback is learning to control physiologic responses to stimuli or thoughts. D) Homeopathy is not a mind-body therapy, but uses the concept of like curing like.

1) The 19-year-old pregnant woman begins a job to "save money for the baby." What is the most significant developmental task the nurse understands this statement to demonstrate? A) Striving for gaining autonomy and independence B) Completed development of a sense of identity C) Attainment of a sense of achievement D) Having developed an intimate relationship

Answer: A Explanation: A) Having a job is how most teens develop financial independence and autonomy. B) Having a job can give teens a sense of identity, but it is not an indication that the development of a sense of identity is completed. C) Having a job does give teens a sense of achievement, but this is not the most significant developmental task. D) Having a job does not indicate that the teen has developed an intimate relationship.

1) The nurse obtains a health history from four clients. To which client should she give priority for teaching about cervical cancer prevention? A) Age 37, multiple partners B) Age 22, abstains from sexual intercourse C) Age 32, pregnant with twins D) Age 27, uses female condom

Answer: A Explanation: A) Having multiple partners increases the client's risk of contracting sexually transmitted infections, including possible exposure to human papilloma virus (H P V). Contracting H P V increases the risk of abnormal cervical cell changes and cervical cancer. B) Practicing abstinence does not increase risks of cervical cancer. C) Pregnancy of any type does not increase risks of cervical cancer. Use of a female condom does not increase risks of cervical cancer

1) The nurse at the prenatal clinic has four calls to return. Which call should the nurse return first? A) Client at 32 weeks, reports headache and blurred vision. B) Client at 18 weeks, reports no fetal movement in this pregnancy. C) Client at 16 weeks, reports increased urinary frequency. D) Client at 40 weeks, reports sudden gush of fluid and contractions.

Answer: A Explanation: A) Headache and blurred vision are signs of preeclampsia, which is potentially life-threatening for both mother and fetus. This client has top priority. B) Fetal movement should be felt by 19-20 weeks. The lack of fetal movement prior to 20 weeks is considered normal. This client is a lower priority. C) Increased urinary frequency is common during pregnancy as the increased size of the uterus puts pressure on the urinary bladder. D) A full-term client who is experiencing contractions and a sudden gush of fluid is in labor. Although laboring clients should be in contact with their provider for advice on when to go to the hospital, labor at full term is an expected finding. This client is a lower priority.

1) A woman has come to the emergency department with multiple bruises on her body and a small laceration over her upper lip. She says she fell down the stairs while doing housework. Which observation would most cause the nurse to suspect that the client has been a victim of battering? A) The client is hesitant to provide details about how the injuries occurred. B) The client was accompanied to the emergency department by her mother instead of her partner. C) The client has sought care quickly after the incident. D) The client does not seem to be in pain.

Answer: A Explanation: A) Hesitation to provide detailed information about the injury and how it occurred is a common sign of abuse. B) Who accompanies the client to the emergency department is not a significant sign for abuse. C) Often a woman delays seeking care when there has been abuse. D) Pain level is not indicative of abuse. The experience of pain and how it is expressed is often a cultural issue.

There have been a number of historical trends that have contributed to the existing wage gap, including which of the following? A) There was a perception that men were the sole breadwinners. B) Women who were competitive in the work environment were viewed positively. C) Women in past generations generally were not limited to certain occupations. D) Increase in societal importance of women's intellectual traits.

Answer: A Explanation: A) Historically, there was a perception that men were the sole breadwinners, thus higher salaries for men were justified to support a family. B) Historically, women who were competitive in the work environment were viewed negatively. C) Women in past generations generally were limited to certain occupations. D) There was an increase in societal importance focusing on women's appearance, with intellectual traits being viewed less favorably.

1) The nurse is caring for a pregnant woman who admits to using cocaine and ecstasy on a regular basis. The client states, "Everybody knows that alcohol is bad during pregnancy, but what's the big deal about ecstasy?" What is the nurse's best response? A) "Ecstasy can cause a high fever in you and therefore cause the baby harm." B) "Ecstasy leads to deficiencies of thiamine and folic acid, which help the baby develop." C) "Ecstasy produces babies with small heads and short bodies with brain function alterations." D) "Ecstasy produces intrauterine growth restriction and meconium aspiration."

Answer: A Explanation: A) Hyperthermia (elevated temperature) is a side effect of M D M A (ecstasy). B) Alcohol, not ecstasy, causes deficiencies of thiamine and folic acid. Folic acid helps prevent neural tube defects. C) Cocaine, not ecstasy, causes these fetal effects (small heads and short bodies with brain function alterations). D) Heroin, not ecstasy, causes these fetal effects (intrauterine growth restriction and meconium aspiration).

1) The nurse is caring for the newborn of a diabetic mother whose blood glucose level is 39 m g/d L. What should the nurse include in the plan of care for this newborn? A) Offer early feedings with formula or breast milk. B) Provide glucose water exclusively. C) Evaluate blood glucose levels at 12 hours after birth. D) Assess for hyperthermia.

Answer: A Explanation: A) I D Ms whose serum glucose falls below 40 m g/d L should have early feedings with formula or breast milk (colostrum). B) If normal glucose levels cannot be maintained with oral feeding, an intravenous (I V) infusion of glucose will be necessary. C) Blood glucose determinations should be performed by heel stick hourly during the first 4 hours after birth and at 4-hour intervals until the risk period (about 48 hours) has passed. D) Hypothermia is a potential problem for the S G A newborn due to decreased brown fat stores and minimal subcutaneous tissues.

When analyzing data collected during a sexual history, the nurse notes that a patient has limited information about contraception. What should the nurse do to address this patient's need? A) Provide the patient with the information B) Suggest that the patient talk with the nurse practitioner C) Schedule an appointment for the patient to see the midwife D) Discuss the implications if contraception is not used correctly

Answer: A Explanation: A) If a deficiency in knowledge is identified the nurse can identify a plan of care to address this deficiency and provide the teaching. The patient does not need to talk with a nurse practitioner or a midwife to discuss contraception. The implications of inappropriately used contraception can be included when discussing the individual types with the patient. B) If a deficiency in knowledge is identified the nurse can identify a plan of care to address this deficiency and provide the teaching. The patient does not need to talk with a nurse practitioner or a midwife to discuss contraception. The implications of inappropriately used contraception can be included when discussing the individual types with the patient. C) If a deficiency in knowledge is identified the nurse can identify a plan of care to address this deficiency and provide the teaching. The patient does not need to talk with a nurse practitioner or a midwife to discuss contraception. The implications of inappropriately used contraception can be included when discussing the individual types with the patient. If a deficiency in knowledge is identified the nurse can identify a plan of care to address this deficiency and provide the teaching. The patient does not need to talk with a nurse practitioner or a midwife to discuss contraception. The implications of inappropriately used contraception can be included when discussing the individual types with the patient

1) A woman's history and appearance suggest drug abuse. What is the nurse's best approach? A) Ask the woman directly, "Do you use any street drugs?" B) Ask the woman whether she would like to talk to a counselor. C) Ask some questions about over-the-counter medications and avoid mention of illicit drugs. D) Explain how harmful drugs can be for her baby.

Answer: A Explanation: A) If drug abuse is suspected, the nurse should ask direct questions and be matter-of-fact and nonjudgmental to elicit honest responses. B) Advising the client to seek counseling would not occur until the nurse verifies that the woman uses street drugs. C) If abuse is suspected, the nurse should ask direct questions, beginning with less threatening questions about the use of tobacco, caffeine, and over-the-counter medications, but not avoid the mention of illicit drugs. D) When talking to clients in a therapeutic manner, it is important not to be threatening or judgmental; an example of judgmental behavior would be stating that the drugs will harm the baby.

1) An analgesic medication has been administered intramuscularly to a client in labor. How would the nurse evaluate if the medication was effective? A) The client dozes between contractions. B) The client is moaning during contractions. C) The contractions decrease in intensity. D) The contractions decrease in frequency.

Answer: A Explanation: A) If the client dozes between contractions, the analgesic is effective. Analgesics decrease discomfort and increase relaxation. B) Analgesics decrease the discomfort of contractions. C) Contractions will not decrease in intensity. D) Contractions will not decrease in frequency.

1. The postpartum multipara is breastfeeding her new baby. The client states that she developed mastitis with her first child, and asks whether there is something she can do to prevent mastitis this time. What would the best response of the nurse be? A) "Massage your breasts on a daily basis, and if you find a hardened area, massage it towards the nipple." B) "Most first-time moms experience mastitis. It is really quite unusual for a woman having her second baby to get it again." C) "Apply cabbage leaves to any areas that feel thickened or firm to relieve the swelling." D) "Take your temperature once a day. This will help you to pick up the infection early, before it becomes severe."

Answer: A Explanation: A) If the mother finds that one area of her breast feels distended or lumpy, she can massage the lumpy area toward the nipple as the infant nurses. B) This statement is not accurate. Most first-time moms do not experience mastitis. C) Cabbage leaves are applied to suppress lactation, not prevent mastitis. The onset of mastitis is quite rapid, and taking the temperature daily is not likely to be helpful for catching early onset of the infection

1) The client has been a victim of a violent, sadistic rape. She is crying and asks the nurse, "Why would someone do something like that?" The nurse should explain that which of the following is the primary purpose of sadistic rape? A) Take pleasure from the victim's struggle and pain B) Express feelings of rage C) Feel a sense of power or mastery D) Relieve intolerable anxiety

Answer: A Explanation: A) In sadistic rape, the assailant has an antisocial personality and delights in torture and mutilation. In this type of rape, the victim and assailant are generally strangers, and the assault is planned. Sadistic rapes cause the most injuries, including homicide. B) In anger rape, the sexual assault is used to express feelings of rage and to retaliate for what the attacker perceives as wrongs against him. These perceived wrongs most often have nothing to do with the rape victim. Considerable brutality and degradation can characterize this type of rape. Attacks on older women often are a form of anger rape. C) In power rape, the purpose of the assault is control or mastery. The assailant uses sexual intercourse to place a woman in a powerless position so that he can feel dominant, potent, and strong. He often believes that his victim enjoys the assault, and he exerts only the amount of force necessary to subdue his victim. Often power rape is a planned stranger attack, but most acquaintance rapes are also power rapes. The vast majority of all rapes are motivated by this need for power and control. D) Anxiety is not associated with a type of rape.

A patient and her partner are being treated for trichomoniasis. What should the nurse emphasize when teaching the couple about this infection? A) Avoid intercourse until symptom free B) Ensure a repeat test is completed in 3 months C) Limit alcohol intake while taking metronidazole D) Have annual screening for recurrence of the infection

Answer: A Explanation: A) In trichomoniasis, partners should avoid intercourse until both are cured. Retesting for gonorrhea is recommended 3 months following treatment secondary to increasing prevalence and the potential for P I D. Alcohol should be avoided, not limited, for 48 hours after taking metronidazole because of an effect similar to that of alcohol and disulfiram (Antabuse). Annual screening for sexually active individuals up to age 25 is recommended for chlamydia.

1) The client gave birth to a 7 pound, 14 ounce female 30 minutes ago. The placenta has not yet delivered. Manual removal of the placenta is planned. What should the nurse prepare to do? A) Start an I V of lactated Ringer's. B) Apply anti-embolism stockings. C) Bottle-feed the infant. Send the placenta to pathology

Answer: A Explanation: A) In women who do not have an epidural in place, intravenous sedation may be required because of the discomfort caused by the procedure. An I V is necessary. B) Anti-embolism stockings are used after major surgery that leads to immobility, thus increasing the risk of embolism. However, anti-embolism stockings are not needed for this client. C) The client's partner or family member, or a nursery nurse, can feed the infant. Preparation for manual removal of the placenta is a higher priority at this time. D) The placenta might be sent to pathology after it is removed, but preparing the client for manual removal of the placenta now is a higher priority.

1) A 28-year-old woman has been an insulin-dependent diabetic for 10 years. At 36 weeks' gestation, she has an amniocentesis. A lecithin/sphingomyelin (L/S) ratio test is performed on the sample of her amniotic fluid. Because she is a diabetic, what would an obtained 2:1 ratio indicate for the fetus? A) The fetus may or may not have immature lungs. B) The amniotic fluid is contaminated. C) The fetus has a neural tube defect. D) There is blood in the amniotic fluid.

Answer: A Explanation: A) Infants of diabetic mothers (I D Ms) have a high incidence of false-positive results (i.e., the L/S ratio is thought to indicate lung maturity, but after birth the baby develops R D S). B) Meconium contaminates the amniotic fluid and does not indicate fetal lung maturity. C) Neural tube defects are screened with the maternal serum alpha-fetoprotein test, and diagnosed with ultrasound. D) L B Cs testing, not a lecithin/sphingomyelin (L/S) ratio test, tests diabetic women and can be performed when there is blood in the amniotic fluid.

1) Infants of women with preeclampsia during pregnancy tend to be small for gestational age (SGA) because of which condition? A) Intrauterine growth restriction B) Oliguria C) Proteinuria D) Hypertension

Answer: A Explanation: A) Infants of women with preeclampsia during pregnancy tend to be small for gestational age (SGA) because of intrauterine growth restriction. The cause is related specifically to maternal vasospasm and hypovolemia, which result in fetal hypoxia and malnutrition. B) Oliguria is a sign of preeclampsia, but does not cause infants to be small for gestational age (SGA). C) Proteinuria is a sign of preeclampsia, but does not cause infants to be small for gestational age (SGA). D) Hypertension is a sign of preeclampsia, but does not cause infants to be small for gestational age (SGA).

1) The nurse is performing an assessment on an infant whose mother states that she feeds the infant in a supine position by propping the bottle. Based on this information, what would the nurse include in the assessment? A) Otoscopic exam of the eardrum B) Bowel sounds C) Vital signs D) Skin assessment

Answer: A Explanation: A) Infants who bottle feed in a supine position have an increased risk of otitis media and dental caries in the older infant. B) Bowel sounds are not affected by the position of the feeding. C) Vital signs are not affected by the position of the feeding. D) The skin is not affected by the position of the feeding.

1) The nurse is caring for a client with fetal heart rate monitoring, and the fetus is discovered to have tachycardia. Which complication should the nurse anticipate in the fetus? A) Infection B) Umbilical cord compression C) Vagus nerve stimulation D) Hypoxemia

Answer: A Explanation: A) Infection is one of the most common causes of fetal tachycardia. B) Umbilical cord compression may result in bradycardia, not tachycardia. C) Vagus nerve stimulation may result in bradycardia, not tachycardia. D) Hypoxemia may result in bradycardia, not tachycardia.

1) The home care nurse is visiting a newborn-and-mother couplet. Which nursing action has the highest priority? A) Establish rapport with the family members. B) Review the hospital medical records. C) Determine the newborn's sleeping arrangements. D) Examine the umbilical cord stump.

Answer: A Explanation: A) It is critical to establish rapport with family members prior to beginning any assessments. The nurse can rely on the same characteristics of a caring relationship that have been integral to hospital-based practice-regard for patients, genuineness, empathy, and establishment of trust and rapport. B) Although this is important, record review should be done prior to arriving at the home, so that the nurse is prepared. C) Determining the newborn's sleeping arrangements is less important than establishing rapport with the family members. D) Examining the umbilical cord stump is less important than establishing rapport with the family members.

1) Which assessment findings by the nurse would require obtaining a blood glucose level on the newborn? A) Jitteriness B) Sucking on fingers C) Lusty cry D) Axillary temperature of 98°F

Answer: A Explanation: A) Jitteriness of the newborn is associated with hypoglycemia. Aggressive treatment is recommended after a single low blood glucose value if the infant shows this symptom. B) Sucking on the fingers is a normal finding. C) A lusty cry is a normal finding. An axillary temperature of 98°F is a normal finding.

1) The nurse is admitting a Hispanic woman scheduled for a cholecystectomy. The nurse uses a cultural assessment tool during the admission. Which question would be most important for the nurse to ask? A) "What other treatments have you used for your abdominal pain?" B) "In what country were you were born?" C) "When you talk to family members, how close do you stand?" D) "How would you describe your role within your family?"

Answer: A Explanation: A) Knowing what other treatments the client has used for pain is most important because some traditional or folk remedies include the use of herbs, which can have medication interactions. B) Although information about the country of birth is helpful, it is not a physiological issue. Asking other questions is a higher priority. C) Although understanding the client's perception of appropriate personal space is helpful, it is not a physiological issue. Asking other questions is a higher priority. D) Although understanding the client's family roles is helpful, it is not a physiological issue. Asking other questions is a higher priority.

1) The nurse is responding to phone calls. Whose call should the nurse return first? A) A client at 37 weeks' gestation reports no fetal movement for 24 hours. B) A client at 29 weeks' gestation reports increased fetal movement. C) A client at 32 weeks' gestation reports decreased fetal movement X 2 days. D) A client at 35 weeks' gestation reports decreased fetal movement X 4 hours.

Answer: A Explanation: A) Lack of fetal movement can be an indication of nonreassuring fetal status or even fetal death. This client is the highest priority. B) Increased fetal movement is not indicative of a problem. C) Although decreased fetal movement can indicate intrauterine growth restriction or fetal hypoxia, this client is not the highest priority. D) Although decreased fetal movement can indicate intrauterine growth restriction or fetal hypoxia, 4 hours is a very short period of time to assess decreased fetal movement.

25) The nurse is preparing a prenatal class about infant feeding methods. The maternal nutritional requirements for breastfeeding and formula-feeding will be discussed. What statement should the nurse include? A) "Breastfeeding requires a continued high intake of protein and calcium." B) "Formula-feeding mothers should protect their health with a lot of calcium." C) "Producing breast milk requires calories, but any source of food is fine." D) "Formula-feeding mothers need a high protein intake to avoid fatigue."

Answer: A Explanation: A) Lactation requires calories, along with increased protein and calcium intake. B) Formula-feeding mothers do not need additional nutrients. C) Although any food source would provide the additional calories, an adequate protein intake is essential while breastfeeding because protein is an important component of breast milk and calcium is an important nutrient in milk production, and increases over non-pregnancy needs are expected. D) Formula-feeding moms do not need additional nutrients.

A premenopausal patient is experiencing vaginal dryness. What pharmacological intervention should the nurse suggest for this patient's symptom? A) Local low-dose vaginal estrogen B) Testosterone replacement therapy C) Menopausal hormone therapy with testosterone D) Menopausal hormone therapy with estrogen alone

Answer: A Explanation: A) Local low-dose vaginal estrogen is generally recommended to treat vaginal dryness or dyspareunia. Women are not provided with testosterone alone replacement therapy. Menopausal hormone therapy with testosterone helps to improve libido. Hormone therapy containing estrogen only is given to women who have undergone a hysterectomy. B) Local low-dose vaginal estrogen is generally recommended to treat vaginal dryness or dyspareunia. Women are not provided with testosterone alone replacement therapy. Menopausal hormone therapy with testosterone helps to improve libido. Hormone therapy containing estrogen only is given to women who have undergone a hysterectomy. C) Local low-dose vaginal estrogen is generally recommended to treat vaginal dryness or dyspareunia. Women are not provided with testosterone alone replacement therapy. Menopausal hormone therapy with testosterone helps to improve libido. Hormone therapy containing estrogen only is given to women who have undergone a hysterectomy. D) Local low-dose vaginal estrogen is generally recommended to treat vaginal dryness or dyspareunia. Women are not provided with testosterone alone replacement therapy. Menopausal hormone therapy with testosterone helps to improve libido. Hormone therapy containing estrogen only is given to women who have undergone a hysterectomy.

1) The nurse is evaluating the effectiveness of phototherapy on a newborn. Which evaluation indicates a therapeutic response to phototherapy? A) The newborn maintains a normal temperature B) An increase of serum bilirubin levels C) Weight loss D) Skin blanching yellow

Answer: A Explanation: A) Maintenance of temperature is an important aspect of phototherapy because the newborn is naked except for a diaper during phototherapy. The isolette helps the infant maintain his or her temperature while undressed. B) Phototherapy is a primary intervention that is used for the prevention of hyperbilirubinemia, to halt bilirubin levels from climbing dangerously high. C) Weight loss is a sign of developing dehydration in the newborn. The newborn should be weighed daily. Yellowing in the skin should disappear with effective phototherapy

After a pelvic examination, a patient is scheduled for tests to diagnose pelvic inflammatory disease. Which finding from the physical examination suggested to the nurse practitioner that further testing is required? A) Cervical tenderness B) Greenish vaginal discharge C) Open sores along the vagina D) Condylomata acuminata on the vulva

Answer: A Explanation: A) Manifestations of pelvic inflammatory disease include cervical tenderness or the chandelier sign. Greenish vaginal discharge is associated with gonorrhea. Open sores along the vagina might be associated with genital herpes. Condylomata acuminata on the vulva are genital warts.

1) The primiparous client has told the nurse that she is afraid she will develop hemorrhoids during pregnancy because her mother did. Which statement would be best for the nurse to make? A) "It is not unusual for women to develop hemorrhoids during pregnancy." B) "Most women don't have any problem until after they've delivered." C) "If your mother had hemorrhoids, you will get them, too." D) "If you get hemorrhoids, you probably will need surgery to get rid of them."

Answer: A Explanation: A) Many pregnant women will develop hemorrhoids. Hemorrhoids are varicosities of the veins in the lower end of the rectum and anus. During pregnancy, the gravid uterus presses on the veins and interferes with venous circulation. As the pregnancy progresses, the straining that accompanies constipation can contribute to the development of hemorrhoids. B) It is not true that most women have no hemorrhoids until after the birth. Many women develop hemorrhoids during pregnancy. C) A family history does not automatically mean that a client will develop the condition. D) Hemorrhoids that occur in pregnancy or at birth usually become asymptomatic after the early postpartum period.

1) The nurse seeks to involve the adolescent father in the prenatal care of his girlfriend. What is the rationale for this nursing strategy? A) Having the father more involved with the birth B) Avoiding conflict between the adolescent father and pregnant teenager C) Including his name on the birth certificate D) Avoiding legal action by the adolescent father's family

Answer: A Explanation: A) Many young fathers genuinely want to be involved with their children and would have more contact and input if they could. B) Issues such as conflicts with the teen mother or maternal grandparents and a lack of financial resources may act as additional barriers for the young father, but is not the reason for involving the father in prenatal care for his girlfriend. C) Fathers are being included on birth certificates far more frequently today than in the past. This helps ensure the father's rights and encourages him to meet his responsibilities to his child, but this is not the reason for involving the father in prenatal care for his girlfriend. D) Avoiding legal action by the adolescent father's family is not the reason for involving the father in the prenatal care of his girlfriend.

1) The nurse educator is teaching a group of students about maternal medical conditions that contribute to stillbirth. Which student statement indicates correct understanding of the material presented? A) "Maternal hypertensive disorders often contribute to stillbirth." B) "Maternal congenital anomalies contribute to stillbirth." C) "Maternal chromosomal abnormalities contribute to stillbirth." D) "Maternal malformations contribute to stillbirth."

Answer: A Explanation: A) Maternal hypertensive disorders are a contributing factor to stillbirth. This statement indicates correct understanding by the student. B) Fetal, not maternal, congenital anomalies contribute to stillbirth. This statement indicates the need for additional education. C) Fetal, not maternal, chromosomal abnormalities contribute to stillbirth. This statement indicates the need for additional education. Fetal, not maternal, malformations contribute to stillbirth. This statement indicates the need for additional education

1) The nurse would expect a physician to prescribe which medication to a postpartum client with heavy bleeding and a boggy uterus? A) Methylergonovine maleate (Methergine) B) Rh immune globulin (R h o G A M) C) Terbutaline (Brethine) D) Docusate (Colace)

Answer: A Explanation: A) Methylergonovine maleate is the drug used for the prevention and control of postpartum hemorrhage. B) R h o G A M is a medicine given by intramuscular injection that is used to prevent the immunological condition known as R h disease (or hemolytic disease of the newborn). C) Terbutaline (Brethine) is a β2-adrenergic receptor agonist used as a "reliever" inhaler in the management of asthma. D) Docusate is a stool softener.

1) A new mother at 36 hours post-delivery has asked to be discharged to home. The nurse explains that criteria for discharge before the newborn is 48 hours old include which of the following? A) The newborn's respiratory rate is less than 60/min. B) Singleton birth at a minimum 35 weeks' gestation. C) The newborn has passed at least three spontaneous stools. D) The newborn has normal and stable vital signs for 24 hours before discharge.

Answer: A Explanation: A) Minimal criteria include a respiratory rate in the newborn less than 60/min. B) Minimal criteria include a singleton birth at 37 to 41 weeks' gestation. C) Minimal criteria include the newborn's passing at least one spontaneous stool. D) Vital signs must be normal and stable for 12 hours prior to discharge.

1) A Navajo client who is 36 weeks pregnant meets with a traditional healer as well as her physician. What does the nurse understand this to mean? A) The client is seeking spiritual direction. B) The client does not trust her physician. C) The client will not adapt well to mothering. D) The client is experiencing complications of pregnancy.

Answer: A Explanation: A) Navajo clients are aware of the mind-soul connection, and might try to follow certain practices to have a healthy pregnancy and birth. Practices could include focus on peace and positive thoughts as well as certain types of prayers and ceremonies. A traditional healer may assist them. B) Seeing a traditional healer does not indicate mistrust of the provider. C) Seeking a spiritual healer does not indicate the client's lack of parenting ability. D) Seeking a spiritual healer does not indicate any type of pathology or complications.

1) The nurse is caring for the newborn of a diabetic mother. Which of the following should be included in the nurse's plan of care for this newborn? A) Offer early feedings. B) Administer an intravenous infusion of glucose. C) Assess for hypercalcemia. D) Assess for hyperbilirubinemia immediately after birth.

Answer: A Explanation: A) Newborns of diabetic mothers may benefit from early feedings, as they are extremely valuable in maintaining normal metabolism and lowering the possibility of such complications as hypoglycemia and hyperbilirubinemia. B) If normal glucose levels cannot be maintained with oral feeding, an intravenous (I V) infusion of glucose will be necessary. C) The newborn should be assessed for hypocalcemia. D) Hyperbilirubinemia can occur 48 to 72 hours after birth.

1) The nurse has completed a community education session on growth patterns of infants. Which statement by a participant indicates that additional teaching is needed? A) "Newborns should regain their birth weight by 1 week of age." B) "Breastfed and formula-fed babies have different growth rates." C) "Formula-fed infants regain their birth weight earlier than breastfed infants." D) "Healthcare providers consider breastfeeding to be the 'gold standard' for neonatal nutrition."

Answer: A Explanation: A) Newborns should gain at least 10 g/kg/day and be back to birth weight no later than day 14 of life. B) Breastfed and formula-fed babies have different growth rates because the compositions of human milk and formula are different. C) Formula-fed infants tend to regain their birth weight earlier than breastfed infants because of consistent fluid intake in the first few days of life, whereas the breastfed infant's fluid intake depends on the mother's milk supply and breastfeeding efficiency. D) Most healthcare providers (as well as representatives of formula companies) consider breastfeeding to be the "gold standard" for neonatal nutrition, and the outcomes associated with its use are the norm to which other forms of nutrition should be compared.

1) In planning care for the fetal alcohol syndrome (F A S) newborn, which intervention would the nurse include? A) Allow extra time with feedings. B) Assign different personnel to the newborn each day. C) Place the newborn in a well-lit room. D) Monitor for hyperthermia.

Answer: A Explanation: A) Newborns with fetal alcohol syndrome have feeding problems. Because of their feeding problems, these infants require extra time and patience during feedings. B) It is important to provide consistency in the staff working with the baby and parents and to keep personnel and visitors to a minimum at any one time. C) The F A S D baby is most comfortable in a quiet, minimally stimulating environment. D) Nursing care of the F A S D newborn is aimed at avoiding heat loss.

1) The nurse is assessing a new mother 2 days after a normal vaginal delivery. The mother has chosen not to breastfeed. What would an abnormal finding be? A) Weight loss of 3 pounds B) Small amount of breast milk expressed C) Pink striae on the abdomen D) Lochia serosa

Answer: A Explanation: A) Normal weight loss postpartum is in the range of 12 to 20-plus pounds. B) At 2 days after the birth, a non-breastfeeding mother might express a small amount of milk. C) Pink, obvious striae on the abdomen are normal. D) At 2 days postpartum, lochia rubra or lochia serosa in decreasing quantities is normal.

1) The nurse is caring for a client pregnant with twins. Which statement indicates that the client needs additional information? A) "Because both of my twins are boys, I know that they are identical." B) "If my twins came from one fertilized egg that split, they are identical." C) "If I have one boy and one girl, I will know they came from two eggs." D) "It is rare for both twins to be within the same amniotic sac."

Answer: A Explanation: A) Not all same-sex twins are identical or monozygotic, because fraternal, or dizygotic, twins can be the same gender or different genders. B) Identical, or monozygotic, twins develop from a single fertilized ovum. They are of the same sex and have the same phenotype (appearance). C) The only way to have twins of different sexes is if they come from two separate fertilized ova. D) If the amnion has already developed approximately 8 to 12 days after fertilization, division results in two embryos with a common amniotic sac and a common chorion (monochorionic-monoamniotic placenta). This type occurs rarely.

Care delivered by nurse-midwives can be safe and effective and can represent a positive response to the healthcare provider shortage. Nurse-midwives tend to use less technology, which often results in which of the following? A) There is less trauma to the mother. B) More childbirth education classes are available. C) They are instrumental in providing change in the birth environment at work. D) They advocate for more home healthcare agencies.

Answer: A Explanation: A) Nurse-midwife models of care can be one way to ensure that mothers receive excellent prenatal and intrapartum care. B) It is appropriate for nurse-midwives, in conjunction with doctors and hospitals, to provide childbirth classes for expectant families. C) By working with other staff members and doctors, the nurse-midwife is able to implement changes as needed within the birthing unit. D) Clients are increasingly going home sooner, so there needs to be more follow-up in the home.

1) The mother of a premature newborn questions why a gavage feeding catheter is placed in the mouth of the newborn and not in the nose. What is the nurse's best response? A) "Most newborns are nose breathers." B) "The tube will elicit the sucking reflex." C) "A smaller catheter is preferred for feedings." D) "Most newborns are mouth breathers."

Answer: A Explanation: A) Orogastric insertion is preferable to nasogastric because most infants are obligatory nose breathers. B) The tube or gavage feeding method is used with preterm infants who lack or have a poorly coordinated suck-swallow-breathing pattern. C) A small catheter is used for a nasogastric tube to minimize airway obstruction. D) Orogastric insertion is preferable to nasogastric because most infants are obligatory nose breathers.

1) Which client in the gynecology clinic should the nurse see first? A) 32-year-old taking gonadotropins, reporting extremity edema B) 15-year-old, no menses for past 4 months C) 18-year-old seeking information on contraception methods D) 31-year-old, taking progestins, reports increasing dyspareunia

Answer: A Explanation: A) Ovarian hyperstimulation syndrome (O H S S) is a potentially life-threatening complication of ovulation induction by gonadotropin therapy, manifested by third-spacing. This client should be seen first. B) Secondary amenorrhea can be caused by pregnancy; teen pregnancy is a high risk. This client does not take priority of care. C) Unplanned pregnancy and sexually transmitted infections can be problematic in the future, but this client exhibits no signs or symptoms of a life-threatening condition at this time and does not need to be seen first. D) Although this client might have endometriosis, dyspareunia is not a life-threatening condition.

The nurse administered oxytocin 20 units at the time of placental delivery. Why was this primarily done? A) To contract the uterus and minimize bleeding B) To decrease breast milk production C) To decrease maternal blood pressure To increase maternal blood pressure

Answer: A Explanation: A) Oxytocin is given to contract the uterus and minimize bleeding. B) Oxytocin does not have an effect on breast milk production. C) Oxytocin does not have an effect on maternal blood pressure. D) Oxytocin does not have an effect on maternal blood pressure.

1) A woman has been having contractions since 4 a.m. At 8 a.m., her cervi × is dilated to 5 c m. Contractions are frequent, and mild to moderate in intensity. Cephalopelvic disproportion (C P D) has been ruled out. After giving the mother some sedation so she can rest, what would the nurse anticipate preparing for? A) Oxytocin induction of labor B) Amnioinfusion C) Increased intravenous infusion D) Cesarean section

Answer: A Explanation: A) Oxytocin is the drug of choice for labor augmentation or labor induction and may be administered as needed for hypotonic labor patterns. B) Amnioinfusion would not change the ineffective labor pattern. C) Increasing the I V infusion would not change the ineffective labor pattern. D) Because C P D has been ruled out, a cesarean section is not anticipated.

1) An infant is diagnosed with an atrial-septal defect. When teaching the parents of this infant about the disorder, which diagram should the nurse use?

Answer: A Explanation: A) Patent ductus arteriosus is a vascular connection that, during fetal life, bypasses the pulmonary vascular bed and directs blood from the pulmonary artery to the aorta. After birth, blood shunts through the ductus from the aorta to the pulmonary artery (left-to-right shunting). Coarctation of the aorta is characterized by a narrowed aortic lumen. The lesion produces an obstruction to the flow of blood through the aorta, causing an increased left ventricular pressure and workload, minimizing systemic circulation of blood. In tetralogy of Fallot, the severity of symptoms depends on the degree of pulmonary stenosis, the size of the ventricular septal defect, and the degree to which the aorta overrides the septal defect. Complete transposition of great vessels is an embryologic defect caused by a straight division of the bulbar trunk without normal spiraling. As a result, the aorta originates from the right ventricle, and the pulmonary artery from the left ventricle resulting in a parallel circulatory system. An abnormal communication between the two circulations must be present to sustain life.

1) The nurse is preparing to assess the fetus of a laboring client. Which assessment should the nurse perform first? A) Perform Leopold maneuvers to determine fetal position. B) Count the fetal heart rate between, during, and for 30 seconds following a uterine contraction (U C). C) Dry the maternal abdomen before using the Doppler. D) The diaphragm should be cooled before using the Doppler.

Answer: A Explanation: A) Performing Leopold maneuvers is the first step. B) This is how to auscultate the fetal heart rate, but it is not the first step in assessment. C) Prior to using the Doppler device, a water-based gel is applied to the skin. D) The diaphragm should be warmed before using the Doppler.

1) The home care nurse is examining a 3-day-old infant. The child's skin on the sternum is yellow when blanched with a finger. The parents ask the nurse why jaundice occurs. What is the best response from the nurse? A) "The liver of an infant is not fully mature, and doesn't conjugate the bilirubin for excretion." B) "The infant received too many red blood cells after delivery because the cord was not clamped immediately." C) "The yellow color of your baby's skin indicates that you are breastfeeding too often." D) "This is an abnormal finding related to your baby's bowels not excreting bilirubin as they should."

Answer: A Explanation: A) Physiologic jaundice is a common occurrence, and peaks at 3 to 5 days in term infants. The reduction in hepatic activity, along with a relatively large bilirubin load, decreases the liver's ability to conjugate bilirubin and increases susceptibility to jaundice. B) The conjugation of bilirubin has nothing to do with cord clamping. C) Prevention and treatment of early breastfeeding jaundice includes encouraging frequent (every 2 to 3 hours) breastfeeding. D) Direct bilirubin is excreted into the bile ducts and duodenum. The conjugated bilirubin then progresses down the intestines, where bacteria transform it into urobilinogen and stercobilinogen. Stercobilinogen is not reabsorbed, but is excreted as a yellow-brown pigment in the stools.

1) The mother of a 3-day-old infant calls the clinic and reports that her baby's skin is turning slightly yellow. What should the nurse explain to the mother? A) Physiologic jaundice is normal, and peaks at this age. B) The newborn's liver is not working as well as it should. C) The baby is yellow because the bowels are not excreting bilirubin. D) The yellow color indicates that brain damage might be occurring.

Answer: A Explanation: A) Physiologic jaundice occurs soon after birth. Bilirubin levels peak at 3 to 5 days in term infants. B) The liver of an infant is not fully mature at this point. C) The liver of an infant conjugates the bilirubin, which is then excreted through the bowels. D) Unmonitored and untreated severe hyperbilirubinemia may progress to excessive levels that are associated with bilirubin neurotoxicity. An infant with severe jaundice would have a high level of yellow skin color, but this infant is only slightly yellow.

1) The nurse knows that a contraindication to the induction of labor is which of the following? A) Placenta previa B) Isoimmunization C) Diabetes mellitus D) Premature rupture of membranes

Answer: A Explanation: A) Placenta previa is a contraindication to the induction of labor. B) Isoimmunization is an indication for induction. C) Diabetes mellitus is an indication for induction. D) Premature rupture of membranes is an indication for induction.

1) A female college student comes into the student health clinic, concerned about being pregnant from unprotected intercourse the evening before. What should the school nurse counsel this student? A) "Take Plan B One Step now." B) "Take 1 pill of Plan B now and the second pill in 5 days." C) "Take one half of Plan B One Step now and the second half in 3 days." D) "Wait 5 days and take 1 pill of Plan B followed by the second pill in 2 days."

Answer: A Explanation: A) Plan B One-Step is one pill containing 1.5 m g of levonorgestrel which should be taken as one dose within 72 hours of unprotected intercourse. Plan B comes as two tablets of 0.75 m g levonorgestrel to be taken at once and should be taken as soon after coitus as possible. The pill in Plan B One-Step does not need to be split or taken days apart. The Plan B pills should be taken immediately and not after waiting for 5 days. B) Plan B One-Step is one pill containing 1.5 m g of levonorgestrel which should be taken as one dose within 72 hours of unprotected intercourse. Plan B comes as two tablets of 0.75 m g levonorgestrel to be taken at once and should be taken as soon after coitus as possible. The pill in Plan B One-Step does not need to be split or taken days apart. The Plan B pills should be taken immediately and not after waiting for 5 days. C) Plan B One-Step is one pill containing 1.5 m g of levonorgestrel which should be taken as one dose within 72 hours of unprotected intercourse. Plan B comes as two tablets of 0.75 m g levonorgestrel to be taken at once and should be taken as soon after coitus as possible. The pill in Plan B One-Step does not need to be split or taken days apart. The Plan B pills should be taken immediately and not after waiting for 5 days. D) Plan B One-Step is one pill containing 1.5 m g of levonorgestrel which should be taken as one dose within 72 hours of unprotected intercourse. Plan B comes as two tablets of 0.75 m g levonorgestrel to be taken at once and should be taken as soon after coitus as possible. The pill in Plan B One-Step does not need to be split or taken days apart. The Plan B pills should be taken immediately and not after waiting for 5 days.

1) A breastfeeding postpartum client reports sore nipples to the nurse during a home visit. What intervention would be the highest priority? A) Infant positioning B) Use of the breast shield C) Use of breast pads D) Type of soap used

Answer: A Explanation: A) Poor latch and/or suck are the primary causes of nipple soreness and the baby's position at the breast is a critical factor in nipple soreness. Encouraging the mother to rotate positions when feeding the infant may decrease nipple soreness. Changing positions alters the focus of greatest stress and promotes more complete breast emptying. B) Using a breast shield is not a critical factor in alleviating nipple soreness. C) Using breast pads is not a critical factor in alleviating nipple soreness. D) The type of soap used is not a critical factor in alleviating nipple soreness.

1) The labor and delivery nurse is caring for a client whose labor is being induced due to fetal death in utero at 35 weeks' gestation. In planning intrapartum care for this client, which nursing diagnosis is most likely to be applied? A) Powerlessness B) Urinary Elimination, Impaired C) Coping: Family, Readiness for Enhanced D) Skin Integrity, Impaired

Answer: A Explanation: A) Powerlessness is commonly experienced by families who face fetal loss. Powerlessness is related to a sense of lack of control in the current situational crisis. B) Impaired urinary elimination is not related to fetal loss. C) Ineffective family coping is more likely to occur at this point than readiness for enhanced family coping. Impaired skin integrity is not related to fetal loss

1) The nurse is inducing the labor of a client with severe preeclampsia. As labor progresses, fetal intolerance of labor develops. The induction medication is turned off, and the client is prepared for cesarean birth. Which statement should the nurse include in her preoperative teaching? A) "Because of your preeclampsia, you are at higher risk for hypotension after an epidural anesthesia." B) "Because of your preeclampsia, you might develop hypertension after a spinal anesthesia." C) "Because of your preeclampsia, your baby might have decreased blood pressure after birth." D) "Because of your preeclampsia, your husband will not be allowed into the operating room."

Answer: A Explanation: A) Pregnancies complicated by preeclampsia are high-risk situations. The woman with mild preeclampsia usually may have the analgesia or anesthesia of choice, although the incidence of hypotension with epidural anesthesia is increased. If hypotension occurs with the epidural block, it provides further stress on an already compromised cardiovascular system. B) Hypertension is not associated with spinal anesthesia. C) Preeclampsia does not affect the baby's blood pressure. D) The presence of preeclampsia does not mean the husband cannot be present at the birth.

1) A client is admitted to the birth setting in early labor. She is 3 c m dilated, -2 station, with intact membranes, and F H R of 150 b p m. Her membranes rupture spontaneously, and the F H R drops to 90 b p m with variable decelerations. What would the nurse's initial response be? A) Perform a vaginal exam B) Notify the physician C) Place the client in a left lateral position D) Administer oxygen at 2 L per nasal cannula

Answer: A Explanation: A) Prolapsed umbilical cord can occur when the membranes rupture. The fetus is more likely to experience variable decelerations because the amniotic fluid is insufficient to keep pressure off the umbilical cord. A vaginal exam is the best way to confirm. B) A vaginal exam should be performed before the physician is notified. C) Positioning will not relieve the decreased heart rate if the cord is compromised. D) Oxygen will not relieve the decreased heart rate if the cord is compromised.

1) The nurse is completing the discharge teaching of a young first-time mother. Which statement by the mother requires immediate intervention? A) "I will put my baby to bed with his bottle so he doesn't get hungry during the night." B) "My baby will probably have a bowel movement each breastfeeding, and will wet often." C) "Nursing every 2 to 3 hours is normal, for a total of 8 to 12 feedings every day." D) "I will drink fenugreek tea from my grandmother to prevent my milk from coming in."

Answer: A Explanation: A) Putting a baby to bed with a propped bottle is a choking hazard, and should never be done. B) Breastfed infants have more frequent bowel movements than do bottle-fed infants. The infant will have 4 wet diapers, 3 to 4 bowel movements on day 4; 5 wet diapers, 3 to 4 bowel movements on day 5; and 6 to 8 wet diapers, 3 to 4 bowel movements every day thereafter during the first month of life. C) Breast milk is easier to digest than formula, therefore infants eat more frequently. Infants will arouse to feed at least every 3 hours and will stay awake until the end of each feeding. The infant will breastfeed 8 to 12 times per day. D) Herbal galactogogues can be consumed as a tea or can be taken as capsules or as a tincture added to liquid to drink. Fenugreek is probably the most well-known herbal galactogogue among lactation consultants in the United States.

1) The nurse is reviewing charts of clients who underwent cesarean births by request in the last two years. The hospital is attempting to decrease costs of maternity care. What findings contribute to increased healthcare costs in clients undergoing cesarean birth by request? A) Increased abnormal placenta implantation in subsequent pregnancies B) Decreased use of general anesthesia with greater use of epidural anesthesia C) Prolonged anemia, requiring blood transfusions every few months D) Coordination of career projects of both partners leading to increased income

Answer: A Explanation: A) Repeat cesarean births are associated with greater risks, including increased incidence of abnormal placentation in subsequent pregnancies and increased risk of mortality secondary to surgery, which would contribute to increased healthcare costs. B) Which anesthesia method is used is not a significant factor in healthcare costs of cesarean birth by request. C) Prolonged anemia is not a complication of cesarean birth by request. D) The income of the couple does not affect healthcare costs directly.

1) Upon the client's admission to the birthing unit, the nurse performs a careful assessment to determine whether the client has a history of physical dependence on narcotics. For which complication related to analgesic administration is the nurse preparing? A) Respiratory depression B) Urinary retention C) Fetal depression D) Pruritis

Answer: A Explanation: A) Respiratory depression may require the use of naloxone (Narcan) to reverse the effects of narcotic agents. If the client has a history of physical dependence on narcotics, naloxone (Narcan) may induce withdrawal symptoms, which will adversely affect the client and her baby. B) Naloxone (Narcan) is given to correct respiratory depression, sedation, and hypotension, but not for urinary retention. C) Naloxone (Narcan) is given to correct respiratory depression, sedation, and hypotension, but not for fetal depression. D) Naloxone (Narcan) is given to correct respiratory depression, sedation, and hypotension, but not for pruritis.

1) The community nurse is working with a client from Southeast Asia who has delivered her first child. Her mother has come to live with the family for several months. The nurse understands that the main role of the grandmother while visiting is to do which of the following? A) Help the new mother by allowing her to focus on resting and caring for the baby. B) Teach her son-in-law the right way to be a father because this is his first child. C) Make sure that her daughter does not become abusive towards the infant. D) Pass on cultural values and beliefs to the newborn grandchild.

Answer: A Explanation: A) Rest, seclusion, and dietary restraint practices in many traditional non-Western cultures (South Asian groups) are designed to assist the woman and her baby during postpartum vulnerable periods. B) The new father might be taught some skills either directly or indirectly through observation, but this is not the most important role of the grandmother. C) The new grandmother does not assume that her daughter will be abusive toward the infant. D) An infant is too young to be taught values and beliefs.

1) The client with blood type A, R h-negative, delivered yesterday. Her infant is blood type AB, R h-positive. Which statement indicates that teaching has been effective? A) "I need to get R h o G A M so I don't have problems with my next pregnancy." B) "Because my baby is R h-positive, I don't need R h o G A M." C) "If my baby had the same blood type I do, it might cause complications." D) "Before my next pregnancy, I will need to have a R h o G A M shot."

Answer: A Explanation: A) Rh-negative mothers who give birth to R h-positive infants should receive R h immune globulin (R h o G A M) to prevent alloimmunization. B) If the baby is R h-negative, the mother does not need R h o G A M. C) It is specifically the R h factor that causes complications; ABO grouping does not cause alloimmunization. D) The injection must be given within 72 hours after delivery to prevent alloimmunization.

1) What is the most important aspect of care for the nurse to remember when screening a woman for partner abuse? A) Ensuring privacy and confidentiality B) Conveying warmth and empathy C) Asking specific, direct questions about abuse D) Clarifying her myths about battering

Answer: A Explanation: A) Screening for women experiencing domestic violence must be done privately, with only the nurse and the client present, in a safe and quiet place. B) Warmth and empathy are helpful, but confidentiality is more important. C) General questions about possible abuse both will facilitate trust building and are more likely to obtain accurate information, but privacy to obtain this information is the first priority. D) Clarifying myths is not essential during screening.

1) A pregnant client asks the nurse, "What is this "knuckle test" that is supposed to tell whether my baby has a genetic problem?" What does the nurse correctly explain? A) "In the first trimester, the nuchal translucency measurement is used to improve the detection rate for Down syndrome and trisomy 18." B) "You will need to ask the physician for an explanation." C) "It tests for hemophilia A or B." D) "It tests for Duchenne muscular dystrophy."

Answer: A Explanation: A) Screening tests, such as nuchal translucency ultrasound are designed to gather information about the risk that the pregnancy could have chromosome abnormalities or open spina bifida. B) This is not an appropriate response. The nurse must be aware of laboratory tests performed during pregnancy to intelligently answer clients' questions. C) D N A testing tests for hemophilia. DNA testing tests for Duchenne muscular dystrophy

1) The nurse is interviewing an adolescent client. The client reports a weight loss of 50 pounds over the last 4 months, and reports running at least 5 miles per day. The client asserts that her menarche was 5 years ago. Her menses are usually every 28 days, but her last menstrual period was 4 months ago. The client denies any sexual activity. Which is the best statement for the nurse to make? A) "Your lack of menses might be related to your rapid weight loss." B) "It is common and normal for runners to stop having any menses." C) "Increase your intake of iron-rich foods to reestablish menses." D) "Adolescents rarely have regular menses, even if they used to be regular."

Answer: A Explanation: A) Secondary amenorrhea can be caused by rapid weight loss, including the development of the eating disorders anorexia and bulimia. Runners with low body fat might have irregular menses, but amenorrhea is not a normal condition. B) It is common for runners to have amenorrhea, but it is not normal. C) Iron deficiency does not impact menstrual regularity. D) Although the first year or two after menarche might be characterized by irregular menses, once menses are established and regular, a lack of menses is secondary amenorrhea.

1) Which of the following conditions would predispose a client for thrombophlebitis? A) Severe anemia B) Cesarean delivery C) Anorexia D) Hypocoagulability

Answer: A Explanation: A) Severe anemia would predispose a client for thrombophlebitis. B) A traumatic delivery would predispose a client for thrombophlebitis. C) Obesity, not anorexia, would predispose a client for thrombophlebitis. D) Hypercoagulability would predispose a client for thrombophlebitis.

1) The introduction of a new baby into the family is often the beginning of which of the following? A) Sibling rivalry B) Inconsistent childrearing C) Toilet training D) Weaning

Answer: A Explanation: A) Sibling rivalry results from children's fear of change in the security of their relationships with their parents, which comes with the birth of a sibling. B) Consistency is important in dealing with young children. They need reassurance that certain people, special things, and familiar places will continue to exist after the new baby arrives. C) Parents should know that the older, toilet-trained child may regress to wetting or soiling because he or she sees the new baby getting attention for such behavior. D) The older, weaned child may want to drink from the breast or bottle again after the new baby comes.

1) The nurse notes that a lesbian client who recently found a breast lump on self-examination has not had a mammogram for 10 years. When asked about this delay the client states that she was not made to feel comfortable during the last mammogram. What should the nurse recognize is the underlying problem that this client is describing? A) Social barrier B) Emotional barrier C) Fear of finding a health problem D) Discomfort with the examination

Answer: A Explanation: A) Since the patient is lesbian and did not feel comfortable during her last mammogram, the nurse suspects that the patient experienced discrimination and social barriers related to the client's sexual orientation. Lesbian women are at greater risk for health and social disparities such as lack of screening for female-related cancers (breast cancer and cervical cancer). There is no information to support fear of finding a health problem, or discomfort with the examination. Although the client is likely emotional over the experience, the underlying problem described is a social barrier or discrimination.

1) The parents of a newborn are receiving discharge teaching. The nurse explains that the infant should have several wet diapers per day. Which statement by the parents indicates that further education is necessary? A) "Our baby was born with kidneys that are too small." B) "A baby's kidneys don't concentrate urine well for several months." C) "Feeding our baby frequently will help the kidneys function." D) "Kidney function in an infant is very different from that in an adult."

Answer: A Explanation: A) Size of the kidneys is rarely an issue. B) The ability to concentrate urine fully is attained by 3 months of age. C) Feeding practices may affect the osmolarity of the urine but have limited effect on concentration of the urine. D) The neonate's ability to dilute urine is fully developed, but concentrating ability is limited.

1) The nurse is teaching a class on vaginal birth after cesarean (V B A C). Which statement by a participant indicates that additional information is needed? A) "Because the scar on my belly goes down from my navel, I am not a candidate for a V B A C." B) "My first baby was in a breech position, so for this pregnancy, I can try a V B A C if the baby is head-down." C) "Because my hospital is so small and in a rural area, they won't let me attempt a V B A C." D) "The rate of complications from V B A C is lower than the rate of complications from a cesarean."

Answer: A Explanation: A) Skin incision is not indicative of uterine incision. Only the uterine incision is a factor in deciding whether V B A C is advisable. The classic vertical incision was commonly done in the past and is associated with increased risk of uterine rupture in subsequent pregnancies and labor. B) Nonrecurring conditions such as any non-vertex presentation might make V B A C a viable option as long as this pregnancy is vertex. C) For a V B A C to be safely attempted, the facilities must have in-house anesthesia personnel available for emergency cesarean births if warranted. Many small and rural hospitals do not have surgical and anesthesia staff available at night or on weekends and holidays, and therefore do not allow clients to have V B A Cs. D) The incidence of uterine rupture is 0.1% to 0.7%.

1) After teaching a pregnant client about the effects of smoking on pregnancy, the nurse knows that the client needs further education when she makes which statement? A) "I am at increased risk for preeclampsia." B) "I am at increased risk for preterm birth." C) "I am at increased risk for placenta previa." D) "I am at increased risk for abruptio placentae."

Answer: A Explanation: A) Smoking is not associated with increased risk for preeclampsia. B) Smoking is associated with increased risk for preterm birth. C) Smoking is associated with increased risk for placenta previa. D) Smoking is associated with increased risk for abruptio placentae.

1) The nurse notes purplish stretch marks on the pregnant client's breasts during the physical assessment. Which term will the nurse use when documenting this finding in the medical record? A) Striae B) Colostrum C) Linea nigra D) Chadwick's sign

Answer: A Explanation: A) Striae is the term used to document the purplish stretch marks that may be noted on the breasts during pregnancy. B) Colostrum is not the term used to document the purplish stretch marks that may be noted on the breasts during pregnancy. C) Linea nigra is not the term used to document the purplish stretch marks that may be noted on the breasts during pregnancy. D) Chadwick's sign is not the term used to document the purplish stretch marks that may be noted on the breasts during pregnancy.

1) A client tells you that her mother was a twin, two of her sisters have twins, and several cousins either are twins or gave birth to twins. The client, too, is expecting twins. Because there is a genetic predisposition to twins in her family, there is a good chance that the client will have what type of twins? A) Dizygotic twins B) Monozygotic twins C) Identical twins D) Nonzygotic twins

Answer: A Explanation: A) Studies indicate that dizygotic twins tend to occur in certain families, perhaps because of genetic factors that result in elevated serum gonadotropin levels leading to double ovulation. B) Monozygotic twins, known also as identical twins, are not familial. C) Identical twins, known also as monozygotic twins, are not familial. D) Nonzygotic twins do not exist.

1) The parents of a newborn have just been told their infant has tetralogy of Fallot. The parents do not seem to understand the explanation given by the physician. What statement by the nurse is best? A) "With this defect, not enough of the blood circulates through the lungs, leading to a lack of oxygen in the baby's body." B) "The baby's aorta has a narrowing in a section near the heart that makes the left side of the heart work harder." C) "The blood vessels that attach to the ventricles of the heart are positioned on the wrong sides of the heart." D) "Your baby's heart doesn't circulate blood well because the left ventricle is smaller and thinner than normal."

Answer: A Explanation: A) Tetralogy of Fallot is a cyanotic heart defect that comprises four abnormalities: pulmonary stenosis, ventricular septal defect, overriding aorta, and right ventricle hypertrophy. The severity of symptoms depends on the degree of pulmonary stenosis, the size of the ventricular septal defect, and the degree to which the aorta overrides the septal defect. B) This describes coarctation of the aorta and is characterized by a narrowed aortic lumen. The lesion produces an obstruction to the flow of blood through the aorta, causing an increased left ventricular pressure and workload, minimizing systemic circulation of blood. C) This describes complete transposition of great vessels and is an embryologic defect caused by a straight division of the bulbar trunk without normal spiraling. As a result, the aorta originates from the right ventricle, and the pulmonary artery from the left ventricle resulting in a parallel circulatory system. An abnormal communication between the two circulations must be present to sustain life. D) This describes hypoplastic left heart syndrome which is the underdevelopment of the left side of the heart including aortic valve atresia, severe mitral valve stenosis, and small left ventricle.

The nurse is completing the health history for a client desiring the Essure method of permanent sterilization. What should the nurse specifically ask when assessing this client? A) "Are you allergic to any metals?" B) "How many children do you have?" C) "When was your last menstrual period?" D) "Is your spouse aware of the procedure?"

Answer: A Explanation: A) The Essure® method of permanent sterilization uses a nickel-titanium alloy. The client should be asked about having an allergy to nickel. The number of children, last menstrual period, and spouse's knowledge about the procedure are not essential when completing the client's health history prior to this method of sterilization. B) The Essure® method of permanent sterilization uses a nickel-titanium alloy. The client should be asked about having an allergy to nickel. The number of children, last menstrual period, and spouse's knowledge about the procedure are not essential when completing the client's health history prior to this method of sterilization. C) The Essure® method of permanent sterilization uses a nickel-titanium alloy. The client should be asked about having an allergy to nickel. The number of children, last menstrual period, and spouse's knowledge about the procedure are not essential when completing the client's health history prior to this method of sterilization. D) The Essure® method of permanent sterilization uses a nickel-titanium alloy. The client should be asked about having an allergy to nickel. The number of children, last menstrual period, and spouse's knowledge about the procedure are not essential when completing the client's health history prior to this method of sterilization.

1) he nurse discussing different pregnancy programs explains that which of the following is critical to the success of an adolescent pregnancy-prevention program? A) Including role models from the same cultural and racial backgrounds as the participants B) Focusing on the adolescent female C) Having short-term, informal programs available twice per year D) Focusing on the expectations of the adolescents' parents

Answer: A Explanation: A) The National Campaign to Prevent Teen and Unplanned Pregnancy's task forces found that the programs most effective at preventing teen pregnancy include models from the same cultural and racial backgrounds as the participants. B) The National Campaign to Prevent Teen and Unplanned Pregnancy's task forces found that the programs most effective at preventing teen pregnancy include focusing on the adolescent male. C) The National Campaign to Prevent Teen and Unplanned Pregnancy's task forces found that the programs most effective at preventing teen pregnancy are both long-term and intensive. D) The National Campaign to Prevent Teen and Unplanned Pregnancy's task forces found that the programs most effective at preventing teen pregnancy focus on adolescent males.

1) The nurse assesses the postpartum client to have moderate lochia rubra with clots. Which nursing intervention would be appropriate? A) Assess fundus and bladder status. B) Catheterize the client. C) Administer Methergine I M per order. D) Contact the physician immediately.

Answer: A Explanation: A) The amount, consistency, color, and odor of the lochia are monitored on an ongoing basis. Increased bleeding is most often related to uterine atony and responds to fundal massage, expression of any clots, and emptying the bladder. B) Catheterizing the client might be an intervention if the bladder is full and the client is unable to void, but it is not the initial intervention. C) It is not necessary to administer Methergine I M per order; the situation does not warrant this intervention. D) It is not necessary to contact the physician immediately; the situation does not warrant this intervention.

1) While performing a uterine assessment on a client in the birthing unit, the nurse notes a loss of fetal station and a change in uterine shape. The client reports constant abdominal pain, uterine tenderness, and is exhibiting signs of shock. Which condition should the nurse suspect? A) Uterine rupture B) Anaphylactoid syndrome of pregnancy C) Circumvallate placenta D) Breech presentation

Answer: A Explanation: A) The assessment findings are consistent with uterine rupture, which may also include a nonreassuring fetal heart rate, hematuria, and cessation of contractions. B) Anaphylactoid syndrome of pregnancy is characterized by shortness of breath, hypoxia, cyanosis, and cardiovascular and respiratory collapse. C) Circumvallate placenta may result in antepartum hemorrhage, prematurity, and abnormal bleeding during or following the third stage of labor. Assessment findings consistent with breech presentation include palpation of the fetal sacrum in the lower part of the maternal abdomen and fetal heart tones present above the umbilicus on auscultation

1) The nurse is making an initial assessment of the newborn. Which of the following data would be considered normal? A) Chest circumference 31.5 c m, head circumference 33.5 c m B) Chest circumference 30 c m, head circumference 29 c m C) Chest circumference 38 c m, head circumference 31.5 c m D) Chest circumference 32.5 c m, head circumference 36 c m

Answer: A Explanation: A) The average circumference of the head at birth is 32 to 37 c m. Average chest circumference ranges from 30 to 35 c m at birth. The circumference of the head is approximately 2 c m greater than the circumference of the chest at birth. Answer 1 is the only choice in which both the chest and head circumferences fall within the norm in terms of actual size and comparable size. B) The average circumference of the head at birth is 32 to 37 c m. Average chest circumference ranges from 30 to 35 c m at birth. The circumference of the head is approximately 2 c m greater than the circumference of the chest at birth. Answer 1 is the only choice in which both the chest and head circumferences fall within the norm in terms of actual size and comparable size. C) The average circumference of the head at birth is 32 to 37 c m. Average chest circumference ranges from 30 to 35 c m at birth. The circumference of the head is approximately 2 c m greater than the circumference of the chest at birth. Answer 1 is the only choice in which both the chest and head circumferences fall within the norm in terms of actual size and comparable size. D) The average circumference of the head at birth is 32 to 37 c m. Average chest circumference ranges from 30 to 35 c m at birth. The circumference of the head is approximately 2 c m greater than the circumference of the chest at birth. Answer 1 is the only choice in which both the chest and head circumferences fall within the norm in terms of actual size and comparable size.

1) A client describes breast swelling and tenderness. What piece of data would be most important for the nurse to gather initially? A) Timing of the symptoms B) Birth control method C) Method of breast self-examination D) Diet history

Answer: A Explanation: A) The breast undergoes regular cyclical changes in response to hormonal stimulation. The nurse will want to determine when the swelling and tenderness occur within the menstrual cycle. B) The birth control method can contribute to the database, but does not have priority. C) The method of B S E can contribute to the database, but does not have priority. D) Diet history can contribute to the database, but does not have priority.

A client in the women's clinic asks the nurse, "How is the cervical mucus method of contraception different from the rhythm method?" The appropriate response by the nurse is that the cervical mucus method is which of the following? A) More effective for women with irregular cycles B) Not acceptable to women of many different religions C) Harder to work with than the rhythm method D) Requires an artificial substance or device

Answer: A Explanation: A) The cervical mucus method (Billings Ovulation Method) can be used by women with irregular cycles. B) The cervical mucus method (Billings Ovulation Method) can be used by women of many religions, and is safe and free. C) The cervical mucus method (Billings Ovulation Method) is easier to implement than is the rhythm method for most women. D) The cervical mucus method (Billings Ovulation Method) does not require any artificial device.

A patient schedules an appointment to be seen in the community clinic for dysuria, urgency, frequency, blood in the urine, and low back pain. For which health problem should the nurse provide care for this patient? A) Cystitis B) Pyelonephritis C) Glomerulonephritis D) Asymptomatic bacteriuria

Answer: A Explanation: A) The classic initial symptoms of cystitis include dysuria, urgency, frequency, low back pain and hematuria. Manifestations of acute pyelonephritis include a sudden onset with chills, high temperature, costovertebral angle tenderness or flank pain, nausea, vomiting, and general malaise. Manifestations of glomerulonephritis include periorbital edema, elevated blood pressure, and urinary changes. Asymptomatic bacteriuria has no characteristic manifestations.

1. The client delivered by cesarean birth 3 days ago and is being discharged. Which statement should the nurse include in the discharge teaching? A) "If your incision becomes increasingly painful, call the doctor." B) "It is normal for the incision to ooze greenish discharge in a few days." C) "Increasing redness around the incision is a part of the healing process." D) "A fever is to be expected because you had a surgical delivery."

Answer: A Explanation: A) The client should call the doctor if the incision becomes increasingly painful. After cesarean delivery, wound infection is most often associated with concurrent endometritis. The wound is typically red, indurated, tender at the margins, and draining purulent exudate. Some women have cellulitis without actual purulent drainage. B) Green drainage is not an acceptable symptom. C) The client should call the doctor if the incision becomes increasingly painful. Cesarean wound infections are characterized by increasing redness and tenderness at the margins. D) A fever could be a symptom of infection and this client should call the doctor

1) While completing the medical and surgical history during the initial prenatal visit, the 16-year-old primigravida interrupts with "Why are you asking me all these questions? What difference does it make?" Which statement would best answer the client's questions? A) "We ask these questions to detect anything that happened in your past that might affect the pregnancy." B) "We ask these questions to see whether you can have prenatal visits less often than most clients do." C) "We ask these questions to make sure that our paperwork and records are complete and up to date." D) "We ask these questions to look for any health problems in the past that might affect your parenting."

Answer: A Explanation: A) The course of a pregnancy depends on a number of factors, including the past pregnancy history (if this is not a first pregnancy), prepregnancy health of the woman, presence of disease/illness states, family history, emotional status, and past healthcare. B) Prenatal visits follow a set schedule for normal clients without complications. C) Paperwork is a lower priority than client care. D) The psychological history of a client, not the medical or surgical history, can indicate potential problems with parenting.

1) The nurse is working with a woman who is undergoing chemotherapy for breast cancer. The client states, "First, the cancer seemed unreal. Now I feel like I can cope." What is the nurse's best response? A) "Women with breast cancer often go through several stages of adjustment." B) "Women with breast cancer cope better than their partners cope." C) "Women with breast cancer seek multiple opinions before starting treatment." D) "Women with breast cancer become angry after treatment begins."

Answer: A Explanation: A) The course of adjustment confronting the woman with cancer has been described in four phases: shock, reaction, recovery, and reorientation. The client's statement indicates shock followed by reaction. B) Partners are often the primary support person during treatment, but might also have difficulty adapting to the diagnosis. C) Some clients seek multiple opinions; some do not. D) Anger is not a stage of adjustment.

1) The school nurse is planning a presentation on pregnancy for 13- and 14-year-olds who are currently pregnant. When planning the content of this presentation, what should the nurse keep in mind about these teens? A) They are working on independence and autonomy. B) They are no longer developing a sense of achievement. C) They are confident in their own identity. D) They are in control of their impulses.

Answer: A Explanation: A) The developmental tasks of adolescence include developing an identity, gaining autonomy and independence, developing intimacy in a relationship, developing comfort with one's own sexuality, and developing a sense of achievement. Teens in early adolescence will not have achieved all of these tasks yet. B) One developmental task of adolescence is developing a sense of achievement. C) One developmental task of adolescence is developing an identity. D) Teens in early adolescence may lack impulse control, which can result in risk-taking behaviors.

1. To identify the duration of a contraction, the nurse would do which of the following? A) Start timing from the beginning of one contraction to the completion of the same contraction. B) Time between the beginning of one contraction and the beginning of the next contraction. C) Palpate for the strength of the contraction at its peak. D) Time from the beginning of the contraction to the peak of the same contraction.

Answer: A Explanation: A) The duration of each contraction is measured from the beginning of the contraction to the completion of the contraction. B) Duration is not measured this way. C) Duration is not measured this way. D) Duration is not measured this way.

1) The nurse is planning care for three newly delivered adolescents and their babies. What should the nurse keep in mind when planning their care? A) The baby's father should be encouraged to participate when the nurse is providing instruction. B) A class for all the adolescents would decrease teaching effectiveness. C) The schools that the adolescents attend will provide teaching on bathing. D) Adolescents understand the danger signals in newborns.

Answer: A Explanation: A) The father, if he is involved, should be included as much as possible. If classes are offered in the hospital during the postpartum stay, the adolescent mother and father should be strongly encouraged to attend and participate. B) If classes are offered in the hospital during the postpartum stay, the adolescent mother and father should be strongly encouraged to attend and participate. C) The nurse should never assume that basic newborn care education will be provided to a client elsewhere. D) Group classes for adolescent mothers should include infant care skills, information about growth and development, infant feeding, well-baby care, and danger signals in the ill newborn.

1) While caring for a client in labor, the nurse notes the following persistent rhythm on the fetal heart rate monitor: Which action should the nurse take first? A) Notify the provider. B) Prepare for expedient delivery. C) Reposition the client. D) Discontinue oxytocin therapy.

Answer: A Explanation: A) The fetal heart rate monitor demonstrates a persistent sinusoidal pattern. The provider should be notified immediately. B) Expeditious birth of the baby is a priority after the healthcare provider is notified. C) Repositioning the client is not the immediate action that should be taken to address a persistent sinusoidal pattern. D) Discontinuing oxytocin therapy is not the immediate action that should be taken to address a persistent sinusoidal pattern.

1) The fetal heart rate baseline is 140 beats/min. When contractions begin, the fetal heart rate drops suddenly to 120, and rapidly returns to 140 before the end of the contraction. Which nursing intervention is best? A) Assist the client to change position. B) Apply oxygen to the client at 2 liters per nasal cannula. C) Notify the operating room of the need for a cesarean birth. D) Determine the color of the leaking amniotic fluid.

Answer: A Explanation: A) The fetus is exhibiting variable decelerations, which are caused by cord compression. Sometimes late or variable decelerations are due to the supine position of the laboring woman. In this case, the decrease in uterine blood flow to the fetus may be alleviated by raising the woman's upper trunk or turning her to the side to displace pressure of the gravid uterus on the inferior vena cava. B) A nasal cannula is rarely used in labor and birth. C) There is no indication that a cesarean delivery is needed. D) There is no indication that the amniotic fluid is meconium-stained or bloody.

The nurse is telling a new client how advanced technology has permitted the physician to do which of the following? A) Treat the fetus and monitor fetal development. B) Deliver at home with a nurse-midwife and doula. C) Have the father act as the coach and cut the umbilical cord. Breastfeed a new baby on the delivery table.

Answer: A Explanation: A) The fetus is increasingly viewed as a patient separate from the mother, although treatment of the fetus necessarily involves the mother. B) A nurse-midwife and a doula are not examples of technological care. C) Fathers being present during labor and coaching their partners represents nontechnological care during childbirth. D) Breastfeeding is not an example of technology impacting care.

1) The nurse is assessing the baseline fetal heart rate for a client in labor. What action should the nurse take first? A) Measure the fetal heart rate for 10 minutes B) Round the heart rate to increments of 5 beats/minute C) Exclude periods of marked variation D) Calculate the mean (average) heart rate

Answer: A Explanation: A) The first action the nurse should take is measuring the fetal heart rate for 10 minutes. B) The nurse should round the fetal heart rate to increments of 5 beats/minute after taking a 10-minute measurement of the fetal heart rate. C) The nurse should exclude periods of marked variation after taking a 10-minute measurement of the fetal heart rate. D) The nurse should calculate the mean (average) heart rate after taking a 10-minute measurement of the fetal heart rate.

1) The prenatal clinic nurse has received four phone calls. Which client should the nurse call back first? A) Pregnant woman at 28 weeks with history of asthma who is reporting difficulty breathing and shortness of breath B) Pregnant woman at 6 weeks with a seizure disorder who is inquiring which foods are good folic acid sources for her C) Pregnant woman at 35 weeks with a positive H B s A G who is wondering what treatment her baby will receive after birth D) Pregnant woman at 11 weeks with untreated hyperthyroidism who is describing the onset of vaginal bleeding

Answer: A Explanation: A) The goal of therapy is to prevent maternal exacerbations, because even a mild exacerbation can cause severe hypoxia-related complications in the fetus. B) Women with seizure disorders should be started on folic acid supplements prior to pregnancy, and should continue throughout pregnancy. However, this client is not the highest priority. C) A client with a positive H B s A G is contagious for hepatitis B. Within 12 hours of birth, infants born to women with H B V infection should receive hepatitis B immune globulin and the first dose of the H B V vaccine series. However, this client is not the highest priority. D) Pregnant women with untreated hyperthyroidism have increased rates of abortion, intrauterine death, and stillbirth. Vaginal bleeding at 11 weeks could indicate that spontaneous abortion is taking place. However, this client is not the highest priority.

1) The nurse is preparing a program about osteoporosis for a group of community members. What should the nurse emphasize as being the greatest risk factor for the development of this disorder? A) Family history B) Caucasian race C) Sedentary lifestyle D) Low lifetime intake of calcium

Answer: A Explanation: A) The greatest influencing factor for the development of osteoporosis is a family history of osteoporosis. Although Caucasian race, sedentary lifestyle, and low lifetime intake of calcium are risk factors, the greatest factor is family history. B) The greatest influencing factor for the development of osteoporosis is a family history of osteoporosis. Although Caucasian race, sedentary lifestyle, and low lifetime intake of calcium are risk factors, the greatest factor is family history. C) The greatest influencing factor for the development of osteoporosis is a family history of osteoporosis. Although Caucasian race, sedentary lifestyle, and low lifetime intake of calcium are risk factors, the greatest factor is family history. D) The greatest influencing factor for the development of osteoporosis is a family history of osteoporosis. Although Caucasian race, sedentary lifestyle, and low lifetime intake of calcium are risk factors, the greatest factor is family history.

1) The nurse is presenting a class to women who are currently pregnant or are planning pregnancy in the near future. Which client statement indicates that additional teaching is required? A) "The older a woman is when she conceives, the safer the pregnancy is." B) "Pregnant teens can have additional nutritional needs." C) "A woman whose sisters all had hypertension will be watched carefully." D) "Pregnancy may be more difficult to achieve in my 40s."

Answer: A Explanation: A) The health risks associated with pregnancy vary by age. The risk for maternal death is significantly higher for women over age 35 and even higher for women age 40 and older. The incidence of low-birth-weight infants, preterm births, miscarriage, stillbirth, and perinatal morbidity and mortality is higher among women age 35 or older. B) The health risks associated with pregnancy vary by age. Young teens who are still growing need additional calories and protein. C) The health risks associated with pregnancy vary by age. The risk of pregnancy complications is higher in women over age 35 who have a chronic condition such as hypertension or diabetes, or who are in poor general health. D) The decreased fertility of women over age 35 may make conception more difficult.

1) The mother of a 2-day-old male has been informed that her child has sepsis. The mother is distraught and says, "I should have known that something was wrong. Why didn't I see that he was so sick?" What is the nurse's best reply? A) "Newborns have immature immune function at birth, and illness is very hard to detect." B) "Your mothering skills will improve with time. You should take the newborn class." C) "Your baby didn't get enough active acquired immunity from you during the pregnancy." D) "The immunity your baby gets in utero doesn't start to function until he is 4 to 8 weeks old."

Answer: A Explanation: A) The immune responses in neonates are usually functionally impaired when compared with adults. B) This response does not address the physiology of neonatal infection, and is not therapeutic because it is blaming. C) The pregnant woman forms antibodies in response to illness or immunization called active acquired immunity. Neonatal defense against infections in utero or after delivery is dependent on maternal immunity. D) When antibodies are transferred to the fetus in utero, passive acquired immunity results because the fetus does not produce the antibodies itself.

1) What is the major adverse side effect of epidural anesthesia? A) Maternal hypotension B) Decrease in variability of the F H R C) Vertigo D) Decreased or absent respiratory movements

Answer: A Explanation: A) The major adverse effect of epidural anesthesia is maternal hypotension caused by a spinal blockade, which lowers peripheral resistance, decreases venous return to the heart, and subsequently lessens cardiac output and lowers blood pressure. B) A decrease in variability of the F H R is a fetal side effect of benzodiazepines. C) Vertigo is a side effect of the drug Nubain. D) Meperidine has multiple fetal side effects, including decreased or absent respiratory movements.

1) What is one of the most common initial signs of nonreassuring fetal status? A) Meconium-stained amniotic fluid B) Cyanosis C) Dehydration D) Arrest of descent

Answer: A Explanation: A) The most common initial signs of nonreassuring fetal status are meconium-stained amniotic fluid and changes in the fetal heart rate (F H R). B) Cyanosis is not a common sign of nonreassuring fetal status. C) Dehydration is not a common sign of nonreassuring fetal status. D) Arrest of descent is not a common sign of nonreassuring fetal status.

1) Which of the following behaviors noted in the postpartum client would require the nurse to assess further? A) Responds hesitantly to infant cries. B) Expresses satisfaction about the sex of the baby. C) Friends and family visit the client and give advice. D) Talks to and cuddles with the infant frequently.

Answer: A Explanation: A) The mother tends to respond verbally to any sounds emitted by the newborn, such as cries, coughs, sneezes, and grunts. Responding hesitantly to infant cries suggests further assessment is needed to determine what the mother is feeling. B) Expressing satisfaction about the sex of the baby is usually a positive sign. C) Birth centers are especially geared toward the inclusion of the family in the birth process. In the hospital setting, the advent of open visiting hours and rooming-in permits siblings and grandparents to participate in the attachment process. D) Talking to and cuddling with the infant frequently is a behavior that facilitates attachment.

1) The nurse is providing care to a client in labor who admits to using heroin throughout the pregnancy. Which will the neonate be at risk for following birth? A) Seizure activity B) Congenital anomalies C) Large for gestational age D) Cardiac anomalies

Answer: A Explanation: A) The neonate who has been exposed to heroin in utero is at an increased risk for seizure activity following birth. B) The neonate who has been exposed to heroin in utero is not at an increased risk for congenital anomalies. This is expected if the neonate was exposed to lithium carbonate while in utero. C) The neonate who has been exposed to heroin in utero is not an increased risk for being large for gestational age. This is expected for the neonate who is born to a mother who is diagnosed with gestational diabetes mellitus. D) The neonate who has been exposed to heroin in utero is not at an increased risk for cardiac anomalies. This is expected for the neonate who is exposed to cocaine in utero.

1) A woman at 7 weeks' gestation is diagnosed with hyperemesis gravidarum. Which nursing diagnosis would receive priority? A) Fluid Volume: Deficient B) Cardiac Output, Decreased C) Injury, Risk for D) Nutrition, Imbalanced: Less than Body Requirements

Answer: A Explanation: A) The newly admitted client with hyperemesis gravidarum has been experiencing excessive vomiting, and is in a fluid volume-deficit state. B) Because no preexisting cardiac condition is present, the body has compensated for this fluid loss. C) The risk for injury is present due to the symptoms of fluid volume deficit; however, it is not the priority. D) The nutrition status of the client is compromised until the emesis and the fluid volume status are corrected. But it is not the first priority.

1) Which action by the nurse is appropriate when providing care to a female client who is the victim of domestic violence? A) Providing adequate time for the client to tell her story B) Reporting the incident to the police to protect the client C) Telling the spouse about the client's accusations of abuse D) Stressing to the client that the abuse could have been avoided

Answer: A Explanation: A) The nurse should allow the client adequate time to work through her story, problems, and situation at her own pace. B) The nurse would not report the incident to the police to protect the client. Reporting the abuse is associated with the risk for further abuse towards the client. C) It is not appropriate for the nurse to tell the client's spouse about the accusations of abuse. This is a breach of confidentiality. D) It is not therapeutic for the nurse to stress to the client that the abuse could have been avoided.

1) The nurse is collecting information during the health history assessment for the client profile during the initial prenatal visit. Which question is appropriate when assessing the current pregnancy? A) "What was the date of your last menstrual period?" B) "How many times have you been pregnant?" C) "What were your children's birth weights?" D) "How many living children do you have?"

Answer: A Explanation: A) The nurse would ask the client for the date of the last menstrual period when assessing the current pregnancy as part of the client profile. B) The nurse would ask the client how many times she has been pregnant when assessing past pregnancies as part of the client profile. C) The nurse would assess the birth weights of the client's children when assessing past pregnancies as part of the client profile. D) The nurse would ask the client how many living children she has when assessing past pregnancies as part of the client profile.

1) The nurse is teaching a group of new parents about their infants. The infants are all 4 weeks of age or younger. Which statement should the nurse include? A) "Your baby will respond to you the most if you look directly into his eyes and talk to him." B) "Each baby is different. Don't try to compare your infant's behavior with any other child's behavior." C) "If the sound level around your baby is high, the baby will wake up and be fussy or cry." D) "If your baby is a cuddler, it is because you rocked and talked to her during your pregnancy."

Answer: A Explanation: A) The parents' visual (en face) and auditory (soft, continuous voice) presence stimulates their infant to orient to them. B) Although each infant is unique, there are certain predictable norms to observe for when assessing for neurological normalcy or impairment. C) Some infants become overstimulated when excessive noise is present, but more habituate to the sound and sleep. Cuddling is a

1) The nurse prepares to admit to the nursery a newborn whose mother had meconium-stained amniotic fluid. The nurse knows this newborn might require which of the following? A) Initial resuscitation B) Vigorous stimulation at birth C) Phototherapy immediately D) An initial feeding of iron-enriched formula

Answer: A Explanation: A) The presence of meconium in the amniotic fluid indicates that the fetus may be suffering from asphyxia. Meconium-stained newborns or newborns who have aspirated particulate meconium often have respiratory depression at birth and require resuscitation to establish adequate respiratory effort. B) Stimulation at birth should be avoided to minimize respiratory movements. C) Phototherapy is not required immediately. D) Mild or chronic anemia in an infant may be treated adequately with iron supplements alone or with iron-fortified formulas.

1) At 32 weeks' gestation, a woman is scheduled for a second nonstress test (following one she had at 28 weeks' gestation). Which statement by the client would indicate an adequate understanding of this procedure? A) "I can't get up and walk around during the test." B) "I'll have an I V started before the test." C) "I can still smoke before the test." D) "I need to have a full bladder for this test."

Answer: A Explanation: A) The purpose of the nonstress test is to determine the results of movement on fetal heart rate. The N S T is typically performed with the woman in the semi-Fowler's position with a small pillow or blanket under the right hip to displace the uterus to the left. B) No I V is needed to administer medications. C) Cigarette smoking can adversely affect the test results, so the woman should be counseled to avoid smoking prior to testing. D) Clients usually are asked to have their bladders full only for ultrasounds.

1) A woman gave birth last week to a fetus at 18 weeks' gestation after her first pregnancy. She is in the clinic for follow-up, and notices that her chart states she has had one abortion. The client is upset over the use of this word. How can the nurse best explain this terminology to the client? A) "Abortion is the obstetric term for all pregnancies that end before 20 weeks." B) "Abortion is the word we use when someone has miscarried." C) "Abortion is how we label babies born in the second trimester." D) "Abortion is what we call all babies who are born dead."

Answer: A Explanation: A) The term abortion means a birth that occurs before 20 weeks' gestation or the birth of a fetus-newborn who weighs less than 500 g. An abortion may occur spontaneously, or it may be induced by medical or surgical means. B) This explanation is only partially correct. C) This explanation is only partially correct. D) This is not a true statement.

1) The nurse is present when a mother and her partner are told that their 35-week fetus has died. Which nursing intervention should the nurse perform first? A) Encourage open communication with the family and the healthcare team. B) Ask the family to withhold questions until the next day. C) Request that another nurse come and care for this family. Contact a local funeral home to help the family with funeral plans

Answer: A Explanation: A) The top priority for the nurse is to encourage open communication. The nurse functions as an advocate for the family in organizing interdisciplinary involvement, maintaining continuity of care, offering the opportunity for open communication, and ensuring that the family's wishes regarding their loss experience are honored. B) Requesting that the couple not ask questions does not facilitate communication. The nurse should provide ongoing opportunities for the couple to ask questions. C) Requesting that another nurse care for the family does not facilitate communication. When possible, the same nurse should provide care for the couple so that a therapeutic relationship can be established. D) Funeral plans can be made later and are usually made by the family.

1) After being in labor for several hours with no progress, a client is diagnosed with C P D (cephalopelvic disproportion), and must have a cesarean section. The client is worried that she will not be able to have any future children vaginally. After sharing this information with her care provider, the nurse would anticipate that the client would receive what type of incision? A) Transverse B) Infraumbilical midline C) Classic D) Vertical

Answer: A Explanation: A) The transverse incision is made across the lowest and narrowest part of the abdomen and is the most common lower uterine segment incision. B) The infraumbilical midline incision is a type of vertical incision, which carries an increased risk of uterine rupture with subsequent pregnancy, labor, and birth. C) The classic vertical incision, which is made into the upper uterine segment, was the method of choice for many years but is nearly never performed in modern obstetrics. It carries an increased risk of uterine rupture with subsequent pregnancy, labor, and birth. D) The classic vertical incision, which is made into the upper uterine segment, was the method of choice for many years but is nearly never performed in modern obstetrics. It carries an increased risk of uterine rupture with subsequent pregnancy, labor, and birth.

1) The nurse is observing a student nurse who is caring for a neonate undergoing intensive phototherapy. Which action by the student nurse indicates an understanding of how to provide this care? A) Urine specific gravity is assessed at each voiding. B) Eye coverings are left off to help keep the baby calm. C) Temperature is checked every 6 hours. The infant is taken out of the isolette for diaper changes

Answer: A Explanation: A) This action is correct. Specific gravity provides one measure of urine concentration. Highly concentrated urine is associated with a dehydrated state. Weight loss is also a sign of developing dehydration in the newborn. B) Eyes should be covered at all times. C) Six hours is too long. Vital signs should be monitored every 4 hours with axillary temperatures. The isolette helps the infant maintain his or her temperature while undressed. The diaper should be changed while the infant is under the lights in the isolette, as care activities should be clustered

1) The nurse is caring for several pregnant clients. Which client should the nurse anticipate is most likely to have a newborn at risk for mortality or morbidity? A) 37-year-old, with a history of multiple births and preterm deliveries who works in a chemical factory B) 23-year-old of low socioeconomic status, unmarried C) 16-year-old who began prenatal care at 30 weeks D) 28-year-old with a history of gestational diabetes

Answer: A Explanation: A) This client is at greatest risk because she has multiple risk factors: age over 35, high parity, history of preterm birth, and exposure to chemicals that might be toxic. B) The main risk factor for this client is her low socioeconomic status. C) This client has two risk factors: young age and late onset of prenatal care. D) This client's only risk factor is the history of gestational diabetes.

1. The client delivered vaginally 2 hours ago after receiving an epidural analgesia. She has a slight tingling sensation in both lower extremities, but normal movement. She sustained a second-degree perineal laceration. Her perineum is edematous and ecchymotic. What should the nurse include in the plan of care for this client? A) Assist the client to the bathroom in 2 hours to void. B) Place a Foley catheter now. C) Apply warm packs to the perineum three times a day. D) Allow the client to rest for the next 8 hours.

Answer: A Explanation: A) This client is at risk for urinary retention and bladder overdistention. Overdistention occurs postpartum when the woman is unable to empty her bladder, usually because of trauma or the effects of anesthesia. After the effects of anesthesia have worn off, if the woman cannot void, postpartum urinary retention is highly indicative of a urinary tract infection (U T I). Assisting the client to the bathroom is the most likely intervention that will prevent urinary retention. B) A Foley catheter is not indicated at this time. C) Cold packs will help decrease the perineal edema; warm packs would increase the edema. D) Waiting 8 hours to reassess the bladder is too long.

1) While caring for a client in labor, the nurse notes the following F H R pattern: Which action should the nurse perform? A) Continue to monitor the client B) Fetal scalp stimulation C) Palpate contraction strength D) Discontinue oxytocin

Answer: A Explanation: A) This is a benign finding; there is no slowing of fetal heart rate with contractions. B) Fetal scalp stimulation is not indicated based on this finding. C) Contraction strength assessment is not indicated based on this finding. D) Discontinuing oxytocin should not be performed based on this finding.

1) The mother of a 16-week-old infant is concerned because she cannot feel the posterior fontanelle on her infant. Which response by the nurse would be most appropriate? A) "It is normal for the posterior fontanelle to close by 8 to 12 weeks after birth." B) "Bring your infant to the clinic immediately." C) "This is due to overriding of the cranial bones during labor." D) "Your baby must be dehydrated."

Answer: A Explanation: A) This is a normal finding at 16 weeks. The posterior fontanelle closes within 8 to 12 weeks. B) There is no reason to bring the infant to the clinic. C) Overriding of the cranial bones is referred to as molding, and diminishes within a few days following birth. D) Fontanelles can be depressed when the infant is dehydrated.

1) The nurse is answering phone calls at the pediatric clinic. Which call should the nurse return first? A) Mother of a 2-week-old infant who doesn't make eye contact when talked to B) Father of a 1-week-old infant who sleeps through the noise of an older sibling C) Father of a 6-day-old infant who responds more to mother's voice than to father's voice D) Mother of a 3-week-old infant who has begun to suck on the fingers of the right hand

Answer: A Explanation: A) This is an abnormal finding. Orientation to the environment is determined by an ability to respond to cues given by others and by a natural ability to fix on and to follow a visual object horizontally and vertically. Inability or lack of response may indicate visual or auditory problems. B) Sleeping though noise is habituation, and is an expected behavior. C) The newborn can discriminate the individual characteristics of the human voice and is especially sensitive to sound levels within the normal conversation range. D) Self-consolatory behaviors such as sucking on fists, thumbs, or fingers are normal findings.

1) The nurse is seeing prenatal clients in the clinic. Which client is exhibiting expected findings? A) 12 weeks' gestation, with fetal heart tones heard by Doppler fetoscope B) 22 weeks' gestation, client reports no fetal movement felt yet C) 16 weeks' gestation, fundus three finger-breadths above umbilicus D) Marked edema

Answer: A Explanation: A) This is an expected finding because fetal heart tones should be heard by 12 weeks using a Doppler fetoscope. B) At 22 weeks, no fetal movement is an abnormal finding. Fetal movement should be felt by 20 weeks. C) This is an abnormal finding. The fundus should be three finger-breadths above umbilicus at 28 weeks. D) This is an abnormal finding. There may be some edema of hands and ankles in late pregnancy, but marked edema could indicate preeclampsia.

1) A client admitted to the birthing unit with placenta previa asks the nurse, "What is the cause of my condition?" Which statement should be included in the nurse's response? A) "The placenta is improperly implanted in the lower uterus." B) "The placenta has separated prematurely." C) "The placenta has grown too large." D) "The placenta has prolapsed and is being compressed."

Answer: A Explanation: A) This statement correctly describes placenta previa, when the placenta implants low in the uterus or over the cervix. B) This statement describes abuptio placentae, another placental complication. C) Placenta previa is not when the placenta has grown too large; this statement should not be used by the nurse. D) Placenta previa is not when the placenta becomes prolapsed and is being compressed; this statement should not be used by the nurse.

1) The client at 14 weeks' gestation has undergone a transvaginal ultrasound to assess cervical length. The ultrasound revealed cervical funneling. How should the nurse explain these findings to the client? A) "Your cervix has become cone-shaped and more open at the end near the baby." B) "Your cervix is lengthened, and you will deliver your baby prematurely." C) "Your cervix is short, and has become wider at the end that extends into the vagina." D) "Your cervix was beginning to open but now is starting to close up again."

Answer: A Explanation: A) Transvaginal ultrasound can most accurately identify shortened cervical length and cervical funneling, which is a cone-shaped indentation in the cervical os indicating cervical insufficiency or risk of preterm labor. B) Transvaginal ultrasound can most accurately identify shortened cervical length and cervical funneling, not lengthening of the cervix. C) Transvaginal ultrasound can most accurately identify shortened cervical length and cervical funneling, which is a cone-shaped indentation in the cervical os indicating cervical insufficiency or risk of preterm labor. D) Cervical change in pregnancy is progressive, and the cervix does not spontaneously constrict or close again until after delivery.

1) The nurse is caring for a 2-hour-old newborn whose mother is diabetic. The nurse assesses that the newborn is experiencing tremors. Which nursing action has the highest priority? A) Obtain a blood calcium level. B) Take the newborn's temperature. C) Obtain a bilirubin level. D) Place a pulse oximeter on the newborn.

Answer: A Explanation: A) Tremors are a sign of hypocalcemia. Diabetic mothers tend to have decreased serum magnesium levels at term. This could cause secondary hypoparathyroidism in the infant. B) Body temperature might be necessary to monitor, but obtaining a blood calcium level takes priority for this newborn. C) Bilirubin level might be necessary to monitor, but obtaining a blood calcium level takes priority for this newborn. D) Oxygen saturation might be necessary to monitor, but obtaining a blood calcium level takes priority for this newborn.

6) By inquiring about the expectations and plans that a laboring woman and her partner have for the labor and birth, the nurse is primarily doing which of the following? A) Recognizing the client as an active participant in her own care. B) Attempting to correct any misinformation the client might have received. C) Acting as an advocate for the client. D) Establishing rapport with the client.

Answer: A Explanation: A) Understanding the couple's expectations and plans helps the nurse provide optimal nursing care and facilitate the best possible birth experience. B) Any misinformation the family has can be corrected, but that is not the primary focus. C) The nurse might use the information about plans and expectations to act as an advocate for the client as the labor progresses, but this is not the primary rationale for inquiring about them. D) Rapport and a therapeutic relationship are important for all nurse-client interactions, but are not best addressed by asking about plans and expectations for the birth.

The nurse manager is examining the descriptive statistics of increasing teen pregnancy rates in the community. Which inferential statistical research question would the nurse manager find most useful in investigating the reasons for increased frequency of teen pregnancy? A) What providers do pregnant teens see for prenatal care? B) What are the ages of the parents of pregnant teens in the community? C) Do pregnant teens drink caffeinated beverages? D) What do pregnant teens do for recreation?

Answer: A Explanation: A) Understanding which providers pregnant teens are most likely to seek out for prenatal care can lead to further investigation on why prenatal care with that provider is more acceptable to teens, which in turn can lead to greater understanding of the issue of teen pregnancy. B) A question about the age of parents of pregnant teens might prove useful in seeking causes of teen pregnancy, but it is not the most useful question in understanding the increased frequency of teen pregnancy. C) Whether pregnant teens drink caffeinated beverages gives no further insight into the issues of teen pregnancy. D) Understanding the recreational activities of pregnant teens would not lead to an understanding of the issues surrounding increasing teen pregnancy rates.

1. The client has experienced a hemorrhage at 6 hours postpartum. After controlling the hemorrhage, the client's partner asks what would cause a hemorrhage. How should the nurse respond? A) "Sometimes the uterus relaxes and excessive bleeding occurs." B) "The blood collected in the vagina and poured out when your partner stood up." C) "Bottle-feeding prevents the uterus from getting enough stimulation to contract." D) "The placenta had embedded in the uterine tissue abnormally."

Answer: A Explanation: A) Uterine atony (relaxation of the uterus) is the leading cause of early postpartum hemorrhage, accounting for over 50% of postpartum hemorrhage cases. B) Although blood can pool in the vagina and thus pour out when the client stands, this does not constitute a hemorrhage. C) Although breastfeeding stimulates the release of oxytocin, which causes the uterus to contract, bottle-feeding does not cause hemorrhage. D) Had the placenta embedded abnormally (as in placenta accreta), the hemorrhage would have occurred immediately after the placenta delivered.

1) The client tells the nurse that she has come to the hospital so that her baby's position can be changed. The nurse would begin to organize the supplies needed to perform which procedure? A) A version B) An amniotomy C) Leopold maneuvers D) A ballottement

Answer: A Explanation: A) Version, or turning the fetus, is a procedure used to change the fetal presentation by abdominal or intrauterine manipulation. B) Amniotomy is the artificial rupture of membranes. C) Leopold maneuvers are a series of palpations performed to determine fetal position. D) Ballottement occurs when the fetus floats away and then returns to touch an examiner's hand during a vaginal exam.

1) The nurse is providing care to a female client who is the victim of domestic violence. Which referral by the nurse is most appropriate? A) Group therapy B) Physical therapy C) Nutrition therapy D) Occupational therapy

Answer: A Explanation: A) Victims of domestic violence require counseling and advocacy from the nurse. The most appropriate referral for this client is group therapy. B) Physical therapy is not an appropriate referral for this client. C) Nutrition therapy is not an appropriate referral for this client. D) Occupational therapy is not an appropriate referral for this client.

1) The student nurse attempts to take a newborn's vital signs, but the newborn is crying. What nursing action would be appropriate? A) Place a gloved finger in the newborn's mouth. B) Take the vital signs. C) Wait until the newborn stops crying. D) Place a hot water bottle in the isolette.

Answer: A Explanation: A) Vital sign assessments are most accurate if the newborn is at rest, so measure pulse and respirations first if the baby is quiet. To soothe a crying baby, the nurse should place a moistened, unpowdered, gloved finger in the baby's mouth, and then complete the assessment while the baby suckles. B) Crying increases heart rate and respiratory rate, so vitals should not be taken when the newborn is crying. C) Assessment of vitals needs to be done at regularly timed intervals, so waiting until the newborn stops crying might cause too long a delay. D) A hot water bottle should not be placed next to the newborn because of the risk for burns.

1) After telling a mother that her 13-year-old daughter is pregnant, the nurse would expect the mother to respond with which statement? A) "We had such high hopes for you." B) "But you have always been a happy child." C) "I've always liked that boy." D) "This is just one of those things that happen."

Answer: A Explanation: A) When an adolescent pregnancy is first revealed to the teen's mother, the result is often anger, shame, or sorrow. The degree of negative response will be determined by the age of the teen, the family expectations for the teen, and the presence or absence of other teen pregnancies in the family or support network. B) This response is not necessarily typical of parental responses to their young teen daughter's pregnancy. C) When an adolescent pregnancy is first revealed to the teen's mother, the result is often anger and shame and not expressions of fondness for the father. D) This attitude of indifference is not typical of the response of most parents of teen daughters who are pregnant.

1) A client is concerned because she has been told that her blood type and her baby's are incompatible. What is the nurse's best response? A) "This is called ABO incompatibility. It is somewhat common but rarely causes significant hemolysis, a breakdown of red blood cells." B) "This is a serious condition, and additional blood studies are currently in process to determine whether you need a medication to prevent it from occurring with a future pregnancy." C) "This is a condition caused by a blood incompatibility between you and your husband, but does not affect the baby." D) "This type of condition is very common, and the baby can receive a medication to prevent jaundice from occurring."

Answer: A Explanation: A) When blood types, not R h, are incompatible, it is called ABO incompatibility. The incompatibility occurs as a result of the maternal antibodies present in her serum and interaction between the antigen sites on the fetal RBCs. B) Although this can be serious, additional blood studies are not typically done. There is no medication that can be given to the mother to prevent this from occurring. C) The incompatibility is not between the mother's and the father's blood, but between the mother's and the infant's blood. D) ABO incompatibility is common, but there is no medication to give the baby that will prevent jaundice.

15) Two hours after delivery, a client's fundus is boggy and has risen to above the umbilicus. What is the first action the nurse would take? A) Massage the fundus until firm B) Express retained clots C) Increase the intravenous solution D) Call the physician

Answer: A Explanation: A) When the uterus becomes boggy, pooling of blood occurs within it, resulting in the formation of clots. Anything left in the uterus prevents it from contracting effectively. Thus if it becomes boggy or appears to rise in the abdomen, the fundus should be massaged until firm. B) Expressing retained clots is not the nurse's first action. C) Increasing the intravenous solution is not a priority in this case. D) The physician does not need to be notified unless either the uterus does not respond to massage or the bleeding does not decrease.

1. The client has asked the nurse why her cervix has only changed from 1 to 2 c m in 3 hours of contractions occurring every 5 minutes. What is the nurse's best response to the client? A) "Your cervix has also effaced, or thinned out, and that change in the cervix is also labor progress." B) "When your perineal body thins out, your cervix will begin to dilate much faster than it is now." C) "What did you expect? You've only had contractions for a few hours. Labor takes time." D) "The hormones that cause labor to begin are just getting to be at levels that will change your cervix."

Answer: A Explanation: A) With each contraction, the muscles of the upper uterine segment shorten and exert a longitudinal traction on the cervix, causing effacement. Effacement is the taking up (or drawing up) of the internal os and the cervical canal into the uterine side walls. B) As the fetal head descends to the pelvic floor, the pressure of the presenting part causes the perineal structure, which was once 5 c m in thickness, to change to a structure less than 1 c m thick. The perineal body's thinning primarily occurs during later stages of labor; it is not expected now. C) This reply is not therapeutic. The nurse must always be therapeutic in all communication. D) The hormones that cause labor contractions do not directly cause cervical change; the contractions cause the cervix to change.

1) The pregnant client employed at a factory asks the nurse whether exposure to chemicals can cause harm to her fetus. The nurse should advise the client that exposure to which substance can lead to neurological damage? A) Lead B) Latex C) Formaldehyde D) Benzene

Answer: A Explanation: A) Women exposed to lead during pregnancy are at risk for spontaneous abortion, prematurity, low birth weight, intrauterine growth restriction, and brain, kidney, and nervous system dysfunction. B) Exposure to latex is not known to cause neurological damage. C) Exposure to formaldehyde is not known to cause neurological damage. Exposure to benzene is not known to cause neurological damage

1) A 26-year-old client is having her initial prenatal appointment. The client reports to the nurse that she suffered a pelvic fracture in a car accident 3 years ago. The client asks whether her pelvic fracture might affect her ability to have a vaginal delivery. What response by the nurse is best? A) "It depends on how your pelvis healed." B) "You will need to have a cesarean birth." C) "Please talk to your doctor about that." D) "You will be able to delivery vaginally."

Answer: A Explanation: A) Women with a history of pelvic fractures may also be at risk for cephalopelvic disproportion (C P D). B) Not all clients will be able to deliver vaginally, but not all will need cesarean birth. C) It is not therapeutic to tell a client to talk to someone else. Not all clients will be able to deliver vaginally, but not all will need cesarean birth

1) The nurse is caring for a female client with a history of pelvic inflammatory disease (P I D) who reports having difficulty conceiving after unprotected sex for over 2 years. Which deviation from the norm does the nurse recognize is most likely the cause of the client's infertility? A) Non-patent fallopian tube B) Unfavorable cervical mucus C) Absence of ovulation D) Abnormal endometrial preparation

Answer: A Explanation: A) Women with a history of pelvic inflammatory disease (P I D) have the risk of developing infertility. Specifically, obstruction and non-patent fallopian tubes often occur in P I D and are a common source of infertility among women with the disease. Unfavorable cervical mucus, absence of ovulation, and abnormal endometrial preparation may all lead to infertility; however, these are not most common among women diagnosed with P I D. B) Women with a history of pelvic inflammatory disease (P I D) have the risk of developing infertility. Specifically, obstruction and non-patent fallopian tubes often occur in P I D and are a common source of infertility among women with the disease. Unfavorable cervical mucus, absence of ovulation, and abnormal endometrial preparation may all lead to infertility; however, these are not most common among women diagnosed with P I D. C) Women with a history of pelvic inflammatory disease (P I D) have the risk of developing infertility. Specifically, obstruction and non-patent fallopian tubes often occur in P I D and are a common source of infertility among women with the disease. Unfavorable cervical mucus, absence of ovulation, and abnormal endometrial preparation may all lead to infertility; however, these are not most common among women diagnosed with P I D. D) Women with a history of pelvic inflammatory disease (P I D) have the risk of developing infertility. Specifically, obstruction and non-patent fallopian tubes often occur in P I D and are a common source of infertility among women with the disease. Unfavorable cervical mucus, absence of ovulation, and abnormal endometrial preparation may all lead to infertility; however, these are not most common among women diagnosed with P I D.

A female client with an intrauterine device calls the clinic because she is unable to locate the strings after her last menstrual period. What should the nurse counsel this client? A) Schedule an appointment immediately B) Wait a few days and recheck for the strings C) Take a tub bath and then recheck for the strings D) Perform a douche and then recheck for the strings

Answer: A Explanation: A) Women with intrauterine contraception should contact their healthcare providers if the strings are missing. The client should not wait a few days, take a tub bath, or douche before rechecking for the strings. B) Women with intrauterine contraception should contact their healthcare providers if the strings are missing. The client should not wait a few days, take a tub bath, or douche before rechecking for the strings. C) Women with intrauterine contraception should contact their healthcare providers if the strings are missing. The client should not wait a few days, take a tub bath, or douche before rechecking for the strings. D) Women with intrauterine contraception should contact their healthcare providers if the strings are missing. The client should not wait a few days, take a tub bath, or douche before rechecking for the strings.

1) The nurse is working with a group of recent immigrants from a country in which female genital mutilation (F G M) is practiced. In order to be effective in teaching about gynecologic care in the U.S., the nurse must keep which issues in mind? Note: Credit will be given only if all correct choices and no incorrect choices are selected. Select all that apply. A) Women might undergo F G M willingly to support the status quo of society. B) Women might undergo the procedure to be considered for marriage. C) Women who immigrate to other countries feel a sense pride once the procedure has been performed. D) Women might undergo the procedure to gain greater sexual pleasure. Women might undergo the procedure to lose their virginity

Answer: A, B Explanation: 1. The procedure is performed by various ethnic groups for a variety of reasons, including perceived improved social acceptance. 2. The procedure is performed by various ethnic groups for a variety of reasons, including marriageability. 3. Women who immigrate to other countries may feel a sense of shame or embarrassment once the procedure has been performed. 4. The procedure is performed by various ethnic groups for a variety of reasons, including the reduction of female sexual desire. 5. Women might undergo the procedure to preserve their virginity.

1) In caring for the premature newborn, the nurse must assess hydration status continually. Assessment parameters should include which of the following? Note: Credit will be given only if all correct choices and no incorrect choices are selected. Select all that apply. A) Volume of urine output B) Weight C) Blood p H D) Head circumference E) Bowel sounds

Answer: A, B Explanation: A) In order to assess hydration status, volume of urine output must be evaluated. B) In order to assess hydration status, the infant's weight must be evaluated. C) Blood p H is not an indicator of hydration. D) Head circumference is not an indicator of hydration. E) Bowel sounds are not an indicator of hydration.

1) The nurse is caring for a client who has experienced premature rupture of membranes. For which maternal implication(s) should the nurse monitor? Note: Credit will be given only if all correct choices and no incorrect choices are selected. Select all that apply. A) Infection B) Preterm labor C) Dyspnea D) Discomfort E) E) Uterine distention

Answer: A, B Explanation: A) Premature rupture of membranes places the client at an increased risk for infection. B) Premature rupture of membranes places the client at an increased risk for preterm labor. C) Hydramnios, not premature rupture of membranes, increases dyspnea. D) Hydramnios, not premature rupture of membranes, increases discomfort. E) Multiple gestation, not premature rupture of membranes, increases uterine distention.

1) Why is it important for the nurse to understand the type of family that a client comes from? Note: Credit will be given only if all correct choices and no incorrect choices are selected. Select all that apply. A) Family structure can influence finances. B) Some families choose to conceive or adopt without a life partner. C) The nurse can anticipate which problems a client will experience based on the type of family the client has. D) Understanding if the client's family is nuclear or blended will help the nurse teach the client the appropriate information. E) The values of the family will be predictable if the nurse knows what type of family the client is a part of.

Answer: A, B Explanation: A) Single-parent families often face difficulties because the sole parent may lack social and emotional support, need assistance with childrearing issues, and face financial strain. B) In the single mother by choice family, the mother is typically older, college-educated, and financially stable and has contemplated pregnancy significantly prior to conceiving. C) Each client and family must be assessed as individuals, without making assumptions. Although generalities can be drawn based on the type of family that a client comes from or currently is part of, stereotypes must be avoided. D) Each client and family must be assessed as individuals, without making assumptions. Although generalities can be drawn based on the type of family that a client comes from or currently is part of, stereotypes must be avoided. E) Each client and family must be assessed as individuals, without making assumptions. Although generalities can be drawn based on the type of family that a client comes from or currently is part of, stereotypes must be avoided.

1) The nurse is assessing the comfort of the parents during the third stage of labor. Which finding(s) indicate that the parents feel comfortable during this stage? Note: Credit will be given only if all correct choices and no incorrect choices are selected. Select all that apply. A) Talking to the newborn B) Verbally expressing feelings of pride C) Requesting to dim the lights D) Preferring limited contact with the newborn initially E) Immediately placing phone calls

Answer: A, B Explanation: A) Talking to the newborn and verbally expressing feelings of pride are indications that the parents feel comfortable in the environment. B) Talking to the newborn and verbally expressing feelings of pride are indications that the parents feel comfortable in the environment. C) Requesting to dim the lights does not necessarily indicate whether or not the parents feel comfortable in the environment. D) The parents may prefer to limit contact with the newborn initially, and it is important for the nurse to support the wishes of the parent. However, this does not necessarily indicate whether or not the parents feel comfortable in the environment. E) Immediately placing phone calls after the newborn is delivered does not necessarily indicate whether or not the parents feel comfortable in the environment.

28) The nurse has taken a detailed social history from a client admitted to the birthing unit. Which insights may the nurse gain as a result of this assessment? Note: Credit will be given only if all correct choices and no incorrect choices are selected. Select all that apply. A) Social habits B) Psychologic factors C) Presence of H I V D) Readiness for discharge E) Need for bed rest

Answer: A, B Explanation: A) The nurse may gain a view of both the woman's social habits and psychologic factors that may affect her birth experience. B) The nurse may gain a view of both the woman's social habits and psychologic factors that may affect her birth experience. C) The presence of H I V may only be detected by specific testing. D) The readiness for discharge cannot be determined at the time of obtaining a detailed social history. E) The need for bed rest is determine by the physical, not social, assessment.

1) A cesarean section is ordered for the laboring client with whom the nurse has worked all shift. The client will receive general anesthesia. The nurse knows that potential complications of general anesthesia include which of the following? Note: Credit will be given only if all correct choices and no incorrect choices are selected. Select all that apply. A) Fetal depression that is directly proportional to the depth and duration of the anesthesia B) Poor fetal metabolism of anesthesia, which inhibits use with preterm infants C) Uterine relaxation D) Increased gastric motility E) Itching of the face and neck

Answer: A, B, C Explanation: A) A primary danger of general anesthesia is fetal depression, because the medication reaches the fetus in about 2 minutes. The depression is directly proportional to the depth and duration of anesthesia. B) The poor fetal metabolism of general anesthetic agents is similar to that of analgesic agents administered during labor. General anesthesia is not advocated when the fetus is considered to be at high risk, particularly in preterm birth. C) Most general anesthetic agents cause some degree of uterine relaxation. D) Pregnancy results in decreased gastric motility, and the onset of labor halts the process almost entirely. Food eaten hours earlier may remain undigested in the stomach. Itching of the face and neck is not associated with general anesthesia

1) Lesbian, transgendered, and bisexual women are at greater risk for health and social disparities, including which of the following? Note: Credit will be given only if all correct choices and no incorrect choices are selected. Select all that apply. A) Increased suicide risk B) Increased risk of homelessness C) Lack of screening for female-related cancers D) Lack of screening for lung cancers E) Increased divorce rates

Answer: A, B, C Explanation: A) Healthy People 2020 recognizes that health disparities continue to exist for lesbian, gay, bisexual, and transgendered individuals, which includes increased suicide risk. B) Healthy People 2020 recognizes that health disparities continue to exist for lesbian, gay, bisexual, and transgendered individuals, which includes increased risk of homelessness. C) Healthy People 2020 recognizes that health disparities continue to exist for lesbian, gay, bisexual, and transgendered individuals, which includes lack of screening for female-related cancers. D) Lack of screening for lung cancer is not a specific risk factor for this demographic. E) Increased divorce rates is not a specific factor for this demographic.

1. The labor nurse would not encourage a mother to bear down until the cervix is completely dilated, to prevent which of the following? Note: Credit will be given only if all correct choices and no incorrect choices are selected. Select all that apply. A) Maternal exhaustion B) Cervical edema C) Tearing and bruising of the cervix D) Enhanced perineal thinning E) Having to perform an episiotomy

Answer: A, B, C Explanation: A) If the cervix is not completely dilated, maternal exhaustion can occur. B) If the cervix is not completely dilated, cervical edema can occur. C) If the cervix is not completely dilated, tearing and bruising of the cervix can occur. D) Cervical dilation has nothing to do with perineal thinning. E) Not bearing down until the cervix is completely dilated has nothing to do with needing an episiotomy.

1) In working with immigrants in an inner-city setting, the nurse recognizes that acculturation of immigrants often brings with it which of the following benefits? Note: Credit will be given only if all correct choices and no incorrect choices are selected. Select all that apply. A) Improved socioeconomic status B) Use of preventive care services C) Better nutrition D) Increase in substance abuse over time E) More physician visits due to language barriers

Answer: A, B, C Explanation: A) Improvement of socioeconomic status is a benefit of acculturation in the United States. B) Acculturation of immigrants increases the likelihood that the family members will use preventive health services. C) Improved socioeconomic status leads to better nutrition and access to health care. D) Substance abuse tends to increase over time as immigrants acculturate, especially among Hispanics. E) Language barriers with physicians tend to decrease the use of healthcare services.

1. A nurse suspects that a postpartum client has mastitis. Which data support this assessment? Note: Credit will be given only if all correct choices and no incorrect choices are selected. Select all that apply. A) Shooting pain between breastfeedings B) Late onset of nipple pain C) Pink, flaking, pruritic skin of the affected nipple D) Nipple soreness when the infant latches on E) Pain radiating to the underarm area from the breast

Answer: A, B, C Explanation: A) Mastitis is characterized by shooting pain between feedings, often radiating to the chest wall. B) Mastitis is characterized by late-onset nipple pain. C) The skin of the affected breast becomes pink, flaking, and pruritic. D) Nipple soreness when the infant latches on is not a symptom of mastitis. E) The pain from mastitis does not radiate to the underarm area.

1) A client is admitted to the labor and delivery unit in active labor. What nursing diagnoses might apply to the client with suspected abruptio placentae? Note: Credit will be given only if all correct choices and no incorrect choices are selected. Select all that apply. A) Fluid Volume, Deficient, related to hypovolemia secondary to excessive blood loss B) Tissue Perfusion: Peripheral, Ineffective, related to blood loss secondary to uterine atony following birth C) Anxiety related to concern for own personal status and the baby's safety D) Knowledge, Deficient related to lack of information about inherited genetic defects E) Alteration in Respiratory Function related to blood loss

Answer: A, B, C Explanation: A) Maternal and perinatal fetal mortality are concerns due to hypoxia. B) Maternal and perinatal fetal mortality are concerns due to blood loss. C) This mother would be anxious for herself and her baby. D) Abruptio placentae is a premature separation of the placenta, not a genetic abnormality. E) Respiratory function is not related to the blood loss. Also, this is not a nursing diagnosis.

1) In assessing a family, the community nurse uses a family assessment tool, which provides an organized framework to collect data concerning which of the following? Note: Credit will be given only if all correct choices and no incorrect choices are selected. Select all that apply. A) Access to laundry and grocery facilities B) Access to health care C) Sharing of religious beliefs and values D) Acculturation to traditional lifestyles E) Ability to include a new spouse into the family unit

Answer: A, B, C Explanation: A) Measuring access to laundry, grocery, and recreational facilities that meet the physical, emotional, and spiritual needs of members is part of the family assessment tool. B) Measuring access to healthcare that meets the physical, emotional, and spiritual needs of members is part of the family assessment tool. C) Learning about shared religious beliefs and values, which meet the physical, emotional, and spiritual needs of members, is part of the family assessment tool. D) Acculturation to traditional lifestyles is not measured with the family assessment tool. E) The ability to include a new spouse into the family unit is a developmental task/stage of those who are divorced, and is not a part of the family assessment tool.

Nursing research is vital to do which of the following? Note: Credit will be given only if all correct choices and no incorrect choices are selected. Select all that apply. A) Expand the science of nursing. B) Foster evidence-based practice. C) Improve client care. D) Visually depict nursing management. Plan and organize care

Answer: A, B, C Explanation: A) Research is vital to expanding the science of nursing. B) Research is vital to fostering evidence-based practice. C) Research is vital to improving client care. D) The nursing process is research-based, but is not a part of the clinical pathway. Visually depicting nursing management is part of concept mapping, not nursing research. E) Organizing patient care is an aspect of the nursing process. Planning and organizing care is part of nursing care plans, not nursing research.

The Quality and Safety Education for Nurses (Q S E N) project focused on competencies in which areas? Note: Credit will be given only if all correct choices and no incorrect choices are selected. Select all that apply. A) Client-centered care B) Teamwork and collaboration C) Evidence-based practice D) Family planning E) Injury and violence prevention

Answer: A, B, C Explanation: A) The Quality and Safety Education for Nurses (Q S E N ) project is designed "to meet the challenge of preparing future nurses who will have the knowledge, skills, and attitudes (K S A s) necessary to continuously improve the quality and safety of the healthcare systems within which they work," which includes client-centered care. B) The Quality and Safety Education for Nurses (Q S E N ) project, is designed "to meet the challenge of preparing future nurses who will have the knowledge, skills, and attitudes (KSAs) necessary to continuously improve the quality and safety of the healthcare systems within which they work," which includes teamwork and collaboration. C) The Quality and Safety Education for Nurses (Q S E N ) project, is designed "to meet the challenge of preparing future nurses who will have the knowledge, skills, and attitudes (K S A s) necessary to continuously improve the quality and safety of the healthcare systems within which they work," which includes evidence-based practice. D) Healthy People 2020 focuses on family planning. E) Healthy People 2020 focuses on injury and violence prevention.

1) What self-care measures would a nurse recommend for a client in her first trimester to reduce the discomfort of nausea and vomiting? Note: Credit will be given only if all correct choices and no incorrect choices are selected. Select all that apply. A) Avoid odors or causative factors. B) Have small but frequent meals. C) Drink carbonated beverages. D) Drink milk before arising in the morning. E) Eat highly seasoned food.

Answer: A, B, C Explanation: A) The nurse would recommend for a client in her first trimester to avoid odors and causative factors to reduce the discomfort of nausea and vomiting. B) The nurse would recommend for a client in her first trimester to have small but frequent meals to reduce the discomfort of nausea and vomiting. C) The nurse would recommend for a client in her first trimester to drink carbonated beverages to reduce the discomfort of nausea and vomiting. D) The nurse would recommend for a client in her first trimester to eat dry crackers or toast before arising in the morning to reduce the discomfort of nausea and vomiting. E) The nurse would recommend for a client in her first trimester to avoid greasy or highly seasoned foods to reduce the discomfort of nausea and vomiting.

1) The prenatal period should be used to expose the prospective parents to up-to-date, evidence-based information about which of the following topics? Note: Credit will be given only if all correct choices and no incorrect choices are selected. Select all that apply. A) Breastfeeding B) Pain relief C) Obstetric complications and procedures D) Toddler care E) Antepartum adjustment

Answer: A, B, C Explanation: A) The prenatal period should expose prospective parents to up-to-date, evidence-based information about breastfeeding. B) The prenatal period should expose prospective parents to up-to-date, evidence-based information about pain relief. C) The prenatal period should expose prospective parents to up-to-date, evidence-based information about obstetric complications and procedures. D) The prenatal period should expose prospective parents to up-to-date, evidence-based information about normal newborn care. E) The prenatal period should expose prospective parents to up-to-date, evidence-based information about postpartum adjustment.

1) The nurse is caring for a client who is about to receive an amnioinfusion. For which complication(s) should the nurse monitor the client? Note: Credit will be given only if all correct choices and no incorrect choices are selected. Select all that apply. A) Umbilical cord prolapse B) Amniotic fluid embolism C) Uterine rupture D) Amnionitis E) Abruptio placentae

Answer: A, B, C Explanation: A) Umbilical cord prolapse is a rare, but serious risk factor for the client receiving an amnioinfusion. B) Amniotic fluid embolism is a rare, but serious risk factor for the client receiving an amnioinfusion. C) Uterine rupture is a rare, but serious risk factor for the client receiving an amnioinfusion. D) The presence of amnionitis is a contraindication to amnioinfusion, but is not a condition that results from amnioinfusion. E) The presence of abruptio placentae is a contraindication to amnioinfusion, but is not a condition that results from amnioinfusion.

1. A postpartal client recovering from deep vein thrombosis is being discharged. What areas of teaching on self-care and anticipatory guidance should the nurse discuss with the client? Note: Credit will be given only if all correct choices and no incorrect choices are selected. Select all that apply. A) Avoid crossing the legs. B) Avoid prolonged standing or sitting. C) Take frequent walks. D) Take a daily aspirin dose of 650 m g. E) Avoid long car trips.

Answer: A, B, C Explanation: A) Women should be taught to avoid prolonged standing or sitting in one position or sitting with legs crossed. B) Women should be taught to avoid prolonged standing or sitting in one position or sitting with legs crossed. C) Women should be advised to avoid a sedentary lifestyle and to exercise as much as possible (walking is ideal). D) Taking a daily aspirin increases anticoagulant activity, and should be avoided if the client is being treated with other anticoagulants. E) Avoiding long car trips is not necessary. The client should be encouraged to take frequent breaks during long car trips and to walk around, thereby preventing prolonged venous stasis.

The nurse is discharging a client after hospitalization for pelvic inflammatory disease (P I D). Which statements indicate that teaching was effective? Note: Credit will be given only if all correct choices and no incorrect choices are selected. Select all that apply. A) "I might have infertility because of this infection." B) "It is important for me to finish my antibiotics." C) "Tubal pregnancy could occur after P I D." D) "My P I D was caused by a yeast infection." E) "I am going to have an I U D placed for contraception."

Answer: A, B, C Explanation: A) Women sometimes become infertile because of scarring in the fallopian tubes as a result of the inflammation of P I D. B) Antibiotic therapy should always be completed when a client is diagnosed with any infection. C) The tubal scarring that occurs from tubal inflammation during P I D can prevent a fertilized ovum from passing through the tube into the uterus, causing an ectopic or tubal pregnancy. D) P I D is caused by bacteria, most commonly Chlamydia trachomatis or Neisseria gonorrhoeae. E) An intrauterine device (I U D) in place increases the risk of developing P I D; a client who has a history of P I D is not a good candidate for an I U D.

15) The nurse is reviewing the contents of the birthing unit's emergency pack for use in case of a precipitous birth. Which item(s) should the nurse ensure is (are) included in the pack? Note: Credit will be given only if all correct choices and no incorrect choices are selected. Select all that apply. A) Sterile drape B) Bulb syringe C) Two sterile clamps D) Sterile gloves E) Forceps

Answer: A, B, C, D Explanation: A) A small drape is included that can be placed under the woman's buttocks to provide a sterile field. B) A bulb syringe is needed to clear mucus from the newborn's mouth. C) Two sterile clamps (Kelly or Rochester) are needed to clamp the umbilical cord before applying a cord clamp. D) Sterile gloves are a basic element of a typical birthing unit emergency pack. E) Forceps are not required during a precipitous birth.

1) The nurse is performing an assessment on a client in the birthing unit who has acquired cervical insufficiency. Which other finding(s) may contribute to the client's condition? Note: Credit will be given only if all correct choices and no incorrect choices are selected. Select all that apply. A) Inflammation B) Infection C) Cervical trauma D) Cone biopsy E) H P V positivity

Answer: A, B, C, D Explanation: A) Acquired cervical insufficiency may be related to inflammation. B) Acquired cervical insufficiency may be related to infection. C) Acquired cervical insufficiency may be related to cervical trauma. D) Acquired cervical insufficiency may be related to cone biopsy. E) Acquired cervical insufficiency is not related to H P V positivity.

A client at 10 weeks' gestation is scheduled for a surgical abortion. Which approaches may be used to dilate the cervix for the procedure? Note: Credit will be given only if all correct choices and no incorrect choices are elected. Select all that apply. A) Misoprostol B) Mifepristone C) Metal dilators D) Sterile seaweed E) Paracervical block

Answer: A, B, C, D Explanation: A) After 8 weeks' gestation, the cervix is dilated with misoprostol or mifepristone, mechanically with metal dilators, or osmotically with sterile seaweed. A paracervical block is used to anesthetize the cervix. B) After 8 weeks' gestation, the cervix is dilated with misoprostol or mifepristone, mechanically with metal dilators, or osmotically with sterile seaweed. A paracervical block is used to anesthetize the cervix. C) After 8 weeks' gestation, the cervix is dilated with misoprostol or mifepristone, mechanically with metal dilators, or osmotically with sterile seaweed. A paracervical block is used to anesthetize the cervix. D) After 8 weeks' gestation, the cervix is dilated with misoprostol or mifepristone, mechanically with metal dilators, or osmotically with sterile seaweed. A paracervical block is used to anesthetize the cervix. E) After 8 weeks' gestation, the cervix is dilated with misoprostol or mifepristone, mechanically with metal dilators, or osmotically with sterile seaweed. A paracervical block is used to anesthetize the cervix.

1) The nurse is performing a preoperative assessment on a client who is in the second trimester of pregnancy. For which finding(s) should the nurse monitor? Note: Credit will be given only if all correct choices and no incorrect choices are selected. Select all that apply. A) Respiratory infection B) Fever C) Urinary tract infection D) Anemia E) ABO incompatibility

Answer: A, B, C, D Explanation: A) Assessing for respiratory infections is an important part of the preoperative assessment for the client who is pregnant. B) Assessing for fever is an important part of the preoperative assessment for the client who is pregnant. C) Assessing for urinary tract infection is an important part of the preoperative assessment for the client who is pregnant. D) Assessing for anemia is an important part of the preoperative assessment for the client who is pregnant. E) ABO incompatibility is not routinely assessed as part of the preoperative assessment for the client who is pregnant, as it rarely has serious life-threatening consequences and is almost exclusively seen after the birth of the newborn.

1) The nurse is performing a comprehensive assessment on a client admitted to the birthing unit with abruptio placentae. Which finding(s) contribute(s) to this condition? Note: Credit will be given only if all correct choices and no incorrect choices are selected. Select all that apply. A) History of domestic violence B) Presence of uterine fibroids C) Alcohol consumption during pregnancy D) Hypertension E) Gestational diabetes mellitus

Answer: A, B, C, D Explanation: A) Domestic violence contributes to the development of abruptio placentae. B) The presence of fibroids contributes to the development of abruptio placentae. C) Alcohol consumption contributes to the development of abruptio placentae. D) Maternal hypertension is the most common cause of abruptio placentae. E) Gestational diabetes mellitus is not considered a finding that contributes to abruptio placentae.

1) The nurse is caring for a client undergoing fetal heart rate monitoring, and the F H R is greater than 162 beats/min for 12 minutes. For what cause(s) should the nurse anticipate treatment? Note: Credit will be given only if all correct choices and no incorrect choices are selected. Select all that apply. A) Maternal anxiety B) Fetal asphyxia C) Prematurity D) Fetal anemia E) Maternal hypotension

Answer: A, B, C, D Explanation: A) Maternal anxiety may result in fetal tachycardia. B) Fetal asphyxia may result in fetal tachycardia. C) Prematurity may result in fetal tachycardia. D) Fetal anemia may result in fetal tachycardia. E) Maternal hypotension may result in fetal bradycardia, not fetal tachycardia.

1) The nurse on the birthing unit is collecting the obstetric history of a client at risk for cervical insufficiency. Which findings increase the client's risk for this condition? Note: Credit will be given only if all correct choices and no incorrect choices are selected. Select all that apply. A) Multiple gestations B) Previous preterm births C) Progressively earlier births with each subsequent pregnancy D) Cervical manipulation E) Prolonged labors

Answer: A, B, C, D Explanation: A) Multiple gestations increase the risk for cervical insufficiency. B) Previous preterm births increase the risk for cervical insufficiency. C) Progressively earlier births with each subsequent pregnancy increase the risk for cervical insufficiency. D) Cervical manipulation increases the risk for cervical insufficiency. E) Short labors, not prolonged labors, increase the risk for cervical insufficiency.

1) During a labor and delivery class, a client asks the nurse, "Why would I be placed under general anesthesia during delivery?" What should the nurse include in the response? Note: Credit will be given only if all correct choices and no incorrect choices are selected. Select all that apply. A) Perceived lack of time for regional anesthesia B) Contraindications for regional anesthesia C) Failure of regional anesthesia D) Refusal of regional anesthesia E) Provider preference over regional anesthesia

Answer: A, B, C, D Explanation: A) Perceived lack of time is a common indication for general anesthesia. B) When regional anesthesia is contraindicated, general anesthesia is commonly used. C) Failure of regional anesthesia is a common indication for general anesthesia. D) Patient refusal of regional anesthesia is a common indication for general anesthesia. Provider preference is not a common indication of general anesthesia

1) Pesticide exposure can be linked to a variety of adverse health outcomes, including which of the following? Note: Credit will be given only if all correct choices and no incorrect choices are selected. Select all that apply. A) Increased risk of cancer B) Endocrine abnormalities C) Liver damage D) Birth defects E) Cardiovascular diseases

Answer: A, B, C, D Explanation: A) Pesticide exposure can be linked to a variety of adverse health outcomes, including an increased risk of cancer. B) Pesticide exposure can be linked to a variety of adverse health outcomes, including endocrine abnormalities. C) Pesticide exposure can be linked to a variety of adverse health outcomes, including liver damage. D) Pesticide exposure can be linked to a variety of adverse health outcomes, including birth defects. E) Cardiovascular diseases in not one of the adverse health outcomes linked to pesticide exposure.

1) A 17-year-old high school student comes into the nurse's office to find out what to do about severe menstrual cramps. What should the nurse recommend to this student? Note: Credit will be given only if all correct choices and no incorrect choices are selected. Select all that apply. A) Rest B) Good nutrition C) Regular exercise D) Application of heat E) D & C of the uterus

Answer: A, B, C, D Explanation: A) Treatment of primary dysmenorrhea includes rest, good nutrition, regular exercise, and application of heat. D & C of the uterus is a treatment for secondary dysmenorrhea. B) Treatment of primary dysmenorrhea includes rest, good nutrition, regular exercise, and application of heat. D & C of the uterus is a treatment for secondary dysmenorrhea. C) Treatment of primary dysmenorrhea includes rest, good nutrition, regular exercise, and application of heat. D & C of the uterus is a treatment for secondary dysmenorrhea. D) Treatment of primary dysmenorrhea includes rest, good nutrition, regular exercise, and application of heat. D & C of the uterus is a treatment for secondary dysmenorrhea. E) Treatment of primary dysmenorrhea includes rest, good nutrition, regular exercise, and application of heat. D & C of the uterus is a treatment for secondary dysmenorrhea.

A 40-year-old patient is being seen in the clinic for gynecological changes. Which approaches should the nurse use when completing this patient's health interview? Note: Credit will be given only if all correct choices and no incorrect choices are selected. Select all that apply. A) Avoid writing B) Clarify terms used C) Maintain eye contact D) Analyze body language E) Use simple yes-no questions

Answer: A, B, C, D Explanation: A) When conducting a sexual history the nurse should avoid writing, clarify terms being used, maintain eye contact unless it is culturally inappropriate, and analyze the patient's body language. Using closed questions will limit the amount of information collected and should be avoided. B) When conducting a sexual history the nurse should avoid writing, clarify terms being used, maintain eye contact unless it is culturally inappropriate, and analyze the patient's body language. Using closed questions will limit the amount of information collected and should be avoided. C) When conducting a sexual history the nurse should avoid writing, clarify terms being used, maintain eye contact unless it is culturally inappropriate, and analyze the patient's body language. Using closed questions will limit the amount of information collected and should be avoided. D) When conducting a sexual history the nurse should avoid writing, clarify terms being used, maintain eye contact unless it is culturally inappropriate, and analyze the patient's body language. Using closed questions will limit the amount of information collected and should be avoided. E) When conducting a sexual history the nurse should avoid writing, clarify terms being used, maintain eye contact unless it is culturally inappropriate, and analyze the patient's body language. Using closed questions will limit the amount of information collected and should be avoided.

1. Childbirth preparation offers several advantages including which of the following? Note: Credit will be given only if all correct choices and no incorrect choices are selected. Select all that apply. A) It helps a pregnant woman and her support person understand the choices in the birth setting. B) It promotes awareness of available options. C) It provides tools for a pregnant woman and her support person to use during labor and birth. D) Women who receive continuous support during labor require more analgesia, and have more cesarean and instrument births. E) Each method has been shown to shorten labor.

Answer: A, B, C, E Explanation: A) Childbirth preparation offers several advantages. It helps a pregnant woman and her support person understand the choices in the birth setting, promotes awareness of available options, and provides tools for them to use during labor and birth. B) Childbirth preparation offers several advantages. It helps a pregnant woman and her support person understand the choices in the birth setting, promotes awareness of available options, and provides tools for them to use during labor and birth. C) Childbirth preparation offers several advantages. It helps a pregnant woman and her support person understand the choices in the birth setting, promotes awareness of available options, and provides tools for them to use during labor and birth. D) This is not true. Women who receive continuous support during labor require less analgesia, and have fewer cesarean and instrument births. Childbirth preparation offers several advantages. Each method has been shown to shorten labor

1) When blood pressure and other signs indicate that preeclampsia is worsening, hospitalization is necessary to monitor the woman's condition closely. At that time, which of the following should be assessed? Note: Credit will be given only if all correct choices and no incorrect choices are selected. Select all that apply. A) Fetal heart rate B) Blood pressure C) Temperature D) Urine color E) Pulse and respirations

Answer: A, B, C, E Explanation: A) Determine the fetal heart rate along with blood pressure, or monitor continuously with the electronic fetal monitor if the situation indicates. B) Determine blood pressure every 1 to 4 hours, or more frequently if indicated by medication or other changes in the woman's status. C) Determine temperature every 4 hours, or every 2 hours if elevated or if premature rupture of the membranes (PROM) has occurred. D) Urine color is not monitored. However, urine output, urine protein, and urine specific gravity are monitored. E) Determine pulse rate and respirations along with blood pressure.

1) Upon assessing the F H R tracing, the nurse determines that there is fetal tachycardia. The fetal tachycardia would be caused by which of the following? Note: Credit will be given only if all correct choices and no incorrect choices are selected. Select all that apply. A) Early fetal hypoxia B) Prolonged fetal stimulation C) Fetal anemia D) Fetal sleep cycle E) Infection

Answer: A, B, C, E Explanation: A) Early fetal hypoxia can cause fetal tachycardia. B) Prolonged fetal stimulation can cause fetal tachycardia. C) Fetal anemia can cause fetal tachycardia. D) The fetal sleep cycle does not cause fetal tachycardia. E) Infection can cause fetal tachycardia.

1) For what common side effects of epidural anesthesia should the nurse watch? Note: Credit will be given only if all correct choices and no incorrect choices are selected. Select all that apply. A) Elevated maternal temperature B) Urinary retention C) Nausea D) Long-term back pain E) Local itching

Answer: A, B, C, E Explanation: A) Elevated maternal temperature is a potential side effect of epidural anesthesia. B) Urinary retention is a potential side effect of epidural anesthesia. C) Nausea is a potential side effect of epidural anesthesia. D) Long-term back pain should not result from an epidural. E) Pruritus may occur at any time during the epidural infusion. It usually appears first on the face, neck, or torso and is generally the result of the agent used in the epidural infusion. Benadryl, an antihistamine, can be administered to manage pruritus.

1) The public health nurse is working with a student nurse. The student nurse asks which of the six groups of people they have seen today are considered to be families. How should the nurse respond? Note: Credit will be given only if all correct choices and no incorrect choices are selected. Select all that apply. A) "The married heterosexual couple without children" B) "The gay couple with two adopted children" C) "The unmarried heterosexual couple with two biological children" D) "The lesbian couple not living together that have no children" E) "The married heterosexual couple with three children, living with grandparents"

Answer: A, B, C, E Explanation: A) Families take many forms in today's society. The basis for people to be considered a family is a commitment to one another and the sharing of responsibilities, chores, and expenses. A couple without children is still a family. B) Families take many forms in today's society. The basis for people to be considered a family is a commitment to one another and the sharing of responsibilities, chores, and expenses. Gay and lesbian families are those in which two or more people who share a same-sex orientation live together, or in which a gay or lesbian single parent rears a child. C) Families take many forms in today's society. The basis for people to be considered a family is a commitment to one another and the sharing of responsibilities, chores, and expenses. A family may be formed without a legal marriage. D) A couple not living together and without children together are considered dating and not yet a family. E) Families take many forms in today's society. The basis for people to be considered a family is a commitment to one another and the sharing of responsibilities, chores, and expenses. Extended family members, including parents or grandparents, will often live with their adult children or grandchildren, creating intergenerational families.

1) Which of the following symptoms, if progressive, are indicative of C H F, the heart's signal of its decreased ability to meet the demands of pregnancy? Note: Credit will be given only if all correct choices and no incorrect choices are selected. Select all that apply. A) Palpitations B) Heart murmurs C) Dyspnea D) Frequent urination E) Rales

Answer: A, B, C, E Explanation: A) Palpitations are indicative of C H F. B) Heart murmurs are indicative of C H F. C) Dyspnea is indicative of C H F. D) Frequent urination is not indicative of C H F. Rales are indicative of CHF

1. Premonitory signs of labor include which of the following? Note: Credit will be given only if all correct choices and no incorrect choices are selected. Select all that apply. A) Braxton Hicks contractions B) Cervical softening and effacement C) Weight gain D) Rupture of membranes E) Sudden loss of energy

Answer: A, B, D Explanation: A) A premonitory sign of labor includes Braxton Hicks contractions. B) A premonitory sign of labor includes cervical softening and effacement. C) A premonitory sign of labor includes weight loss, not weight gain. D) A premonitory sign of labor includes rupture of membranes. E) A premonitory sign of labor includes a sudden burst of energy, not a loss of energy.

1) Marked changes that occur in the cardiopulmonary system at birth include which of the following? Note: Credit will be given only if all correct choices and no incorrect choices are selected. Select all that apply. A) Closure of the foramen ovale B) Closure of the ductus venosus C) Mean blood pressure of 31 to 61 m m H g in full-term resting newborns D) Increased systemic vascular resistance and decreased pulmonary vascular resistance E) Opening of the ductus arteriosus

Answer: A, B, D Explanation: A) Closure of the foramen ovale is a function of changing arterial pressures. B) Closure of the ductus venosus is related to mechanical pressure changes that result from severing the cord, redistribution of blood, and cardiac output. C) The average mean blood pressure of 31 to 61 m m H g in full-term resting newborns is a normal finding, but not a marked change in the cardiopulmonary system. D) Increased systemic vascular resistance and decreased pulmonary vascular resistance; with the loss of the low-resistance placenta, systemic vascular resistance increases, resulting in greater systemic pressure. The combination of vasodilation and increased pulmonary blood flow decreases pulmonary vascular resistance. E) Functional closure, not opening, of the ductus arteriosus in the well newborn starts at 10 to 15 hours after birth.

1) The client is undergoing an emergency cesarean birth for fetal bradycardia. The client's partner has not been allowed into the operating room. What can the nurse do to alleviate the partner's emotional distress? Note: Credit will be given only if all correct choices and no incorrect choices are selected. Select all that apply. A) Allow the partner to wheel the baby's crib to the newborn nursery. B) Allow the partner to be near the operating room where the newborn's first cry can be heard. C) Have the partner wait in the client's postpartum room. D) Encourage the partner to be in the nursery for the initial assessment. E) Teach the partner how to take the client's blood pressure.

Answer: A, B, D Explanation: A) Effective measures include allowing the partner to take the baby to the nursery. B) Effective measures include allowing the partner to be in a place near the operating room, where the newborn's first cry can be heard. C) The nurse should involve the partner in postpartum care in the recovery room, not have the partner wait in the client's postpartum room. D) Effective measures include involving the partner in postpartum care, such as being present for the initial assessment. E) The nurse must take the blood pressure as part of assessing the client.

1) The client with polycystic ovarian syndrome (P C O S) has been prescribed metformin (Glucophage). The nurse tells the client that the medication will do which of the following? Note: Credit will be given only if all correct choices and no incorrect choices are selected. Select all that apply. A) "Decrease your excessive hair growth." B) "Make it easier to lose weight." C) "Increase your acne." D) "Improve your chances of pregnancy." E) "Make your menstrual periods irregular."

Answer: A, B, D Explanation: A) Polycystic ovarian syndrome (P C O S) treatment with metformin decreases hirsutism. B) Polycystic ovarian syndrome (P C O S) treatment with metformin improves weight loss success. C) Polycystic ovarian syndrome (P C O S) treatment with metformin decreases acne. D) Polycystic ovarian syndrome (P C O S) treatment with metformin increases ovulation and therefore menstrual regularity and fertility. Polycystic ovarian syndrome (PCOS) treatment with metformin increases ovulation and therefore menstrual regularity and fertility

1) The nurse tells a mother that the doctor is preparing to circumcise her newborn. The mother expresses concern that the infant will be uncomfortable during the procedure. The nurse explains that the physician will numb the area before the procedure. Additional methods of comfort often used during the procedure include which of the following? Note: Credit will be given only if all correct choices and no incorrect choices are selected. Select all that apply. A) Providing a pacifier B) Stroking the head C) Restraining both arms and legs D) Talking to the infant E) Giving the infant a sedative before the procedure

Answer: A, B, D Explanation: A) Providing a pacifier is an accepted method of soothing during the circumcision. B) Stroking the head is an accepted method of soothing during the circumcision. C) Only the legs are restrained during circumcision. D) Talking to the infant is an accepted method of soothing during the circumcision. E) The infant is never given a sedative.

1) Psychologic elder abuse includes, but is not limited to, which of the following? Note: Credit will be given only if all correct choices and no incorrect choices are selected. Select all that apply. A) Verbal assaults B) Humiliation C) Desertion D) Intimidation E) Failure to provide reasonable care

Answer: A, B, D Explanation: A) Psychologic abuse includes verbal assaults. B) Psychologic abuse includes humiliation. C) Abandonment is the desertion of an elder by any person responsible for the care and custody of that elder. D) Psychologic abuse includes intimidation. E) Failure on the part of a caregiver, or any person having custody of an elder, to provide reasonable care, is consider neglect.

1) What are the three functions of the fallopian tubes? Note: Credit will be given only if all correct choices and no incorrect choices are selected. Select all that apply. A) Provide transport for the ovum from the ovary to the uterus B) Serve as a warm, moist, nourishing environment for the ovum or zygote C) Secrete large amounts of estrogens D) Provide a site for fertilization to occur E) Support and protect the pelvic contents

Answer: A, B, D Explanation: A) The fallopian tubes provide transport for the ovum from the ovary to the uterus. B) The fallopian tubes serve as a warm, moist, nourishing environment for the ovum or zygote. C) The ovaries, not the fallopian tubes, secrete large amounts of estrogens. D) The fallopian tubes provide a site for fertilization to occur. E) The female bony pelvis, not the fallopian tubes, supports and protects the pelvic contents.

1) The nurse educator is teaching student nurses what a fetus will look like at various weeks of development. Which descriptions would be typical of a fetus at 20 weeks' gestation? Note: Credit will be given only if all correct choices and no incorrect choices are selected. Select all that apply. A) The fetus has a body weight of 435-465 g. B) Nipples appear over the mammary glands. C) The kidneys begin to produce urine. D) Nails are present on fingers and toes. E) Lanugo covers the entire body.

Answer: A, B, D, E Explanation: A) A fetus at 20 weeks' gestation has a body weight of 435-465 g. B) A fetus at 20 weeks' gestation has nipples appear over the mammary glands. C) Kidneys of a fetus begin to produce urine at 12 weeks' gestation. D) A fetus at 20 weeks' gestation has nails present on fingers and toes. E) A fetus at 20 weeks' gestation has lanugo that covers the entire body.

1) The nurse is discussing the use of contraception with a client who has just become sexually active. What factors should the nurse include when educating the client on contraceptive methods? Note: Credit will be given only if all correct choices and no incorrect choices are selected. Select all that apply. A) Contraindications in the client's health history B) Religious or moral beliefs C) Partner's belief in the effectiveness of the choice D) Personal preferences to use method E) Future childbearing plans

Answer: A, B, D, E Explanation: A) Decisions about contraception should take into consideration any contraindications the client might have. B) Religious or moral beliefs often impact which choices are acceptable. C) The partner's belief in the effectiveness has no bearing on the actual effectiveness. D) Personal preferences need to be considered when deciding on a contraceptive method. E) Plans for future children should be considered before determining whether sterilization should be performed.

4) The nurse is planning an early-pregnancy class session on nutrition. Which information should the nurse include? Note: Credit will be given only if all correct choices and no incorrect choices are selected. Select all that apply. A) Protein is important for fetal development. B) Iron helps both mother and baby maintain the oxygen-carrying capacity of the blood. C) Calcium prevents constipation at the end of pregnancy. D) Zinc facilitates synthesis of R N A and D N A. E) Vitamin A promotes development of the baby's eyes.

Answer: A, B, D, E Explanation: A) During pregnancy, the woman needs increased amounts of protein to provide amino acids for fetal development. B) Iron deficiency anemia is associated with an increased incidence of preterm birth, low-birth-weight infants, and maternal and infant mortality. C) Calcium is not related to constipation. Calcium is involved in the mineralization of fetal bones and teeth, energy and cell production, and acid-base buffering. D) Zinc is involved in R N A and D N A synthesis, and milk production during lactation. E) Vitamin A promotes healthy formation and development of the fetal eyes.

1) When caring for a laboring client with oligohydramnios, what should the nurse be aware of? Note: Credit will be given only if all correct choices and no incorrect choices are selected. Select all that apply. A) Increased risk of cord compression B) Decreased variability C) Labor progress is often more rapid than average D) Presence of periodic decelerations E) During gestation, fetal skin and skeletal abnormalities can occur

Answer: A, B, D, E Explanation: A) During the labor and birth, the lessened amounts of fluid reduce the cushioning effect for the umbilical cord, and cord compression is more likely to occur. B) The nurse should evaluate the E F M tracing for the presence of nonperiodic decelerations or other nonreassuring signs (such as increasing or decreasing baseline, decreased variability, or presence of periodic decelerations). C) Labor progress may be slower, not faster, than average due to the decreased amniotic fluid volume. Fetal movement can be impaired as a result of inadequate amniotic fluid volume. D) The nurse should evaluate the E F M tracing for the presence of nonperiodic decelerations or other nonreassuring signs (such as increasing or decreasing baseline, decreased variability, or presence of periodic decelerations). E) During the gestational period, fetal skin and skeletal abnormalities may occur because fetal movement is impaired as a result of inadequate amniotic fluid volume.

1) A 30-year-old patient who experiences severe premenstrual syndrome every month asks for nonpharmacologic suggestions to treat this disorder. What should the nurse recommend? Note: Credit will be given only if all correct choices and no incorrect choices are selected. Select all that apply. A) Eat more frequent meals B) Engage in aerobic activity C) Limit alcohol to two drinks per day D) Restrict the intake of chocolate and coffee E) Increase the intake of fruits and vegetables

Answer: A, B, D, E Explanation: A) Nonpharmacologic approaches for treating premenstrual syndrome include eating more frequent meals, engaging in aerobic activity, restricting the intake of chocolate and coffee, and increasing the intake of fruits and vegetables. Alcohol should be restricted and not limited to two drinks per day. B) Nonpharmacologic approaches for treating premenstrual syndrome include eating more frequent meals, engaging in aerobic activity, restricting the intake of chocolate and coffee, and increasing the intake of fruits and vegetables. Alcohol should be restricted and not limited to two drinks per day. C) Nonpharmacologic approaches for treating premenstrual syndrome include eating more frequent meals, engaging in aerobic activity, restricting the intake of chocolate and coffee, and increasing the intake of fruits and vegetables. Alcohol should be restricted and not limited to two drinks per day. D) Nonpharmacologic approaches for treating premenstrual syndrome include eating more frequent meals, engaging in aerobic activity, restricting the intake of chocolate and coffee, and increasing the intake of fruits and vegetables. Alcohol should be restricted and not limited to two drinks per day. Nonpharmacologic approaches for treating premenstrual syndrome include eating more frequent meals, engaging in aerobic activity, restricting the intake of chocolate and coffee, and increasing the intake of fruits and vegetables. Alcohol should be restricted and not limited to two drinks per day

1. Which findings would indicate the presence of a perineal wound infection? Note: Credit will be given only if all correct choices and no incorrect choices are selected. Select all that apply. A) Redness B) Tender at the margins C) Vaginal bleeding D) Hardened tissue E) Purulent drainage

Answer: A, B, D, E Explanation: A) Redness is a classic sign of a perineal wound infection. B) The wound is typically red, indurated, tender at the margins, and draining purulent exudate. C) Vaginal bleeding is nonspecific to identifying a perineal wound infection. D) The wound is typically red, indurated, tender at the margins, and draining purulent exudate. E) Purulent drainage is a classic sign of a perineal wound infection.

1) The nurse is preparing an educational session for high school female students on self-care during menstruation. What should the nurse include regarding care when using a tampon? Note: Credit will be given only if all correct choices and no incorrect choices are selected. Select all that apply. A) Wash hands before inserting a tampon B) Wash hands after inserting the tampon C) Change the tampon every 8 to 12 hours D) Use tampons with the minimum amount of absorbency E) Avoid touching the part that will be inserted into the vagina

Answer: A, B, D, E Explanation: A) Teaching about the use of tampons should include washing the hands before and after inserting the tampon, using tampons with the least amount of absorbency, and to avoid touching the part of the tampon that will be inserted into the vagina. Tampons should be changed every 3 to 6 hours. B) Teaching about the use of tampons should include washing the hands before and after inserting the tampon, using tampons with the least amount of absorbency, and to avoid touching the part of the tampon that will be inserted into the vagina. Tampons should be changed every 3 to 6 hours. C) Teaching about the use of tampons should include washing the hands before and after inserting the tampon, using tampons with the least amount of absorbency, and to avoid touching the part of the tampon that will be inserted into the vagina. Tampons should be changed every 3 to 6 hours. D) Teaching about the use of tampons should include washing the hands before and after inserting the tampon, using tampons with the least amount of absorbency, and to avoid touching the part of the tampon that will be inserted into the vagina. Tampons should be changed every 3 to 6 hours. E) Teaching about the use of tampons should include washing the hands before and after inserting the tampon, using tampons with the least amount of absorbency, and to avoid touching the part of the tampon that will be inserted into the vagina. Tampons should be changed every 3 to 6 hours.

1) Remedies for back pain in pregnancy that are supported by research evidence and may safely be taught to any pregnant woman by the nurse include which of the following? Note: Credit will be given only if all correct choices and no incorrect choices are selected. Select all that apply. A) Pelvic tilt B) Water aerobics C) Sit-ups D) Proper body mechanics E) Good posture is important because it allows more room for the stomach to function.

Answer: A, B, D, E Explanation: A) The pelvic tilt can help restore proper body alignment and relieve back pain. B) Exercise is an effective treatment for lower back pain. Exercise in water seems to provide benefits while being physically comfortable for expectant mothers. C) Sit-ups require back-lying. Because of the pressure of the enlarging uterus on the vena cava, the woman should not lie flat on her back after about the fourth month. D) The use of proper posture and good body mechanics throughout pregnancy is important. E) Good posture is important because it allows more room for the stomach to function.

1) The nurse is caring for a newborn with jaundice. The parents question why the newborn is not under phototherapy lights. The nurse explains that the fiber-optic blanket is beneficial because of which of the following? Note: Credit will be given only if all correct choices and no incorrect choices are selected. Select all that apply. A) Lights can stay on all the time. B) The eyes do not need to be covered. C) The lights will need to be removed for feedings. D) Newborns do not get overheated. E) Weight loss is not a complication of this system.

Answer: A, B, D, E Explanation: A) With the fiber-optic blanket, the light stays on at all times. B) The eyes do not have to be covered with a fiber optic blanket. C) With the fiber-optic blanket, the light stays on at all times, and the newborn is accessible for care, feeding, and diaper changes. D) With the fiber-optic blanket, greater surface area is exposed and there are no thermoregulation issues. E) Fluid and weight loss are not complications of fiber-optic blankets.

1) In learning about Duvall's life-cycle stages ascribed to traditional families, the nursing student recognizes that developmental tasks of each stage include which of the following? Note: Credit will be given only if all correct choices and no incorrect choices are selected. Select all that apply. A) Adjusting to new roles as mother and father B) Working out authority and socialization roles with the school C) Becoming a single parent with custodial responsibilities D) Becoming a couple and dating E) Adjusting to the loss of a spouse

Answer: A, B, E Explanation: A) Adjusting to new roles as mother and father occurs in Stage Ⅱ, which describes childbearing families with infants. B) Working out authority and socialization roles with schools occurs in Stage Ⅳ, which describes families with school-age children. C) Traditional family life-cycle stages do not include those in which divorce occurs. D) Becoming a couple and dating occurs before marriage, and is not a part of the traditional family life-cycle stages. E) Stage Ⅷ includes adjusting to the loss of a spouse.

1) The client and her partner are carriers of sickle cell disease. They are considering prenatal diagnosis with either amniocentesis or chorionic villus sampling (C V S). Which statements indicate that further teaching is needed on these two diagnostic procedures? Note: Credit will be given only if all correct choices and no incorrect choices are selected. Select all that apply. A) "Chorionic villus sampling carries a lower risk of miscarriage." B) "Amniocentesis can be done earlier in my pregnancy than C V S." C) "Neither test will conclusively diagnose sickle cell disease in our baby." D) "The diagnosis comes sooner if we have C V S, not amniocentesis." E) "Amniocentesis is more accurate in diagnosis than the C V S."

Answer: A, B, E Explanation: A) C V S has a risk of spontaneous abortion of 0.3% in cases. This rate is higher than second trimester amniocentesis. B) C V S is performed in some medical centers for first trimester diagnosis after 9 completed weeks. Amniocentesis is performed between 15 and 20 weeks' gestation. C) This is a true statement. D) C V S is performed in some medical centers for first trimester diagnosis after 9 completed weeks. Amniocentesis is performed between 15 and 20 weeks' gestation. E) Much like amniocentesis, chorionic villus sampling (C V S) is a procedure that is used to detect genetic, metabolic, and D N A abnormalities. C V S permits earlier diagnosis than can be obtained by amniocentesis.

1. Risk factors associated with increased risk of thromboembolic disease include which of the following? Note: Credit will be given only if all correct choices and no incorrect choices are selected. Select all that apply. A) Diabetes mellitus B) Varicose veins C) Hypertension D) Adolescent pregnancy E) Malignancy

Answer: A, B, E Explanation: A) Diabetes mellitus is a risk factor for thromboembolic disease. B) Varicose veins are a risk factor for thromboembolic disease. C) Hypertension is not a risk factor for thromboembolic disease. D) Advanced maternal age, not adolescence, is a risk factor for thromboembolic disease. E) Malignancy is a risk factor for thromboembolic disease.

1) The nurse is teaching the pregnant client about the symptoms of preeclampsia. Which clinical manifestations will the nurse include in the teaching session? Note: Credit will be given only if all correct choices and no incorrect choices are selected. Select all that apply. A) Dizziness B) Blurred vision C) Abdominal pain D) Vaginal bleeding E) Severe headache

Answer: A, B, E Explanation: A) Dizziness is a clinical manifestation associated with preeclampsia. B) Blurred vision is a clinical manifestation associated with preeclampsia. C) Abdominal pain is a clinical manifestation of premature labor or abruptio placentae, not preeclampsia. D) Vaginal bleeding is a clinical manifestation of abruptio placentae or placenta previa, not preeclampsia. E) Severe headache is a clinical manifestation associated with preeclampsia.

6) The client presents to the labor and delivery unit stating that her water broke 2 hours ago. Indicators of normal labor include which of the following? Note: Credit will be given only if all correct choices and no incorrect choices are selected. Select all that apply. A) Fetal heart rate of 130 with average variability B) Blood pressure of 130/80 C) Maternal pulse of 160 D) Protein of +1 in urine E) Odorless, clear fluid on underwear

Answer: A, B, E Explanation: A) Fetal heart rate (F H R) of 110-160 with average variability is a normal indication. B) Maternal vital sign of blood pressure below 140/90 is a normal indication. C) A pulse of 60-100 is a normal indication. D) Proteinuria of +1 or more could be a sign of preeclampsia. E) Fluid clear and without odor if membranes ruptured is a normal indication.

1) The 22-year-old client is scheduled for her first gynecologic examination. What can the nurse do to make the client more comfortable during this exam? Note: Credit will be given only if all correct choices and no incorrect choices are selected. Select all that apply. A) Create a trusting atmosphere. B) Show the client what the speculum looks like. C) Avoid telling the client what the exam involves. D) Ask the client why she has delayed her first Pap test this long. E) Provide a mirror for the client.

Answer: A, B, E Explanation: A) It is important to create a trusting atmosphere and incorporate practices that help the client maintain a sense of control. B) Show the client all of the equipment to be used. C) To reduce fear and improve the client's sense of control, create a trusting atmosphere by explaining everything involved in the exam. D) Asking why the client has delayed her first Pap test is being judgmental, which does not foster a therapeutic relationship. Provide a mirror to increase learning about anatomy and to create a trusting atmosphere

1) Fetal factors that possibly indicate electronic fetal monitoring include which of the following? Note: Credit will be given only if all correct choices and no incorrect choices are selected. Select all that apply. A) Meconium passage B) Multiple gestation C) Preeclampsia D) Grand multiparity E) Decreased fetal movement

Answer: A, B, E Explanation: A) Meconium passage is an indicator for electronic fetal monitoring. B) Multiple gestation is an indicator for electronic fetal monitoring. C) Preeclampsia is a maternal indicator for electronic fetal monitoring. D) Grand multiparity is a maternal indicator for electronic fetal monitoring. E) Decreased fetal movement is an indicator for electronic fetal monitoring.

1) Which of the following are potential disadvantages to breastfeeding? Note: Credit will be given only if all correct choices and no incorrect choices are selected. Select all that apply. A) Pain with breastfeeding B) Leaking milk C) Equal feeding responsibilities with fathers D) Vaginal wetness E) Embarrassment

Answer: A, B, E Explanation: A) Nipple tenderness is the most common source of discomfort and is usually related to improper positioning and/or not obtaining a proper attachment of the infant on the breast. Pain can also be related to engorgement or infection. B) Some women will leak milk when their breasts are full and it is nearly time to breastfeed again or whenever they experience let-down. Mothers should be given reassurance that this problem diminishes over time. C) There are unequal feeding responsibilities and fathers are left out in the first 3 to 4 weeks. The parents should be informed that it is advisable for the father to wait to bottle feed the baby with expressed breast milk until after the milk supply and breastfeeding are established. D) Vaginal dryness is associated with breastfeeding. Some mothers experience vaginal dryness related to a low level of estrogen while lactating. This is only a temporary side effect. E) Some mothers feel uncomfortable about breastfeeding because they are modest or may feel embarrassed because our society views breasts as sexual objects and/or an unfriendly social environment makes it difficult to breastfeed in public. This is not an easy issue to overcome.

1) Ovarian hormones include which of the following? Note: Credit will be given only if all correct choices and no incorrect choices are selected. Select all that apply. A) Estrogens B) Progesterone C) Parathyroid hormone D) Luteinizing hormone E) Testosterone

Answer: A, B, E Explanation: A) Ovarian hormones include the estrogens, progesterone, and testosterone. B) Ovarian hormones include the estrogens, progesterone, and testosterone. C) Ovarian hormones do not include the parathyroid hormone. D) Ovarian hormones do not include the luteinizing hormone, although the ovary is sensitive to it. E) Ovarian hormones include the estrogens, progesterone, and testosterone.

1) Which statements by a breastfeeding class participant indicate that teaching by the nurse was effective? Note: Credit will be given only if all correct choices and no incorrect choices are selected. Select all that apply. A) "Breastfed infants get more skin-to-skin contact and sleep better." B) "Breastfeeding raises the level of a hormone that makes me feel good." C) "Breastfeeding is complex and difficult, and I probably won't succeed." D) "Breastfeeding is worthwhile, even if it costs more overall." E) "Breastfed infants have fewer digestive and respiratory illnesses."

Answer: A, B, E Explanation: A) Skin-to-skin contact after birth helps the baby maintain his or her body temperature, helps with self-regulation, increases maternal oxytocin levels, helps the mother to notice subtle feeding cues, and promotes bonding. B) Hormones of lactation promote maternal feelings and sense of well-being. C) Breastfeeding is a natural process but requires a certain knowledge base. Breastfeeding with proper technique should not hurt and these mothers should be encouraged to seek assistance from a knowledgeable person skilled in lactation. D) Breastfeeding actually costs less than formula-feeding. E) This is a true statement. The immunologic advantages of human milk include varying degrees of protection from respiratory tract and gastrointestinal tract infections.

1) The nurse is administering erythromycin (Ilotycin) ointment to a newborn. What factors are associated with administration of this medication? Note: Credit will be given only if all correct choices and no incorrect choices are selected. Select all that apply. A) The medication should be instilled in the lower conjunctival sac of each eye. B) The eyelids should be massaged gently to distribute the ointment. C) The medication must be given immediately after delivery. D) The medication does not cause any discomfort to the infant. E) The medication can interfere with the baby's ability yo focus on the parents' faces

Answer: A, B, E Explanation: A) Successful eye prophylaxis requires that the medication be instilled in the lower conjunctival sac of each eye. B) After administration, the nurse massages the eyelid gently to distribute the ointment. C) Instillation may be delayed up to 1 hour after birth to allow eye contact during parent-newborn bonding. D) Eye prophylaxis medication can cause chemical conjunctivitis, which gives the newborn some discomfort and may interfere with the baby's ability to focus on the parents' faces and can result in edema, inflammation, and discharge. E) Eye prophylaxis medication can cause chemical conjunctivitis, which gives the newborn some discomfort and can interfere with the baby's ability to focus on the parents' faces.

1) To maintain a healthy temperature in the newborn, which of the following actions should be taken? Note: Credit will be given only if all correct choices and no incorrect choices are selected. Select all that apply. A) Keep the newborn's clothing and bedding dry. B) Reduce the newborn's exposure to drafts. C) Do not use the radiant warmer during procedures. D) Do not wrap the newborn. E) Encourage the mother to snuggle with the newborn under blankets.

Answer: A, B, E Explanation: A) To maintain a healthy temperature in the newborn, keep the newborn's clothing and bedding dry. B) To maintain a healthy temperature in the newborn, reduce the newborn's exposure to drafts. C) To maintain a healthy temperature in the newborn, use the radiant warmer during procedures. D) To maintain a healthy temperature in the newborn, double-wrap the newborn. E) To maintain a healthy temperature in the newborn, encourage the mother to snuggle with the newborn under blankets.

1) During labor, the client at 4 c m suddenly becomes short of breath, cyanotic, and hypoxic. The nurse must prepare or arrange immediately for which of the following? Note: Credit will be given only if all correct choices and no incorrect choices are selected. Select all that apply. A) Intravenous access B) Cesarean delivery C) Immediate vaginal delivery D) McRoberts maneuver E) A crash cart

Answer: A, B, E Explanation: A) When an amniotic fluid embolism is suspected, intravenous access is obtained as quickly as possible. B) Shortness of breath, cyanosis, and hypoxia are symptoms of an amniotic fluid embolus, which necessitates immediate cesarean delivery. C) The client is only 4 c m, so vaginal delivery will not take place immediately. D) McRoberts maneuver is used with shoulder dystocia. E) The chances of a code are high, so the crash cart needs to be available.

1) During the history, the client admits to being H I V-positive and says she knows that she is about 16 weeks pregnant. Which statements made by the client indicate an understanding of the plan of care both during the pregnancy and postpartally? Note: Credit will be given only if all correct choices and no incorrect choices are selected. Select all that apply. A) "During labor and delivery, I can expect the zidovudine (Z D V) to be given in my Ⅳ." B) "After delivery, the dose of zidovudine (Z D V) will be doubled to prevent further infection." C) "My baby will be started on zidovudine (Z D V) for six weeks following the birth." D) "My baby's zidovudine (Z D V) will be given in a cream form." E) "My baby will not need zidovudine (Z D V) if I take it during my pregnancy."

Answer: A, C Explanation: A) A R T therapy generally includes oral Zidovudine (Z D V) daily, Ⅳ Z D V during labor and until birth, and Z D V therapy for the infant for 6 weeks following birth. B) The mother will continue with her oral dosage of zidovudine (Z D V) after delivery just as prior to delivery. C) A R T therapy generally includes oral Zidovudine (Z D V) daily, Ⅳ Z D V during labor and until birth, and Z D V therapy for the infant for 6 weeks following birth. D) The initial treatment is zidovudine (Z D V) orally every day. E) The infant will be started on oral zidovudine (Z D V) after birth for 6 weeks.

1) A couple requests to see their stillborn infant. How should the nurse prepare the infant? Note: Credit will be given only if all correct choices and no incorrect choices are selected. Select all that apply. A) Wrapping the infant in a blanket B) Removing all blankets from the infant C) Placing a hat on the infant D) Removing any identification from the infant E) Placing a diaper on the infant is not necessary.

Answer: A, C Explanation: A) After bathing, the infant should be placed in a suitable-sized gown and then wrapped in a blanket. Many parents will eventually remove the covering to inspect the infant; however, applying a covering allows them time to adjust to the appearance at their own pace. B) Removing blankets from the infant would not be appropriate. C) A hat can be applied to cover birth defects. This allows the parents an opportunity to view the infant before seeing the birth defect. D) Removing any identification from the infant would not be appropriate.

3) The pregnant client states she does not want "to take all these supplements." What recommendations could the nurse make for the client? Note: Credit will be given only if all correct choices and no incorrect choices are selected. Select all that apply. A) "Folic acid has been found to be essential for minimizing the risk of neural tube defects." B) "You do not have to take these supplements if you think you are healthy enough." C) "Most women do not have adequate intake of iron pre-pregnancy, and iron needs increase with pregnancy." D) "These medications do the same thing. I will call your physician to cancel one of your medications." E) "You should take the folic acid, but the vitamins are not that important."

Answer: A, C Explanation: A) An inadequate intake of folic acid has been associated with neural tube defects (N T Ds) (e.g., spina bifida, anencephaly). B) This answer is incorrect because prenatal vitamins with iron and folic acid are necessary. C) Iron is essential because many pregnant women do not have adequate intake of iron before pregnancy. D) It is not the nurse's role to tell the physician to cancel any prescribed medication. E) Both folic acid and other vitamins and minerals are necessary for a successful pregnancy and a healthy baby.

1. A client is experiencing excessive bleeding immediately after the birth of her newborn. After speeding up the I V fluids containing oxytocin, with no noticeable decrease in the bleeding, the nurse should anticipate the physician requesting which medications? Note: Credit will be given only if all correct choices and no incorrect choices are selected. Select all that apply. A) Methergine B) Coumadin C) Misoprostol D) Serotonin reuptake inhibitors (S S R Is) E) Nonsteroidal anti-inflammatory drugs

Answer: A, C Explanation: A) Methergine is commonly used orally for postpartum hemorrhage. B) Coumadin (warfarin) is an anticoagulant and is not used for postpartum hemorrhage. C) Misoprostol is commonly used rectally for postpartum hemorrhage. D) Serotonin reuptake inhibitors (S S R Is) are antidepressants and would not be used for postpartum hemorrhage. E) Nonsteroidal anti-inflammatory drugs increase anticoagulant activity and would not be used for postpartum hemorrhage.

1) The client at 34 weeks' gestation has been stabbed in the low abdomen by her boyfriend. She is brought to the emergency department for treatment. Which statements indicate that the client understands the treatment being administered? Note: Credit will be given only if all correct choices and no incorrect choices are selected. Select all that apply. A) "The baby needs to be monitored to check the heart rate." B) "My bowel has probably been lacerated by the knife." C) "I might need an ultrasound to look at the baby." D) "The catheter in my bladder will prevent urinary complications." E) "The Ⅳ in my arm will replace the amniotic fluid if it is leaking."

Answer: A, C Explanation: A) Ongoing assessments of trauma include evaluation of uterine tone, contractions and tenderness, fundal height, fetal heart rate, intake and output and other indicators of shock, normal postoperative evaluation in those women requiring surgery, determination of neurologic status, and assessment of mental outlook and anxiety level. B) The pregnancy usually sustains the majority of the damage, sparing the bowel from injury. C) In cases of noncatastrophic trauma, where the mother's life is not directly threatened, fetal monitoring for 4 hours should be sufficient if there is no vaginal bleeding, uterine tenderness, contractions, or leaking amniotic fluid. D) The Foley catheter is placed to assess for hematuria. E) The Ⅳ will replace intravascular volume, not amniotic fluid.

1) The nurse is at the home of a postpartum client for an initial assessment. The client gave birth by cesarean section 1 week earlier. Which statements should the nurse include? Note: Credit will be given only if all correct choices and no incorrect choices are selected. Select all that apply. A) "Because you had a cesarean, I'd like to assess your incision." B) "You aren't having any problems nursing, right?" C) "How rested do you feel since you came home?" D) "Because you are bottle-feeding, I won't assess your breasts." E) "You should remain at home for the first 3 weeks after delivery."

Answer: A, C Explanation: A) The nurse should assess the cesarean incision. B) Therapeutic communication prohibits asking leading questions. C) The nurse should talk with the mother about her fatigue level and ability to rest and sleep. D) Breasts should be assessed for engorgement even for bottle-feeding mothers. E) The nurse should talk with the mother about her activity level and self-care abilities.

1) What are the primary complications of placenta accreta? Note: Credit will be given only if all correct choices and no incorrect choices are selected. Select all that apply. A) Maternal hemorrhage B) Insomnia C) Failure of the placenta to separate following birth of the infant D) Autonomic dysreflexia E) Shoulder dystocia

Answer: A, C Explanation: A) The primary complications of placenta accreta are maternal hemorrhage and failure of the placenta to separate following birth of the infant. B) Insomnia is not a complication of placenta accreta. C) The primary complications of placenta accreta are maternal hemorrhage and failure of the placenta to separate following birth of the infant. D) Autonomic dysreflexia is a rare complication that can occur in women with a spinal cord injury. E) The most significant complication in macrosomia is shoulder dystocia.

1) Before performing Leopold maneuvers, what would the nurse do? Note: Credit will be given only if all correct choices and no incorrect choices are selected. Select all that apply. A) Have the client empty her bladder. B) Place the client in Trendelenburg position. C) Have the client lie on her back with her feet on the bed and knees bent. D) Turn the client to her left side. E) This is not the optimal position for the client when performing Leopold maneuvers.

Answer: A, C Explanation: A) The woman should have recently emptied her bladder before performing Leopold maneuvers. B) Placing the client in Trendelenburg position is not consistent with accurately performing Leopold maneuvers. C) The woman should lie on her back with her abdomen uncovered. To aid in relaxation of the abdominal wall, the shoulders should be raised slightly on a pillow and the knees drawn up a little. D) Placing the client on her left side is not consistent with accurately performing Leopold maneuvers.

A client is being prepared to take the oral mifepristone-vaginal misoprostol treatment for an abortion. For which reasons should the nurse instruct the client to contact the healthcare provider within 24 hours? Note: Credit will be given only if all correct choices and no incorrect choices are selected. Select all that apply. A) Nausea B) Itchy skin C) Weakness D) Stomach pain E) Bloody discharge

Answer: A, C, D Explanation: A) A woman who has taken the oral mifepristone-vaginal misoprostol regimen and within 24 hours develops nausea, weakness, or stomach pain should contact the healthcare provider immediately. Itchy skin and bloody discharge are not identified as needing to be reported to the healthcare provider. B) A woman who has taken the oral mifepristone-vaginal misoprostol regimen and within 24 hours develops nausea, weakness, or stomach pain should contact the healthcare provider immediately. Itchy skin and bloody discharge are not identified as needing to be reported to the healthcare provider. C) A woman who has taken the oral mifepristone-vaginal misoprostol regimen and within 24 hours develops nausea, weakness, or stomach pain should contact the healthcare provider immediately. Itchy skin and bloody discharge are not identified as needing to be reported to the healthcare provider. D) A woman who has taken the oral mifepristone-vaginal misoprostol regimen and within 24 hours develops nausea, weakness, or stomach pain should contact the healthcare provider immediately. Itchy skin and bloody discharge are not identified as needing to be reported to the healthcare provider. E) A woman who has taken the oral mifepristone-vaginal misoprostol regimen and within 24 hours develops nausea, weakness, or stomach pain should contact the healthcare provider immediately. Itchy skin and bloody discharge are not identified as needing to be reported to the healthcare provider.

1) The nurse encourages a new mother to feed the newborn as soon as the newborn shows interest. The nurse bases this recommendation on which benefits of early feedings? Note: Credit will be given only if all correct choices and no incorrect choices are selected. Select all that apply. A) Early feedings stimulate peristalsis. B) Colostrum is thinner than mature milk. C) Early feedings enhance maternal-infant bonding. D) Early feedings promote the passage of meconium. Colostrum contains a high number of calories

Answer: A, C, D Explanation: A) Early breastfeeding stimulates the newborn's peristalsis. B) Colostrum is a thick, creamy, yellowish fluid with concentrated amounts of protein, fat-soluble vitamins, and minerals. C) Early breastfeeding enhances maternal-infant bonding. D) Early breastfeeding promotes the passage of meconium. E) Colostrum contains fewer calories than mature milk does.

1) A postpartum mother questions whether the environmental temperature should be warmer in the baby's room at home. The nurse responds that the environmental temperature should be warmer for the newborn. This response is based on which newborn characteristics that affect the establishment of thermal stability? Note: Credit will be given only if all correct choices and no incorrect choices are selected. Select all that apply. A) Newborns have less subcutaneous fat than do adults. B) Infants have a thick epidermis layer. C) Newborns have a large body surface to weight ratio. D) Infants have increased total body water. Newborns have more subcutaneous fat than do adults

Answer: A, C, D Explanation: A) Heat transfer from neonatal organs to skin surface is increased compared to adults due to the neonate's decreased subcutaneous fat. B) Preterm infants have increased heat loss via evaporation due to thin skin. C) Heat transfer from neonatal organs to skin surface is increased compared to adults due to the neonate's large body surface to weight ratio. D) Preterm infants have increased heat loss via evaporation due to increased total body water. E) Newborns do not have more subcutaneous fat than adults.

1) Which statements might a pregnant teenager be likely to make at her initial prenatal examination? Note: Credit will be given only if all correct choices and no incorrect choices are selected. Select all that apply. A) "I didn't know I could get pregnant the first time I had sex." B) "Several of my friends go to clinics to get contraception." C) "It's no big deal; two of my best friends have babies, too." D) "I was 13 years old when I had sex the first time." E) "My family and my boyfriend are really happy and supportive."

Answer: A, C, D Explanation: A) Many factors contribute to the high teenage pregnancy rate, and one of them is lack of knowledge about conception. B) A teen whose friends go to clinics for contraception would be more likely to use contraception herself. C) Many factors contribute to the high teenage pregnancy rate, and one of them is a decreased social stigma of being a young and single mother. D) Many factors contribute to the high teenage pregnancy rate, and one of them is a younger age at onset of sexual activity. E) Lack of support from the partner and parents is the norm, especially when the pregnancy is first diagnosed and disclosed.

9) Postpartum nutritional status is determined primarily by assessing which of the following? Note: Credit will be given only if all correct choices and no incorrect choices are selected. Select all that apply. A) Dietary history B) Menstrual history C) Mother's weight D) Hemoglobin levels E) Mother's height

Answer: A, C, D Explanation: A) Postpartum nutritional status is determined by assessing the new mother's dietary history. B) Postpartum nutritional status is not determined by assessing the new mother's menstrual history. C) Postpartum nutritional status is determined by assessing the new mother's weight. D) Postpartum nutritional status is determined by assessing the new mother's hemoglobin levels. E) Postpartum nutritional status is not determined by assessing the new mother's height.

The nurse suspects that a client is experiencing adverse effects from the progestin within a combined oral contraceptive. What did the nurse assess to make this clinical determination? Note: Credit will be given only if all correct choices and no incorrect choices are selected. Select all that apply. A) Pruritus B) Headache C) Hirsutism D) Weight gain E) Hypertension

Answer: A, C, D Explanation: A) Pruritus, hirsutism (facial hair), and weight gain are adverse effects of progestin within a combined oral contraceptive. Headache and hypertension are adverse effects of estrogen within a combined oral contraceptive. B) Pruritus, hirsutism (facial hair), and weight gain are adverse effects of progestin within a combined oral contraceptive. Headache and hypertension are adverse effects of estrogen within a combined oral contraceptive. C) Pruritus, hirsutism (facial hair), and weight gain are adverse effects of progestin within a combined oral contraceptive. Headache and hypertension are adverse effects of estrogen within a combined oral contraceptive. D) Pruritus, hirsutism (facial hair), and weight gain are adverse effects of progestin within a combined oral contraceptive. Headache and hypertension are adverse effects of estrogen within a combined oral contraceptive. E) Pruritus, hirsutism (facial hair), and weight gain are adverse effects of progestin within a combined oral contraceptive. Headache and hypertension are adverse effects of estrogen within a combined oral contraceptive.

1) The nurse is assessing a client in the third trimester of pregnancy. What physiologic changes in the client are expected? Note: Credit will be given only if all correct choices and no incorrect choices are selected. Select all that apply. A) The client's chest circumference has increased by 6 cm during the pregnancy. B) The client has a narrowed subcostal angle. C) The client is using thoracic breathing. D) The client may have epistaxis. E) The client has a productive cough.

Answer: A, C, D Explanation: A) The chest increase compensates for the elevated diaphragm. B) The diaphragm is elevated and the subcostal angle is increased as a result of pressure from the enlarging uterus. C) Breathing changes from abdominal to thoracic as pregnancy progresses. D) Epistaxis (nosebleeds) may occur and are primarily the result of estrogen-induced edema and vascular congestion of the nasal mucosa. E) A productive cough is never a normal finding.

1) The nurse is reviewing the spermicidal agent nonoxynol-9 (N-9) with a client planning to use the barrier method to prevent pregnancy. What should the nurse emphasize when teaching about this preparation? Note: Credit will be given only if all correct choices and no incorrect choices are selected. Select all that apply. A) It does not cause toxicity. B) It is inserted after intercourse. C) It has no systemic side effects. D) It can be purchased over-the-counter. E) It reduces the risk of sexually transmitted infections.

Answer: A, C, D Explanation: A) The major advantages of spermicidal preparations include low local toxicity, lack of systemic side effects, and ease of obtaining through an over-the-counter purchase. Spermicides are inserted before intercourse. Nonoxyol-9 does not offer protection against the organisms that cause gonorrhea, chlamydia, or H I V/A I D S and may actually increase a woman's risk of H I V infection. B) The major advantages of spermicidal preparations include low local toxicity, lack of systemic side effects, and ease of obtaining through an over-the-counter purchase. Spermicides are inserted before intercourse. Nonoxyol-9 does not offer protection against the organisms that cause gonorrhea, chlamydia, or H I V/A I D S and may actually increase a woman's risk of H I V infection. C) The major advantages of spermicidal preparations include low local toxicity, lack of systemic side effects, and ease of obtaining through an over-the-counter purchase. Spermicides are inserted before intercourse. Nonoxyol-9 does not offer protection against the organisms that cause gonorrhea, chlamydia, or H I V/A I D S and may actually increase a woman's risk of H I V infection. D) The major advantages of spermicidal preparations include low local toxicity, lack of systemic side effects, and ease of obtaining through an over-the-counter purchase. Spermicides are inserted before intercourse. Nonoxyol-9 does not offer protection against the organisms that cause gonorrhea, chlamydia, or H I V/A I D S and may actually increase a woman's risk of H I V infection. E) The major advantages of spermicidal preparations include low local toxicity, lack of systemic side effects, and ease of obtaining through an over-the-counter purchase. Spermicides are inserted before intercourse. Nonoxyol-9 does not offer protection against the organisms that cause gonorrhea, chlamydia, or H I V/A I D S and may actually increase a woman's risk of H I V infection.

1) The nurse initiates newborn admission procedures and evaluates the newborn's need to remain under observation by assessing which of the following? Note: Credit will be given only if all correct choices and no incorrect choices are selected. Select all that apply. A) Respiratory rate B) Skin texture C) Airway clearance D) Ability to feed E) Head weight

Answer: A, C, D Explanation: A) The nurse initiates newborn admission procedures and evaluates the newborn's need to remain under observation by assessing vital signs (body temperature, heart rate, respiratory rate). B) The nurse initiates newborn admission procedures and evaluates the newborn's need to remain under observation by assessing skin color, not skin texture. C) The nurse initiates newborn admission procedures and evaluates the newborn's need to remain under observation by assessing airway clearance. D) The nurse initiates newborn admission procedures and evaluates the newborn's need to remain under observation by assessing ability to feed. E) The nurse initiates newborn admission procedures and evaluates the newborn's need to remain under observation by assessing circumference and body weight, not head weight.

1) The nurse is teaching the parents of a newborn who has been exposed to H I V how to care for the newborn at home. Which instructions should the nurse emphasize? Note: Credit will be given only if all correct choices and no incorrect choices are selected. Select all that apply. A) Use proper hand-washing technique. B) Provide three feedings per day. C) Place soiled diapers in a sealed plastic bag. D) Cleanse the diaper changing area with a 1:10 bleach solution after each diaper change. E) Take the temperature rectally.

Answer: A, C, D Explanation: A) The nurse should instruct the parents on proper hand-washing technique. B) Small, frequent meals are recommended. C) The nurse should instruct parents to that soiled diapers are to be placed in plastic bags, sealed, and disposed of daily. D) The nurse should instruct parents that the diaper-changing areas should be cleaned with a 1:10 dilution of household bleach after each diaper change. E) Taking rectal temperatures is to be avoided because it could stimulate diarrhea.

1) A nurse is instructing nursing students about the procedure for vitamin K administration. What information should be included? Note: Credit will be given only if all correct choices and no incorrect choices are selected. Select all that apply. A) Gently massage the site after injection. B) Use a 22-gauge, 1-inch needle. C) Inject in the vastus lateralis muscle. D) Cleanse the site with alcohol prior to injection. E) Inject at a 45-degree angle.

Answer: A, C, D Explanation: A) The nurse would remove the needle and massage the site with an alcohol swab. B) Vitamin K is given I M using a 25-gauge, 5/8-inch needle. C) Vitamin K is given intramuscularly in the vastus lateralis muscle. D) Before injecting, the nurse must thoroughly clean the newborn's skin site for the injection with a small alcohol swab. E) Vitamin K is given I M at a 90-degree angle.

1) The nurse is conducting an initial prenatal assessment for a pregnant client. Which screenings should the nurse prepare the client for during this visit? Note: Credit will be given only if all correct choices and no incorrect choices are selected. Select all that apply. A) Complete blood count (CBC) B) Glucose tolerance test (GT T) C) A B O and Rh typing D) H I V screening E) Urinalysis

Answer: A, C, D, E Explanation: A) A C B C is drawn during the initial prenatal visit. B) A G T T is not done until the second trimester of the pregnancy. C) A B O and Rh typing are drawn during the initial prenatal visit. D) An H I V screening is drawn during the initial prenatal visit. E) A urinalysis is conducted during the initial prenatal visit and for every subsequent prenatal visit.

1) Which of the following are important behaviors to assess in the neurologic assessment? Note: Credit will be given only if all correct choices and no incorrect choices are selected. Select all that apply. A) State of alertness B) Active posture C) Quality of muscle tone D) Cry E) Motor activity

Answer: A, C, D, E Explanation: A) Important behaviors to assess are the state of alertness, resting posture, cry, and quality of muscle tone and motor activity. B) Resting posture is assessed, not active posture. C) Important behaviors to assess are the state of alertness, resting posture, cry, and quality of muscle tone and motor activity. D) Important behaviors to assess are the state of alertness, resting posture, cry, and quality of muscle tone and motor activity. E) Important behaviors to assess are the state of alertness, resting posture, cry, and quality of muscle tone and motor activity.

1) The nurse is caring for a client and her spouse during the third stage of labor. Which action(s) support initial parental-newborn attachment at this time? Note: Credit will be given only if all correct choices and no incorrect choices are selected. Select all that apply. A) Minimizing assessments B) Delaying ophthalmic antibiotics for 2 hours C) Dimming the room lights D) Talking quietly E) Providing privacy

Answer: A, C, D, E Explanation: A) Minimizing assessments enhances parental newborn attachment during this time. B) Ophthalmic antibiotics may be delayed during the first hour, but not up to 2 hours. C) Dimming the room lights enhances parental-newborn attachment during this time. D) Talking quietly enhances parental-newborn attachment during this time. E) Providing privacy enhances parental-newborn attachment during this time.

1) The incidence of complications and discomforts in the first year postpartum is common and women may experience which of the following? Note: Credit will be given only if all correct choices and no incorrect choices are selected. Select all that apply. A) Pain B) Excess energy C) Urinary incontinence D) Changes in mental health status E) Sleep deprivation

Answer: A, C, D, E Explanation: A) Pain can be a discomfort in the first year postpartum. B) Fatigue, not excess energy, can be a discomfort in the first year postpartum. C) Urinary incontinence can be a complication in the first year postpartum. D) Changes in mental health status can be a complication in the first year postpartum. E) Sleep deprivation can be a complication in the first year postpartum.

1) The nurse should inform the parents of a newborn that they should call their healthcare provider when which of the following occurs? Note: Credit will be given only if all correct choices and no incorrect choices are selected. Select all that apply. A) Continual rise in temperature B) Decreased frequency of stools C) Absence of breathing longer than 20 seconds D) Lethargy E) Refusal of two feedings in a row

Answer: A, C, D, E Explanation: A) Parents should call their healthcare provider due to a continual rise in temperature. B) Parents should call their healthcare provider when there are two consecutive green watery or black stools or increased frequency of stools. C) Parents should call their healthcare provider in the absence of breathing longer than 20 seconds. D) Parents should call their healthcare provider if the newborn exhibits lethargy and listlessness. E) Parents should call their healthcare provider if the newborn has refused two feedings in a row.

1) When general anesthesia is necessary for a cesarean delivery, what should the nurse be prepared to do? Note: Credit will be given only if all correct choices and no incorrect choices are selected. Select all that apply. A) Administer an antacid to the client. B) Place a wedge under the client's thigh. C) Apply cricoid pressure to the client during anesthesia intubation. D) Preoxygenate the client for 3-5 minutes before anesthesia. Place a Foley catheter in the client's bladder

Answer: A, C, D, E Explanation: A) Prophylactic antacid therapy is given to reduce the acidic content of the stomach before general anesthesia. B) Before the induction to anesthesia, the woman should have a wedge placed under her hip to displace the uterus and avoid vena caval compression in the supine position. C) During the process of rapid induction of anesthesia, the nurse applies cricoid pressure. D) The woman should be preoxygenated with 3 to 5 minutes of 100% oxygen. E) Urinary retention can be treated with the placement of an indwelling Foley catheter.

1) During an assessment, the nurse determines that a female patient is at risk for developing osteoporosis. Which information did the nurse use to make this clinical determination? Note: Credit will be given only if all correct choices and no incorrect choices are selected. Select all that apply. A) Body weight of 120 lb B) Plays tennis twice a week C) Smokes 2 packs per day of cigarettes D) Ingests 2 to 3 cocktails every day E) Mother diagnosed with osteoporosis

Answer: A, C, D, E Explanation: A) Risk factors for the development of osteoporosis include body weight less than 127 lbs., smoking cigarettes, ingestion of alcohol, and having a family history of osteoporosis. An active lifestyle is an action to reduce the risk of developing osteoporosis. B) Risk factors for the development of osteoporosis include body weight less than 127 lbs., smoking cigarettes, ingestion of alcohol, and having a family history of osteoporosis. An active lifestyle is an action to reduce the risk of developing osteoporosis. C) Risk factors for the development of osteoporosis include body weight less than 127 lbs., smoking cigarettes, ingestion of alcohol, and having a family history of osteoporosis. An active lifestyle is an action to reduce the risk of developing osteoporosis. D) Risk factors for the development of osteoporosis include body weight less than 127 lbs., smoking cigarettes, ingestion of alcohol, and having a family history of osteoporosis. An active lifestyle is an action to reduce the risk of developing osteoporosis. Risk factors for the development of osteoporosis include body weight less than 127 lbs., smoking cigarettes, ingestion of alcohol, and having a family history of osteoporosis. An active lifestyle is an action to reduce the risk of developing osteoporosis

1) If a woman returns to an abusive situation, the nurse should encourage her to develop an exit, or safety, plan for herself and her children, if she has any. What should the plan include? Note: Credit will be given only if all correct choices and no incorrect choices are selected. Select all that apply. A) Identify friends and family who know about the situation and will help her. B) Call the police if violence begins. C) Pack a change of clothes for herself and the children. D) Have a plan for where she will go. E) Have a planned escape route.

Answer: A, C, D, E Explanation: A) She should identify friends and family who know about the situation and will help her. Ask that she establish a code word for danger with those family and friends. B) She should ask a neighbor to call the police if violence begins. C) She should pack a change of clothes for herself and the children, including toilet articles and an extra set of car and house keys stored away from her house with a friend or neighbor. D) She should have a plan for where she will go, regardless of the day or time. E) She should have a planned escape route and emergency telephone numbers she can call.

1) The nurse is caring for a client who had a cesarean birth 4 hours ago. Which interventions would the nurse implement at this time? Note: Credit will be given only if all correct choices and no incorrect choices are selected. Select all that apply. A) Administer analgesics as needed. B) Encourage the client to ambulate to the bathroom to void. C) Encourage leg exercises every 2 hours. D) Encourage the client to cough and deep-breathe every 2 to 4 hours. E) Encourage the use of breathing, relaxation, and distraction.

Answer: A, C, D, E Explanation: A) The nurse continues to assess the woman's pain level and provide relief measures as needed. B) Ambulation should begin no later than 24 hours postoperatively and should be encouraged at least 2 to 3 times a day, but not in the first 4 hours. C) Within the first 12 hours postoperatively, unless medically contraindicated, the woman should be assisted to dangle her legs off the side of the bed. D) The woman is encouraged to cough and breathe deeply and to use incentive spirometry every 2 to 4 hours while awake for the first few days following cesarean birth. The nurse should encourage the use of breathing, relaxation, and distraction techniques

1) The nurse understands that a client's pregnancy is progressing normally when what physiologic changes are documented on the prenatal record of a woman at 36 weeks' gestation? Note: Credit will be given only if all correct choices and no incorrect choices are selected. Select all that apply. A) The joints of the pelvis have relaxed, causing a waddling gait. B) The cervix is firm and blue-purple in color. C) The uterus vasculature contains one sixth of the total maternal blood volume. D) Gastric emptying time is delayed, and the client complains of constipation and bloating. Supine hypotension

Answer: A, C, D, E Explanation: A) The sacroiliac, sacrococcygeal, and pubic joints of the pelvis relax in the later part of the pregnancy, presumably as a result of hormonal changes. This often causes a waddling gait. B) Cervical changes during pregnancy include softening and blue-purple discoloration. C) By the end of pregnancy, one sixth of the total maternal blood volume is contained within the vascular system of the uterus. D) Gastric emptying time and intestinal motility are delayed, leading to frequent complaints of bloating and constipation, which can be aggravated by the smooth muscle relaxation and increased electrolyte and water reabsorption in the large intestine. E) The enlarging uterus may exert pressure on the vena cava when the woman lies supine, causing a drop in blood pressure. This is called the vena caval syndrome, or supine hypotension.

The nurse manager is consulting with a certified nurse-midwife about a client. What is the role of the C N M? Note: Credit will be given only if all correct choices and no incorrect choices are selected. Select all that apply. A) Be prepared to manage independently the care of women at low risk for complications during pregnancy and birth. B) Give primary care for high-risk clients who are in hospital settings. C) Give primary care for healthy newborns. D) Obtain a physician consultation for any technical procedures at delivery. E) Be educated in two disciplines of nursing.

Answer: A, C, E Explanation: A) A C N M is prepared to manage independently the care of women at low risk for complications during pregnancy and birth and the care of healthy newborns. B) CNMs cannot give primary care for high-risk clients who are in hospital settings. The physician provides the primary care. C) A C N M is prepared to manage independently the care of women at low risk for complications during pregnancy and birth and the care of healthy newborns. D) The C N M does not need to obtain a physician consultation for any technical procedures at delivery. E) The C N M is educated in the disciplines of nursing and midwifery.

1) Among women who have been sexually assaulted, which of the following are the most frequently diagnosed sexually transmitted infections (S T Is)? Note: Credit will be given only if all correct choices and no incorrect choices are selected. Select all that apply. A) Bacterial vaginosis B) H I V C) Chlamydia D) Syphilis E) Gonorrhea

Answer: A, C, E Explanation: A) Among women who have been sexually assaulted, trichomoniasis, bacterial vaginosis, gonorrhea, and chlamydia are the most frequently diagnosed sexually transmitted infections (S T Is). B) H I V is not one of the most frequently diagnosed S T Is following a sexual assault. C) Among women who have been sexually assaulted, trichomoniasis, bacterial vaginosis, gonorrhea, and chlamydia are the most frequently diagnosed sexually transmitted infections (S T Is). D) Syphilis is not one of the most frequently diagnosed S T Is following a sexual assault. E) Among women who have been sexually assaulted, trichomoniasis, bacterial vaginosis, gonorrhea, and chlamydia are the most frequently diagnosed sexually transmitted infections (S T Is).

1) When a woman seeks care for an injury, the nurse should be alert to which clues of abuse? Note: Credit will be given only if all correct choices and no incorrect choices are selected. Select all that apply. A) Defensive injuries B) Immediate reporting of symptoms or seeking care for injuries C) Lack of eye contact D) Providing too much detailed information about the injury E) Vague complaints without accompanying pathology

Answer: A, C, E Explanation: A) Defensive injuries may be a sign of abuse. B) Delayed reporting of symptoms or seeking care for injuries may be a sign of abuse, not immediate reporting and seeking care. C) Lack of eye contact may be a sign of abuse. D) Hesitation in providing detailed information about the injury and how it occurred may be a sign of abuse. E) Vague complaints without accompanying pathology may be a sign of abuse.

1) Student nurses in their obstetrical rotation are learning about fertilization and implantation. The process of implantation is characterized by which statements? Note: Credit will be given only if all correct choices and no incorrect choices are selected. Select all that apply. A) The trophoblast attaches itself to the surface of the endometrium. B) The most frequent site of attachment is the lower part of the anterior uterine wall. C) Between days 7 and 10 after fertilization, the zona pellucida disappears, and the blastocyst implants itself by burrowing into the uterine lining. D) The lining of the uterus thins below the implanted blastocyst. E) The cells of the trophoblast grow down into the uterine lining, forming the chorionic villi.

Answer: A, C, E Explanation: A) During implantation, the trophoblast attaches itself to the surface of the endometrium for further nourishment. B) The most frequent site of attachment is the upper part of the posterior uterine wall. C) Between days 7 and 10 after fertilization, the zona pellucida disappears, and the blastocyst implants itself by burrowing into the uterine lining and penetrating down toward the maternal capillaries until it is completely covered. D) The lining of the uterus thickens, not thins. E) The cells of the trophoblast grow down into the thickened lining, forming the chorionic villi.

The nurse is serving on a panel to evaluate the hospital staff's reliance on evidence-based practice in their decision-making processes. Which practices characterize the basic competencies related to evidence-based practice? Note: Credit will be given only if all correct choices and no incorrect choices are selected. Select all that apply. A) Recognizing which clinical practices are supported by good evidence B) Recognizing and including clinical practice supported by intuitive evidence C) Using data in clinical work to evaluate outcomes of care D) Including quality-improvement measures in clinical practice E) Appraising and integrating scientific bases into practice

Answer: A, C, E Explanation: A) Recognizing which clinical practices are supported by sound evidence is a basic competency related to evidence-based practice. B) Including clinical practice supported by intuitive evidence is not a basic competency related to evidence-based practice. C) Using data in clinical work to evaluate outcomes of care is one of the basic competencies related to evidence-based practice. D) Including quality-improvement measures is a form of evidence that can be useful in making clinical practice decisions, but it is not a basic competency related to evidence-based practice. E) Appraising and integrating scientific bases into practice is one of the characteristics of the basic competencies related to evidence-based practice.

26) Women with eating disorders who become pregnant are at risk for a variety of complications including which of the following? Note: Credit will be given only if all correct choices and no incorrect choices are selected. Select all that apply. A) Premature birth B) Too many nutrients available for the fetus C) Miscarriage D) High birth weight E) Perinatal mortality

Answer: A, C, E Explanation: A) Risks to the mother and baby include premature birth. B) Risks to the mother and baby include lack of nutrients available for the fetus. C) Risks to the mother and baby include miscarriage. D) Risks to the mother and baby include low birth weight. E) Risks to the mother and baby include perinatal mortality.

1) The nurse is teaching experienced postpartum nurses about home care visits. Which statements indicate that teaching was effective? Note: Credit will be given only if all correct choices and no incorrect choices are selected. Select all that apply. A) "I should tell the family to put any guns or knives away." B) "It is best to blend in with the community and not bring attention to myself on visits." C) "If I encounter a crime in progress, I should leave the area." D) "Wearing jewelry is a good way to demonstrate my professionalism." E) "Ignoring my 'gut' feelings might lead to an unsafe situation."

Answer: A, C, E Explanation: A) The nurse should leave the home immediately if a weapon is visible and the patient or a family member refuses requests to put it away. B) The nurse should wear a name tag and carry identification. C) Nurses should avoid entering areas where violence is in progress. In such cases, they should return to the car and contact the appropriate authorities by calling 911. D) The nurse should avoid wearing expensive jewelry. E) The nurse should terminate the visit if a situation arises that feels unsafe, or if the previous requests are not honored.

1) A new mother is concerned about a mass on the newborn's head. The nurse assesses this to be a cephalohematoma based on which characteristics? Note: Credit will be given only if all correct choices and no incorrect choices are selected. Select all that apply. A) The mass appeared on the second day after birth. B) The mass appears larger when the newborn cries. C) The head appears asymmetrical. D) The mass appears on only one side of the head. E) The mass overrides the suture line.

Answer: A, D Explanation: A) A cephalohematoma is a collection of blood resulting from ruptured blood vessels between the surface of a cranial bone and the periosteal membrane. These areas emerge as defined hematomas between the first and second days. B) A cephalohematoma does not increase in size when the newborn cries. C) Molding causes the head to appear asymmetrical because of the overriding of cranial bones during labor and birth. D) Cephalohematomas can be unilateral or bilateral, but do not cross the suture lines. E) Cephalohematomas can be unilateral or bilateral, but do not cross the suture lines.

1. A clinic nurse is preparing diagrams of pelvic shapes. Which pelvic shapes are considered least adequate for vaginal childbirth? Note: Credit will be given only if all correct choices and no incorrect choices are selected. Select all that apply. A) Android B) Anthropoid C) Gynecoid D) Platypelloid E) Lambdoidal suture

Answer: A, D Explanation: A) In the android and platypelloid types, the pelvic diameters are diminished. Labor is more likely to be difficult (longer) and a cesarean birth is more likely. B) The anthropoid pelvis type is considered favorable for vaginal childbirth. C) The gynecoid pelvis type is considered favorable for vaginal childbirth. D) In the android and platypelloid types, the pelvic diameters are diminished. Labor is more likely to be difficult (longer) and a cesarean birth is more likely. E) This is not a pelvis type.

1) The nurse is caring for a postpartum client who is experiencing afterpains following the birth of her third child. Which comfort measure should the nurse implement to decrease her pain? Note: Credit will be given only if all correct choices and no incorrect choices are selected. Select all that apply. A) Offer a warm water bottle for her abdomen. B) Call the physician to report this finding. C) Inform her that this is not normal, and she will need an oxytocic agent. D) Administer a mild analgesic to help with breastfeeding. E) Administer a mild analgesic at bedtime to ensure rest.

Answer: A, D, E Explanation: A) A warm water bottle placed against the low abdomen may reduce the discomfort of afterpains. B) It is not necessary to report this finding to the physician. C) Afterpains are not abnormal in multiparas. Oxytocic agents stimulate uterine contraction and increase the discomfort of the afterpains. D) The breastfeeding mother may find it helpful to take a mild analgesic agent approximately 1 hour before feeding her infant. An analgesic agent such as ibuprofen is also helpful at bedtime if the afterpains interfere with the mother's rest

1) The nurse knows that the Bishop scoring system for cervical readiness includes which of the following? Note: Credit will be given only if all correct choices and no incorrect choices are selected. Select all that apply. A) Fetal station B) Fetal lie C) Fetal presenting part D) Cervical effacement E) Cervical softness

Answer: A, D, E Explanation: A) Fetal station is one of the components evaluated by the Bishop scoring system. B) Fetal lie is not one of the components evaluated by the Bishop scoring system. C) The presenting part is not one of the components evaluated by the Bishop scoring system. D) Cervical effacement is one of the components evaluated by the Bishop scoring system. E) Cervical consistency is one of the components evaluated by the Bishop scoring system.

1) The nurse's goals for prenatal classes may include which of the following? Note: Credit will be given only if all correct choices and no incorrect choices are selected. Select all that apply. A) Increasing self-esteem B) Preparing the participants for childrearing C) Offering information on the risks of breastfeeding D) Providing anticipatory guidance about pregnancy E) Helping participants develop more adaptive coping skills

Answer: A, D, E Explanation: A) Goals for prenatal classes would include increasing self-esteem. B) The nurse would be preparing the participants for labor and birth, not childrearing. C) Goals for prenatal classes would include information on the benefits of breastfeeding. D) Goals for prenatal class would include providing anticipatory guidance about pregnancy. E) Goals for prenatal class would include helping participants develop more adaptive coping skills.

A female patient asks what can be done to control vaginal odor. How should the nurse respond? Note: Credit will be given only if all correct choices and no incorrect choices are selected. Select all that apply. A) Wear cotton underwear B) Use a mild vaginal deodorant C) Schedule douching to occur weekly D) Cleanse from front to back when toileting E) Use soap and water to cleanse the perineum

Answer: A, D, E Explanation: A) To control vaginal odor the patient should be instructed to wear cotton underwear, cleanse from front to back when toileting, and to use soap and water to cleanse the perineum. Vaginal deodorants and douching are not recommended. B) To control vaginal odor the patient should be instructed to wear cotton underwear, cleanse from front to back when toileting, and to use soap and water to cleanse the perineum. Vaginal deodorants and douching are not recommended. C) To control vaginal odor the patient should be instructed to wear cotton underwear, cleanse from front to back when toileting, and to use soap and water to cleanse the perineum. Vaginal deodorants and douching are not recommended. D) To control vaginal odor the patient should be instructed to wear cotton underwear, cleanse from front to back when toileting, and to use soap and water to cleanse the perineum. Vaginal deodorants and douching are not recommended. E) To control vaginal odor the patient should be instructed to wear cotton underwear, cleanse from front to back when toileting, and to use soap and water to cleanse the perineum. Vaginal deodorants and douching are not recommended.

1. A client who is having false labor most likely would have which of the following? Note: Credit will be given only if all correct choices and no incorrect choices are selected. Select all that apply. A) Contractions that do not intensify while walking B) An increase in the intensity and frequency of contractions C) Progressive cervical effacement and dilation D) Pain in the abdomen that does not radiate E) contractions lessen with rest and warm tub baths.

Answer: A, D, E Explanation: A) True labor contractions intensify while walking. B) The contractions of true labor produce progressive dilation and effacement of the cervix. They occur regularly and increase in frequency, duration, and intensity. C) True labor results in progressive dilation, increased intensity and frequency of contractions, and pain in the back that radiates to the abdomen. D) True labor results in progressive dilation, increased intensity and frequency of contractions, and pain in the back that radiates to the abdomen. E) In true labor, contractions do not lessen with rest and warm tub baths.

1) In assessing a new family coming to the clinic, the nurse determines they are an extended kin family because the family exhibits what as characteristics of an extended kin network family? Note: Credit will be given only if all correct choices and no incorrect choices are selected. Select all that apply. A) A sharing of a social support network B) Each family establishes their own sources of goods and services C) Elderly parents share housing D) Children are members of two nuclear families E) A sharing of goods and services

Answer: A, E Explanation: A) Extended kin family networks share a social support network. B) Extended kin family networks share goods and services, rather than establishing their own sources of goods and services. C) Elderly parents sharing a household is a feature of the extended family system. D) Children being members of two nuclear families applies to the binuclear family. E) Extended kin family networks share goods and services.

1) A 7-year-old client tells the nurse that "Grandpa, Mommy, Daddy, and my brother live at my house." The nurse identifies this as what type of family? A) Binuclear B) Extended C) Gay or lesbian D) Traditional

Answer: B Explanation: A) A binuclear family includes divorced parents with joint custody of their biologic children, who alternate spending varying amounts of time in the home of each parent. B) An extended family consists of a couple who share the house with their parents, siblings, or other relatives. C) A gay or lesbian family is composed of two same-sex domestic partners; they might not have children. D) The traditional nuclear family consists of a husband provider, a wife who stays home, and the biologic children of this union.

15) Before applying a cord clamp, the nurse assesses the umbilical cord. The mother asks why the nurse is doing this. What should the nurse reply? A) "I'm checking the blood vessels in the cord to see whether it has one artery and one vein." B) "I'm checking the blood vessels in the cord to see whether it has two arteries and one vein." C) "I'm checking the blood vessels in the cord to see whether it has two veins and one artery." D) "I'm checking the blood vessels in the cord to see whether it has two arteries and two veins."

Answer: B Explanation: A) A normal umbilical cord does not have one artery and one vein. B) Two arteries and one vein are present in a normal umbilical cord. C) A normal umbilical cord does not have two veins and one artery. D) A normal umbilical cord does not have two arteries and two veins.

The nurse is explaining the difference between descriptive statistics and inferential statistics to a group of student nurses. To illustrate descriptive statistics, what would the nurse use as an example? A) A positive correlation between breastfeeding and infant weight gain B) The infant mortality rate in the state of Oklahoma C) A causal relationship between the number of sexual partners and sexually transmitted infections D) The total number of spontaneous abortions in drug-abusing women as compared with non-drug-abusing women

Answer: B Explanation: A) A positive correlation between two or more variables is an inferential statistic. B) The infant mortality rate in the state of Oklahoma is a descriptive statistic, because it describes or summarizes a set of data. C) A causal relationship between the number of sexual partners and sexually transmitted infections is an inferential statistic. D) The total number of spontaneous abortions in drug-abusing women is an inferential statistic.

1) The nurse is providing care to a laboring pregnant client who is diagnosed with sickle cell disease. What will the nurse prepare for upon delivery of the client's newborn, based on this diagnosis? A) Hemorrhage B) Intrauterine growth restriction C) Neonatal resuscitation D) Cesarean birth

Answer: B Explanation: A) A pregnant client diagnosed with sickle cell anemia is not at an increased risk for hemorrhage. B) When providing care for a pregnant client diagnosed with sickle cell anemia, the nurse will prepare for a neonate experiencing intrauterine growth restriction, as this is a known consequence during pregnancy. C) The newborn of a client diagnosed with sickle cell anemia is not at an increased risk for requiring neonatal resuscitation. D) A pregnant client diagnosed with sickle cell anemia is not at an increased for cesarean birth.

1) The client delivered 30 minutes ago. Her blood pressure and pulse are stable. Vaginal bleeding is scant. The nurse should prepare for which procedure? A) Abdominal hysterectomy B) Manual removal of the placenta C) Repair of perineal lacerations D) Foley catheterization

Answer: B Explanation: A) Abdominal hysterectomy is not required. B) Retention of the placenta beyond 30 minutes after birth is termed retained placenta. Manual removal of the placenta is then performed. C) Repair of perineal lacerations would not ensue until after the placenta was delivered. D) There is no indication of urinary retention that requires a Foley catheter.

The nurse is preparing an education session for women on the prevention of urinary tract infections (U T Is). Which statement should be included? A) Lower urinary tract infections rarely occur in women. B) The most common causative organism of cystitis is E. coli. C) Wiping from back to front after a B M will help prevent a U T I. D) Back pain often develops with a lower urinary tract infection.

Answer: B Explanation: A) About 60% of women will experience an episode of cystitis during their lifetime. B) E. coli is present in 75% to 90% of women with U T Is. C) Wiping from back to front increases the risk of U T Is because the E. coli of the bowel is being drawn toward the urethra. Women should be instructed always to wipe from front to back. D) Low back or flank pain is a sign of pyelonephritis, which is an upper urinary tract infection.

1) The nurse is teaching a new mother how to encourage a sleepy baby to breastfeed. Which of the following instructions would not be included in that teaching? A) Providing skin-to-skin contact B) Swaddling the newborn in a blanket C) Unwrapping the newborn D) Allowing the newborn to feel and smell the mother's breast

Answer: B Explanation: A) Activities that encourage a sleepy newborn to breastfeed include providing skin-to-skin contact, which enhances bonding. B) Remove the baby's blanket and clothing so that the infant is wearing only a diaper and T-shirt. Babies feed better when they are not bundled, and they can achieve better attachment without the bulk of extra clothing and blankets. Swaddling the newborn has the opposite effect. C) Remove the baby's blanket and clothing so that the infant is wearing only a diaper and T-shirt. Babies feed better when they are not bundled, and they can achieve better attachment without the bulk of extra clothing and blankets. D) If the newborn falls asleep after the first few suckles, encourage the mother to use tactile stimulation while the newborn is still attached to the breast. The mother can also be encouraged to use breast compression or breast massage while the infant is breastfeeding.

1) The nurse manager in a hospital with a large immigrant population is planning an in-service. Aware of how ethnocentrism affects nursing care, the nurse manager asks, "The belief that one's own values and beliefs are the only or the best values has which of the following results?" A) It implies newcomers to the United States should adopt the norms and values of the country. B) It can create barriers to communication through misunderstanding. C) It leads to an expectation that all clients will exhibit pain the same way. It improves the quality of care provided to culturally diverse client bases

Answer: B Explanation: A) Although acculturation involves adoption of some of the majority culture's practices and beliefs, each cultural group will continue to hold and express its own set of values and beliefs. B) When the nurse assumes that a client has the same values and beliefs as the nurse, misunderstanding will frequently occur, which in turn can negatively impact nurse-client communication. Ethnocentrism is the conviction that the values and beliefs of one's own cultural group are the best or only acceptable ones. C) Expression of pain is one area that varies greatly from one culture to another. D) The belief that one's own values and beliefs are the best will not improve the quality of care provided to culturally diverse client bases.

1) The adolescent client reports to the clinic nurse that her period is late, but her home pregnancy test is negative. What should the nurse explain that these findings most likely indicate? A) "This means you are not pregnant." B) "You might be pregnant, but it might be too early for your home test to be accurate." C) "We don't trust home tests. Come to the clinic for a blood test." D) "Most people don't use the tests correctly. Did you read the instructions?"

Answer: B Explanation: A) Although it might be true that she is not pregnant, this is not the best statement because the pregnancy might be too early for a urine pregnancy test to detect. B) This is a true statement. Most home pregnancy tests have low false-positive rates, but the false-negative rate is slightly higher. Repeating the test in a week is recommended. C) This statement is not worded therapeutically. A clinic pregnancy test is usually a urine test. D) Although this statement gets at the need to read the instructions for the test, it is not worded therapeutically.

1) The home care nurse is examining a newborn who is sleeping on a pillow in a basket, covered with a fluffy blanket. There is also a stuffed animal in the basket. The most important nursing action to do is which of the following? A) Remove the stuffed animal from the basket and place it on the floor. B) Teach the parents the risk of S I D S from soft items in the infant's bed. C) Make certain that the blanket is firmly tucked under the baby. D) Ask whether the color of the blanket has cultural significance.

Answer: B Explanation: A) Although loose bedding and soft objects should be removed from the infant's crib during sleep, the highest priority is teaching the parents that these items are safety hazards and about the risk of S I D S. B) Teaching the parents about the risk of sudden infant death syndrome (S I D S) is the highest priority. C) Loose bedding and soft objects should be removed from the infant's crib during sleep. D) Cultural significance is important, but the newborn's physical safety is a higher priority.

1) One day after giving birth vaginally, a client develops painful vesicular lesions on her perineum and vulva. She is diagnosed with a primary herpes simplex 2 infection. What is the expected care for her neonate? A) Meticulous hand washing and antibiotic eye ointment administration. B) Intravenous acyclovir (Zovirax) and contact precautions. C) Cultures of blood and C S F and serial chest x-rays every 12 hours. Parental rooming-in and four intramuscular injections of penicillin

Answer: B Explanation: A) Although meticulous hand washing by staff and parents is important, antibiotic eye ointment is used for conjunctivitis of gonorrhea or chlamydia. B) Administering intravenous acyclovir (Zovirax) and contact precautions are appropriate measures for an infant at risk for developing herpes simplex 2 infection. C) Cultures of blood and C S F cultures are appropriate, but chest X-rays are not indicated. Chest X-rays are obtained if the neonate is thought to have group B strep pneumonia. D) Parental rooming-in is encouraged, but penicillin does not treat viral illness.

1) A new grandmother comments that when her children were born, they stayed in the nursery. The grandmother asks the nurse why her daughter's baby stays mostly in the room instead of the nursery. How should the nurse respond? A) "Babies like to be with their mothers more than they like to be in the nursery." B) "Contact between parents and babies increases attachment." C) "Budget cuts have decreased the number of nurses in the nursery." D) "Why do you ask? Do you have concerns about your daughter's parenting?"

Answer: B Explanation: A) Although most newborns cry less when held than when in their cribs, this is not the most important reason for encouraging mothers to spend time with their babies. B) In a mother-baby unit, the newborn's crib is placed near the mother's bed, where she can see her baby easily; this is conducive to an on-demand feeding schedule for both breastfeeding and formula-feeding infants. C) Budget cuts are not a reason for babies' being in the nursery less than in the past. D) It is not therapeutic to use the word "why." The grandmother has not indicated that she has any concerns about her daughter's parenting.

1) A client comes to the reproductive health clinic and reports that she woke up in a strange room this morning, her perineal area is sore, and she can't clearly remember what happened the previous evening. The client says she is afraid that she was a victim of a drug-facilitated sexual assault. Which statement should the nurse include when discussing this possibility with the client? A) "Drinking alcohol can lead to uninhibited sexual behavior, which is not the same as rape." B) "Some men use drugs mixed into a drink to subdue a potential victim prior to a rape." C) "It is rare that a woman doesn't remember what happened if she is actually raped." D) "We need to check for forensic evidence of rape before we can be sure what happened."

Answer: B Explanation: A) Although one effect of alcohol consumption is decreased inhibition, which can lead to less cautious sexual behavior, if a woman is drugged, the sexual act is nonconsensual and is therefore classified as rape. B) Drug-facilitated sexual assault occurs when a drug such as Rohypnol, which dissolves easily and is odorless, is slipped into the drink of an unsuspecting woman. C) Rohypnol, which dissolves easily and is odorless, can be slipped into the drink of an unsuspecting woman and causes amnesia of the attack. D) Forensic evidence is collected for possible legal prosecution of the attacker, but the absence of collectable evidence does not eliminate the possibility of rape.

1) The nurse is assessing a primiparous client who indicates that her religion is Judaism. Why is this information pertinent for the nurse to assess? A) Religious and cultural background can impact what a client eats during pregnancy. B) It provides a baseline from which to ask questions about the client's religious and cultural background. C) Knowing the client's beliefs and behaviors regarding pregnancy is not important. D) Clients sometimes encounter problems in their pregnancies based on what religion they practice.

Answer: B Explanation: A) Although this can be true, much more than diet is impacted by religious and cultural background. B) Nurses have an obligation to be aware of other cultures and develop a culturally sensitive plan of care to meet the needs of the childbearing woman and her family. C) It is especially helpful if the nurse is familiar with common practices of various religious and cultural groups who reside in the community. D) How a client observes her religion occasionally will cause problems with pregnancy, but this is not the most important reason for obtaining this information.

1) A new mother is holding her 2-hour-old son. The delivery occurred on the due date. His Apgar score was 9 at both 1 and 5 minutes. The mother asks the nurse why her son was so wide awake right after birth, and now is sleeping so soundly. What is the nurse's best response? A) "Don't worry. Babies go through a lot of these little phases." B) "Your son is in the sleep phase. He'll wake up soon." C) "Your son is exhausted from being born, and will sleep 6 more hours." D) "Your breastfeeding efforts have caused excessive fatigue in your son."

Answer: B Explanation: A) Although this infant's behavior is expected, nurses must avoid using clichés in therapeutic communication. B) The first period of reactivity lasts approximately 30 minutes after birth. During this period the newborn is awake and active and may appear hungry and have a strong sucking reflex. After approximately half an hour, the newborn's activity gradually diminishes, and the heart rate and respirations decrease as the newborn enters the sleep phase. The sleep phase may last from a few minutes to 2 to 4 hours. C) Six hours of sleep at this point is not an expected finding. D) Breastfeeding does not cause fatigue in a normal term newborn.

1) The client with blood type O R h-negative has given birth to an infant with blood type O R h-positive. The infant has become visibly jaundiced at 12 hours of age. The mother asks why this is happening. What is the best response by the nurse? A) "The R h o G A M you received at 28 weeks' gestation did not prevent alloimmunization." B) "Your body has made antibodies against the baby's blood that are destroying her red blood cells." C) "The red blood cells of your baby are breaking down because you both have type O blood." D) "Your baby's liver is too immature to eliminate the red blood cells that are no longer needed."

Answer: B Explanation: A) Although this statement is true, the term "alloimmunization" is not likely to be understood by the client. It is better to explain what is happening using more understandable terminology. B) This explanation is accurate and easy for the client to understand. Newborns of R h-negative and O blood type mothers are carefully assessed for blood type status, appearance of jaundice, and levels of serum bilirubin. C) Mother and baby's both having type O blood is not a problem. A B O incompatibility occurs if mother is O and baby is A or B. D) The infant's liver is indeed too immature to eliminate red blood cells, but the hemolysis from the maternal antibodies is the cause of the jaundice.

1) Which of the following tests provides information about the fetal number? A) Amniocentesis B) Standard second-trimester sonogram C) Beta h C G D) Maternal serum alpha-fetoprotein

Answer: B Explanation: A) Amniocentesis can make chromosomal and biochemical determinations and can validate abnormalities detected by ultrasound. B) A standard (comprehensive) second trimester sonogram provides information about the fetus, placenta, and uterine conditions, including fetal number. C) Serial quantitative beta h C G testing can be used to distinguish a normally developing fetus from an ectopic pregnancy. D) Maternal serum alpha-fetoprotein (M S A F P) is a component of the screening test, the "quadruple check" that utilizes the multiple markers, including A F P, h C G, diameric inhibin-A, and estriol, to screen pregnancies for N T D, trisomy 21 (Down syndrome), and trisomy 18.

1) The client presents for cervical ripening in anticipation of labor induction tomorrow. What should the nurse include in her plan of care for this client? A) Apply an internal fetal monitor. B) Monitor the client using electronic fetal monitoring. C) Withhold oral intake and start intravenous fluids. D) Place the client in an upright, sitting position.

Answer: B Explanation: A) An internal fetal monitor cannot be applied until adequate cervical dilatation has occurred and the membranes are ruptured. B) The client should be monitored using electronic fetal monitoring for at least 30 minutes and up to 2 hours after placement to assess the contraction pattern and the fetal status. C) Until labor begins, there is no rationale for withholding oral intake. D) The client is placed in a reclining position and bed rest is maintained to prevent the medication from leaking from the vagina.

1) What is the primary carbohydrate in mammalian milk that plays a crucial role in the nourishment of the newborn? A) Colostrum B) Lactose C) Lactoferrin D) Secretory I g A

Answer: B Explanation: A) Another term for human milk is colostrum. B) Lactose is the primary carbohydrate in mammalian milk. C) Lactoferrin is an iron-binding protein found only in breast milk. Secretory IgA is an immunoglobulin present in colostrum and mature breast milk

1) Every time the nurse enters the room of a postpartum client who gave birth 3 hours ago, the client asks something else about her birth experience. What action should the nurse take? A) Answer questions quickly and try to divert her attention to other subjects. B) Review the documentation of the birth experience and discuss it with her. C) Contact the physician to warn him the client might want to file a lawsuit, based on her preoccupation with the birth experience. D) Submit a referral to Social Services because of possible obsessive behavior.

Answer: B Explanation: A) Answering questions quickly and trying to divert the client's attention trivializes her questions and does not allow her to sort out the reality from her fantasized experience. B) The client may talk about her labor and birth experience. The nurse should provide opportunities to discuss the birth experience in a nonjudgmental atmosphere if the woman desires to do so. C) Asking questions about the birth experience is normal behavior. Contacting the physician is not warranted. D) Submitting a referral to Social Services is inappropriate, because this client's behavior is normal.

1) The client at 9 weeks' gestation has been told that her H I V test was positive. The client is very upset, and tells the nurse, "I didn't know I had H I V! What will this do to my baby?" The nurse knows teaching has been effective when the client makes which statement? A) "I cannot take the medications that control H I V during my pregnancy, because they will harm the baby." B) "My baby can get H I V during the pregnancy and through my breast milk." C) "The pregnancy will increase the progression of my disease and will reduce my C D4 counts." D) "The H I V won't affect my baby, and I will have a low-risk pregnancy without additional testing."

Answer: B Explanation: A) Antiretroviral therapy is recommended to all infected pregnant women, regardless of whether or not they are symptomatic, to reduce the rate of perinatal transmission. B) H I V transmission can occur during pregnancy and through breast milk; however, it is believed that the majority of all infections occur during labor and birth. C) There is no evidence to indicate that pregnancy increases the progression of H I V/A I D S. D) A pregnancy complicated by H I V infection, even if asymptomatic, is considered high risk, and the fetus is monitored closely. Weekly non-stress testing (N S T) is begun at 32 weeks' gestation.

1) The nurse at a women's clinic is reviewing a new client health information questionnaire. Which question does she find to be insulting and discriminatory toward lesbian clients? A) Who should be contacted in case of emergency? B) What method of birth control do you use? C) How often do you drink alcohol? D) Do you feel safe in your relationship?

Answer: B Explanation: A) Asking who should be contacted in an emergency is not an insulting or discriminatory question. Emergency contact is important for all clients. B) The assumption that all women are in need of contraception for birth control is often cited as a reason that lesbian women may conceal their sexual orientation. C) Lesbians should be assessed for chemical dependency like all clients. D) Asking whether the client feels safe in her relationship and assessing for domestic partner violence are common interventions for all clients who come into the clinic.

1. The nurse is assisting a multiparous woman to the bathroom for the first time since her delivery 3 hours ago. When the client stands up, blood runs down her legs and pools on the floor. The client turns pale and feels weak. What would be the first action of the nurse? A) Assist the client to empty her bladder B) Help the client back to bed to check the fundus C) Assess her blood pressure and pulse D) Begin an I V of lactated Ringer's solution

Answer: B Explanation: A) Assisting the client to empty her bladder is not the first action the nurse would take. B) Massaging the fundus is the top priority because of the excessive blood loss. If the fundus is not firm, gentle fundal massage is performed until the uterus contracts. C) Blood pressure and pulse do not change until 1000 to 2000 m L of blood has been lost. D) An I V might need to be started if the client becomes symptomatic.

1) The true moment of fertilization occurs when what happens? A) Cortical reaction occurs B) Nuclei unite C) Spermatozoa propel themselves up the female tract D) Sperm surrounding the ovum release their enzymes

Answer: B Explanation: A) At the moment of penetration by a fertilizing sperm, the zona pellucida undergoes a reaction that prevents additional sperm from entering a single ovum, known as the block to polyspermy. This cellular change is mediated by release of materials from the cortical granules, organelles found just below the ovum's surface, and is called the cortical reaction. B) The true moment of fertilization occurs as the nuclei unite. Their individual nuclear membranes disappear, and their chromosomes pair up to produce the diploid zygote. C) Fertilization has not yet occurred when the spermatozoa are still in the female reproductive tract. This is part of the acrosomal reaction and occurs prior to fertilization

1) A female client presents in the emergency department (E D) after being sexually assaulted at a party. Which assessment finding indicates that the client may have been drugged? A) Attending the party with a large group of friends B) Accepting a beverage from a stranger at the party C) Dancing and kissing several men during the party D) Drinking large amounts of alcohol during the party

Answer: B Explanation: A) Attending a party with a large group of friends is not an assessment finding that would indicate the client may have been drugged. B) Accepting a drink from someone else or drinking a drink that was left unattended would indicate the client may have been drugged. C) Dancing and kissing several men during the party is not an assessment finding that would indicate the client may have been drugged. D) Drinking large amounts of alcohol at the party is not an assessment finding that would indicate the client may have been drugged. Having one or two drinks and then suddenly feeling very drunk would be an indicator that the client had been drugged.

1) The clinic nurse is returning phone calls. Which call should the nurse return first? A) The call from a 22-year-old reporting that she has menstrual cramps and vomiting every month B) The call from a 17-year-old asking whether there is a problem with using one tampon for a whole day C) The call from a 46-year-old mother of a teen wondering if her daughter should be on birth control D) The call from a 34-year-old requesting information on douching after intercourse

Answer: B Explanation: A) Because vomiting can lead to dehydration, this client is not completely normal or stable, but is not the top priority. B) Using a single tampon for an entire day can lead to toxic shock syndrome, a potentially life-threatening condition. This client needs education on the danger of using one tampon longer than 3-6 hours. C) A sexually active teen could be at risk for unintended pregnancy, as well as sexually transmitted infections. However, it is unclear whether the daughter is sexually active. This call is a low priority. D) This client requires education, but is not a top priority.

The nurse is reviewing care of clients on a mother-baby unit. Which situation should be reported to the supervisor? A) A 2-day-old infant has breastfed every 2-3 hours and voided four times. B) An infant was placed in the wrong crib after examination by the physician. C) The client who delivered by cesarean birth yesterday received oral narcotics. A primiparous client who delivered today is requesting discharge within 24 hours

Answer: B Explanation: A) Breastfeeding every 2 hours and voiding four times is within normal limits for a 2-day-old infant. There is no negligence in this situation. B) Placing an infant in the wrong crib is malpractice. Malpractice is negligent action by a professional person. C) Receiving oral narcotics at this point in the client's stay is within normal limits. There is no negligence in this situation. D) If the client is feeling well and able to care for her infant, it is normal to be discharged at this time. The mother and baby both must be within normal limits to be discharged.

1) Placing the baby at mother's breast facilitates early latch and promotes successful breastfeeding. When should breastfeeding be initiated? A) 6 to 12 hours after birth B) Within 1 hour of birth C) 24 hours after birth D) 48 hours after birth

Answer: B Explanation: A) Breastfeeding should be initiated within the first hour of life, not 6 to 12 hours after birth. B) Breastfeeding should be initiated within the first hour of life unless medically contraindicated. C) Breastfeeding should be initiated within the first hour of life, not 24 hours after birth. Breastfeeding should be initiated within the first hour of life, not 48 hours after birth

1) What is required for any woman receiving oxytocin (Pitocin)? A) C P R B) Continuous electronic fetal monitoring C) Administering oxygen by mask D) Nonstress test

Answer: B Explanation: A) C P R is not required for a woman receiving oxytocin. B) Continuous electronic fetal monitoring (E F M) is required for any woman receiving oxytocin (Pitocin). C) Administering oxygen by mask is not required for a woman receiving oxytocin. D) Nonstress test is not required for a woman receiving oxytocin.

A nurse is providing guidance to a group of parents of children in the infant-to-preschool age group. After reviewing statistics on the most common cause of death in this age group, the nurse includes information about prevention of which of the following? A) Cancer by reducing the use of pesticides in the home B) Accidental injury by reducing the risk of pool and traffic accidents C) Heart disease by incorporating heart-healthy foods into the child's diet D) Pneumonia by providing a diet high in vitamin C from fruits and vegetables

Answer: B Explanation: A) Cancer due to pesticide use is not a large cause of death in this age group. B) Unintentional injuries cause death in infants more often than cancer, heart disease, and pneumonia. C) Heart disease is not a large cause of death in this age group. Pneumonia does not cause a large number of deaths

1) The nurse is anticipating the arrival of a couple in the labor unit. It has been determined that the 37-week fetus has died in utero from unknown causes. What should the nurse include in the plan of care for this couple? A) Allow the couple to adjust to the labor unit in the waiting area. B) Place the couple in a labor room at the end of the hall with an empty room next door. C) Encourage the father to go home and rest for a few hours. D) Contact the mother's emergency contact person and explain the situation.

Answer: B Explanation: A) Care should be taken not to leave the couple in the waiting room with other expectant parents or visitors waiting for news from other women in labor. B) Upon arrival at the facility, the couple with a known or suspected fetal demise should immediately be placed in a private room. When possible, the woman should be in the room that is farthest away from other laboring women. C) The couple should be allowed to remain together as much as they wish. Provide privacy as needed and maintain a supportive environment. D) Some couples may want outside support, such as family members or friends, to be present during the labor. The nurse needs to facilitate the couple's wishes, including not contacting anyone if that wish is expressed.

1) A newly diagnosed insulin-dependent type 1 diabetic with good blood sugar control is at 20 weeks' gestation. She asks the nurse how her diabetes will affect her baby. What would the best explanation include? A) "Your baby could be smaller than average at birth." B) "Your baby will probably be larger than average at birth." C) "As long as you control your blood sugar, your baby will not be affected at all." D) "Your baby might have high blood sugar for several days."

Answer: B Explanation: A) Characteristically, infants of mothers with diabetes are large for gestational age (L G A). B) Characteristically, infants of mothers with diabetes are large for gestational age (L G A), as a result of high levels of fetal insulin production stimulated by the high levels of glucose crossing the placenta from the mother. Sustained fetal hyperinsulinism and hyperglycemia ultimately lead to excessive growth, called macrosomia, and deposition of fat. C) The demands of pregnancy will make it difficult for the best of clients to control blood sugar on a regular basis. D) After birth, the umbilical cord is severed, and, thus, the generous maternal blood glucose supply is eliminated.

1) The nurse is completing the gestational age assessment on a newborn while in the mother's postpartum room. During the assessment, the mother asks what aspects of the baby are being checked. What is the nurse's best response? A) "I'm checking to make sure the baby has all of its parts." B) "This assessment looks at both physical aspects and the nervous system." C) "This assessment checks the baby's brain and nerve function." D) "Don't worry. We perform this check on all the babies."

Answer: B Explanation: A) Clinical gestational age assessment tools have two components: external physical characteristics and neurologic or neuromuscular development evaluations. B) Clinical gestational age assessment tools have two components: external physical characteristics and neurologic or neuromuscular development evaluations. C) Clinical gestational age assessment tools have two components: external physical characteristics and neurologic or neuromuscular development evaluations. D) Nurses must always use therapeutic communication and giving a "don't worry" answer dismisses the client's question or concern.

1) A couple who came to the United States two years ago with their two children are seeing the nurse in the community clinic. The nurse knows their family is acculturating when the mother makes which statement? A) "The children are much less well-behaved than they used to be." B) "Our diet now includes hamburgers and French fries." C) "We celebrate the same holidays that we used to at home." D) "When the children leave the house, I worry about them."

Answer: B Explanation: A) Concern about behavior of the children is nearly universal, and is not an indicator of a family's acculturation. B) Inclusion of fast food in the diet is an indication of acculturation, because it shows a belief in the nutritional value of these foods and an acceptance of purchasing fast food as equivalent in value to home-cooked meals. C) The holidays that are celebrated might not change as a part of acculturation. Concern about the children leaving the home is universal, and is not an indicator of a family's acculturation

1) The nurse is working with a client whose religious beliefs differ from those of the general population. What is the best nursing intervention to use to meet the specific spiritual needs of this family? A) Ask how important the client's religious and spiritual beliefs are when making decisions about health care. B) Show respect while allowing time and privacy for religious rituals. C) Ask for the client's opinion on what caused the illness. D) Identify healthcare practices forbidden by religious or spiritual beliefs.

Answer: B Explanation: A) Considering the impact of religious and spiritual beliefs might be part of the spiritual assessment process but is not an intervention. B) Providing spiritually sensitive care involves determining the current spiritual and religious beliefs and practices that will affect the mother and baby, accommodating these practices where possible, and examining one's own spiritual or religious beliefs to be more aware and able to provide nonjudgmental care. C) Asking what caused the client's illness is not an intervention, and does nothing to meet the spiritual needs specific to the family. D) Identifying what health practices might be forbidden by the family's beliefs might be part of the spiritual assessment process, but is not an intervention.

1. The labor and delivery nurse is reviewing charts. The nurse should inform the supervisor about which client? A) Client at 5 c m requesting labor epidural analgesia B) Client whose cervix remains at 6 c m for 4 hours C) Client who has developed nausea and vomiting D) Client requesting her partner to stay with her

Answer: B Explanation: A) Contacting the supervisor is required when an abnormal situation is present. Requests for medication are not abnormal. B) Average cervical change in the active phase of the first stage of labor is 1.2 c m/hour; thus, this client's lack of cervical change is unexpected, and should be reported to the supervisor. C) Nausea and vomiting are common during the transitional phase of the first stage of labor. Contacting the supervisor is required only when an abnormal situation is present. Clients in the transitional phase of the first stage of labor often fear being left alone; this is an expected finding. Contacting the supervisor is required only when an abnormal situation is present

1) The nurse notes that a 36-hour-old newborn's serum bilirubin level has increased from 14 m g/d L to 16.6 m g/d L in an 8-hour period. What nursing intervention would be included in the plan of care for this newborn? A) Continue to observe B) Begin phototherapy C) Begin blood exchange transfusion D) Stop breastfeeding

Answer: B Explanation: A) Continued observation is only appropriate with normal findings. B) Neonatal hyperbilirubinemia must be considered pathologic if the serum bilirubin concentration is rising by more than 0.2 m g/d L per hour. If the newborn is over 24 hours old, which is past the time where an increase in bilirubin would result from pathologic causes, phototherapy may be the treatment of choice to prevent the possible complications of kernicterus. C) If a newborn has hemolysis with an unconjugated bilirubin level of 14 m g/d L, weighs less than 2500 g (birth weight), and is 24 hours old or less, an exchange transfusion may be the best management. This newborn is 36-hours-old. D) The newborn may continue to breastfeed.

1) The nurse is making a postpartum home visit in the summer. The new father asks about taking the baby to a family outing this weekend. The nurse should encourage the father to do which of the following? A) Cover the infant with dark blankets to block the sun. B) Keep the infant in the shade. C) Uncover the infant's head to prevent hyperthermia. D) Avoid taking the infant outdoors for 6 months.

Answer: B Explanation: A) Covering the infant with dark blankets would cause overheating and is not necessary. The infant should wear a light layer of clothing. B) To prevent sunburn, the newborn should remain shaded, wear a light layer of clothing, or be protected with sunscreen specifically formulated for infants. C) Newborns should wear a head covering outdoors to protect their sensitive ears from drafts and to prevent heat loss. D) Avoiding taking the infant outdoors for 6 months is neither necessary nor practical.

1) A mother who is H I V-positive has given birth to a term female. What plan of care is most appropriate for this infant? A) Test with an H I V serologic test at 8 months. B) Begin prophylactic A Z T (Zidovudine) administration. C) Provide 4 to 5 large feedings throughout the day. D) Encourage the mother to breastfeed the child.

Answer: B Explanation: A) Currently available H I V serologic tests (enzyme-linked immunosorbent assay [E L I S A] and Western blot test) cannot distinguish between maternal and infant antibodies; therefore, they are inappropriate for infants up to 18 months of age. B) For infants, A Z T is started prophylactically 2 m g/k g/dose P O every 6 hours beginning as soon after birth as possible and continuing for 6 weeks. C) Nutrition is essential because failure to thrive and weight loss are common. Small, frequent feedings and food supplementation are helpful. D) Breastfeeding should be avoided with an H I V-positive mother, as transmission of the H I V virus to the newborn in breast milk is well documented.

The nurse is providing follow-up education to a client just diagnosed with vaginal herpes. What statement by the client verifies correct knowledge about vaginal herpes? A) "I should douche daily to prevent infection." B) "I could have another breakout during my period." C) "I am more likely to develop cancer of the cervix." D) "I should use sodium bicarbonate on the lesions to relieve discomfort."

Answer: B Explanation: A) Douching does not prevent infection. B) Menstruation seems to trigger recurrences of herpes. C) There is no relation between herpes and cancer of the cervix. Burow's (aluminum acetate) solution, not sodium bicarbonate, relieves discomfort

1. A client is admitted to the labor unit with contractions 1-2 minutes apart lasting 60-90 seconds. The client is apprehensive and irritable. This client is most likely in what phase of labor? A) Active B) Transition C) Latent D) Second

Answer: B Explanation: A) During the active phase, the cervix dilates from about 4 to 7 c m. When the woman enters the early active phase, her anxiety tends to increase as she senses the intensification of contractions and pain. B) During transition, contractions have a frequency of 1 1/2 to 2 minutes, a duration of 60 to 90 seconds, and are strong in intensity. When the woman enters the transition phase, she may demonstrate significant anxiety. C) The latent phase is characterized by mild contractions lasting 20 to 40 seconds with a frequency of 3 to 30 minutes. In the latent stage, the woman may be relieved that labor has finally started. D) The second stage is the pushing stage, and the woman might feel relieved that the birth is near and she can push. There is no second phase of labor.

1) A client is concerned about her risk for breast cancer. Following the initial history, the nurse identifies which of the following as a high risk factor for breast cancer? A) History of late menarche and early menopause B) Sister who has had breast cancer C) Mother with fibrocystic breast disease D) Multiparity

Answer: B Explanation: A) Early menarche combined with late menopause is a breast cancer risk. B) Family history of first-degree relative (mother, sister, or daughter) with breast cancer increases the risk of breast cancer with the number of first-degree relatives with breast cancer. C) Fibrocystic breast disease is not a breast cancer risk factor. D) Multiparity is not a breast cancer risk factor.

1) The female and male reproductive organs are homologous, which means what? A) They are believed to cause vasoconstriction and muscular contraction B) They are fundamentally similar in function and structure C) They are rich in sebaceous glands D) They are target organs for estrogenic hormones

Answer: B Explanation: A) Efferent sympathetic motor nerves are believed to cause vasoconstriction and muscular contraction. B) The female and male reproductive organs are homologous; that is, they are fundamentally similar in function and structure. C) The labia minora are rich in sebaceous glands. D) The female internal reproductive organs are target organs for estrogenic hormones.

1) A pregnant woman tells the nurse-midwife, "I've heard that if I eat certain foods during my pregnancy, the baby will be a boy." The nurse-midwife should explain that this is a myth, and that the sex of the baby is determined at what time? A) At the time of ejaculation B) At the time of fertilization C) At the time of implantation D) At the time of differentiation

Answer: B Explanation: A) Ejaculation is the release of sperm from the male, and does not necessarily cause a pregnancy. B) Fertilization is the point at which the sex of the zygote is determined. C) Implantation is when the fertilized ovum is implanted in the uterine endometrium. The sex of the zygote has already been determined at this stage. D) Differentiation refers to a cell division process.

1) The laboring client participated in childbirth preparation classes that strongly discouraged the use of medications and intervention during labor. The client has been pushing for two hours, and is exhausted. The physician requests that a vacuum extractor be used to facilitate the birth. The client first states that she wants the birth to be normal, then allows the vacuum extraction. Following this, what should the nurse assess the client for after the birth? A) Elation, euphoria, and talkativeness B) A sense of failure and loss C) Questions about whether or not to circumcise D) Uncertainty surrounding the baby's name

Answer: B Explanation: A) Elation, euphoria, and talkativeness are expected after birth. B) Clients who participate in childbirth classes that stress the normalcy of birth may feel a sense of loss or failure if an intervention is used during their labor or birth. C) Decisions on circumcision and naming are often encountered after birth, and are not correlated with the use of intervention. D) Decisions on circumcision and naming are often encountered after birth, and are not correlated with the use of intervention.

The nurse is caring for a client hospitalized for pelvic inflammatory disease. Which nursing intervention would have priority? A) Encourage oral fluids B) Administer cefotetan Ⅳ C) Enforce bed rest D) Remove I U C, if present

Answer: B Explanation: A) Encouraging oral fluids is not a priority. B) Administration of medications to treat the disease is the first priority. C) Bed rest is not a priority. D) Removal of an I U C is not a nursing intervention.

1. A client calls the labor and delivery unit and tells the nurse that she is 39 weeks pregnant and over the last 4 or 5 days, she has noticed that although her breathing has become easier, she is having leg cramps, a slight amount of edema in her lower legs, and an increased amount of vaginal secretions. The nurse tells the client that she has experienced which of the following? A) Engagement B) Lightening C) Molding D) Braxton Hicks contractions

Answer: B Explanation: A) Engagement of the presenting part occurs when the largest diameter of the fetal presenting part reaches or passes through the pelvic inlet. B) Lightening describes the effect occurring when the fetus begins to settle into the pelvic inlet. C) The fetal cranial bones overlap under pressure of the powers of labor and the demands of the unyielding pelvis. This overlapping is called molding. D) Braxton Hicks contractions occur before the onset of labor.

1) A nurse teaches newly pregnant clients that if an ovum is fertilized and implants in the endometrium, the hormone the fertilized egg begins to secrete is which of the following? A) Estrogen B) Human chorionic gonadotropin (h C G) C) Progesterone D) Luteinizing hormone

Answer: B Explanation: A) Estrogen and progesterone are ovarian hormones. B) If the ovum is fertilized and implants in the endometrium, the fertilized egg begins to secrete human chorionic gonadotropin (h C G), which is needed to maintain the corpus luteum. C) Estrogen and progesterone are ovarian hormones. D) Luteinizing hormone is excreted by the anterior pituitary gland.

1) A woman has been unable to complete a full-term pregnancy because the fertilized ovum failed to implant in the uterus. This is most likely due to a lack of which hormone? A) Estrogen B) Progesterone C) F S H D) L H

Answer: B Explanation: A) Estrogens are associated with characteristics contributing to femaleness. B) Progesterone is often called the hormone of pregnancy because it inhibits uterine contractions and relaxes smooth muscle to cause vasodilation, allowing pregnancy to be maintained. C) F S H is a hormone secreted by the pituitary gland, and its lack would not affect the ability of the uterus to be prepared for implantation of the fertilized ovum. D) L H is a hormone secreted by the pituitary gland, and its lack would not affect the ability of the uterus to be prepared for implantation of the fertilized ovum.

1. Four minutes after the birth of a baby, there is a sudden gush of blood from the mother's vagina, and about 8 inches of umbilical cord slides out. What action should the nurse take first? A) Place the client in McRoberts position. B) Watch for the emergence of the placenta. C) Prepare for the delivery of an undiagnosed twin. D) Place the client in a supine position.

Answer: B Explanation: A) Exercises aimed at adducting the legs into an extended McRoberts position, which is performed by flexing the mother's thighs toward her shoulders while she is lying on her back, help enable the woman to stretch her hamstring muscles, a task usually required during the second stage of labor. B) Signs of placental separation usually appear around 5 minutes after birth of the infant, but can take up to 30 minutes to manifest. These signs are (1) a globular-shaped uterus, (2) a rise of the fundus in the abdomen, (3) a sudden gush or trickle of blood, and (4) further protrusion of the umbilical cord out of the vagina. C) The first placenta usually does not deliver before the birth of the second twin. D) Blood pressure may drop precipitously when the pregnant woman lies in a supine position and experiences aortocaval compression.

1) The physician has determined the need for forceps. The nurse should explain to the client that the use of forceps is indicated because of which of the following? A) Her support person is exhausted B) Premature placental separation C) To shorten the first stage of labor D) To prevent fetal distress

Answer: B Explanation: A) Exhaustion of the support person is not an indication for use of forceps. B) Fetal conditions indicating the need for forceps include premature placental separation, prolapsed umbilical cord, and nonreassuring fetal status. C) Forceps may be used electively to shorten the second stage of labor and spare the woman's pushing effort, or when regional anesthesia has affected the woman's motor innervation, and she cannot push effectively. D) Indications for the use of forceps include premature placental separation and a nonreassuring fetal heart rate. Using forceps does not prevent fetal distress.

1) Extended use of combined oral contraceptives (C O Cs) reduces the side effects of C O Cs such as which of the following? A) Cramping B) Hypertension C) Breast tenderness D) Bloating

Answer: B Explanation: A) Extended use of C O Cs reduces the side effects of C O Cs such as bloating, headache, breast tenderness, cramping, and swelling. B) Complications of C O Cs include: myocardial infarction, stroke, blood clots, and hypertension. C) Extended use of C O Cs reduces the side effects of C O Cs such as bloating, headache, breast tenderness, cramping, and swelling. D) Extended use of C O Cs reduces the side effects of C O Cs such as bloating, headache, breast tenderness, cramping, and swelling.

1) The nurse is caring for a jaundiced infant receiving bank light phototherapy in an isolette. Which finding requires an immediate intervention? A) Eyes are covered, no clothing on, diaper in place B) Axillary temperature 99.7°F C) Infant removed from the isolette for breastfeeding D) Loose bowel movement

Answer: B Explanation: A) Eye coverings are used because it is not known if phototherapy injures delicate eye structures, particularly the retina. Because the tissue absorbs the light, best results are obtained when there is maximum skin surface exposure. B) Temperature assessment is indicated to detect hypothermia or hyperthermia. Normal temperature ranges are 97.7°F-98.6°F. Vital signs should be monitored every 4 hours with axillary temperatures. C) Breastfeeding should continue during phototherapy; removing the infant for feedings repositions the infant to prevent pressure areas. Infants undergoing phototherapy treatment have increased water loss and loose stools as a result of bilirubin excretion

1. The nurse is calling clients at 4 weeks postpartum. Which of the following clients should be seen immediately? A) The client who describes feeling sad all the time B) The client who reports hearing voices talking about the baby C) The client who states she has no appetite and wants to sleep all day D) The client who says she needs a refill on her sertraline (Zoloft) next week

Answer: B Explanation: A) Feeling sad is an indication the client is experiencing postpartum blues, and is not the highest priority. B) Hearing voices is an indication the client is experiencing postpartum psychosis, and is the highest priority because the voices might tell her to harm her baby. C) Having no appetite and sleeping all day is an indication the client is experiencing postpartum depression, but is not the highest priority. D) A client on medications needs refills on time, but right now she has medication, and therefore is not a high priority.

1) In utero, what is the organ responsible for gas exchange? A) Umbilical vein B) Placenta C) Inferior vena cava D) Right atrium

Answer: B Explanation: A) From the placenta, highly oxygenated blood flows through the umbilical vein. B) In utero, the placenta is the organ of gas exchange. C) From the placenta, highly oxygenated blood flows through the umbilical vein. A small amount of blood perfuses the liver, with the majority of blood volume flowing through the inferior vena cava and to the right atrium. D) From the placenta, highly oxygenated blood flows through the umbilical vein. A small amount of blood perfuses the liver, with the majority of blood volume flowing through the inferior vena cava and to the right atrium.

23) The school nurse is planning a class about nutrition for pregnant teens, several of whom have been diagnosed with iron-deficiency anemia. In order to increase iron absorption, the nurse would encourage the teens to consume more of what beverage? A) Gatorade B) Orange juice C) Milk D) Green tea

Answer: B Explanation: A) Gatorade does not contain vitamin C, which increases iron absorption. B) Vitamin C is found in citrus fruits and juices, and is known to enhance the absorption of iron from meat and non-meat sources. C) Milk does not contain vitamin C, which increases iron absorption. D) Green tea does not contain vitamin C, which increases iron absorption.

1) The nurse anticipates that the physician will most likely order a cervicovaginal fetal fibronectin test for which client? A) The client at 34 weeks' gestation with gestational diabetes B) The client at 32 weeks' gestation with regular uterine contractions C) The client at 37 weeks' multi-fetal gestation D) The client at 20 weeks' gestation with ruptured amniotic membranes

Answer: B Explanation: A) Gestational diabetes does not predispose a client to preterm labor. B) The absence of cervicovaginal f F N between 20 and 34 weeks' gestation has been shown to be a strong predictor of a woman not experiencing preterm birth due to spontaneous preterm labor or premature rupture of membranes. Positive findings indicate a 99% probability of birth within the next 2 weeks. C) Multi-fetal pregnancies often experience preterm delivery, but the client at 37 weeks is at term. D) A pregnancy at 20 weeks has not reached the point of viability, and is treated as an impending spontaneous abortion, not preterm labor.

1) The student nurse notices that the newborn seems to focus on the mother's eyes. The nursing instructor explains that this newborn behavior is which of the following? A) Habituation B) Orientation C) Self-quieting D) Reactivity

Answer: B Explanation: A) Habituation is the newborn's ability to process and respond to complex stimulation. B) Orientation is the newborn's ability to be alert to, follow, and fixate on complex visual stimuli that have a particular appeal and attraction. The newborn prefers the human face and eyes, and bright shiny objects. C) Self-quieting is the ability of newborns to use their own resources to quiet and comfort themselves. D) The newborn usually shows a predictable pattern of behavior during the first several hours after birth, characterized by two periods of reactivity separated by a sleep phase.

The nurse is planning teaching for a patient diagnosed with hepatitis A. What should the nurse emphasize when instructing the patient about this disease process? A) It is a chronic illness B) It is not a chronic illness C) A vaccination is not available D) It occurs in East and South Asia

Answer: B Explanation: A) Hepatitis A is self-limiting and is not a chronic condition. Hepatitis B, C, and D infections are chronic. There is a vaccination available for hepatitis A. Hepatitis E is common worldwide and occurs primarily in East and South Asia.

1) The nurse is admitting a client with possible hydramnios. When is hydramnios most likely suspected? A) Hydramnios is most likely suspected when there is less amniotic fluid than normal for gestation. B) Hydramnios is most likely suspected when the fundal height increases disproportionately to the gestation. C) Hydramnios is most likely suspected when the woman has a twin gestation. D) Hydramnios is most likely suspected when the quadruple screen comes back positive.

Answer: B Explanation: A) Hydramnios occurs when there is more amniotic fluid than normal for gestation. B) Hydramnios should be suspected when the fundal height increases out of proportion to the gestational age. C) Hydramnios is not suspected simply because of a twin gestation. D) A positive quadruple screen is not indicative of hydramnios.

1) The client at 38 weeks' gestation has been diagnosed with oligohydramnios. Which statement indicates that teaching about the condition has been effective? A) "My gestational diabetes might have caused this problem to develop." B) "When I go into labor, I should come to the hospital right away." C) "This problem was diagnosed with blood and urine tests." D) "Women with this condition usually do not have a cesarean birth."

Answer: B Explanation: A) Hydramnios, not oligohydramnios, is associated with such maternal disorders as diabetes. B) The incidence of cord compression and resulting fetal distress is high when there is an inadequate amount of amniotic fluid. The client with oligohydramnios should come to the hospital in early labor. C) Oligohydramnios is diagnosed when the largest vertical pocket of amniotic fluid visible on ultrasound examination is 5 c m or less. D) The fetus is continually monitored during labor and birth. In the event that the fetal heart rate tracing is nonreassuring, late decelerations occur, and birth is not imminent, a cesarean birth may be performed.

1. Which client requires immediate intervention by the labor and delivery nurse? A) Client at 8 c m, systolic blood pressure has increased 35 m m H g B) Client who delivered 1 hour ago with W B C of 50,000/m m3 C) Client at 5 c m with a respiratory rate of 22 between contractions D) Client in active labor with polyuria

Answer: B Explanation: A) In the first stage, systolic pressure may increase by 35 m m H g, and there may be further increases in the second stage during pushing efforts. B) The white blood cell (W B C) count increases to between 25,000/mm3 to 30,000/mm3 during labor and early postpartum. This count is abnormally high, and requires further assessment and provider notification. C) Oxygen demand and consumption increase at the onset of labor because of the presence of uterine contractions. This client requires no further intervention. D) Polyuria is common during labor. This results from the increase in cardiac output, which causes an increase in the glomerular filtration rate and renal plasma flow, and requires no further intervention.

1. The client at 40 weeks' gestation reports to the nurse that she has had increased pelvic pressure and increased urinary frequency. Which response by the nurse is best? A) "Unless you have pain with urination, we don't need to worry about it." B) "These symptoms usually mean the baby's head has descended further." C) "Come in for an appointment today and we'll check everything out." D) "This might indicate that the baby is no longer in a head-down position."

Answer: B Explanation: A) Increased pelvic pressure and urinary frequency are premonitory signs of labor. These are not signs of a urinary tract infection. B) This is the best response because it most directly addresses what the client has reported. C) There is no need for an additional appointment. D) The fetus's changing to a breech presentation would be experienced as fetal movement that was formerly felt in the upper abdomen but now is down in the pelvis.

The nurse is preparing a report on the number of births by three service providers at the facility (certified nurse-midwives, family practitioners, and obstetricians). What is this an example of? A) Inferential statistics B) Descriptive statistics C) Evidence-based practice D) Secondary use of data

Answer: B Explanation: A) Inferential statistics allow the investigator to draw conclusions from data to either support or refute causation. B) Descriptive statistics concisely describe phenomena such as births by providers. C) Evidence-based practice is the use of research conclusions to improve nursing care. D) Secondary use of data is analyzing data in a different way than was originally undertaken, or looking at different variables from a data set.

1) The need for forceps has been determined. The client's cervix is dilated to 10 c m, and the fetus is at +2 station. What category of forceps application would the nurse anticipate? A) Input B) Low C) Mid D) Outlet

Answer: B Explanation: A) Input is not a term associated with the use of forceps. B) Low forceps are applied when the leading edge of the fetal head is at +2 station. C) Midforceps are applied when the fetal head is engaged. D) Outlet forceps are applied when the fetal skull has reached the perineum.

1) The community nurse has identified that the mother who gave birth to a stillborn baby last week is an intuitive griever. Which behavior has the nurse encountered that would lead to this assessment? A) The mother verbalized that her problem-solving skills have been helpful during this process. B) The mother repeatedly talks about her thoughts, feelings, and emotions about losing her child. C) The mother talks little about her experience, and appears detached and unaffected by the loss of her child. D) The mother has asked close friends, co-workers, and relatives not to call or visit.

Answer: B Explanation: A) Instrumental grievers tend to use problem-solving skills in the grief process. B) Intuitive grievers tend to feel their way through the loss and seek emotional and psychosocial support. C) Instrumental grievers can appear detached and unaffected by the experience. D) Intuitive grievers seek emotional and psychosocial support from friends and family.

1) A woman is 32 weeks pregnant. She is H I V-positive but asymptomatic. The nurse knows that what would be important in managing her pregnancy and delivery? A) An amniocentesis at 30 and 36 weeks B) Weekly non-stress testing beginning at 32 weeks' gestation C) Application of a fetal scalp electrode as soon as her membranes rupture in labor D) Administration of intravenous antibiotics during labor and delivery

Answer: B Explanation: A) Invasive procedures such as amniocentesis are avoided when possible to prevent the contamination of a noninfected infant. B) Weekly non-stress testing (N S T) is begun at 32 weeks' gestation and serial ultrasounds are done to detect I U G R. C) Invasive procedures are avoided when possible to prevent the contamination of a noninfected infant. D) Antibiotics would be ineffective for either the mother or the infant who was H I V-positive.

1) A nurse is teaching a class on the different types of uterine bleeding. The nurse explains that which of the following is one of the causes of abnormal uterine bleeding? A) Iron-deficiency anemia B) Polyps C) Heavy periods every 2 months D) Spotting between periods

Answer: B Explanation: A) Iron-deficiency anemia is a symptom of abnormal uterine bleeding, not a cause. B) A classification system has been developed for the causes of A U B using the acronym P A L M-C O E I N. The P stands for Polyps. C) Heavy periods every 2 months could be one of the symptoms of abnormal uterine bleeding, not a cause. D) Spotting between periods is a symptom of abnormal uterine bleeding, not a cause.

1) A 26-year-old client is 28 weeks pregnant. She has developed gestational diabetes. She is following a program of regular exercise, which includes walking, bicycling, and swimming. What instructions should be included in a teaching plan for this client? A) "Exercise either just before meals or wait until 2 hours after a meal." B) "Carry hard candy (or other simple sugar) when exercising." C) "If your blood sugar is 120 m g/d L, eat 20 g of carbohydrate." D) "If your blood sugar is more than 120 m g/d L, drink a glass of whole milk."

Answer: B Explanation: A) It is best to exercise just after the meal, in order to utilize the glucose. B) The nurse should advise her to carry a simple sugar such as hard candy, because of the possibility of exercise-induced hypoglycemia. C) A finger stick result of 120 m g/d L is considered to be normal. D) Such clients need no additional carbohydrate or protein intake.

1) The pregnant client at 14 weeks' gestation is in the clinic for a regular prenatal visit. Her mother also is present. The grandmother-to-be states that she is quite uncertain about how she can be a good grandmother to this baby because she works full time. Her own grandmother was retired, and was always available when needed by a grandchild. What is the nurse's best response to this concern? A) "Don't worry. You'll be a wonderful grandmother. It will all work out fine." B) "What are your thoughts on what your role as grandmother will include?" C) "As long as there is another grandmother available, you don't have to worry." "Grandmothers are supposed to be available. You should retire from your job

Answer: B Explanation: A) It is important to avoid clichés in order to promote effective therapeutic communication. B) Although relationships with parents can be very complex, the expectant grandparents often become increasingly supportive of the expectant couple, even if conflicts previously existed. But it can be difficult for even sensitive grandparents to know how deeply to become involved in the childrearing process. In some areas, classes for grandparents provide information about changes in birthing and parenting practices. C) It is important to avoid placing guilt on clients in order to promote effective therapeutic communication. D) It is important to avoid placing guilt on clients in order to promote effective therapeutic communication.

1) The nurse is preparing to assess the pregnant client's fundal height during a routine prenatal visit. Which nursing action is appropriate in this situation? A) Telling the client not to eat or drink for one hour after the procedure B) Asking the client to empty her bladder prior to the procedure C) Obtaining informed consent for the procedure D) Assessing blood pressure after the procedure

Answer: B Explanation: A) It is not necessary for the client to abstain from eating or drinking for one hour after the procedure. This action might be appropriate for a client who is having a glucose tolerance test, not for one undergoing assessment of fundal height. B) It is appropriate for the nurse to ask the client to empty her bladder prior to assessing fundal height. A full bladder may impact the accuracy of the measurement. C) Informed consent is not needed, as assessing fundal height is not an invasive procedure. D) There is no reason to assess the client's blood pressure after measuring fundal height.

A maternity client is in need of surgery. Which healthcare member is legally responsible for obtaining informed consent for an invasive procedure? A) The nurse B) The physician C) The unit secretary D) The social worker

Answer: B Explanation: A) It is not the nurse's legal responsibility to obtain informed consent. B) Informed consent is a legal concept designed to allow clients to make intelligent decisions regarding their own health care. Informed consent means that a client, or a legally designated decision maker, has granted permission for a specific treatment or procedure based on full information about that specific treatment or procedure as it relates to that client under the specific circumstances of the permission. The individual who is ultimately responsible for the treatment or procedure should provide the information necessary to obtain informed consent. In most instances, this is a physician. C) Unit secretaries are not responsible for obtaining informed consent. D) It is not within a social worker's scope of practice to obtain informed consent.

1) The laboring client brought a written birth plan indicating that she wanted to avoid pain medications and an epidural. She is now at 6 c m and states, "I can't stand this anymore! I need something for pain! How will an epidural affect my baby?" What is the nurse's best response? A) "The narcotic in the epidural will make both you and the baby sleepy." B) "It is unlikely that an epidural will decrease your baby's heart rate." C) "Epidurals tend to cause low blood pressure in babies after birth." D) "I can't get you an epidural, because of your birth plan."

Answer: B Explanation: A) It is rare for sedation to occur from absorption of the medications of a continuous epidural. B) Maternal hypotension results in uteroplacental insufficiency in the fetus, which is manifested as late decelerations on the fetal monitoring strip. The risk of hypotension can be minimized by hydrating the vascular system with 500 to 1000 m L of I V solution before the procedure and changing the woman's position and/or increasing the I V rate afterward. C) Mothers can experience hypotension after the epidural is administered, but babies do not develop hypotension after birth as a result of a labor epidural. D) A birth plan is what the client hopes for prior to the onset of labor, but it can be modified at any time.

6) The laboring client is having moderately strong contractions lasting 60 seconds every 3 minutes. The fetal head is presenting at a -2 station. The cervix is 6 c m and 100% effaced. The membranes spontaneously ruptured prior to admission, and clear fluid is leaking. Fetal heart tones are in the 140s with accelerations to 150. Which nursing action has the highest priority? A) Encourage the husband to remain in the room. B) Keep the client on bed rest at this time. C) Apply an internal fetal scalp electrode. D) Obtain a clean-catch urine specimen.

Answer: B Explanation: A) It is unknown from the given information whether it is culturally appropriate for the client's husband to remain in the room for the labor and birth. B) Because the membranes are ruptured and the head is high in the pelvis at a -2 station, the client should be maintained on bed rest to prevent cord prolapse. C) An internal fetal scalp electrode is placed when there are signs of fetal intolerance of labor. This client has normal fetal heart tones and clear amniotic fluid; no signs of fetal intolerance of labor are present. D) A clean-catch urine specimen is usually obtained upon admission, but amniotic fluid contamination might falsely increase the reading of protein present.

1) The nurse is preparing an educational session on phenylketonuria for a family whose neonate has been diagnosed with the condition. Which statement by a parent indicates that teaching was effective? A) "This condition occurs more frequently among Japanese people." B) "We must be very careful to avoid most proteins to prevent brain damage." C) "Carbohydrates can cause our baby to develop cataracts and liver damage." D) "Our baby's thyroid gland isn't functioning properly."

Answer: B Explanation: A) Japanese people have a very low rate of P K U disease; it is most common among northern Europeans. B) P K U is the inability to metabolize phenylalanine, an amino acid found in most dietary protein sources. Excessive accumulation of phenylalanine and its abnormal metabolites in the brain tissue leads to progressive, irreversible intellectual disability. C) Galactosemia is a carbohydrate metabolism disease. D) Congenital hypothyroidism is the disorder of low thyroid function at birth.

8) What would the nurse do to accurately assess a pregnant client's food intake? A) Assess her most recent laboratory values. B) Ask her to complete a nutritional questionnaire. C) Observe for signs of hunger. D) Ask about her cooking facilities.

Answer: B Explanation: A) Laboratory values may provide information on the nutritional status of the client, but do not indicate what foods she has eaten. B) Diet may be evaluated using a food frequency questionnaire, which lists common categories of foods and asks the woman how frequently in a day (or week) she consumes foods from the list. C) Hunger alone is not an adequate indicator of nutritional status. D) Cooking facilities are not related to food intake.

1) The student nurse has performed a gestational age assessment of an infant, and finds the infant to be at 32 weeks. On which set of characteristics is the nurse basing this assessment? A) Lanugo mainly gone, little vernix across the body B) Prominent clitoris, enlarging minora, anus patent C) Full areola, 5 to 10 m m bud, pinkish-brown in color D) Skin opaque, cracking at wrists and ankles, no vessels visible

Answer: B Explanation: A) Lanugo and vernix disappear as the infant approaches term. B) At 30 to 32 weeks' gestation, the clitoris is prominent, and the labia majora are small and widely separated. As gestational age increases, the labia majora increase in size. At 36 to 40 weeks, they nearly cover the clitoris. At 40 weeks and beyond, the labia majora cover the labia minora and clitoris. C) Areolas develop greater size with advancing gestational age. D) The skin of a preterm infant is translucent, and vessels are visible through the skin.

1) Late preterm infants have higher infant morbidity and mortality rates than term infants. Which of the following complications can they experience? A) Hyperglycemia B) Jaundice C) Motor difficulties D) Sensory complications

Answer: B Explanation: A) Late preterm infants can experience hypoglycemia, not hyperglycemia. B) Late preterm infants can experience jaundice. C) Motor difficulties are not one of the complications that late preterm infants experience. D) Sensory complications are not one of the issues that late preterm infants experience.

A nursing student investigating potential career goals is strongly considering becoming a nurse practitioner (N P). The major focus of the N P is on which of the following? A) Leadership B) Physical and psychosocial clinical assessment C) Independent care of the high-risk pregnant client D) Tertiary prevention

Answer: B Explanation: A) Leadership might be a quality of the N P, but it is not the major focus. B) Physical and psychosocial clinical assessment is the major focus of the N P. C) N P s cannot provide independent care of the high-risk pregnant client, but must work under a physician's supervision. The NP cannot do tertiary prevention as a major focus

1) The nurse is conducting an initial prenatal appointment for a client who believes she is pregnant. Which is considered a positive sign of pregnancy? A) Linea nigra B) Fetal heartbeat C) Breast tenderness D) Urinary frequency

Answer: B Explanation: A) Linea nigra is a probable, not positive, sign of pregnancy. B) A fetal heartbeat is a positive sign of pregnancy. C) Breast tenderness is a probable, not positive, sign of pregnancy. D) Urinary frequency is a probable, not positive, sign of pregnancy.

1) The nurse is working with a laboring woman who has a known intrauterine fetal demise. To facilitate the family's acceptance of the fetal loss, after delivery the nurse should do which of the following? A) Encourage the parents to look at the infant from across the room. B) Offer the parents the choice of holding the infant in their arms. C) Take the infant to the morgue immediately. Call family members and inform them of the birth

Answer: B Explanation: A) Looking at the infant from a distance would not be helpful. B) The nurse should offer the couple the opportunity to see and hold the infant and reassure the couple that any decision they make for themselves is the right one. C) Taking the infant to the morgue before the parents have a chance to see and hold it would not be appropriate. Calling family members would not be helpful or appropriate for the nurse to do

1) If oligohydramnios occurs in the first part of pregnancy, the nurse knows that there is a danger of which of the following? A) Major congenital anomalies B) Fetal adhesions C) Maternal diabetes D) Rh sensitization

Answer: B Explanation: A) Major congenital anomalies are associated with hydramnios. B) If oligohydramnios occurs in the first part of pregnancy, there is a danger of fetal adhesions (one part of the fetus may adhere to another part). C) Maternal diabetes is associated with hydramnios. Rh sensitization is associated with hydramnios

1) Dystocia encompasses many problems in labor. What is the most common? A) Meconium-stained amniotic fluid B) Dysfunctional uterine contractions C) Cessation of contractions D) Changes in the fetal heart rate

Answer: B Explanation: A) Meconium-stained amniotic fluid is a sign of nonreassuring fetal status. B) The most common problem is dysfunctional (or uncoordinated) uterine contractions that result in a prolongation of labor. C) Cessation of contractions is a symptom of possible uterine rupture. D) Changes in the fetal heart rate (F H R) are a sign of nonreassuring fetal status.

1) The nurse is preparing a class on reproduction. What is the cell division process called that results in two identical cells, each with the same number of chromosomes as the original cell? A) Meiosis B) Mitosis C) Oogenesis D) Gametogenesis

Answer: B Explanation: A) Meiosis is a process of cell division that leads to the development of ova and sperm. B) Mitosis results in the production of diploid body (somatic) cells, which are exact copies of the original cell. C) Oogenesis is the process that produces the female gamete, called an ovum (egg). D) Gametogenesis is the process by which germ cells, or gametes (ova and sperm), are produced.

1) The nurse teaching a high school class explains that during the menstrual cycle, the endometrial glands begin to enlarge under the influence of estrogen and cervical mucosal changes occur; the changes peak at ovulation. In which phase of the menstrual cycle does this occur? A) Menstrual B) Proliferative C) Secretory D) Ischemic

Answer: B Explanation: A) Menstruation occurs during the menstrual phase. B) The proliferative phase begins when the endometrial glands begin to enlarge under the influence of estrogen and cervical mucosal changes occur; the changes peak at ovulation. C) The secretory phase occurs after ovulation. The ischemic phase occurs if fertilization does not occur

1) The nurse is teaching a community education class on complementary and alternative therapies. To assess learning, the nurse asks, "In traditional Chinese medicine, what is the invisible flow of energy in the body that maintains health and ensures physiologic functioning?" Which answer indicates that teaching was successful? A) Meridians B) Chi C) Yin D) Yang

Answer: B Explanation: A) Meridians are the 14 pathways along which energy flows, connecting all parts of the body. B) Chi is the energy that flows through the body along meridians, or pathways, to maintain health. C) Yin and yang are opposites. Yin is the female force: passive, cool, wet, and close to the earth. D) Yin and yang are opposites. Yang is the masculine force: aggressive, hot, dry, and celestial.

1) A multiparous client delivered her first child vaginally 2 years ago, and delivered an infant by cesarean yesterday due to breech presentation. Which statement would the nurse expect the client to make? A) "I can't believe how much more tired I was with the first baby." B) "I'm having significantly more pain this time than with my last birth." C) "It is disappointing that I can't breastfeed because of the cesarean." D) "Getting in and out of bed feels more comfortable than last time."

Answer: B Explanation: A) Mothers who have experienced cesareans, particularly unanticipated ones that follow lengthy labors, may be fatigued, sleep deprived, and under the influence of medications that alter their level of consciousness. B) Women with cesarean births have special needs: increased need for rest and sleep; incisional care; self-care; and relief of pain and discomfort. C) Breastfeeding is not contraindicated by cesarean birth. D) Getting in and out of bed is more painful after cesarean birth than after vaginal birth. The nurse can assist the woman in identifying interventions to relieve discomfort or pain. The woman should be encouraged to take pain medication regularly, engage in frequent rest periods, avoid prolonged activity, and observe for signs of "overdoing it."

1) The nurse is teaching a newborn care class to parents who are about to give birth to their first babies. Which statement by a parent indicates that teaching was effective? A) "My baby will be able to focus on my face when she is about a month old." B) "My baby might startle a little if a loud noise happens near him." C) "Newborns prefer sour tastes." D) "Our baby won't have a sense of smell until she is older."

Answer: B Explanation: A) Newborns can focus on faces, eyes, and shiny objects at birth. B) Swaddling, placing a hand on the abdomen, or holding the arms to prevent a startle reflex are ways to soothe the newborn. The settled newborn is then able to attend to and interact with the environment. C) Newborns can distinguish between sweet and sour at 3 days of age. Sugar, for example, increases sucking, and newborns tend to have a preference for sweet tastes. D) Newborns develop the sense of smell rapidly and can differentiate their mother by smell within the first week of life.

1) To promote infant security in the hospital, the nurse instructs the parents of a newborn to do which of the following? A) Keep the baby in the room at all times. B) Check the identification of all personnel who transport the newborn. C) Place a "No Visitors" sign on the door. D) Keep the baby in the nursery at all times.

Answer: B Explanation: A) Newborns need to return to the nursery at times. B) Parents should be instructed to allow only people with proper birthing unit identification to remove the baby from the room. If parents do not know the staff person, they should call the nurse for assistance. C) A "No Visitors" sign would not ensure safety. D) Newborns will need to return to the nursery at times, but the newborn is not required to be there at all times.

1) The nurse receives a phone call from a client who claims she is pregnant. The client reports that she has regular menses that occur every 28 days and last 5 days. The first day of her last menses was April 10. What would the client's estimated date of delivery (E D D) be if she is pregnant? A) Nov. 13 B) Jan. 17 C) Jan. 10 D) Dec. 3

Answer: B Explanation: A) Nov. 13 is not correct according to Nagele's rule. B) The due date is Jan. 17. Nagele's rule is to add 7 days to the last menstrual period and subtract 3 months. The last menstrual period is April 10, therefore Jan. 17 is the E D D. C) Jan. 10 is not correct according to Nagele's rule. D) Dec. 3 is not correct according to Nagele's rule.

1) The nurse is planning care for four infants who were born on this shift. The infant who will require the most detailed assessment is the one whose mother has which of the following? A) A history of obsessive-compulsive disorder (O C D) B) Chlamydia C) Delivered six other children by cesarean section D) A urinary tract infection (U T I)

Answer: B Explanation: A) Obsessive-compulsive disorder (O C D) is not a risk factor for the infant. B) Infants born to mothers with chlamydia infections are at risk for neonatal pneumonia and conjunctivitis, and require close observation of the respiratory status and eyes. C) Having multiple siblings, regardless of how they were delivered, is not a risk factor for the infant. D) An infant whose mother has an untreated urinary tract infection might have been exposed to pathogens, but it is not known whether the mother in this question is on antibiotics.

1) The nurse is reviewing amniocentesis results. Which of the following would indicate that client care was appropriate? A) The client who is R h-positive received R h immune globulin after the amniocentesis. B) The client was monitored for 30 minutes after completion of the test. C) The client began vaginal spotting before leaving for home after the test. D) The client identified that she takes insulin before each meal and at bedtime.

Answer: B Explanation: A) Only R h-negative clients receive R h immune globulin after amniocentesis. The R h-positive client should not ever receive R h o G A M. B) 20 to 30 minutes of fetal monitoring is performed after the amniocentesis. C) Vaginal spotting after the amniocentesis is not an expected finding. Instruct the woman to report any changes or symptoms to her primary caregiver regarding vaginal discharge-either clear drainage or bleeding. D) Whether a client takes insulin has nothing to do with amniocentesis.

The nurse who is taking a sexual history from a client should do which of the following? A) Ask questions that the client can answer with "yes" or "no." B) Ask mostly open-ended questions. C) Have the client fill out a comprehensive questionnaire and review it after the client leaves. D) Try not to make much direct eye contact.

Answer: B Explanation: A) Open-ended questions are often useful in eliciting information. Yes-or-no answers will not provide the necessary information. B) Open-ended questions are often useful in eliciting information. C) Asking a client to fill out a questionnaire about sexual history is not appropriate. D) It is helpful to use direct eye contact as much as possible, unless culturally unacceptable.

1) A nurse is evaluating the diet plan of a breastfeeding mother. Which beverage is most likely to cause intolerance in the infant? A) Orange juice B) Milk C) Decaffeinated tea D) Water

Answer: B Explanation: A) Orange juice does not usually pose a problem for the infant. B) Often fussy breastfeeding or cow's milk-based formula-fed infants are switched to a lactose-free formula because of concerns about lactose intolerance. C) Decaffeinated tea does not usually present a problem for the infant. D) Water should not be a problem at any time.

1) The nurse has presented a community education class on recommended health screenings for women. Which statement about the Pap smear by a class member indicates that additional teaching is necessary? A) "It is recommended for women 21 years of age and older." B) "It diagnoses cervical cancer." C) "Intercourse at a young age is a risk factor for an abnormal Pap smear." D) "Detects abnormal cells."

Answer: B Explanation: A) Pap smear screening is recommended for all women 21 years of age and older. B) The focus of the Pap smear is the detection of high-grade cervical disease. It does not diagnose cervical cancer. C) Several factors put a woman at high risk for an abnormal Pap: intercourse at a young age, multiple partners, history of immunotherapy, long-term combined oral contraceptive (C O C) use, smoking, and previous history of dysplasia. The purpose of the Papanicolaou smear (Pap smear) is to screen for the presence of cellular abnormalities by obtaining a sample containing cells from the cervix and the endocervical canal

1) The nurse is working with a new mother who delivered yesterday. The mother has chosen to breastfeed her infant. Which demonstration of skill is the best indicator that the client understands breastfeeding? A) She puts the infant to breast when he is asleep to help wake him up. B) She takes off her gown to achieve skin-to-skin contact. C) She leans toward the infant so that he turns his head to access the nipple. D) The infant is crying when he is brought to the breast.

Answer: B Explanation: A) Parents can be taught techniques to wake their sleepy baby, as breastfeeding is more successful if the infant is awake. With a little help, the baby may be gently roused to breastfeed. B) Skin-to-skin contact after birth helps the baby maintain his or her body temperature, helps with self-regulation, increases maternal oxytocin levels, helps the mother to notice subtle feeding cues, and promotes bonding. C) The mother should bring the baby to her breast, rather than leaning forward to the baby. D) Crying is a late cue of hunger. Newborns should be put to breast when they begin rooting, lip-smacking, or fussing behaviors.

A menopausal woman tells her nurse that she experiences discomfort from vaginal dryness during sexual intercourse, and asks, "What should I use as a lubricant?" The nurse should recommend which of the following? A) Petroleum jelly B) A water-soluble lubricant C) Body cream or body lotion D) Less-frequent intercourse

Answer: B Explanation: A) Petroleum jelly is not a healthy choice for vaginal lubrication. B) A water-soluble jelly should be used. C) Body creams and body lotions are not healthy choices for vaginal lubrication. D) "Less-frequent intercourse" is an inappropriate response.

1) A client who is in the second trimester of pregnancy tells the nurse that she has developed a darkening of the line in the midline of her abdomen from the symphysis pubis to the umbilicus. What other expected changes during pregnancy might she also notice? A) Lightening of the nipples and areolas B) Reddish streaks called striae on her abdomen C) A decrease in hair thickness D) Small purplish dots on her face and arms

Answer: B Explanation: A) Pigmentation of the skin increases in areas already hyperpigmented: areolae, nipples, vulva, perianal area, and linea alba. B) Striae, or stretch marks, are reddish, wavy, depressed streaks that may occur over the abdomen, breasts, and thighs as pregnancy progresses. C) A greater percentage of hair follicles go into the dormant phase, resulting in less hair shedding, which is perceived as thickening of the hair. D) Although bright-red elevations on the skin (vascular spider nevi) are a normal finding, petechiae are not

1) The postpartum home care client asks the nurse why the visit is taking place. Which response is best? A) "We make home care visits to reinforce any teaching that you didn't quite grasp in the hospital." B) "We make home care visits to verify that both you and the baby are safe and doing well." C) "We make home care visits to ensure you are breastfeeding correctly." D) "We make home care visits to thoroughly assess your baby to make sure he is growing."

Answer: B Explanation: A) Postpartum home care provides opportunities for expanding information and reinforcing self- and infant-care techniques initially presented in the birth setting. However, this response is not therapeutically worded. B) Family well-being should be determined through a comprehensive assessment that includes physical, emotional, and social functioning. C) Although this statement is true if the mother is breastfeeding, it is only one component of the entire home healthcare postpartum visit. D) Assessing the baby is only part of the reason home care visits are performed after birth.

1) Couples at risk for having a detectable single gene or chromosomal anomaly may wish to undergo which procedure? A) Preimplantation genetic screening (P G S) B) Preimplantation genetic diagnosis (P G D) C) Intracytoplasmic sperm injection (I C S I) D) Gamete intrafallopian transfer (G I F T)

Answer: B Explanation: A) Preimplantation genetic screening (P G S) is a term used when the embryos are screened for aneuploidy for the purpose of increasing the likelihood of a viable pregnancy with normal chromosomes. B) Preimplantation genetic diagnosis (P G D) is a term used when one or both genetic parents carry a gene mutation and testing is performed to determine whether that mutation or unbalanced chromosomal compliment has been passed to the oocyte or embryo. C) Intracytoplasmic sperm injection (I C S I) is a microscopic procedure to inject a single sperm into the outer layer of an ovum so that fertilization will occur. D) Gamete intrafallopian transfer (G I F T) involves the retrieval of oocytes by laparoscopy.

1) The nurse is working in a teen pregnancy clinic. In order to give the pregnant adolescent a role in her prenatal care, the nurse should allow the teen to do which of the following? A) Choose the type of prenatal vitamin she takes B) Measure and record her weight at each visit C) Choose the schedule of her prenatal visits D) Decide whether she wants her labor to be induced

Answer: B Explanation: A) Prenatal vitamins are prescribed by healthcare professionals. The client would not be given a choice. B) Having the client weigh herself and record her weight provides her the opportunity to participate in her own care plan. C) Prenatal visit schedules are set to detect developing complications of pregnancy. D) Induction of labor is a medical decision, and not one to be made at the client's discretion.

1) Slowly removing some amniotic fluid is a treatment for hydramnios. What consequence can occur with the withdrawal of fluid? A) Preterm labor B) Prolapsed cord C) Preeclampsia D) Placenta previa

Answer: B Explanation: A) Preterm labor is not a known consequence of amniotic fluid reduction. B) A needle or a fetal scalp electrode is used to make a small puncture in the amniotic sac. There is a risk that the force of the fluid could make a larger hole in the amniotic sac, thus increasing the risk of a prolapsed cord. C) Preeclampsia is not a known consequence of amniotic fluid reduction. D) Placenta previa would not be a result of amniotic fluid reduction.

A client comes to the clinic complaining of severe menstrual cramps. She has never been pregnant, has been diagnosed with ovarian cysts, and has had an intrauterine device (I U D) for 2 years. What is the most likely cause for the client's complaint? A) Primary dysmenorrhea B) Secondary dysmenorrhea C) Menorrhagia D) Hypermenorrhea

Answer: B Explanation: A) Primary dysmenorrhea is defined as cramps without underlying disease. B) Secondary dysmenorrhea is associated with pathology of the reproductive tract, and usually appears after menstruation has been established. Conditions that most frequently cause secondary dysmenorrhea include ovarian cysts and the presence of an intrauterine device. C) Menorrhagia is excessive, profuse menstrual flow. D) Hypermenorrhea is an abnormally long menstrual flow.

1) A prenatal educator is asking a partner about normal psychologic adjustment of an expectant mother during the second trimester of pregnancy. Which answer by the partner would indicate a typical expectant mother's response to pregnancy? A) "She is very body-conscious, and hates every little change." B) "She daydreams about what kind of parent she is going to be." C) "I haven't noticed anything. I just found out she was pregnant." D) "She has been having dreams at night about misplacing the baby."

Answer: B Explanation: A) Psychologic adjustment to pregnancy is as significant as the physiologic changes. B) The second trimester brings increased introspection and consideration of how she will parent. She might begin to get furniture and clothing as concrete preparation, and may feel movement and be aware of the fetus as she begins to incorporate it into her identity. C) In the first trimester, pregnant women usually tell their partners of the pregnancy. This answer is incorrect. D) Psychologic adjustment to pregnancy is as significant as the physiologic changes. In the third trimester, dreams of misplacing the baby or being unable to get to the baby are common.

1) The nurse begins a prenatal assessment on a 25-year-old primigravida at 20 weeks' gestation and immediately contacts the healthcare provider because of which finding? A) Pulse 88/minute B) Respirations 30/minute C) Temperature 37.4°C (99.3°F) D) Blood pressure 118/82 m m H g

Answer: B Explanation: A) Pulse rate may increase 10-15 beats per minute during pregnancy, with an average of 60-100 beats per minute. B) Tachypnea is not a normal finding and requires medical care. C) A slightly higher temperature is an expected finding during pregnancy, ranging from 36.2°C-37.6°C (97°F-99.6°F). A blood pressure of less than or equal to 120/80 mmHg is considered normal

1) Which myth regarding rape will the community health nurse include in a teaching session within the community? A) Rape is a type of sexual assault. B) Women lie about rape as an act of revenge. C) Both men and women can be victims of rape. Rape is one of the most underreported violent crimes

Answer: B Explanation: A) Rape is a type of sexual assault. This is not a myth regarding rape. B) One myth regarding rape is that women lie about rape as an act of revenge. This is appropriate for the nurse to include in the teaching session. C) Both men and women can be victims of rape. This is not a myth regarding rape. Rape is one of the most underreported violent crimes. This is not a myth regarding rape

1) The nurse receives a phone call from a 25-year-old woman experiencing breast tenderness in the week prior to her menses, with palpable breast nodularity, without nipple discharge. What is the best response by the nurse? A) "Please make an appointment at the breast cancer center as soon as possible." B) "How much salty food do you regularly consume?" C) "As long as you don't have nipple discharge, it isn't a serious condition." D) "Eliminate caffeine and chocolate from your diet."

Answer: B Explanation: A) Recommending that the client make an appointment at the breast cancer center is unnecessary and might frighten the client. B) The client is describing fibrocystic breast changes. A salt restriction with a mild diuretic taken the week before menstrual bleeding often improves the condition. C) Neither the absence nor presence of nipple discharge is an indicator of the seriousness of a breast condition. D) Research is inconclusive as to whether eliminating methylxanthines from the diet is effective at reducing the symptoms the client describes.

15) Upon delivery of the newborn, what nursing intervention most promotes parental attachment? A) Placing the newborn under the radiant warmer. B) Placing the newborn on the mother's abdomen. C) Allowing the mother a chance to rest immediately after delivery. D) Taking the newborn to the nursery for the initial assessment.

Answer: B Explanation: A) Removing the baby from the mother does not promote attachment. B) As the baby is placed on the mother's abdomen or chest, she frequently reaches out to touch and stroke her baby. When the newborn is placed in this position, the father or partner also has a very clear, close view and can reach out to touch the baby. C) Removing the baby from the mother does not promote attachment. D) Removing the baby from the mother does not promote attachment.

1) The postpartum client expresses concern about getting back to her prepregnant shape, and asks the nurse when she will be able to run again. Which statement by the client indicates that teaching was effective? A) "I can start running in 2 weeks, and can breastfeed as soon as I am done." B) "I should see how my energy level is at home, and increase my activity slowly." C) "Running is not recommended for breastfeeding women." D) "If I am getting 8 hours of sleep per day, I can start running."

Answer: B Explanation: A) Running should not be initiated until after 6 weeks postpartum or with medical approval. The nurse can provide the new mother with suggestions for resuming her normal level of activity. Breastfeeding should take place prior to running to minimize chest discomfort. B) Women should be encouraged to limit the number of activities to prevent excessive fatigue, increase in lochia, and negative psychologic reactions, such as feeling overwhelmed. A regular exercise program including vigorous activities such as running, weight lifting, or competitive sports can usually be initiated after the 6-week postpartum examination or when approved by the client's physician/C N M. C) This statement is not true. It is more comfortable to nurse prior to running, but running is not contraindicated and can usually be initiated after the 6-week postpartum examination or when approved by the client's physician/C N M. D) This response does not address a more important factor, which is encouraging the client to assess her own energy level and to gradually return to previous activity levels.

1) Mild or chronic anemia in an infant may be treated adequately by which of the following? A) Transfusions with O-negative or typed and cross-matched packed red cells B) Iron supplements or iron-fortified formulas C) Steroid therapy D) Antibiotics or antivirals

Answer: B Explanation: A) Severe cases of anemia are treated with transfusions with O-negative or typed and cross-matched packed red cells. B) Mild or chronic anemia in an infant may be treated adequately with iron supplements or iron-fortified formulas. C) Management of anemia of prematurity includes treating the causative factor (e.g., antibiotics or antivirals used for infection or steroid therapy for disorders of erythrocyte production). D) Management of anemia of prematurity includes treating the causative factor (e.g., antibiotics or antivirals used for infection or steroid therapy for disorders of erythrocyte production).

1) The nurse is preparing a class for expectant fathers. Which information should the nurse include? A) Siblings adjust readily to the new baby. B) Sexual activity is safe for normal pregnancy. C) The expectant mother decides the feeding method. D) Fathers are expected to be involved in labor and birth.

Answer: B Explanation: A) Siblings often have difficulty adapting to the arrival of a new baby. Parents who are unprepared for the older child's feelings of anger, jealousy, and rejection may respond inappropriately in their confusion and surprise. B) In a healthy pregnancy, there is no medical reason to limit sexual activity. C) Often, the father wants input in deciding on the feeding method. D) In some cultures, the father is not present at birth. The nurse should recognize the importance of birth practices that are part of a family's tradition and honor these practices when possible.

1) The community nurse is working with poor women who are formula-feeding their infants. Which statement indicates that the nurse's education session was effective? A) "I should use only soy-based formula for the first year." B) "I should follow the instructions for mixing the powdered formula exactly." C) "It is okay to add more water to the formula to make it last longer." D) "The mixed formula can be left on the counter for a day."

Answer: B Explanation: A) Soy protein-based formula is not intended as a first-choice formula except for infants with primary lactase deficiency or galactosemia, for term infants of formula-feeding vegan parents, and for infants who develop secondary (transient) lactase deficiency following an acute bout with diarrhea. B) Powdered formula is the least expensive type of formula. Parents will need to be briefed on safety precautions during formula preparation, and they should be instructed to follow the directions on the formula package label precisely as written. C) Parents should know that adding too much water during preparation dilutes the nutrients and caloric density and may contribute to undernourishment, insufficient weight gain, and possibly water intoxication leading to hyponatremia and seizures. Not adding enough water concentrates nutrients and calories and can damage an infant's immature kidneys and digestive system, as well as cause dehydration. Allow freshly prepared (unused) formula to sit out at room temperature for no longer than 2 hours. Milk left over in the bottle after a feeding should be discarded

14) A client presents to the antepartum clinic with a history of a 20-pound weight loss. Her pregnancy test is positive. She is concerned about gaining the weight back, and asks the nurse if she can remain on her diet. What is the nurse's best response? A) "As long as you supplement your diet with the prenatal vitamin, the amount of weight you gain in pregnancy is not significant." B) "I understand that gaining weight after such an accomplishment might not appeal to you but weight gain during pregnancy is important for proper fetal growth." C) "Dieting during pregnancy is considered child neglect." D) "Excessive weight gain in pregnancy is due to water retention, so weight loss following birth will not be an issue."

Answer: B Explanation: A) Supplementation with vitamins is important, but so is maintaining weight gain within the expected parameters. B) Maternal weight gain is an important factor in fetal growth and in infant birth weight. An adequate weight gain over time indicates an adequate caloric intake. Child neglect can apply only after the child is born D) Weight gain during pregnancy typically is not water-related. Excess weight gain can be difficult to lose.

1) Which of the following functions primarily to provide low-income women and children who are at risk for medical or nutritional problems with nutritious foods to supplement their diets, nutrition education and counseling, and screening and referrals to other health, welfare, and social programs? A) A B M B) W I C C) I L C A D) L L L I

Answer: B Explanation: A) The Academy of Breastfeeding Medicine (A B M) provides many research-based breastfeeding protocols. B) The Supplemental Nutrition Program for Women, Infants, and Children (W I C) functions primarily to provide low-income women and children who are at risk for medical or nutritional problems with nutritious foods to supplement their diets, nutrition education and counseling, and screening and referrals to other health, welfare, and social programs. C) International Lactation Consultant Association (I L C A) has listings of lactation consultants in specific geographic areas. D) La Leche League International (L L L I) is the first (not-for-profit international educational and service organization) mother-to-mother breastfeeding support group formally recognized in the United States.

15) The neonate was born 5 minutes ago. The body is bluish. The heart rate is 150. The infant is crying strongly. The infant cries when the sole of the foot is stimulated. The arms and legs are flexed, and resist straightening. What should the nurse record as this infant's Apgar score? A) 7 B) 8 C) 9 D) 10

Answer: B Explanation: A) The Apgar score would be higher; only the skin color needs to be subtracted from the score. B) The strong cry earns 2 points. The crying with foot sole stimulation earns 2 points. The limb flexion and resistance earn 2 points each. Bluish color earns 0 points. The Apgar score is 8. C) The Apgar score would be lower than 9 because of the skin color. D) Ten is a perfect score. The nurse needs to subtract for skin color.

1) The home care nurse is seeing a client at 6 weeks postpartum. Which statement by the client indicates the need for immediate intervention? A) "The baby sleeps 7 hours each night now." B) "My flow is red, and I need to wear a pad." C) "My breasts no longer leak between feedings." D) "I started back on the pill 2 weeks ago."

Answer: B Explanation: A) The baby sleeping 7 hours a night by this time is an expected finding, and does not require intervention. B) By 6 weeks postpartum, lochia should be absent or minimal in amount, requiring only a pantiliner. Red, heavy flow is not an expected finding, and requires intervention. C) The mother's breasts no longer leaking between feedings is an expected finding, and does not require intervention. D) Six week postpartum is an appropriate time frame for restarting birth control pills, and does not require intervention.

27) Carbohydrates provide the body's primary source of energy as well as fiber necessary for proper bowel functioning. If the carbohydrate intake is not adequate, the body will use which of the following for energy? A) Iron B) Protein C) Vitamin C D) Vitamin D

Answer: B Explanation: A) The body would not use iron as a source of energy. B) If the carbohydrate intake is not adequate, the body uses protein for energy. Protein then becomes unavailable for growth needs. C) A major function of vitamin C is to aid the formation and development of connective tissue and the vascular system. D) Vitamin D is best known for its role in the absorption and utilization of calcium and phosphorus in skeletal development.

10) The breastfeeding mother is concerned that her milk production has decreased. The nurse knows that further client teaching is needed based on which statement? A) "I am drinking a minimum of 8 to 10 glasses of liquid a day." B) "I have started cutting back on my protein intake." C) "At least three times a day, I drink a glass of milk." D) "My calorie intake is higher than during the pregnancy."

Answer: B Explanation: A) The breastfeeding mother must consume a minimum of 8 to 10 glasses of liquid per day. B) An adequate protein intake is essential while breastfeeding because protein is an important component of breast milk. C) The breastfeeding mother must increase her protein and calcium intake. D) An inadequate caloric intake can reduce milk volume. Breastfeeding mothers should increase their caloric intake by 200 kcal over the pregnancy requirements.

1) The laboring client is at 7 c m, with the vertex at a +1 station. Her birth plan indicates that she and her partner took Lamaze prenatal classes, and they have planned on a natural, unmedicated birth. Her contractions are every 3 minutes and last 60 seconds. She has used relaxation and breathing techniques very successfully in her labor until the last 15 minutes. Now, during contractions, she is writhing on the bed and screaming. Her labor partner is rubbing the client's back and speaking to her quietly. Which nursing diagnosis should the nurse incorporate into the plan of care for this client? A) Fear/Anxiety related to discomfort of labor and unknown labor outcome B) Pain, Acute, related to uterine contractions, cervical dilatation, and fetal descent C) Coping: Family, Compromised, related to labor process D) Knowledge, Deficient, related to lack of information about normal labor process and comfort measures

Answer: B Explanation: A) The client is not exhibiting fear or anxiety, but acute pain; therefore, this diagnosis does not fit. B) The client is exhibiting signs of acute pain, which is both common and expected in the transitional phase of labor. C) There is no evidence regarding the family's coping, only the client's coping with the pain. The client used breathing and relaxation techniques earlier in labor, demonstrating knowledge of these techniques

1) A postpartum client has just received a rubella vaccination. The client demonstrates understanding of the teaching associated with administration of this vaccine when she states which of the following? A) "I will need another vaccination in 3 months." B) "I must avoid getting pregnant for 1 month." C) "This will prevent me from getting chickenpox." D) "This will protect my newborn from getting the measles."

Answer: B Explanation: A) The client will not need another vaccination in 3 months. B) The client must avoid pregnancy for at least 1 month after receiving the rubella vaccine. C) The vaccination prevents measles, not chickenpox. D) The vaccination will only protect the client receiving it; therefore, the newborn will not be protected until the child receives his own vaccination.

1) After reviewing approaches for contraception with a female client, the nurse is concerned that barrier methods will not achieve the client's goal to prevent pregnancy. What did the client say to cause the nurse to draw this conclusion? A) "My partner doesn't mind wearing condoms." B) "I don't want to have to put anything in myself." C) "We should use a condom even with a diaphragm." D) "I know that spermicides are inserted before intercourse."

Answer: B Explanation: A) The client's request to not have to insert anything to prevent pregnancy indicates that the barrier method of a female diaphragm would not be an appropriate method of birth control because the client will not be motivated to use it. Wearing condoms and appropriately using spermicides indicates that the client would adhere to the use of a barrier method. B) The client's request to not have to insert anything to prevent pregnancy indicates that the barrier method of a female diaphragm would not be an appropriate method of birth control because the client will not be motivated to use it. Wearing condoms and appropriately using spermicides indicates that the client would adhere to the use of a barrier method. C) The client's request to not have to insert anything to prevent pregnancy indicates that the barrier method of a female diaphragm would not be an appropriate method of birth control because the client will not be motivated to use it. Wearing condoms and appropriately using spermicides indicates that the client would adhere to the use of a barrier method. D) The client's request to not have to insert anything to prevent pregnancy indicates that the barrier method of a female diaphragm would not be an appropriate method of birth control because the client will not be motivated to use it. Wearing condoms and appropriately using spermicides indicates that the client would adhere to the use of a barrier method.

1) A client received epidural anesthesia during the first stage of labor. The epidural is discontinued immediately after delivery. This client is at increased risk for which problem during the fourth stage of labor? A) Nausea B) Bladder distention C) Uterine atony Hypertension

Answer: B Explanation: A) The epidural is discontinued after delivery, decreasing the likelihood of nausea. B) Nursing care following an epidural block includes frequent assessment of the bladder to avoid bladder distention. C) Uterine atony is not a result of epidurals. D) Hypotension, not hypertension, is an early side effect of epidurals.

1) The nurse is doing preconception counseling with a 28-year-old woman with no prior pregnancies. Which statement made by the client indicates to the nurse that the client has understood the teaching? A) "I can continue to drink alcohol until I am diagnosed as pregnant." B) "I need to stop drinking alcohol completely when I start trying to get pregnant." C) "A beer once a week will not damage the fetus." D) "I can drink alcohol while breastfeeding because it doesn't pass into breast milk."

Answer: B Explanation: A) The expectant woman should avoid alcohol completely. B) Women should discontinue drinking alcohol when they start to attempt to become pregnant due to possible effects of alcohol on the fetus. C) The effects of alcohol on the fetus may include a group of signs referred to as fetal alcohol spectrum disorders (F A S D). D) Alcohol is excreted in breast milk; therefore, it should be avoided.

1) A client at 37 weeks' gestation has a mildly elevated blood pressure. Her antenatal testing demonstrates three contractions in 10 minutes, no decelerations, and accelerations four times in 1 hour. What would this test be considered? A) Positive nonstress test B) Negative contraction stress test C) Positive contraction stress test D) Negative nonstress test

Answer: B Explanation: A) The fact that contractions are present rules out the nonstress test. B) A negative C S T shows three contractions of good quality lasting 40 or more seconds in 10 minutes without evidence of late decelerations. This is the desired result. C) A positive C S T shows repetitive persistent late decelerations with more than 50% of the contractions. This is not a desired result. D) The fact that contractions are present rules out the nonstress test.

1) The prenatal clinic nurse is designing a new prenatal intake information form for pregnant clients. Which question is best to include on this form? A) Where was the father of the baby born? B) Do genetic diseases run in the family of the baby's father? C) What is the name of the baby's father? D) Are you married to the father of the baby?

Answer: B Explanation: A) The father's place of birth is not important information to include about the pregnancy. B) This question has the highest priority because it gets at the physiologic issue of inheritable genetic diseases that might directly impact the baby. C) Although it is helpful for the nurse to know the name of the baby's father to include him in the prenatal care, this is psychosocial information. D) Although the marital status of the client might have cultural significance, this is psychosocial information.

1) During the nursing assessment of a woman with ruptured membranes, the nurse suspects a prolapsed umbilical cord. What would the nurse's priority action be? A) To help the fetal head descend faster B) To use gravity and manipulation to relieve compression on the cord C) To facilitate dilation of the cervix with prostaglandin gel D) To prevent head compression

Answer: B Explanation: A) The fetal head's descent would put additional pressure on the umbilical cord and reduce blood flow and oxygenation to the fetus. B) The top priority is to relieve compression on the umbilical cord to allow blood flow to reach the fetus. It is because some obstetric maneuvers to relieve cord compression are complicated that cesarean birth is sometimes necessary. C) Further dilatation of the cervix is unnecessary in light of a possible cesarean section. D) Head compression is not a concern in the case of prolapsed umbilical cord. The cord is what is compressed.

1) The nurse has just assisted the father in bathing the newborn 2 hours after birth. The nurse explains that the newborn must remain in the radiant warmer. This is based on which assessment data? A) Heart rate 120 B) Temperature 96.8°F C) Respiratory rate 50 D) Temperature 99.6°F

Answer: B Explanation: A) The heart rate is within normal limits for a newborn 2 hours old. B) The nurse rechecks the temperature after the bath and, if it is stable, dresses the newborn in a shirt, diaper, and cap; wraps the baby; and places the baby in an open crib at room temperature. If the baby's axillary temperature is below 36.5°C (97.7°F), the nurse returns the baby to the radiant warmer. The rewarming process should be gradual to prevent the possibility of hyperthermia. C) The respiratory rate is within normal limits for a newborn 2 hours old. D) This temperature (99.6°F) does not warrant placing the infant back in the radiant warmer.

1) What indications would lead the nurse to suspect sepsis in a newborn? A) Respiratory distress syndrome developing 48 hours after birth B) Temperature drops from 97.4°F to 97.0 2°F hours after 2 hours of warming. C) Irritability and flushing of the skin at 8 hours of age D) Bradycardia and tachypnea developing when the infant is 36 hours old

Answer: B Explanation: A) The infant may deteriorate rapidly in the first 12 to 24 hours after birth if β-hemolytic streptococcal infection is present. B) Temperature instability is often seen with sepsis. Fever is rare in a newborn. C) Irritability or lethargy with pallor after the first 24 hours might indicate sepsis, and the skin is cool and clammy. Tachycardia and periods of apnea are seen with sepsis

1) Of all the clients who have been scheduled to have a biophysical profile, the nurse should check with the physician to clarify the order for which client? A) A gravida with intrauterine growth restriction B) A gravida with mild hypotension of pregnancy C) A gravida who is postterm D) A gravida who complains of decreased fetal movement for 2 days

Answer: B Explanation: A) The infant who has intrauterine growth problems might be compromised due to placental insufficiency. B) The biophysical profile is used when there is a risk of placental and/or fetal compromise. The gravida with mild hypotension will need to be monitored more closely throughout the pregnancy, but is not a candidate at present for a biophysical profile. C) The infant who is postterm might be compromised due to placental insufficiency. D) The gravida who is experiencing decreased fetal movement for 2 days needs assessment of the placenta and the fetus.

1) The nursing instructor is conducting a class about attachment behaviors. Which statement by a student indicates the need for further instruction? A) "The en face position promotes bonding and attachment." B) "Ideally, initial skin-to-skin contact occurs after the baby has been assessed and bathed." C) "In reciprocity, the interaction of mother and infant is mutually satisfying and synchronous." D) "The needs of the mother and of her infant are balanced during the phase of mutual regulation."

Answer: B Explanation: A) The mother has direct face-to-face and eye-to-eye contact in the en face position, which is crucial to attachment. B) Ideally, initial skin-to-skin contact is immediate. The benefits of this practice are supported by a preponderance of evidence. C) Reciprocity is an interactional cycle that occurs simultaneously between mother and infant. It involves mutual cuing behaviors, expectancy, rhythmicity, and synchrony. D) In the phase of mutual regulation, mother and infant seek to deal with the degree of control to be exerted by each in their relationship.

1) A nurse is evaluating the diet plan of a breastfeeding mother, and determines that her intake of fruits and vegetables is inadequate. The nurse explains that the nutritional composition of the mother's breast milk can be adversely affected by this nutritional inadequacy. Which strategy should the nurse recommend to the mother? A) Stop breastfeeding B) Provide newborn supplements to the newborn C) Offer whole milk D) Supplement with skim milk

Answer: B Explanation: A) The mother may continue to breastfeed. B) The mother may continue to breastfeed, but the caregiver may choose to prescribe additional vitamins for the newborn. Vitamins in human milk are influenced by the mother's vitamin intake, general nutritional status, and genetic differences. C) Cow's milk should not be given to infants before 1 year of age. The use of skim milk or low-fat cow's milk is not recommended for children under 2 years old. D) Cow's milk should not be given to infants before 1 year of age. The use of skim milk or low-fat cow's milk is not recommended for children under 2 years old.

1) The nurse is presenting a class to pregnant clients. The nurse asks, "The fetal brain is developing rapidly, and the nervous system is complete enough to provide some regulation of body function on its own, at which fetal development stage?" It is clear that education has been effective when a participant makes which response? A) "The 17th-20th week" B) "The 25th-28th week" C) "The 29th-32nd week" D) "The 33rd-36th week"

Answer: B Explanation: A) The nervous system function is not developed between the 17th and 20th weeks of gestation. B) Between the 25th and 28th week, the brain is developing rapidly, and the nervous system is complete enough to provide some degree of regulation of body functions. C) The complexity of the nervous system develops long before the 29th-32nd week. D) The complexity of the nervous system develops long before the 33rd-36th week.

1) The nurse is caring for a newborn who was recently circumcised. Which nursing intervention is appropriate following the procedure? A) Keep the infant N P O for 4 hours following the procedure. B) Observe for urine output. C) Wrap dry gauze tightly around the penis. D) Clean with cool water with each diaper change.

Answer: B Explanation: A) The newborn does not need to be N P O. B) It is important to observe for the first voiding after a circumcision to evaluate for urinary obstruction related to penile injury and/or edema. C) Gauze should not be wrapped tightly around the penis. Only if bleeding occurs should the nurse apply light pressure with a sterile gauze pad to stop the bleeding within a short time. The newborn should be cleaned with warm water with each diaper change

1) A client in labor is found to have meconium-stained amniotic fluid upon rupture of membranes. At delivery, the nurse finds the infant to have depressed respirations and a heart rate of 80. What does the nurse anticipate? A) Delivery of the neonate on its side with head up, to facilitate drainage of secretions. B) Direct tracheal suctioning by specially trained personnel. C) Preparation for the immediate use of positive pressure to expand the lungs. D) Suctioning of the oropharynx when the newborn's head is delivered.

Answer: B Explanation: A) The newborn is not delivered on its side. B) If the infant has absent or depressed respirations, heart rate less than 100 beats/min, or poor muscle tone, direct tracheal suctioning by specially trained personnel is recommended. C) Positive pressure is not used to expand the lungs. Current evidence does not support intrapartum oropharyngeal and nasopharyngeal suctioning as they do not prevent or alter the course of MAS

1) The nurse is assisting a mother to bottle-feed her newborn, who has been crying. The nurse suggests that prior to feeding, the mother should do which of the following? A) Offer a pacifier B) Burp the newborn C) Unwrap the newborn D) Stroke the newborn's spine and feet

Answer: B Explanation: A) The newborn's cries are indicative of an issue; a pacifier would not solve the problem. B) Crying results in increased ingestion of air even before the infant has started feeding. Infants who are very hungry also gulp more air. For these situations, instruct the parents to burp their infant frequently. C) Unwrapping stimulates the newborn. D) Stroking the spine and feet stimulates the newborn.

1) Which statement by a nursing student preparing to care for a postpartum lesbian mother would indicate that the student is prepared for the teaching? A) "I can't let the client know I've never worked with lesbian mothers." B) "I will have to adjust some of my discharge instruction for this mother." C) "I don't need to include the partner when I provide care and instruction." D) "Discharge teaching is exactly the same for lesbian mothers as for all others."

Answer: B Explanation: A) The nurse should ask the patient for guidance regarding any special needs or requests that she or her partner may have. B) The nurse should be aware that standardized postpartum instructions, particularly those related to intercourse and contraception, might need to be individualized and amended. C) Providing quality patient-centered care for any postpartum woman involves acknowledging, welcoming, and involving her intimate partner in care and decision making. D) The nurse should be aware that standardized postpartum instructions, particularly those related to intercourse and contraception, might need to be individualized and amended.

1) The pregnant client has asked the nurse what kinds of medications cause birth defects. Which statement would best answer this question? A) "Birth defects are very rare. Don't worry; your doctor will watch for problems." B) "To be safe, don't take any medication without talking to your doctor." C) "Too much vitamin C is one of the most common issues." D) "Almost all medications will cause birth defects in the first trimester."

Answer: B Explanation: A) The nurse should avoid giving a "don't worry" answer to ensure therapeutic communication, but it is appropriate to instruct the client to check with her caregiver about medications. B) The nurse should remind the client of the need to check with her caregiver about medications. If a woman has taken a drug in category D or X, she should be informed of the risks associated with that drug and of alternatives. C) Vitamin C is cited as a category A drug, as long as its use does not exceed the recommended dietary allowance. It has demonstrated no associated fetal risk. D) Not all medications are teratogenic.

1) A Chinese woman who is 12 weeks pregnant reports to the nurse that ginseng and bamboo leaves help reduce her anxiety. How should the nurse respond to this client? A) Advise the client to give up the bamboo leaves but to continue taking ginseng. B) Advise the client to discuss all herbal remedies with the provider. C) Tell the client that the provider thinks the remedies have no scientific foundation. D) Assess where the client obtains her remedy, and investigate the source.

Answer: B Explanation: A) The nurse should find out what medications and home remedies the client is using, and counsel the client regarding overall effects. B) The nurse should advise the client to discuss all medications (including herbal supplements) with the health care provider. C) It is not appropriate to tell the client that the provider thinks the remedies have no scientific foundation. D) It is outside the nurse's scope to assess the source of the herbs.

1) Nurses who are interacting with expectant families from a different culture or ethnic group can provide more effective, culturally sensitive nursing care by doing what? A) Recognizing that ultimately it is the family's right to make a woman's healthcare choices. B) Obtaining a medical interpreter of the language the client speaks. C) Evaluating whether the client's healthcare beliefs have any positive consequences for her health. D) Accepting personal biases, attitudes, stereotypes, and prejudices.

Answer: B Explanation: A) The nurse should recognize that ultimately it is the woman's right to make her own healthcare choices. B) The nurse should provide for the services of an interpreter if language barriers exist. C) The nurse should evaluate whether the client's healthcare beliefs have any potential negative consequences for her health. D) The nurse should identify personal biases, attitudes, stereotypes, and prejudices.

1) A client at 16 weeks' gestation has a hematocrit of 35%. Her prepregnancy hematocrit was 40%. Which statement by the nurse best explains this change? A) "Because of your pregnancy, you're not making enough red blood cells." B) "Because your blood volume has increased, your hematocrit count is lower." C) "This change could indicate a serious problem that might harm your baby." D) "You're not eating enough iron-rich foods like meat."

Answer: B Explanation: A) The pregnancy would not cause a decrease in the production of red blood cells. B) Hemoglobin and hematocrit levels drop in early to mid-pregnancy as a result of pregnancy-associated hemodilution. Because the plasma volume increase (50%) is greater than the erythrocyte increase (25%), the hematocrit decreases slightly. C) This change is referred to as physiologic anemia of pregnancy, and is not harmful to the fetus. D) The decreased hematocrit does not mean that the woman is not eating enough iron-rich foods. It is recommended that an iron supplement during pregnancy of 27 milligrams of iron be taken daily, and iron can be found in most prenatal supplements.

1) A 25-year-old primigravida is at 20 weeks' gestation. The nurse takes her vital signs and notifies the healthcare provider immediately because of which finding? A) Pulse 88/minute B) Rhonchi in both bases C) Temperature 37.4°C (99.3°F) D) Blood pressure 122/78 m m H g

Answer: B Explanation: A) The pulse will increase 10-15 beats/minute during pregnancy, with 60-90 beats/minute being the normal range. B) Any abnormal breath sounds should be reported to the healthcare provider. C) Temperature norms in pregnancy are slightly higher due to fetal metabolism: 36.2-37.6°C (97-99.6°F). D) A blood pressure less than or equal to 120/80 m m H g considered normal.

1) The nurse is admitting a client to the birthing unit. What question should the nurse ask to gain a better understanding of the client's psychosocial status? A) "How did you decide to have your baby at this hospital?" B) "Who will be your labor support person?" C) "Have you chosen names for your baby yet?" D) "What feeding method will you use for your baby?"

Answer: B Explanation: A) The reason the client is delivering at this facility is not an indication of psychosocial status. B) The expectant mother's partner or support person is an important member of the birthing team, and assessments of the couple's coping, interactions, and teamwork are integral to the nurse's knowledge base. The nurse's physical presence with the laboring woman provides the best opportunity for ongoing assessment. C) Naming the infant is influenced by culture, and is not an indicator of psychosocial status. The chosen feeding method is not an indicator of psychosocial status

1) The physician/C N M opts to use a vacuum extractor for a delivery. What does the nurse understand? A) There is little risk with vacuum extraction devices. B) There should be further fetal descent with the first two "pop-offs." C) Traction is applied between contractions. D) The woman often feels increased discomfort during the procedure.

Answer: B Explanation: A) The risk of complications rises with the use of a vacuum extraction device. B) If more than three "pop-offs" occur (the suction cup pops off the fetal head), the procedure should be discontinued. C) The physician/C N M applies traction in coordination with uterine contractions, not between contractions. D) If adequate regional anesthesia has been administered, the woman feels only pressure during the procedure.

1) Which of the following best describes a nuclear family? A) An unmarried woman who chooses to conceive or adopt without a life partner. B) Children live in a household with both biologic parents and no other relatives or persons. C) A couple shares household and childrearing responsibilities with parents, siblings, or other relatives. D) The head of the household is widowed, divorced, abandoned, separated, or most often, the mother remains unmarried.

Answer: B Explanation: A) The single mother by choice family represents a family composed of an unmarried woman who chooses to conceive or adopt without a life partner. B) In the nuclear family, children live in a household with both biologic parents and no other relatives or persons. C) In an extended family, a couple shares household and childrearing responsibilities with parents, siblings, or other relatives. D) In the single-parent family, the head of the household is widowed, divorced, abandoned, separated, or most often, the mother remains unmarried.

The laboring client and her partner have arrived at the birthing unit. Which step of the admission process should be undertaken first? A) The sterile vaginal exam B) Welcoming the couple C) Auscultation of the fetal heart rate D) Checking for ruptured membranes

Answer: B Explanation: A) The sterile vaginal exam should be performed after maternal vital signs have been assessed. B) It is important to establish rapport and to create an environment in which the family feels free to ask questions. The support and encouragement of the nurse in maintaining a caring environment begins with the initial admission. C) The fetal heart rate should be listened to after the client is made comfortable. D) Assessing for intact or ruptured membranes is a part of the admission assessment after the client is made comfortable.

1) A nurse is performing an assessment on a family with a father and mother who both work. What type of family does she record this family as being? A) A traditional nuclear family B) A dual-career/dual-earner family C) An extended family D) An extended kin family

Answer: B Explanation: A) The traditional nuclear family is defined as a husband provider, a wife who stays home, and children. B) A dual-career/dual-earner family is characterized by both parents working, either by choice or necessity. C) An extended family is defined as a couple who share household and childrearing responsibilities with parents, siblings, or other relatives. D) An extended kin family is a specific form of an extended family in which two nuclear families of primary or unmarried kin live in close proximity to each other.

1) The client has been pushing for two hours, and is exhausted. The fetal head is visible between contractions. The physician informs the client that a vacuum extractor could be used to facilitate the delivery. Which statement indicates that the client needs additional information about vacuum extraction assistance? A) "A small cup will be put onto the baby's head, and a gentle suction will be applied." B) "I can stop pushing and just rest if the vacuum extractor is used." C) "The baby's head might have some swelling from the vacuum cup." D) "The vacuum will be applied for a total of ten minutes or less."

Answer: B Explanation: A) The vacuum extractor is a small cup-shaped device that is applied to the fetal scalp. B) Vacuum extraction is an assistive delivery. The physician/C N M applies traction in coordination with uterine contractions. C) The vacuum extractor might leave some swelling ("chignon") on the scalp where the device is placed. D) Research indicates that negative suction applied for more than 10 minutes is associated with a greater incidence of scalp injury.

1) A female client comes into the clinic for a pregnancy test because she took the morning after pill immediately after having unprotected intercourse 3 days ago and has not had a menstrual period. What should the nurse respond to this client? A) "I'll make sure you have one during this visit." B) "You should wait for two weeks before having a pregnancy test." C) "It's unlikely that you are pregnant. Wait a few days and then take a pregnancy test." D) "How long did you wait to take the morning after pill after having unprotected intercourse?"

Answer: B Explanation: A) The woman should have her normal menses 2 weeks after taking emergency contraception. If she does not, she should follow up with a pregnancy test. A pregnancy test is not needed during this visit. The nurse has no way of knowing if the client is pregnant. The morning after pill should ideally be taken within 72 hours after having unprotected intercourse but can be taken up to 5 days after unprotected intercourse. B) The woman should have her normal menses 2 weeks after taking emergency contraception. If she does not, she should follow up with a pregnancy test. A pregnancy test is not needed during this visit. The nurse has no way of knowing if the client is pregnant. The morning after pill should ideally be taken within 72 hours after having unprotected intercourse but can be taken up to 5 days after unprotected intercourse. C) The woman should have her normal menses 2 weeks after taking emergency contraception. If she does not, she should follow up with a pregnancy test. A pregnancy test is not needed during this visit. The nurse has no way of knowing if the client is pregnant. The morning after pill should ideally be taken within 72 hours after having unprotected intercourse but can be taken up to 5 days after unprotected intercourse. The woman should have her normal menses 2 weeks after taking emergency contraception. If she does not, she should follow up with a pregnancy test. A pregnancy test is not needed during this visit. The nurse has no way of knowing if the client is pregnant. The morning after pill should ideally be taken within 72 hours after having unprotected intercourse but can be taken up to 5 days after unprotected intercourse.

1) A fetal weight is estimated at 4490 grams in a client at 38 weeks' gestation. Counseling should occur before labor regarding which of the following? A) Mother's undiagnosed diabetes B) Likelihood of a cesarean delivery C) Effectiveness of epidural anesthesia with a large fetus D) Need for early delivery

Answer: B Explanation: A) There is a possibility of undiagnosed diabetes, but that is not the current concern because the client is close to delivery. B) The likelihood of a cesarean delivery with a fetus over 4000 grams is high. This should be discussed with the client before labor. C) The weight of the fetus has no bearing on the effectiveness of epidural anesthesia. D) The client is already at term, so it is too late to discuss an early delivery.

1) During labor, the fetus was in a brow presentation, but after a prolonged labor, the fetus converted to face presentation and was delivered vaginally with forceps assist. What should the nurse explain to the parents? A) The infant will need to be observed for meconium aspiration. B) Facial edema and head molding will subside in a few days. C) The infant will be given prophylactic antibiotics. D) Breastfeeding will need to be delayed for a day or two.

Answer: B Explanation: A) There is no mention of meconium-stained fluid that would cause the nurse to assess for meconium aspiration. B) Any facial edema and head molding that result from the use of forceps at birth will subside in a few days. C) There is no reason to place the infant on antibiotics. D) There is no reason to delay breastfeeding.

1) A pregnant client who swims 3-5 times per week asks the nurse whether she should stop this activity. What is the appropriate nursing response? A) "You should decrease the number of times you swim per week." B) "Continuing your exercise program would be beneficial." C) "You should discontinue your exercise program immediately." D) "You should consider a less strenuous type of exercise."

Answer: B Explanation: A) There is no need to decrease the number of times the client swims per week, because mild to moderate exercise is beneficial during pregnancy. B) Mild to moderate exercise is beneficial during pregnancy. Regular exercise-at least 30 minutes of moderate exercise daily or at least most days of the week-is preferred. C) Non-weight-bearing exercises, such as swimming or cycling, are recommended because they decrease the risk of injury and provide fitness with comfort. D) A moderate, rhythmic exercise routine involving large muscle groups such as swimming, cycling, or brisk walking is best.

1. A client arrives in the labor and delivery unit and describes her contractions as occurring every 10-12 minutes, lasting 30 seconds. She is smiling and very excited about the possibility of being in labor. On exam, her cervix is dilated 2 c m, 100% effaced, and -2 station. What best describes this labor? A) Second phase B) Latent phase C) Active phase D) Transition phase

Answer: B Explanation: A) There is no phase of labor that is identified as the second phase. B) In the early or latent phase of the first stage of labor, contractions are usually mild. The woman feels able to cope with the discomfort. The woman is often talkative and smiling and is eager to talk about herself and answer questions. C) When the woman enters the early active phase, her anxiety tends to increase as she senses the intensification of contractions and pain. During this phase the cervix dilates from about 4 to 7 c m. D) When the woman enters the transition phase, she may demonstrate significant anxiety. She becomes acutely aware of the increasing force and intensity of the contractions. She may become restless, frequently changing position.

1) The nurse is assessing a 2-hour-old newborn delivered by cesarean at 38 weeks. The amniotic fluid was clear. The mother had preeclampsia. The newborn has a respiratory rate of 80, is grunting, and has nasal flaring. What is the most likely cause of this infant's condition? A) Meconium aspiration syndrome B) Transient tachypnea of the newborn C) Respiratory distress syndrome D) Prematurity of the neonate

Answer: B Explanation: A) There was no meconium in the amniotic fluid, which rules out meconium aspiration syndrome. B) The infant is term and was born by cesarean, and is most likely experiencing transient tachypnea of the newborn. C) The infant is not premature and therefore is not likely to be experiencing respiratory distress syndrome. The infant is not premature

1) The nurse has received the end-of-shift report on the postpartum unit. Which client should the nurse see first? A) Woman who is 2nd day post-cesarean, moderate lochia serosa B) Woman day of delivery, fundus firm 2 c m above umbilicus C) Woman who had a cesarean section, 1st postpartum day, 4 c m diastasis recti abdominis D) Woman who had a cesarean section, 1st postpartum day, hypoactive bowel sounds all quadrants

Answer: B Explanation: A) This client is not experiencing any unexpected findings. The uterus rids itself of the debris remaining after birth through a discharge called lochia, which is classified according to its appearance and contents. Lochia serosa is a pinkish color. B) This client is the top priority. The fundus should not be positioned above the umbilicus after delivery. If the fundus is in the midline but higher than expected, it is usually associated with clots within the uterus. C) Diastasis recti abdominis is normal and is especially pronounced in women who have undergone a cesarean section. D) Bowel sounds are often decreased after delivery and it may take a few days for the bowel to regain its tone. The woman who has had a cesarean or a difficult birth may benefit from stool softeners.

1) A woman is 16 weeks pregnant. She has had cramping, backache, and mild bleeding for the past 3 days. Her physician determines that her cervix is dilated to 2 centimeters, with 10% effacement, but membranes are still intact. She is crying, and says to the nurse, "Is my baby going to be okay?" In addition to acknowledging the client's fear, what should the nurse also say? A) "Your baby will be fine. We'll start IV, and get this stopped in no time at all." B) "Your cervix is beginning to dilate. That is a serious sign. We will continue to monitor you and the baby for now." C) "You are going to miscarry. But you should be relieved because most miscarriages are the result of abnormalities in the fetus." D) "I really can't say. However, when your physician comes, I'll ask her to talk to you about it."

Answer: B Explanation: A) This is a serious situation. The client should not be offered false hope of everything being fine. B) If bleeding persists and abortion is imminent or incomplete, the woman may be hospitalized, Ⅳ therapy or blood transfusions may be started to replace fluid, and dilation and curettage (D&C) or suction evacuation is performed to remove the remainder of the products of conception. C) The nurse should avoid giving a justification of the miscarriage. D) The nurse should not defer the conversation to someone else (e.g., the physician).

1) The client having her second child is scheduled for a cesarean birth because the baby is in a breech presentation. The client states, "I'm wondering what will be different this time compared with my first birth, which was vaginal." What response is best? A) "We'll take good care of you and your baby. You'll be home before you know it." B) "You'll be wearing a sequential compression device until you start walking." C) "You will have a lot of pain, but there are medications that we give when it gets really bad." D) "You won't be able to nurse until the baby is 12 hours old, because of your epidural."

Answer: B Explanation: A) This response focuses on the nurse, and does not provide specific information to answer the client's question. B) The use of sequential compression devices (S C Ds) and early ambulation are essential to the prevention of deep vein thrombosis, especially if the client had a cesarean birth. C) Focusing on the pain is a negative emphasis and pain can also be a factor in a vaginal birth. D) Epidural anesthesia prevents leg function, and therefore ambulation, but does not impact a mother's ability to breastfeed.

The nursing instructor explains to the class that according to the 1973 Supreme Court decision in Roe v. Wade, abortion is legal if induced: A) Before the 30th week of pregnancy. B) Before the period of viability. C) To provide tissue for therapeutic research. Can be done any time if mother, doctor, and hospital all agree

Answer: B Explanation: A) This statement is not true, because the fetus is viable many weeks before the 30th week. B) Abortion can be performed legally until the period of viability. C) Abortion cannot be used for the sole purpose of providing tissue for therapeutic research. D) This is not true. Legal abortion can be done only up until the time of viability.

1) The nurse is presenting a session on intimate partner violence. Which statement by a client indicates a need for further education? A) "My daughter is not to blame for the violence in her marriage." B) "Everyone experiences anger and hitting in a relationship." C) "Abusers can be either husbands or boyfriends or girlfriends." D) "The 'honeymoon period' follows an episode of violence."

Answer: B Explanation: A) This statement recognizes that the blame for her assault lies with her assailant, not with the victim. B) Violence is not a normal part of intimate relationships. Domestic violence, also called intimate partner violence (I P V), is defined as a pattern of coercive behaviors and methods used to gain and maintain power and control by one individual over another in an adult intimate relationship. This statement indicates that the client has likely been a victim of domestic violence herself. C) Batterers come from all racial, ethnic, and religious groups and all professions, occupations, and socioeconomic strata. Batterers can also be either male or female. An acute episode of battering is followed by the tranquil phase, or honeymoon period, which is characterized by extremely loving, kind, and contrite behaviors by the batterer

1) A new mother is concerned about spoiling her newborn. The home care nurse teaches the mother which of the following? A) Newborns can be manipulative, so caution is advised. B) Meeting the infant's needs develops a trusting relationship. C) An infant who is rocked to sleep every night is being spoiled. D) Crying is good for babies, and letting them cry it out is advised.

Answer: B Explanation: A) This would be inappropriate and incorrect advice. B) Picking babies up when they cry teaches them that adults are responsive to their needs. This helps build a sense of trust and security. C) This would be inappropriate and incorrect advice. D) This would be inappropriate and incorrect advice.

A fetus has been diagnosed with myelomeningocele. Which of the following surgeries would be performed to correct this condition? A) Tubal ligation B) Intrauterine fetal surgery C) Cesarean section D) Sterilization

Answer: B Explanation: A) Tubal ligation is not an intrauterine fetal surgery. B) Intrauterine fetal surgery, which is generally considered experimental, is a therapy for anatomic lesions that can be corrected surgically and are incompatible with life if not treated. Examples include surgery for myelomeningocele and some congenital cardiac defects. C) A cesarean birth is not considered an intrauterine fetal surgery. D) Sterilization surgery does not involve the fetus.

1) The nurse working at a homeless shelter is studying case statistics. Of the total homeless population served at the shelter, which group would the nurse's statistics likely uncover as the fastest-growing group? A) Unemployed women B) Families with children C) The mentally ill D) The elderly

Answer: B Explanation: A) Unemployment is one aspect of this trend, but unemployed women are not the fastest-growing group. B) In 2012, the number of homeless individuals fell slightly, by 0.4%, with the exception of homeless families. C) The mentally ill are one part of this equation, but are not the fastest-growing group. D) The elderly are part of the group living in the shelters, but are not the fastest-growing group.

1) A woman in labor asks the nurse to explain the electronic fetal heart rate monitor strip. The fetal heart rate baseline is 150 with accelerations to 165, variable decelerations to 140, and moderate long-term variability. Which statement indicates that the client understands the nurse's teaching? A) "The most important part of fetal heart monitoring is the absence of variable decelerations." B) "The most important part of fetal heart monitoring is the presence of variability." C) "The most important part of fetal heart monitoring is the fetal heart rate baseline." D) "The most important part of fetal heart monitoring is the depth of decelerations."

Answer: B Explanation: A) Variable decelerations indicate cord compression. B) Baseline variability is a reliable indicator of fetal cardiac and neurologic function, and overall well-being. The opposing "push-pull" balancing between the sympathetic nervous system and the parasympathetic nervous system directly affects the F H R. C) The fetal heart rate baseline does not indicate central nervous system function. D) The depth of decelerations does not indicate central nervous system function.

1) The client is in the second stage of labor. The fetal heart rate baseline is 170, with minimal variability present. The nurse performs fetal scalp stimulation. The client's partner asks why the nurse did that. What is the best response by the nurse? A) "I stimulated the top of the fetus's head to wake him up a little." B) "I stimulated the top of the fetus's head to try to get his heart rate to accelerate." C) "I stimulated the top of the fetus's head to calm the fetus down before birth." D) "I stimulated the top of the fetus's head to find out whether he is in distress."

Answer: B Explanation: A) Waking the fetus is not the goal or outcome of fetal scalp stimulation. B) Fetal scalp stimulation is done when there is a question regarding fetal status. An acceleration indicates fetal well-being. C) Calming the fetus is not the goal or outcome of fetal scalp stimulation. D) The nurse wants to assess what the fetus does with stimulation.

1) A nurse is working in a clinic where clients from several cultures are seen. As a first step toward the goal of personal cultural competence, the nurse will do which of the following? A) Enhance cultural skills. B) Gain cultural awareness. C) Seek cultural encounters. Acquire cultural knowledge

Answer: B Explanation: A) Ways to enhance cultural skill include learning a prevalent language and learning how to recognize health-manifesting skin color variations in different races. B) One begins to gain cultural competence by gaining cultural awareness or by gaining an effective and cognitive self-awareness of personal worldview biases, beliefs, etc. C) During daily interactions with clients from diverse backgrounds, these cultural encounters allow the nurse to appreciate the uniqueness of individuals from varying backgrounds. D) Another early step, although not the first step, is acquiring cultural knowledge, and includes studying information about the beliefs, biological variations, and favored treatments of specific cultural groups.

1) The prenatal clinic nurse is caring for a client with hyperemesis gravidarum at 14 weeks' gestation. The vital signs are: blood pressure 95/48, pulse 114, respirations 24. Which order should the nurse implement first? A) Weigh the client. B) Give 1 liter of lactated Ringer's solution Ⅳ. C) Administer 30 m L Maalox (magnesium hydroxide) orally. D) Encourage clear liquids orally.

Answer: B Explanation: A) Weighing the client provides information on weight gain or loss, but is not the top priority in a client with excessive vomiting during pregnancy. The vital signs indicate hypovolemia. B) The vital signs indicate hypovolemia from dehydration, which leads to hypotension and increased pulse rate. Giving this client a liter of lactated Ringer's solution intravenously will reestablish vascular volume and bring the blood pressure up, and the pulse and respiratory rate down. C) The vital signs indicate hypovolemia. There is no indication that the client has dyspepsia. D) Lack of tolerance of oral fluids through excessive vomiting is what has led to the hypovolemia.

1) The clinic nurse is assisting with an initial prenatal assessment. The following findings are present: spider nevi on lower legs; dark pink, edematous nasal mucosa; mild enlargement of the thyroid gland; mottled skin and pallor on palms and nail beds; heart rate 88 with murmur present. What is the best action for the nurse to take based on these findings? A) Document the findings on the prenatal chart. B) Have the physician see the client today. C) Instruct the client to avoid direct sunlight. D) Analyze previous thyroid hormone lab results.

Answer: B Explanation: A) While all of these findings should be documented on the prenatal chart, additional action is indicated. B) Mottling of the skin is indicative of possible anemia. These abnormalities must be reported to the healthcare provider immediately. C) Instructing the client to avoid direct sunlight is not necessary; rather, additional action is indicated. D) The thyroid gland increases in size during pregnancy due to hyperplasia. Additional action is indicated.

1) Which is the criterion that practitioners use to diagnosis fetal demise in utero? A) Absence of fetal movement B) Absence of fetal heartbeat C) Increased serum estriol D) Increased serum testosterone

Answer: B Explanation: A) While the absence of fetal movement often cues the pregnant client that there may be something wrong, this is not the criterion for the diagnosis of fetal demise in utero. B) Absence of a fetal heartbeat is the criterion that practitioners use to diagnose fetal demise in utero. C) A decreased, not increased, level of estriol that accompanies fetal demise will often initiate labor. However, this is not the criterion for diagnosis of fetal demise in utero. Testosterone levels do not cause, nor do they change, as a result of fetal demise; therefore, this is not a criterion used to diagnose fetal demise in utero

1) A pregnant teenage client is diagnosed with iron-deficiency anemia. Which nutrient should the nurse encourage her to take to increase iron absorption? A) Vitamin A B) Vitamin C C) Vitamin D D) Vitamin E

Answer: B Explanation: A) While vitamin A is good for the body, it does not promote the absorption of iron. B) Vitamin C is known to enhance the absorption of iron from meat and nonmeat sources. C) While vitamin D is good for the body, it does not promote the absorption of iron. D) While vitamin E is good for the body, it does not promote the absorption of iron.

A client who has been using transdermal hormonal contraception comes in for a routine wellness visit. Which finding should cause the nurse to question if the client should continue to use this form of contraception? A) Body weight of 179 lb B) Skin breakdown at the site of the patch C) Drinks 2 cups of caffeinated coffee a day D) Bicycles at the gym three evenings a week

Answer: B Explanation: A) Women who have a skin disorder that may result in a reaction at the site of application may not be candidates for the patch. Body weight needs to be below 198 l b s. for the patch. Caffeine and exercise are not contraindications for using the patch for contraception. B) Women who have a skin disorder that may result in a reaction at the site of application may not be candidates for the patch. Body weight needs to be below 198 l b s. for the patch. Caffeine and exercise are not contraindications for using the patch for contraception. C) Women who have a skin disorder that may result in a reaction at the site of application may not be candidates for the patch. Body weight needs to be below 198 l b s. for the patch. Caffeine and exercise are not contraindications for using the patch for contraception. D) Women who have a skin disorder that may result in a reaction at the site of application may not be candidates for the patch. Body weight needs to be below 198 l b s. for the patch. Caffeine and exercise are not contraindications for using the patch for contraception.

1) Women with H I V should be evaluated and treated for other sexually transmitted infections and for what condition occurring more commonly in women with H I V? A) Syphilis B) Toxoplasmosis C) Gonorrhea D) Herpes

Answer: B Explanation: A) Women with H I V should be evaluated and treated for other sexually transmitted infections and for conditions occurring more commonly in women with H I V, such as tuberculosis, cytomegalovirus, toxoplasmosis, and cervical dysplasia. Syphilis occurs more often in men. B) Women with H I V should be evaluated and treated for other sexually transmitted infections and for conditions occurring more commonly in women with H I V, such as tuberculosis, cytomegalovirus, toxoplasmosis, and cervical dysplasia. C) Women with H I V should be evaluated and treated for other sexually transmitted infections and for conditions occurring more commonly in women with H I V, such as tuberculosis, cytomegalovirus, toxoplasmosis, and cervical dysplasia. Gonorrhea affects both men and women. Women with HIV should be evaluated and treated for other sexually transmitted infections and for conditions occurring more commonly in women with HIV, such as tuberculosis, cytomegalovirus, toxoplasmosis, and cervical dysplasia. Herpes affects both men and women

1) An older female patient is concerned about the finances needed to run her home. What event in this person's life should the nurse realize is causing her financial concern? A) Change in the number of prescribed medications B) Recent death of spouse after a long and costly illness C) Participation in activities at the community center D) Relocation of older children to another city

Answer: B Explanation: B) A husband's long, costly illness and the decrease or loss of his pension following his death can negatively impact a woman's financial resources. There is no information about how the patient's medications have changed. She could be prescribed less. Participating in activities and having adult children move away would not directly impact this older person's financial status.

1) The nurse provides the following diagram to a patient who is 10 weeks pregnant and scheduled for a sonogram. Why is this test being used? A) Estimate fetal weight B) Determine gestational age C) Evaluate placenta integrity D) Calculate pelvis dimensions

Answer: B Explanation: B) An early transvaginal or abdominal sonogram is indicated when there is a need to establish an accurate gestational age. When dating a pregnancy using ultrasound the crown-rump length is considered most accurate for 6- to 12-week gestational periods (plus or minus 3 to 5 days). Dotted line shows the measurement from the top of the fetal crown (head) to the bottom of the rump (buttocks). This test is not used to estimate fetal weight, evaluate placental integrity, or calculate pelvis dimensions.

1) The nurse is explaining the processes of infant heat loss to a new mother. Which diagram should the nurse use to describe the process of convection?

Answer: B Explanation: B) Convection is the loss of heat from the warm body surface to cooler air currents, as shown in Diagram 2. Air-conditioned rooms, air currents with a temperature below the infant's skin temperature, unwarmed oxygen by mask, and removal of the infant from an incubator for procedures increase convective heat loss of the newborn. Conduction is the loss of heat to a cooler surface by direct skin contact, as shown in Diagram 1. Chilled hands, cool scales, cold examination tables, and cold stethoscopes can cause heat loss by conduction. Evaporation is the loss of heat incurred when water is converted to a vapor, as shown in Diagram 3. The newborn is particularly prone to heat loss by evaporation immediately after birth (when the baby is wet with amniotic fluid) and during baths; therefore, drying the newborn is critical. Radiation losses occur when body heat is transferred to cooler surfaces and objects that are not in direct contact with the body, as shown in Diagram 4. The walls of a room or of an incubator are potential causes of heat loss by radiation, even if the ambient temperature of the incubator is within the neutral thermal range for the infant.

1) The nurse prepares teaching material for a patient who is experiencing a spontaneous abortion. Based upon the material selected, which type of abortion is the patient experiencing? A) Threatened B) Imminent C) Incomplete D) Complete

Answer: B Explanation: B) For an imminent abortion the placenta has separated from the uterine wall, the cervix has dilated, and the amount of bleeding has increased. For a threatened abortion the cervix is not dilated, and the placenta is still attached to the uterine wall, but some bleeding occurs. For an incomplete abortion the embryo/fetus has passed out of the uterus; however, the placenta remains. In a complete abortion all contents of the pregnancy have been expelled from the uterus.

1) The nurse is performing Leopold maneuvers with a patient in the 3rd trimester of pregnancy. Which maneuver should the nurse perform first?

Answer: B Explanation: B) For the first maneuver, while facing the woman, the nurse palpates the upper abdomen with both hands. Choice 1 is the second maneuver, where the nurse tries to determine the location of the fetal back and notes whether it is on the right or left side of the maternal abdomen. Choice 3 is the third maneuver, where the nurse determines what fetal part is lying above the inlet by gently grasping the lower portion of the abdomen just above the symphysis pubis with the thumb and fingers of the right hand. Choice 4 is the fourth maneuver, where the nurse faces the woman's feet and attempts to locate the cephalic prominence or brow. Location of this landmark assists in assessing the descent of the presenting part into the pelvis. The fingers of both hands are moved gently down the sides of the uterus toward the pubis.

1) The nurse is reviewing laboratory testing completed for a patient with suspected pelvic inflammatory disease. Which test result should the nurse identify as supporting this diagnosis? A) Decreased hematocrit level B) Elevated sedimentation rate C) Decreased hemoglobin level D) Elevated white blood cell count

Answer: B Explanation: B) In pelvic inflammatory disease the woman may have an elevated sedimentation rate. Decreased hematocrit and hemoglobin levels would indicate bleeding. An elevated white blood cell count indicates an infection, which may or may not occur with pelvic inflammatory disease.

28) The nurse determines that a newborn has the following findings: Heart rate: 88 beats per minute Respirations: 24 per minute and irregular Muscle tone: Minor movement of lower extremities Reflex response: Grimace Skin color: Pink body, blue extremities If using the following table, what action should the nurse take at this time? A) Begin resuscitation B) Stimulate the infant C) Document the findings D) Nasopharyngeal suctioning

Answer: B Explanation: B) The infant's Apgar score is 5. An Apgar score between 4 and 7 indicates the need for stimulation. A score under 4 indicates the need for resuscitation. A score of 7 to 10 indicates a newborn in good condition who requires only nasopharyngeal suctioning. The nurse needs to do more than document the findings.

1) The maternity nurse is preparing material for a staff development seminar on ectopic pregnancy. When using the diagram below, which area should the nurse identify as being the most common location for the implantation of an ectopic pregnancy? 1. A Fimbrial 2. B Ampulla 3. C Intramural 4. D Ovary

Answer: B Explanation: B) The most common location for implantation of an ectopic pregnancy is the ampulla of the tube. Fimbrial, intramural, and ovarian are not the most common locations for the implantation of an ectopic pregnancy.

1) The healthcare provider asks for forceps to aid in the delivery of a fetus that is engaged, but the leading edge is at +1. Which type of forceps should be handed to the healthcare provider for this delivery?

Answer: B Explanation: The criterion for midforceps application is that the fetal head must be engaged (largest diameter of the head reaches or passes through the pelvic inlet), but the leading edge (presenting part) of the fetal skull is above a plus 2 (+2) station (for example, +1, 0, —1, —2). When midforceps are used, the goal is to apply traction and, frequently, to rotate the head and facilitate the vaginal birth. Kielland forceps are used for midforceps rotations. Elliot and Simpson forceps are used as outlet forceps. Piper forceps are used to provide traction and flexion of the after-coming head of a fetus in breech presentation.

1) The nurse is working with a new mother who follows Muslim traditions. Which expectations and actions are appropriate for this client? Note: Credit will be given only if all correct choices and no incorrect choices are selected. Select all that apply. A) Make sure she gets a kosher diet. B) Expect that most visitors will be women. C) Uncover only the necessary skin when assessing. D) The father will take an active role in infant care. E) She will prefer a male physician.

Answer: B, C Explanation: A) A kosher diet is appropriate for Jewish women, not Muslims. Although many of the dietary laws are similar, kosher laws are not appropriate for a Muslim client. B) In Muslim cultures, emphasis on childrearing and infant care activities is on the mother and female relatives. C) Women of the Islamic faith may have specific modesty requirements; the woman must be completely covered, with only her feet and hands exposed. D) In Muslim cultures, emphasis on childrearing and infant care activities is on the mother and female relatives, and there is little involvement of the father in newborn care. E) Women of the Islamic faith may have specific modesty requirements; the woman must be completely covered, with only her feet and hands exposed, and no man, other than the husband or a family member, may be alone with her. Because of the culture of modesty, many Muslim women highly prefer female care providers.

1) The nurse is planning an in-service educational program to talk about disseminated intravascular coagulation (D I C). The nurse should identify which conditions as risk factors for developing D I C? Note: Credit will be given only if all correct choices and no incorrect choices are selected. Select all that apply. A) Diabetes mellitus B) Abruptio placentae C) Fetal demise D) Multiparity E) Preterm labor

Answer: B, C Explanation: A) Diabetes does not cause the release of thromboplastin that triggers D I C. B) As a result of the damage to the uterine wall and the retroplacental clotting with covert abruption, large amounts of thromboplastin are released into the maternal blood supply, which in turn triggers the development of disseminated intravascular coagulation (D I C) and the resultant hypofibrinogenemia. C) Perinatal mortality associated with abruptio placentae is approximately 25%. If fetal hypoxia progresses unchecked, irreversible brain damage or fetal demise may result. D) Multiparity does not cause the release of thromboplastin that triggers D I C. E) Preterm labor does not cause the release of thromboplastin that triggers D I C.

1. The postpartum client is suspected of having acute cystitis. Which symptoms would the nurse expect to see in this client? Note: Credit will be given only if all correct choices and no incorrect choices are selected. Select all that apply. A) High fever B) Frequency C) Suprapubic pain D) Chills E) Nausea and vomiting

Answer: B, C Explanation: A) High fever is not usually present in acute cystitis, although it can appear if the cystitis progresses to pyelonephritis. B) Frequency is characteristic of acute cystitis. C) Suprapubic pain is characteristic of acute cystitis. D) Chills are not usually present in acute cystitis, although they can appear if the cystitis progresses to pyelonephritis. E) Nausea and vomiting are not usually present in acute cystitis, although they can appear if the cystitis progresses to pyelonephritis.

1) The nurse is planning an in-service presentation about perinatal loss. Which statements should the nurse include in this presentation? Note: Credit will be given only if all correct choices and no incorrect choices are selected. Select all that apply. A) "Perinatal loss refers to third-trimester fetal death in utero." B) "Perinatal loss occurs more frequently in assisted reproduction." C) "Perinatal loss rates have declined in the United States over the past few years." D) "Perinatal loss includes 25% of stillbirths occurring before the onset of labor." E) "Perinatal loss rarely causes an emotional problem for the family."

Answer: B, C Explanation: A) Perinatal loss is the death of a fetus or infant from the time of conception through the end of the newborn period 28 days after birth. B) Pregnancies conceived by in vitro fertilization have higher rates of pregnancy loss and pregnancy complications. C) Perinatal loss in industrialized countries has declined in recent years as early diagnosis of congenital anomalies and advances in genetic testing techniques have increased the use of elective termination. D) It is estimated that 50% of stillbirths occur before the onset of labor, with more than 24.4% occurring between 20 and 28 weeks' gestation. E) Perinatal losses are grieved by most couples and families like any other death. Like other grief, the grief over perinatal loss can become overwhelming.

1) In planning a conference for adolescents, the nurse researches the most common socioeconomic and cultural factors that contribute to adolescent pregnancy. Which true statements should be included in this conference? Note: Credit will be given only if all correct choices and no incorrect choices are selected. Select all that apply. A) Economic status has a relatively low impact on an adolescent's sexual activity. B) Nearly three quarters of all teens ages 15-19 have had sex at least once. C) Poor self-esteem contributes to adolescent pregnancy. D) 38% percent of sexually active students use a condom. E) Sexual abuse is not an issue with teen mothers.

Answer: B, C Explanation: A) Poverty is a major risk factor for adolescent pregnancy. B) By 19 years of age, 70% of all teens have had intercourse. C) This is a true statement. Poor self-esteem is a major contributing factor in adolescent pregnancy. D) 68% of females aged 15 to 19 and 80% of males use condoms during their first sexual intercourse. E) More teens who become pregnant, compared with teens who have not been pregnant, have been physically, emotionally, or sexually abused.

1) The client presents to the clinic for an initial prenatal examination. She asks the nurse whether there might be a problem for her baby because she has type B R h-positive blood and her husband has type O R h-negative blood, or because her sister's baby had ABO incompatibility. What is the nurse's best answer? Note: Credit will be given only if all correct choices and no incorrect choices are selected. Select all that apply. A) "Your baby would be at risk for R h problems if your husband were R h-negative." B) "R h problems only occur when the mother is R h-negative and the father is not." C) "ABO incompatibility occurs only after the baby is born." D) "We don't know for sure, but we can test for ABO incompatibility." E) "Your husband's being type B puts you at risk for ABO incompatibility."

Answer: B, C Explanation: A) R h incompatibility is a possibility when the mother is R h-negative and the father is R h-positive. B) R h incompatibility is a possibility when the mother is R h-negative and the father is R h-positive. C) ABO incompatibility is limited to type O mothers with a type A or B fetus and occurs after the baby is born. D) ABO incompatibility is limited to type O mothers with a type A or B fetus and occurs after the baby is born. E) The husband's blood type is not an issue for ABO incompatibility.

1) Absolute contraindications to exercise while pregnant include which of the following? Note: Credit will be given only if all correct choices and no incorrect choices are selected. Select all that apply. A) Abruptio placentae B) Placenta previa after 26 weeks' gestation C) Preeclampsia-eclampsia D) Cervical insufficiency (cerclage) E) Intrauterine growth restriction (I U G R)

Answer: B, C, D Explanation: A) Abruptio placentae is not an absolute contraindication to exercise. B) Placenta previa after 26 weeks' gestation is an absolute contraindication to exercise. C) Preeclampsia-eclampsia is an absolute contraindication to exercise. D) Cervical insufficiency (cerclage) is an absolute contraindication to exercise. E) Intrauterine growth restriction (I U G R) is not an absolute contraindication to exercise.

1) Under which circumstances would the nurse remove prostaglandin from the client's cervix? Note: Credit will be given only if all correct choices and no incorrect choices are selected. Select all that apply. A) Contractions every 5 minutes B) Nausea and vomiting C) Uterine tachysystole D) Cardiac tachysystole E) Baseline fetal heart rate of 140-148

Answer: B, C, D Explanation: A) Contractions every 5 minutes are consistent with the plan of induction. B) A reason to remove prostaglandin from a client's cervix is the presence of nausea and vomiting. C) A reason to remove prostaglandin from a client's cervix is uterine tachysystole. D) A reason to remove prostaglandin from a client's cervix is cardiac tachysystole. E) This is a good heart rate and would not warrant removing the prostaglandin.

1) Which questions are appropriate for the nurse to ask during a cultural assessment of a client who is new to the clinic? Note: Credit will be given only if all correct choices and no incorrect choices are selected. Select all that apply. A) What genetic and other biological differences affect caregiving? B) Which family member must be consulted for decisions about care? C) What type of healthcare provider is the most appropriate? D) Does the client have beliefs or traditions that might impact the care plan? E) Are communications patterns established?

Answer: B, C, D Explanation: A) Genetic and biological differences are health concerns, such as hypertension that the nurse must keep in mind, but the nurse would not ask about genetic and biological differences during a cultural assessment. B) It is important that the nurse recognize cultural differences in regard to which family member must be consulted for decisions about care. C) Some cultures do not allow a person of the opposite sex to touch the client. Cultural sensitivity will recognize and allow for this. D) The nurse must be aware of traditions and beliefs that might impact the care plan. Communication patterns will have been established. The nurse must be able to communicate with the client, using the patterns of communication the client uses

1) The nurse is preparing an in-service presentation for a group of middle-school nurses. Which statements by the nurse would indicate that the middle-school nurse understood the role of culture in adolescent pregnancy? Note: Credit will be given only if all correct choices and no incorrect choices are selected. Select all that apply. A) "Studies show that 85% of teen mothers are middle class, and give birth to gain adult status." B) "Teenage pregnancy is the leading reason why adolescent women drop out of school." C) "Teens of color are more likely to become pregnant." D) "Intelligence and academic ability are positively associated with delayed sexual activity, greater use of contraception, and lower rates of pregnancy." E) "Over 50% of adolescents who have had an abortion or recent birth become pregnant again within two years."

Answer: B, C, D Explanation: A) Poverty is a major risk factor for adolescent pregnancy. Studies show that 85% of births to unmarried teens occur to those from poor or low-income families. Adolescents who do not have access to middle-class opportunities tend to maintain their pregnancies, because they see pregnancy as their only option for adult status. B) An estimated 30% to 40% of female teenage dropouts are mothers. C) In the United States, the adolescent birth rate is higher among African American and Hispanic teens than among white teens. To some degree, the higher teenage pregnancy rate in these groups reflects the impact of poverty because a disproportionately higher number of African American and Hispanic youths live in poverty. D) Teens with future goals (i.e., college or job) tend to use birth control more consistently compared with other teens; if they become pregnant, they are also more likely to have abortions. E) Just over one-third (35%) of adolescents who have had an abortion or recent birth become pregnant again within two years.

1) The nurse is preparing a brochure for couples considering pregnancy after the age of 35. Which statements should be included? Note: Credit will be given only if all correct choices and no incorrect choices are selected. Select all that apply. A) There is a decreased risk of Down syndrome. B) Preexisting medical conditions can complicate pregnancy. C) Preterm births are more common. D) Amniocentesis can be performed to detect genetic anomalies. E) The increased fertility of women over age 35 makes conception easier.

Answer: B, C, D Explanation: A) The incidence of Down syndrome increases somewhat in women over age 35 and increases significantly in those over age 40. B) Preexisting medical conditions, such as hypertension or diabetes, probably play a more significant role than age in maternal well-being and the outcome of pregnancy. C) The incidence of low-birth-weight infants, preterm births, miscarriage, stillbirth, and perinatal morbidity and mortality is higher among women age 35 or older. D) Amniocentesis is offered to all women over age 35 to permit the early detection of several chromosomal abnormalities, including Down syndrome; noninvasive analysis of fetal nucleic acid is now commonly recommended to women of advanced maternal age. E) The decreased fertility of women over age 35 may make conception more difficult.

1) Nonreassuring fetal status often occurs with a tachysystole contraction pattern. Intrauterine resuscitation measures may become warranted and can include which of the following measures? Note: Credit will be given only if all correct choices and no incorrect choices are selected. Select all that apply. A) Position the woman on her right side. B) Apply oxygen via face mask. C) Call the anesthesia provider for support. D) Increase intravenous fluids by at least 500 m L bolus. E) Call the physician/C N M to the bedside.

Answer: B, C, D Explanation: A) The nurse would position the woman on her left side. B) The nurse would apply oxygen via face mask. C) The nurse would call the anesthesia provider for support. D) The nurse would increase intravenous fluids by at least 500 m L bolus. E) The nurse would not call the physician/C N M to the bedside.

1) The nurse knows that which of the following are advantages of spinal block? Note: Credit will be given only if all correct answers and no incorrect answers are selected. Select all that apply. A) Intense blockade of sympathetic fibers B) Relative ease of administration C) Maternal compartmentalization of the drug D) Immediate onset of anesthesia E) Larger drug volume

Answer: B, C, D Explanation: A) The primary disadvantage of spinal block is intense blockade of sympathetic fibers, resulting in a high incidence of hypotension. B) One of the advantages of spinal block is the relative ease of administration. C) One of the advantages of spinal block is the maternal compartmentalization of the drug. D) One of the advantages of spinal block is the immediate onset of anesthesia. E) One of the advantages of spinal block is a smaller drug volume, not a larger one.

1) The nurse is caring for a client who delivered by cesarean birth. The client received a general anesthetic. To prevent or minimize abdominal distention, which of the following would the nurse encourage? Note: Credit will be given only if all correct choices and no incorrect choices are selected. Select all that apply. A) Increased intake of cold beverages B) Leg exercises every 2 hours C) Abdominal tightening D) Ambulation E) Using a straw when drinking fluids

Answer: B, C, D Explanation: A) The woman should avoid carbonated or very hot or cold beverages, as they would increase the distention through the increase of gas and constipation. B) Immobility increases the risk of abdominal distention and discomfort. Leg exercises serve to prevent or minimize abdominal distention in a surgical client who received a general anesthetic. C) Abdominal tightening serves to prevent or minimize abdominal distention in a surgical client who received a general anesthetic. D) Early ambulation prevents abdominal distention that can occur with excess accumulation of gas in the intestines. E) The woman should avoid the use of straws to avoid increasing the distention through increase of gas and constipation.

1) The nurse is teaching new parents how to dress their newborn. Which statements indicate that teaching has been effective? Note: Credit will be given only if all correct choices and no incorrect choices are selected. Select all that apply. A) "We should keep our home air-conditioned so the baby doesn't overheat." B) "It is important that we dry the baby off as soon as we give him a bath or shampoo his hair." C) "When we change the baby's diaper, we should change any wet clothing or blankets, too." D) "If the baby's body temperature gets too low, he will warm himself up without any shivering." E) "Our baby will have a much faster rate of breathing if he is not dressed warmly enough."

Answer: B, C, D, E Explanation: A) Because of the risk of hypothermia and possible cold stress, minimizing heat loss in the newborn after birth is essential. B) The newborn is particularly prone to heat loss by evaporation immediately after birth and during baths; thus drying the newborn is critical. C) Changing wet clothing or blankets immediately prevents evaporation, one mechanism of heat loss. D) Nonshivering thermogenesis (N S T), an important mechanism of heat production unique to the newborn, is the major mechanism through which heat is produced. E) A decrease in the environmental temperature of 2°C is a drop sufficient to double the oxygen consumption of a term newborn and can cause the newborn to show signs of respiratory distress.

1) During a follow-up wellness visit, the nurse determines that a female client is experiencing favorable outcomes after starting combined oral contraceptives. What data did the nurse use to determine this? Note: Credit will be given only if all correct choices and no incorrect choices are selected. Select all that apply. A) Reduced appetite B) Reduced menstrual flow C) Fewer menstrual cramps D) No pain with ovulation E) Cycle is regular at 28 days

Answer: B, C, D, E Explanation: A) Combined oral contraceptives have noncontraceptive benefits that include reduced menstrual flow, fewer menstrual cramps, mittelschmerz or pain with ovulation disappearing, and a more regular cycle. Combined oral contraceptives do not affect appetite. B) Combined oral contraceptives have noncontraceptive benefits that include reduced menstrual flow, fewer menstrual cramps, mittelschmerz or pain with ovulation disappearing, and a more regular cycle. Combined oral contraceptives do not affect appetite. C) Combined oral contraceptives have noncontraceptive benefits that include reduced menstrual flow, fewer menstrual cramps, mittelschmerz or pain with ovulation disappearing, and a more regular cycle. Combined oral contraceptives do not affect appetite. D) Combined oral contraceptives have noncontraceptive benefits that include reduced menstrual flow, fewer menstrual cramps, mittelschmerz or pain with ovulation disappearing, and a more regular cycle. Combined oral contraceptives do not affect appetite. E) Combined oral contraceptives have noncontraceptive benefits that include reduced menstrual flow, fewer menstrual cramps, mittelschmerz or pain with ovulation disappearing, and a more regular cycle. Combined oral contraceptives do not affect appetite.

1) The nurse is caring for a client who plans to relinquish her baby for adoption. The nurse would implement which approach to care? Note: Credit will be given only if all correct choices and no incorrect choices are selected. Select all that apply. A) Encourage the client to see and hold her infant. B) Encourage the client to express her emotions. C) Respect any special requests for the birth. D) Acknowledge the grieving process in the client. E) Allow access to the infant, if the client requests it.

Answer: B, C, D, E Explanation: A) Encouraging the client to see and hold her infant does not respect the client's right to refuse interaction. The amount of contact she chooses to have with her newborn should be respected. B) The mother who decides to relinquish her baby needs emotional support and validation of her loss. C) The woman should decide whether to see and hold her baby and should have any special requests regarding the birth honored. D) Perinatal nurses should be aware that relinquishing mothers are at risk for disenfranchised grief, in which they are unable to proceed through the grieving process and come to resolution with the loss. The nurse should acknowledge the woman's loss and support her decision. E) The amount of contact she chooses to have with her newborn should be respected.

1) The nurse knows that the maternal risks associated with postterm pregnancy include which of the following? Note: Credit will be given only if all correct choices and no incorrect choices are selected. Select all that apply. A) Polyhydramnios B) Maternal hemorrhage C) Maternal anxiety D) Forceps-assisted delivery E) Perineal damage

Answer: B, C, D, E Explanation: A) Polyhydramnios is not associated with postterm pregnancy. B) Maternal symptoms and complications in postterm pregnancy may include maternal hemorrhage. C) Maternal symptoms and complications in postterm pregnancy may include maternal anxiety. D) Maternal symptoms and complications in postterm pregnancy may include an operative vaginal birth with forceps or vacuum extractor. E) Maternal symptoms and complications in postterm pregnancy may include perineal trauma and damage.

1) Which of the following symptoms would be an indication of postpartum blues? Note: Credit will be given only if all correct choices and no incorrect choices are selected. Select all that apply. A) Overeating B) Anger C) Mood swings D) Constant sleepiness E) Crying

Answer: B, C, E Explanation: A) Anorexia, not overeating, would be a symptom of postpartum blues. B) Anger would be a symptom of postpartum blues. C) Mood swings would be a symptom of postpartum blues. D) Difficulty sleeping would be a symptom of postpartum blues. E) Weepiness and crying would be a symptom of postpartum blues.

1) Maternal risks of occiput posterior (O P) malposition include which of the following? Note: Credit will be given only if all correct choices and no incorrect choices are selected. Select all that apply. A) Blood loss greater than 1000 m L B) Postpartum infection C) Anal sphincter injury D) Higher rates of vaginal birth E) Instrument delivery

Answer: B, C, E Explanation: A) Blood loss greater than 500 m L is a maternal risk of O P. B) Postpartum infection is a maternal risk of O P. C) Anal sphincter injury is a maternal risk of O P. D) Higher rates of cesarean birth are a maternal risk of O P. E) Instrument delivery is a maternal risk of O P.

1) Nursing interventions that foster the process of becoming a mother include which of the following? Note: Credit will be given only if all correct choices and no incorrect choices are selected. Select all that apply. A) Encouraging detachment from the nurse-patient relationship B) Promoting maternal-infant attachment C) Building awareness of and responsiveness to infant interactive capabilities D) Instruct about promoting newborn independence E) Preparing the woman for the maternal social role

Answer: B, C, E Explanation: A) Encouraging interactive therapeutic nurse-patient relationships is a nursing intervention that fosters the process of becoming a mother. B) Promoting maternal-infant attachment is a nursing intervention that fosters the process of becoming a mother. C) Building awareness of and responsiveness to infant interactive capabilities is a nursing intervention that fosters the process of becoming a mother. D) Instructing about infant caregiving is a nursing intervention that fosters the process of becoming a mother. E) Preparing the woman for the maternal social role is a nursing intervention that fosters the process of becoming a mother.

1) A N I C U nurse plans care for a preterm newborn that will provide opportunities for development. Which interventions support development in a preterm newborn in a N I C U? Note: Credit will be given only if all correct choices and no incorrect choices are selected. Select all that apply. A) Schedule care throughout the day. B) Silence alarms quickly. C) Place a blanket over the top portion of the incubator. D) Do not offer a pacifier. E) Dim the lights.

Answer: B, C, E Explanation: A) Nursing care should be planned to decrease the number of times the baby is disturbed. B) Noise levels can be lowered by replacing alarms with lights or silencing alarms quickly. C) Dimmer switches should be used to shield the baby's eyes from bright lights with blankets over the top portion of the incubator. D) Pacifiers can be offered because they provide opportunities for nonnutritive sucking. E) Dimming the lights may encourage infants to open their eyes and be more responsive to their parents.

1) What signs would indicate that a pregnant client's urinalysis culture was abnormal? Note: Credit will be given only if all correct choices and no incorrect choices are selected. Select all that apply. A) p H 4.6-8 B) Alkaline urine C) Cloudy appearance D) Negative for protein and red blood cells E) Hemoglobinuria

Answer: B, C, E Explanation: A) Urine p H of 4.6-8 is within a normal, healthy range. B) Alkaline urine could indicate metabolic alkalemia, Proteus infection, or an old specimen. C) A cloudy appearance could indicate an infection. D) Positive findings could indicate contaminated specimen, U T I, or kidney disease. Hemoglobinuria would be indicated by an abnormal urine color

1) The nurse interviews a 28-year-old client with a new medical diagnosis of endometriosis. Which question asked by the nurse is appropriate? Note: Credit will be given only if all correct choices and no incorrect choices are selected. Select all that apply. A) "Are you having hot flashes?" B) "Are you experiencing pain during intercourse?" C) "Is a vaginal discharge present?" D) "Are you having pain during your period?" E) "Have you noticed any skin rashes?"

Answer: B, D Explanation: A) Hot flashes are not a symptom of endometriosis. B) The primary symptoms of endometriosis include dyspareunia. C) Vaginal discharge is not a symptom of endometriosis. D) The primary symptoms of endometriosis include dysmenorrhea. Skin rashes are not a symptom of endometriosis

A couple is at the clinic for preconceptual counseling. Both parents are 40 years old. The nurse knows that the education session has been successful when the wife makes which statement(s)? Note: Credit will be given only if all correct choices and no incorrect choices are selected. Select all that apply. A) "We are at low risk for having a baby with Down syndrome." B) "Our children are more likely to have genetic defects." C) "Children born to parents our age have sex-linked disorders." D) "The tests for genetic defects can be done early in pregnancy." E) "It will be almost impossible for us to conceive a child."

Answer: B, D Explanation: A) The risk for trisomy 21 (Down syndrome) is 1 in 385. B) Women 35 or older are at greater risk for having children with chromosome abnormalities. C) Sex-linked disorders are not related to the age of either parent. D) Genetic testing such as amniocentesis and chorionic villus sampling are done in the first trimester. E) Fertility decreases somewhat after age 35, but being over 35 does not mean that conception is impossible.

1) During the first several postpartum weeks, the new mother must accomplish certain physical and developmental tasks, including which of the following? Note: Credit will be given only if all correct choices and no incorrect choices are selected. Select all that apply. A) Establish a therapeutic relationship with her physician B) Adapt to altered lifestyles and family structure resulting from the addition of a new member C) Restore her intellectual abilities D) Restore physical condition E) Develop competence in caring for and meeting the needs of her infant

Answer: B, D, E Explanation: A) During the first several postpartum weeks, the new mother must establish a relationship with her new child, not her physician. B) During the first several postpartum weeks, the new mother must adapt to altered lifestyles and family structure resulting from the addition of a new member. C) During the first several postpartum weeks, the new mother must restore her physical condition, not her intellectual abilities. D) During the first several postpartum weeks, the new mother must restore her physical condition. E) During the first several postpartum weeks, the new mother must develop competence in caring for and meeting the needs of her infant.

1) Benefits of skin-to-skin care as a developmental intervention include which of the following? Note: Credit will be given only if all correct choices and no incorrect choices are selected. Select all that apply. A) Routine discharge B) Stabilization of vital signs C) Increased periods of awake-alert state D) Decline in episodes of apnea and bradycardia E) Increased growth parameters

Answer: B, D, E Explanation: A) Early discharge is a benefit of skin-to-skin care as a developmental intervention. B) Stabilization of vital signs is a benefit of skin-to-skin care as a developmental intervention. C) Increased periods of quiet sleep is a benefit of skin-to-skin care as a developmental intervention. D) A decline in episodes of apnea and bradycardia is a benefit of skin-to-skin care as a developmental intervention. E) Increased growth parameters are a benefit of skin-to-skin care as a developmental intervention.

1) Which of the following are considered risk factors for development of severe hyperbilirubinemia? Note: Credit will be given only if all correct choices and no incorrect choices are selected. Select all that apply. A) Northern European descent B) Previous sibling received phototherapy C) Gestational age 27 to 30 weeks D) Exclusive breastfeeding E) Infection

Answer: B, D, E Explanation: A) East Asian or Mediterranean descent is considered a risk factor for development of severe hyperbilirubinemia. B) Previous sibling received phototherapy is considered a risk factor for development of severe hyperbilirubinemia. C) Gestational age 35 to 36 weeks (late preterm gestational age) is considered a risk factor for development of severe hyperbilirubinemia. D) Exclusive breastfeeding, particularly if nursing is not going well and excessive weight loss is experienced, is considered a risk factor for development of severe hyperbilirubinemia. E) Infection is considered a risk factor for development of severe hyperbilirubinemia.

1) The nurse is caring for a newborn 30 minutes after birth. After assessing respiratory function, the nurse would report which findings as abnormal? Note: Credit will be given only if all correct choices and no incorrect choices are selected. Select all that apply. A) Respiratory rate of 66 breaths per minute B) Periodic breathing with pauses of 25 seconds C) Synchronous chest and abdomen movements D) Grunting on expiration E) Nasal flaring

Answer: B, D, E Explanation: A) Immediately after birth and for the next 2 hours, the normal respiratory rate is 60 to 70 breaths per minute. B) Periodic breathing with pauses longer than 20 seconds (apnea) is an abnormal finding that should be reported to the physician. C) Abdominal movements that are synchronous with the chest movements are normal. D) Grunting on expiration is an abnormal finding that should be reported to the physician. E) Nasal flaring is an abnormal finding that should be reported to the physician.

1) Risk factors for labor dystocia include which of the following? Note: Credit will be given only if all correct choices and no incorrect choices are selected. Select all that apply. A) Tall maternal height B) Labor induction C) Small-for-gestational-age (S G A) fetus D) Malpresentation E) Prolonged latent phase

Answer: B, D, E Explanation: A) Short maternal height, not tall, is a risk factor of dystocia. B) Labor induction is a risk factor of dystocia. C) Large-for-gestational-age (F G A) fetus, not small, is a risk factor of dystocia. D) Malpresentation is a risk factor of dystocia. E) Prolonged latent phase is a risk factor of dystocia.

1) Amniotomy as a method of labor induction has which of the following advantages? Note: Credit will be given only if all correct choices and no incorrect choices are selected. Select all that apply. A) The danger of a prolapsed cord is decreased. B) There is usually no risk of hypertonus or rupture of the uterus. C) The intervention can cause a decrease in pain. D) The color and composition of amniotic fluid can be evaluated. E) The contractions elicited are similar to those of spontaneous labor.

Answer: B, D, E Explanation: A) The danger of a prolapsed cord is increased once the membranes have ruptured, especially if the fetal presenting part is not firmly pressed down against the cervix. B) There is usually no risk of hypertonus or rupture of the uterus, and this is an advantage of amniotomy. C) The intervention can cause an increase in pain, making labor more difficult to manage. D) The color and composition of amniotic fluid can be evaluated, and this is an advantage of amniotomy. E) The contractions elicited are similar to those of spontaneous labor, and this is an advantage of amniotomy.

17) A pregnant client who is a lacto-vegetarian asks the nurse for assistance with her diet. What instruction should the nurse give? Note: Credit will be given only if all correct choices and no incorrect choices are selected. Select all that apply. A) "Protein is important; therefore, the addition of one serving of meat a day is necessary." B) "A daily supplement of vitamin B12 is important." C) "The high fiber in a vegetarian diet is dangerous for pregnant women." D) "Eggs are important to add to your diet. Eat six eggs per week." E) "Milk products contain protein, but they are very low in iron."

Answer: B, E Explanation: A) Lacto-vegetarians do not eat meat, meat by-products, or eggs, and the nurse should not force this issue. Most vegetables must be combined with another food to form complete proteins. B) Supplementation may be recommended for vegans who have difficulty meeting the recommended amounts of vitamin B12 through food sources. C) The high fiber found in vegetarian diets actually is good for the pregnant woman who may be suffering from constipation. D) Lacto-vegetarians do not eat meat, meat by-products, or eggs, and the nurse should not force this issue. Most vegetables must be combined with another food to form complete proteins. E) Milk products will provide needed protein, but are not significant sources of iron.

1) The nurse determines the gestational age of an infant to be 40 weeks. Which characteristics are most likely to be observed? Note: Credit will be given only if all correct choices and no incorrect choices are selected. Select all that apply. A) Lanugo abundant over shoulders and back B) Plantar creases over entire sole C) Pinna of ear springs back slowly when folded. D) Vernix well distributed over entire body E) Testes are pendulous, and the scrotum has deep rugae

Answer: B, E Explanation: A) Vernix and lanugo both disappear as the infant reaches term. B) Sole (plantar) creases are reliable indicators of gestational age in the first 12 hours of life. C) The pinna's springing back slowly indicates prematurity. By term, the newborn's pinna is firm, stands away from the head, and springs back quickly from the folding. D) Vernix and lanugo both disappear as the infant reaches term. E) By term, the testes are generally in the lower scrotum, which is pendulous and covered with rugae.

1) The nurse is planning a community education program on the role of complementary and alternative therapies during pregnancy. Which statement about alternative and complementary therapies should the nurse include? A) "They bring about cures for illnesses and diseases." B) "They are invasive but effective for achieving health." C) "They emphasize prevention and wellness." D) "They prevent pregnancy complications."

Answer: C Explanation: A) These therapies emphasize prevention and wellness, aiming for holistic health rather than cure or treatment. B) Most alternative and complimentary therapies are noninvasive. The only ones that are invasive are acupuncture, herbs, and foods. C) Complementary and alternative therapies have many benefits during pregnancy. They emphasize prevention and wellness, aiming for holistic health rather than cure or treatment. No method of treatment can prevent all pregnancy complications

1) Induction of labor is planned for a 31-year-old client at 39 weeks due to insulin-dependent diabetes. Which nursing action is most important? A) Administer 100 mcg of misoprostol (Cytotec) vaginally every 2 hours. B) Place dinoprostone (Prepidil) vaginal gel and ambulate client for 1 hour. C) Begin Pitocin (oxytocin) 4 hours after 50 mcg misoprostol (Cytotec). D) Prepare to induce labor after administering a tap water enema.

Answer: C Explanation: A) 100 mcg every 2 hours is too much medication administered too frequently. B) The client must remain recumbent for 2 hours after administration of dinoprostone (Prepidil) vaginal gel, during which time she is continuously monitored. C) Pitocin should not administered less than 4 hours after the last Cytotec dose. D) Enemas are not routinely used in labor. This order is not expected.

1) When counseling a newly pregnant client at 8 weeks' gestation with twins, the nurse teaches the woman about the need for increased caloric intake. What would the nurse tell the woman that the minimum recommended intake should be? A) 2500 k c a l and 120 grams protein B) 3000 k c a l and 150 grams protein C) 4000 k c a l and 135 grams protein D) 5000 k c a l and 190 grams protein

Answer: C Explanation: A) 2500 k c a l and 120 grams protein is less than the recommended caloric and protein intake for a twin-gestation pregnancy. B) 3000 k c a l and 150 grams protein is lower in calories but higher in protein than is recommended for a twin-gestation pregnancy. C) 4000 k c a l and 135 grams protein is the recommended caloric and protein intake in a twin-gestation pregnancy. D) 5000 k c a l and 190 grams protein is more than recommended caloric and protein intake for a twin-gestation pregnancy.

1) The nurse is providing care to a Hispanic client who has experienced a stillbirth. Which action by the nurse is appropriate when providing culturally competent care to the client and family members? A) Asking the paternal grandfather if they will sit in shiva B) Asking the oldest male member of the family if he would like to participate in the cremation C) Asking the parents if they wish to have their baby baptized D) Asking the parents if they wish to participate in the ritual of painting the baby's body

Answer: C Explanation: A) A Jewish, not Hispanic, family may sit in shiva after the loss of a baby. This action is not appropriate when providing culturally competent care to the client and family. B) A Hindu, not Hispanic, family, may want the oldest male member of the family to participate in the baby's cremation. This action is not appropriate when providing culturally competent care to the client and family. C) Many Hispanic families will want the baby to be baptized. This action is appropriate when providing culturally competent care to the client and family. D) A Native American, not Hispanic, family may want to participate in a ritual of painting the baby's body. This action is not appropriate when providing culturally competent care to the client and family.

1) A woman has been in labor for 16 hours. Her cervi × is dilated to 3 c m and is 80% effaced. The fetal presenting part is not engaged. The nurse would suspect which of the following? A) Breech malpresentation B) Fetal demise C) Cephalopelvic disproportion (C P D) D) Abruptio placentae

Answer: C Explanation: A) A breech presentation would not prevent the presenting part from becoming engaged. B) Fetal demise would not prevent the presenting part from becoming engaged. C) Cephalopelvic disproportion (C P D) prevents the presenting part from becoming engaged. D) Abruptio placentae has specific complications; however, it would not prevent engagement of the presenting part.

1) The nurse is providing care to a pregnant client diagnosed with a urinary tract infection (U T I) during a routine prenatal visit. What will the nurse educate the client about based on this data? A) Gestational hypertension B) Gestational diabetes mellitus C) Preterm labor D) Anemia

Answer: C Explanation: A) A diagnosis of a U T I during pregnancy does not increase the risk for gestational hypertension. B) A diagnosis of a U T I during pregnancy does not increase the risk for gestational diabetes mellitus. C) The nurse would provide teaching to the client regarding signs and symptoms associated with preterm labor, as the diagnosis of a U T I increases the risk for developing this complication of pregnancy. A diagnosis of a UTI during pregnancy does not increase the risk for anemia

1. The postpartum client who is being discharged from the hospital experienced severe postpartum depression after her last birth. What should the nurse include in the plan of follow-up care for this client? A) One visit from a home care nurse, to take place in 2 days B) Two visits from a public health nurse over the next month C) An appointment with a mental health counselor D) Follow-up with the obstetrician in 6 weeks

Answer: C Explanation: A) A home visit in 2 days will be helpful to assess feeding, but is too early to detect signs of postpartum depression. B) Two home visits in a month are too sporadic to accurately detect postpartum depression. C) Postpartum depression has a high recurrence rate. Women with a history of postpartum psychosis or depression, or other risk factors, may benefit from a referral to a mental health professional for counseling during pregnancy or postpartum. D) Following up with the obstetrician in 6 weeks is too long a wait.

1) The nurse has received a phone call from a multigravida who is 21 weeks pregnant and has not felt fetal movement yet. What is the best action for the nurse to take? A) Reassure the client that this is a normal finding in multigravidas. B) Suggest that she should feel for movement with her fingertips. C) Schedule an appointment for her with her physician for that same day. D) Tell her gently that her fetus is probably dead.

Answer: C Explanation: A) A lack of fetal movement is unusual at 21 weeks, and should be checked. B) Fetal movement can be actively palpated by the client's physician or a trained examiner, but is unlikely to be self-detected by the mother at this stage. C) Quickening, or the mother's perception of fetal movement, occurs about 18 to 20 weeks after the L M P in a primigravida (a woman who is pregnant for the first time) but may occur as early as 16 weeks in a multigravida (a woman who has been pregnant more than once). D) The fetus may or may not have died after or about the 20th week of pregnancy; however, telling the client that the fetus might have died in utero without confirmation of this fact is nontherapeutic.

1) The client requires vacuum extraction assistance. To provide easier access to the fetal head, the physician cuts a mediolateral episiotomy. After delivery, the client asks the nurse to describe the episiotomy. How does the nurse respond? A) "The episiotomy goes straight back toward your rectum." B) "The episiotomy is from your vagina toward the urethra." C) "The episiotomy is cut diagonally away from your vagina." D) "The episiotomy extends from your vagina into your rectum."

Answer: C Explanation: A) A midline episiotomy is straight back from the vagina toward the rectum. B) Episiotomies are not cut anteriorly toward the urethra. C) A mediolateral episiotomy is angled from the vaginal opening toward the buttock. It begins in the midline of the posterior fourchette and extends at a 45-degree angle downward to the right or left. D) Extension into the rectum is a fourth-degree laceration.

1) The nurse is conducting a health maintenance assessment for a new female client who recently moved to the city. Which finding would indicate the need for further assessment for intimate partner violence? A) A miscarriage two years ago noted during the reproductive history. B) A sprained ankle one year ago noted during the health history interview. C) A history of delaying treatment for a concussion and fractured extremity. A scar noted on the abdomen from a previous surgery during the physical examination

Answer: C Explanation: A) A miscarriage two years ago that is noted in the reproductive history would not cause the nurse to further assess the client for intimate partner abuse. B) Sprains and strains are not associated with intimate partner abuse. C) A client who delays treatment for a concussion or fractured extremity would indicate the need for further assessment for intimate partner violence. A scar from an old injury, not from a surgical procedure, would indicate the need for further assessment for intimate partner violence

1) The nurse knows that in some cases, breastfeeding is not advisable. Which mother should be counseled against breastfeeding? A) A mother with a poorly balanced diet B) A mother who is overweight C) A mother who is H I V positive D) A mother who has twins

Answer: C Explanation: A) A newborn whose mother has a poor diet might need to receive supplements. B) Mothers who are overweight can be encouraged to breastfeed. C) Women with H I V or A I D S are counseled against breastfeeding. D) Mothers who have twins can be encouraged to breastfeed.

1) A client in her second trimester is complaining of spotting. Causes for spotting in the second trimester are diagnosed primarily through the use of which of the following? A) A non-stress test B) A vibroacoustic stimulation test C) An ultrasound D) A contraction stress test

Answer: C Explanation: A) A non-stress test is used to assess the well-being of the fetus. B) A vibroacoustic stimulation test is used to assess the well-being of the fetus. C) Indirect diagnosis is made by locating the placenta via tests that require no vaginal examination. The most commonly employed diagnostic test is the transabdominal ultrasound scan. A contraction stress test is used to assess the well-being of the fetus

1) The postpartum client has chosen to bottle-feed her infant. Nursing actions that aid in lactation suppression include which of the following? A) Warm showers B) Pumping milk C) Ice packs to each breast D) Avoiding wearing a bra for 5 to 7 days

Answer: C Explanation: A) A nonbreastfeeding mother should avoid warm showers to decrease the flow of breast milk. B) A nonbreastfeeding mother should avoid pumping milk or any stimulation of breasts to decrease the flow of breast milk. C) A nonbreastfeeding mother should use cooling packs for comfort and to decrease the flow of breast milk. D) A nonbreastfeeding mother should wear a supportive, well-fitted bra to decrease the flow of breast milk.

1) The nurse has received the shift change report on infants born within the previous 4 hours. Which newborn should the nurse see first? A) 37-week male, respiratory rate 45 B) 8 pound 1 ounce female, pulse 150 C) Term male, nasal flaring D) 4-hour-old female who has not voided

Answer: C Explanation: A) A normal respiratory rate is 30 to 60 breaths/min. This infant has no unexpected findings. B) A normal pulse is 110 to 160 beats/min. This infant has no unexpected findings. C) Nasal flaring is an indication of respiratory distress. The nurse must be immediately available to provide appropriate interventions for a newborn in distress. D) The first voiding should occur within 24 hours and first passage of stool within 48 hours. This is not a life-threatening condition.

1) he nurse is making an initial assessment of the newborn. The findings include a chest circumference of 32.5 c m and a head circumference of 33.5 c m. Based on these findings, which action should the nurse take first? A) Notify the physician. B) Elevate the newborn's head. C) Document the findings in the chart. D) Assess for hypothermia immediately.

Answer: C Explanation: A) A physician would be notified only if findings were abnormal. B) There is no indication that the newborn's head should be elevated. C) Documentation is the appropriate first step. The average circumference of the head at birth is 32 to 37 c m, and average chest circumference ranges from 30 to 35 c m. D) None of the findings indicates hypothermia. No data on temperature are given.

1. The postpartum client has developed thrombophlebitis in her right leg. Which finding requires immediate intervention? A) The client reports she had this condition after her last pregnancy. B) The client develops pain and swelling in her left lower leg. C) The client appears anxious, and describes pressure in her chest. D) The client becomes upset that she cannot go home yet.

Answer: C Explanation: A) A risk factor includes recurrent thromboembolic disease, but this is neither a predictor nor an indication of complications. B) Development of thrombophlebitis is a complication, but not the top priority. C) The most common clinical findings of a pulmonary embolism include dyspnea, pleuritic chest pain, cough with or without hemoptysis, cyanosis, tachypnea and tachycardia, panic, syncope, or sudden hypotension and require immediate intervention. D) Becoming upset is a psychosocial issue and far less important than another finding.

1) On the 3rd day postpartum, a client who is not breastfeeding experiences engorgement. To relieve her discomfort, the nurse should encourage the client to do which of the following? A) Remove her bra B) Apply heat to the breasts C) Apply cold packs to the breasts D) Use a breast pump to release the milk

Answer: C Explanation: A) A support bra is recommended. B) Applying heat would stimulate milk production. C) Applying cold packs to the breasts relieves discomfort and helps suppress lactation. D) Using a breast pump would stimulate milk production and delay the suppression process.

1) The nurse has been asked by a community organization to give a presentation on prevention of teen pregnancy. Which statement indicates appropriate steps toward reduction of the local teen pregnancy rate? A) Abstinence-only education will be offered in the school and clinics. B) Classes on how to parent will be mandatory in high school. C) A low-cost reproductive health clinic will be planned. D) Parents will be encouraged to avoid discussing sexual activity.

Answer: C Explanation: A) Abstinence with information on contraception is most effective in reducing teen pregnancy rates. B) Parenting classes for teens who are neither pregnant nor parents do not reduce teen pregnancy. C) Key strategies for prevention of unintended teen pregnancy and sexual health promotion include the provision of services that ensure accessible and high-quality reproductive healthcare; sex education programs that provide developmentally appropriate, evidence-based curricula; and youth development strategies to enhance life skills. A low-cost reproductive health clinic would be able to provide these services. D) Parents are a big influence on teens' decisions to begin or avoid sexual activity. Parents need to talk with their children about sex early and often and be specific in the discussions.

1) The nurse is assessing a client who reports seeing an acupuncturist on a weekly basis to treat back pain. The nurse understands that acupuncture is an example of what? A) A risky practice without evidence of efficacy B) A folk remedy C) A complementary therapy D) An alternative therapy

Answer: C Explanation: A) Acupuncture has been used in traditional Chinese medicine for over 3000 years. Good evidence is available on the efficacy of acupuncture for treatment of chronic pain. B) A folk remedy is a practice of a cultural group that either has no evidence to support efficacy or has been found not to have an effect. C) Acupuncture is a therapy that is used in conjunction with conventional medical treatment, and therefore is an example of a complementary therapy. D) Acupuncture is not categorized as an alternative therapy, because it is used in conjunction with conventional medical treatment.

1) The nurse is providing care to a couple who have experienced a perinatal loss. Which nursing diagnosis is appropriate to include in the plan of care? A) Acute Pain B) Complicated Grieving C) Powerlessness D) Insomnia

Answer: C Explanation: A) Acute pain is not an appropriate nursing diagnosis to include in the couple's plan of care. B) Complicated grieving is not an appropriate nursing diagnosis to include in the couple's plan of care. C) Powerlessness related to the sense of a lack of control in the current situational crisis is an appropriate nursing diagnosis to include in the couple's plan of care. Insomnia is not an appropriate nursing diagnosis to include in the couple's plan of care

18) A pregnant client confides to the nurse that she is eating laundry starch daily. The nurse should assess the client for which of the following? A) Alopecia B) Weight loss C) Iron deficiency anemia D) Fecal impaction

Answer: C Explanation: A) Alopecia, a condition that causes hair loss, is not associated with eating laundry starch. B) Weight gain is related to the client's eating laundry starch. C) Iron deficiency anemia is the most common concern with pica. The ingestion of laundry starch or certain types of clay may contribute to iron deficiency by replacing iron-containing foods from the diet or by interfering with iron absorption. D) Fecal impaction is associated with the eating of clay, not laundry starch.

1) A new father asks the nurse to describe what his baby will experience while sleeping and awake. What is the best response? A) "Babies have several sleep and alert states. Keep watching and you'll notice them." B) "You might have noticed that your child was in an alert awake state for an hour after birth." C) "Newborns have two stages of sleep: deep or quiet sleep and rapid eye movement sleep." D) "Birth is hard work for babies. It takes them a week or two to recover and become more awake."

Answer: C Explanation: A) Although it is true that babies have several sleep and alert states, the wording of this response is condescending and not therapeutic. B) Although this statement is true, it does not respond to the father's question about sleeping now. C) Teaching the parents how to recognize the two sleep stages helps them tune in to their infant's behavioral states. D) Recovery from the birth process only takes a day or two. The newborn usually shows a predictable pattern of behavior during the first several hours after birth.

1) The nurse is performing a postpartum home care visit. Which teaching has the highest priority? A) Teaching or reviewing how to bathe the baby B) Teaching how to thoroughly childproof the house C) How many wet diapers the baby should have daily D) Prevention of plagiocephaly

Answer: C Explanation: A) Although knowing how to bathe a newborn is important, the number of wet diapers is the highest priority. B) Childproofing the home is not necessary until the baby begins to crawl. This is a low priority at this time. C) Assessment of intake, output, weight, and hydration status is imperative. The baby should have at least six diapers that are saturated with clear urine each day by 1 week of age. Wet diapers are an indication of hydration of the newborn. This is the highest priority. D) Preventing flat spots on the back or side of the infant's head is primarily a cosmetic issue. The number of wet diapers would be a higher priority.

1) The nurse is admitting a client to the labor and delivery unit. Which aspect of the client's history requires notifying the physician? A) Blood pressure 120/88 B) Father a carrier of sickle-cell trait C) Dark red vaginal bleeding D) History of domestic abuse

Answer: C Explanation: A) Although the diastolic reading is slightly elevated, this blood pressure reading is not the top priority. B) The infant also might have sickle-cell trait, but it is not life-threatening at this time. C) Third-trimester bleeding is caused by either placenta previa or abruptio placentae. Dark red bleeding usually indicates abruptio placentae, which is life-threatening to both mother and fetus. D) This client is at risk for harm after delivery but is not in a life-threatening situation at this time. This is not the highest priority for the client.

1) The nurse is caring for a 15-year-old client who gave birth to her first child yesterday. What action is the best indicator that the nurse understands the parenting adolescent? A) The client's mother is included in all discussions and demonstrations. B) The father of the baby is encouraged to change a diaper and give a bottle. C) The nurse explains the characteristics and cues of the baby when assessing him. D) A discussion on contraceptive methods is the first topic of teaching.

Answer: C Explanation: A) Although the grandmother or another family member may plan to assist with or provide much of the newborn care in some cases, the nurse should always ensure that the adolescent mother has the knowledge and demonstrates the skills to provide care for her newborn before discharge. B) The father, if he is involved, should be included as much as possible, but having the mother learn more about her new baby is a higher priority. C) A newborn physical examination performed at the bedside gives the parent(s) immediate feedback about the newborn's health and demonstrates methods of handling an infant. This action helps the client learn about her baby as an individual and facilitates maternal-infant attachment. This is the highest priority. The nurse should offer detailed teaching on contraception, as the young woman may have no prior experience with it and may not feel comfortable requesting this information, but establishing rapport and facilitating understanding of and attachment to the newborn is a higher priority

1) The parents of a newborn male ask the nurse whether they should circumcise their son. What is the best response by the nurse? A) "Circumcision should be undertaken to prevent problems in the future." B) "Circumcision might decrease the child's risk of developing a urinary tract infection." C) "Circumcision can sometimes cause complications. What questions do you have?" D) "Circumcision is painful, and should be avoided unless you are Jewish."

Answer: C Explanation: A) Although this is a common reason parents give for requesting circumcision, it is still an opinion not based in medical fact. B) Although circumcision permits exposure of the glans for easier cleaning, getting more information from the parents about their questions or concerns would be better. C) Asking this question allows the nurse to determine what the parents' concerns are, then address them specifically. D) Although circumcision can be painful, most providers administer a penile nerve root block to prevent or minimize procedural pain. Circumcision is practiced in many religions and traditions.

1. The nurse is preparing a client education handout on the differences between false labor and true labor. What information is most important for the nurse to include? A) True labor contractions begin in the back and sweep toward the front. B) False labor often feels like abdominal tightening, or "balling up." C) True labor can be diagnosed only if cervical change occurs. D) False labor contractions do not increase in intensity or duration.

Answer: C Explanation: A) Although this is a true statement, it is not the most important indication of true labor. B) Although this is a true statement, it is not the most important fact about false and true labor. C) Cervical change is the only factor that actually distinguishes false from true labor. The contractions of true labor produce progressive dilation and effacement of the cervix. The contractions of false labor do not produce progressive cervical effacement and dilation. D) Although this is a true statement, it is not the most important fact about true and false labor.

1) The maternal health nurse is caring for a pregnant client with obsessive-compulsive disorder (O C D). Which action will the nurse take when teaching the client, based on the client's psychological disorder? A) Allow the client opportunity for repetitive rituals. B) Ask the client to follow along with written material. C) Repeat instructions as needed. D) Provide multiple handouts with various photographs

Answer: C Explanation: A) Although those with O C D may have repetitive rituals, this does not promote learning and is not the best action for the nurse. B) While written material may be helpful as a supplement to learning, it may be difficult for the client with O C D to follow along. C) The best action for the nurse is to repeat any instructions or teachings as needed. This allows the client more opportunity to learn. While multiple learning techniques and opportunities are helpful for most individuals, those with OCD have a difficult time following along with teaching material

1. The postpartum client is concerned about mastitis because she experienced it with her last baby. Preventive measures the nurse can teach include which of the following? A) Wearing a tight-fitting bra B) Limiting breastfeedings C) Frequent breastfeedings D) Restricting fluid intake

Answer: C Explanation: A) Although wearing a supportive bra is recommended, a tight-fitting bra would tend to suppress lactation. B) The woman should continue to breastfeed; in fact, regular drainage of both breasts actually helps by preventing milk stasis and abscess formation. C) Treatment and prevention of mastitis includes frequent and complete emptying of the breasts. D) Supportive measures include increased fluid intake (at least 2 to 2.5 L/day).

1. Which relief measure would be most appropriate for a postpartum client with superficial thrombophlebitis? A) Urge ambulation B) Apply ice to the leg C) Elevate the affected limb D) Massage her calf

Answer: C Explanation: A) Ambulation would increase discomfort. B) Applying ice to the leg is contraindicated in thrombophlebitis treatment. Treatment includes application of local heat. C) Treatment for superficial thrombophlebitis involves application of local heat, elevation of the affected limb, and analgesic agents. D) Massaging the calf is contraindicated because it can cause a breakup of a clot and put the client at risk for a pulmonary embolus. Pulmonary embolism occurs when a thrombus from a lower extremity or the pelvis lodges in the pulmonary vascular bed and restricts circulation to the corresponding area of the lung vasculature.

1) What is the most significant cause of neonatal morbidity and mortality? A) A) Amenorrhea B) B) Posttraumatic stress disorder C) C) Prematurity D) D) Endometriosis

Answer: C Explanation: A) Amenorrhea the absence of menses in a woman and does not affect neonatal morbidity or mortality. B) Posttraumatic stress disorder does not affect neonatal morbidity or mortality. C) The most significant cause of neonatal morbidity and mortality is prematurity and its associated complications such as respiratory distress syndrome, necrotizing enterocolitis, and intraventricular hemorrhage. D) Endometriosis is a condition of a woman characterized by the presence of endometrial tissue outside the uterine cavity, and it does not affect neonatal morbidity or mortality.

1) The nurse is reviewing preconception questionnaires in charts. Which couple are the most likely candidates for preconceptual genetic counseling? A) Wife is 30 years old, husband is 31 years old B) Wife and husband are both 29 years old, first baby for husband, wife has a normal 4-year-old C) Wife's family has a history of hemophilia D) Single 32-year-old woman is using donor sperm

Answer: C Explanation: A) An age under 35 is not a risk factor for genetic abnormalities. B) An age under 35 is not a risk factor for genetic abnormalities. C) For families in which the woman is a known or possible carrier of an X-linked disorder, such as hemophilia, the risk of having an affected male fetus is 25%. D) Sperm donors are screened for genetic disorders, and men with a possible genetic problem are not accepted for sperm donation.

1) Whether sensitization is the result of a blood transfusion or maternal-fetal hemorrhage for any reason, what test can be performed to determine the amount of R h(D) positive blood present in the maternal circulation, and to calculate the amount of R h immune globulin needed? A) Indirect Coombs' test B) Non-stress test C) Kleihauer-Betke or rosette test D) Direct Coombs' test

Answer: C Explanation: A) An indirect Coombs' test is done on the mother's blood to measure the number of R h-positive antibodies. B) A non-stress test is performed to assess the fetal heart rate in response to fetal movement. C) A Kleihauer-Betke or rosette test can be performed to determine the amount of R h(D) positive blood present in the maternal circulation and to calculate the amount of R h immune globulin needed. D) A direct Coombs' test is done on the infant's blood to detect antibody-coated R h-positive R B Cs.

1) The nurse is assessing a drug-dependent newborn. Which symptom would require further assessment by the nurse? A) Occasional watery stools B) Spitting up after feeding C) Jitteriness and irritability D) Nasal stuffiness

Answer: C Explanation: A) An occasional watery stool can be associated with the normal newborn. B) Spitting up after some feedings can be associated with the normal newborn. C) Jitteriness and irritability can be an indicator of drug withdrawal. D) Nasal stuffiness can be associated with the normal newborn.

1) The nurse is providing prenatal care to an obese client who asks, "How much weight should I gain during my pregnancy?" Which response by the nurse is appropriate? A) "You should gain 15 to 25 pounds." B) "You should gain 25 to 35 pounds." C) "You should gain 11 to 20 pounds." D) "You should gain 28 to 40 pounds."

Answer: C Explanation: A) An overweight client should gain 15 to 25 pounds during pregnancy. B) A pregnant client who has a normal weight before pregnancy should gain 25 to 35 pounds during pregnancy. C) An obese client who becomes pregnant should gain 11 to 20 pounds during pregnancy. D) An underweight client should gain 28 to 40 pounds during pregnancy.

1) The nurse is providing discharge instructions to a client with a diagnosis of vulvovaginal candidiasis (V V C), and knows the client understands when she makes which of the following statements? A) "I need to apply the miconazole for 10 days." B) "I need to douche daily." C) "I need to add yogurt to my diet." D) "I need to wear nylon panties."

Answer: C Explanation: A) Applying miconazole for 10 days does not prevent or assist in treating vulvovaginal candidiasis. B) Douching daily does not prevent or assist in treating vulvovaginal candidiasis. C) Yogurt helps reestablish normal vaginal flora. D) Wearing nylon panties does not prevent or assist in treating vulvovaginal candidiasis.

1) The nurse is helping a victim of domestic abuse to develop a safety plan. Which client action would require intervention by the nurse? A) Asking a neighbor to call police if violence begins B) Establishing a code word for danger with family and friends C) Keeping a bag packed in the home in case the need to leave arises D) Having a planned escape route and emergency phone numbers if violence occurs

Answer: C Explanation: A) Asking a neighbor to call the police if violence begins is an appropriate client action that would not require intervention from the nurse. B) Establishing a code word for danger with family and friends is an appropriate client action that would not require intervention from the nurse. C) Keeping a bag packed in the home if the need arises to leave would require intervention from the nurse. The bag should be kept at the home of a neighbor or family member. If the abuser finds the bag the client's risk for injury may increase. D) Having a planned escape route and emergency phone numbers if violence occurs is an appropriate client action that would not require intervention from the nurse.

1) To prevent sudden infant death syndrome (S I D S), the nurse encourages the parents of a term infant to place the infant in which position when the infant is sleeping? A) On the parents' waterbed B) Swaddled in the infant swing C) On the back D) On the sides

Answer: C Explanation: A) Babies should sleep in the same room as their parent(s) or an adult care provider, but they should not share the bed with anyone. They should also sleep on a firm surface to help prevent S I D S. B) Babies should not sleep with commercial devices marketed to maintain position. Safety has not been established. C) Babies should sleep on their backs every time they are put down for sleep. D) Babies should sleep on their backs every time they are put down for sleep; all caregivers should be informed that side positioning is unsafe.

1) The nurse is preparing a teaching brochure for Spanish-speaking postpartum clients. Which topics are critical for this population? A) Baby baths and birth certificates B) Hygiene practices C) When and how to contact their healthcare provider D) Pain-relief options in labor and after birth

Answer: C Explanation: A) Baby baths and birth certificates are necessary information, but not critical. B) It is important to consider cultural practices and realize that some women may prefer not to shower in the first few days following birth. Some Hispanic women prefer to delay showering. Hygiene practices are important, but not critical. C) Knowing how to contact their healthcare provider at all times is critical so that clients receive appropriate advice and care in case of a problem or emergency. Knowing what to watch for and when to call the healthcare provider also facilitates safety. These are the highest priorities. D) Pain relief is important, but not critical.

A client asks her nurse, "Is it okay for me to take a tub bath during the heavy part of my menstruation?" What is the nurse's correct response? A) "Tub baths are contraindicated during menstruation." B) "You should shower and douche daily instead." C) "Either a bath or a shower is fine at that time." D) "You should bathe and use a feminine deodorant spray during menstruation."

Answer: C Explanation: A) Bathing in a tub is not contraindicated during menses. B) Douching should be avoided during menstruation. C) Bathing, whether it is a tub bath or a shower, is as important (if not more so) during menses as at any other time. D) Bathing is as important (if not more so) during menses as at any other time, but feminine deodorant sprays are unnecessary.

1) The nurse is supervising care in the emergency department. Which situation most requires an intervention? A) Moderate vaginal bleeding at 36 weeks' gestation; client has an Ⅳ of lactated Ringer's solution running at 125 m L/hour B) Spotting of pinkish-brown discharge at 6 weeks' gestation and abdominal cramping; ultrasound scheduled in 1 hour C) Bright red bleeding with clots at 32 weeks' gestation; pulse = 110, blood pressure 90/50, respirations = 20 D) Dark red bleeding at 30 weeks' gestation with normal vital signs; client reports an absence of fetal movement

Answer: C Explanation: A) Bleeding in the third trimester is usually due to placenta previa or placental abruption. The primary intervention for placenta previa or placental abruption is intravenous (Ⅳ) therapy, which the patient already has in place. B) Bleeding in the first trimester can be indicative of spontaneous abortion beginning, or of an ectopic pregnancy. Transvaginal ultrasound is used for diagnosis. C) Bleeding in the third trimester is usually due to placenta previa or placental abruption. Observe the woman for indications of shock, such as pallor, clammy skin, perspiration, dyspnea, or restlessness. Monitor vital signs, particularly blood pressure and pulse, for evidence of developing shock. D) Bleeding in the third trimester can indicate placental abruption. Normal vital signs indicate a normal vascular volume. Decrease in fetal movement or cessation of movement may indicate fetal compromise. The fetus is at greatest risk in this situation; the mother is stable.

1) When is breastfeeding contraindicated? A) Infant has hypertension B) Mother has a history of treated tuberculosis C) Mother is H I V positive or has A I D S D) Mother has a history of treated herpes

Answer: C Explanation: A) Breastfeeding is contraindicated when the infant has galactosemia. B) Breastfeeding is contraindicated when the mother has active, untreated tuberculosis. C) Breastfeeding is contraindicated when the mother is H I V positive or has A I D S and is counseled against breastfeeding. D) Breastfeeding is contraindicated when the mother has active herpes on her breast-the infant may still feed on the unaffected side only, until the lesion has healed.

1) On the first postpartum day, the nurse teaches the client about breastfeeding. Two hours later, the mother seems to remember very little of the teaching. The nurse understands this memory lapse to be related to which of the following? A) The taking-hold phase B) Postpartum hemorrhage C) The taking-in period D) Epidural anesthesia

Answer: C Explanation: A) By the second or third day after birth, the new mother may be observed to be ready to resume control of her body, her mothering, and her life in general. This is called the taking-hold phase. B) Postpartum hemorrhage is a serious complication and needs medical intervention. A low or decreasing blood pressure may reflect hypovolemia secondary to hemorrhage. C) Soon after birth during the taking-in period, the woman tends to be passive and somewhat dependent. She follows suggestions, hesitates about making decisions, and is still rather preoccupied with her needs. D) Epidural anesthesia is a pharmacological approach to pain control. There may be an increased incidence of headache if the woman had spinal or epidural anesthesia.

1) Which of the following diagnostic tests would the nurse question when ordered for a client diagnosed with pelvic inflammatory disease (P I D)? A) C B C (complete blood count) with differential B) Venereal Disease Research Laboratory (V D R L) C) Throat culture for Streptococcus A D) R P R (Rapid Plasma Reagin)

Answer: C Explanation: A) C B C with differential will be ordered to give an indication of the severity of the infection. B) The Venereal Disease Research Laboratory (V D R L) test checks for syphilis. C) Streptococcus of the throat is not associated with P I D. D) R P R is a test for syphilis, a cause of P I D.

1) Which statement regarding cervical mucus is accurate during ovulation and appropriate to include in an educational session with the client? A) Cervical mucus is thicker during ovulation. B) Cervical mucus is opaque during ovulation. C) Cervical mucus is clearer during ovulation. D) Cervical mucus is acidic during ovulation.

Answer: C Explanation: A) Cervical mucus is thinner, not thicker, during ovulation. B) Cervical mucus is clearer, not opaque, during ovulation. C) Cervical mucus is clearer during ovulation. D) Cervical mucus is alkaline, not acidic, during ovulation.

The nurse suspects that a female patient is experiencing amenorrhea because of ovarian failure. For which situation should the nurse assess this patient? A) Severe stress B) Recent head trauma C) Treatment for cancer D) Antianxiety medication

Answer: C Explanation: A) Chemotherapy and radiation are reasons for the development of ovarian failure. Severe stress, antianxiety medication, and head trauma can cause hypothalamic dysfunction as a reason for amenorrhea. B) Chemotherapy and radiation are reasons for the development of ovarian failure. Severe stress, antianxiety medication, and head trauma can cause hypothalamic dysfunction as a reason for amenorrhea. C) Chemotherapy and radiation are reasons for the development of ovarian failure. Severe stress, antianxiety medication, and head trauma can cause hypothalamic dysfunction as a reason for amenorrhea. D) Chemotherapy and radiation are reasons for the development of ovarian failure. Severe stress, antianxiety medication, and head trauma can cause hypothalamic dysfunction as a reason for amenorrhea.

1) A laboring mother has recurrent late decelerations. At birth, the infant has a heart rate of 100, is not breathing, and is limp and bluish in color. What nursing action is best? A) Begin chest compressions. B) Begin direct tracheal suctioning. C) Begin bag-and-mask ventilation. D) Obtain a blood pressure reading.

Answer: C Explanation: A) Chest compressions are not initiated until the heart rate is less than 60 and respirations have been established. B) Direct tracheal suctioning would be appropriate if there were meconium-stained fluid. There is no information about the amniotic fluid in the question. C) Most newborns can be effectively resuscitated by bag-and-mask ventilation. D) Blood pressure is insignificant during resuscitation efforts. This infant needs respirations established.

Client safety goals, which are evaluated and updated regularly, are requirements for what? A) Clinical practice guidelines B) Scope of practice C) Accreditation D) Standards of care

Answer: C Explanation: A) Clinical practice guidelines are adopted within a healthcare setting to reduce variation in care management, to limit costs of care, and to evaluate the effectiveness of care. B) State nurse practice acts protect the public by broadly defining the legal scope of practice within which every nurse must function and by excluding untrained or unlicensed individuals from practicing nursing. C) The Joint Commission has identified client safety as an important responsibility of healthcare providers. D) Standards of care establish minimum criteria for competent, proficient delivery of nursing care.

1) The nurse is providing care to a couple who have experienced a perinatal loss. Which is an expected early physical response to the loss? A) Confusion B) Preoccupation C) Loss of appetite D) Poor concentration

Answer: C Explanation: A) Confusion is an early cognitive, not physical, response to the loss. B) Preoccupation is an early cognitive, not physical, response to the loss. C) Loss of appetite is an early physical response to loss. D) Poor concentration is an early cognitive, not physical, response to loss.

1) The pregnant client in her second trimester states, "I didn't know my breasts would become so large. How do I find a good bra?" The best answer for the nurse to give would be which of the following? A) "Avoid cotton fabrics and get an underwire bra; they fit everyone best." B) "Just buy a bra one cup size bigger than usual, and it will fit." C) "Look for wide straps and cups big enough for all of your breast tissue." D) "There isn't much you can do for comfort. Try not wearing a bra at all."

Answer: C Explanation: A) Cotton is comfortable during pregnancy when perspiration increases, because it does not retain heat and moisture. B) The client should be fitted for a well-fitting, supportive bra of an appropriate size. C) The nurse should instruct the client to get a bra that fits with straps that are wide and do not stretch, and a cup that holds all breast tissue comfortably. D) One can obtain a bra that fits and is comfortable. It is not necessary to be uncomfortable.

While a child is being admitting to the hospital, the parent receives information about the pediatric unit's goals, including the statement that the unit practices family-centered care. The parent asks why that is important. The nurse responds that what communication dynamic is characteristic of the family-centered care paradigm? A) The mother is the principal caregiver in each family. B) The child's physician is the key person in ensuring that the health of a child is maintained. C) The family serves as the constant influence and continuing support in the child's life. D) The father is the leader in each home; thus, all communications should include him.

Answer: C Explanation: A) Culturally competent care recognizes that both matriarchal and patriarchal households exist. B) The physician is not present during the day-to-day routines in a child's life. C) Family-centered care is characterized by an emphasis on the family and family involvement throughout the pregnancy, birth, and postpartum period. D) Culturally competent care recognizes that both matriarchal and patriarchal households exist.

1) The nurse is caring for an infant who was delivered in a car on the way to the hospital and who has developed cold stress. Which finding requires immediate intervention? A) Increased skin temperature and respirations B) Blood glucose level of 45 C) Room-temperature I V running Positioned under radiant warmer

Answer: C Explanation: A) Decreased skin temperature and decreased respirations are signs and symptoms of cold stress. B) A blood glucose level of 45 is an adequate blood sugar in a neonate. A level lower than 40 indicates the infant is hypoglycemic. C) I V fluids should be warmed prior to administration and the newborn can be wrapped in a chemically activated warming mattress immediately following birth to decrease the postnatal fall in temperature that normally occurs. Radiant warmers are used to gradually increase the neonate's temperature

1) When a breastfeeding mother complains that her breasts are leaking milk, the nurse can offer which effective intervention? A) Decrease the number of minutes the newborn is at the breast per feeding. B) Decrease the mother's fluid intake. C) Place absorbent pads in the bra. D) Administer oxytocin.

Answer: C Explanation: A) Decreasing the number of minutes the newborn is at the breast would be contraindicated for a breastfeeding client. B) The mother should consume a nutritionally balanced diet with appropriate caloric and fluid intake to support breastfeeding. Decreasing the mother's fluid intake would be contraindicated for a breastfeeding client. C) The mother can wear nursing pads inside her bra with instructions to change wet pads frequently. D) Early breastfeeding can enhance maternal-infant bonding and facilitate release of oxytocin. Administering oxytocin would be contraindicated for a breastfeeding client.

1) The nurse has been talking to a woman about the reorganization phase following a rape. Which response would indicate that the client understands this phase? A) "By using denial and suppression in this phase, I will eventually be able to accept what has happened to me." B) "During this time, I won't talk much about the rape, because I am examining my inward feelings regarding the rape." C) "During this time, I will repeatedly replay the role of the victim until I come to terms with the experience." D) "My perception of a normal sexual relationship will be similar to my perception prior to the rape."

Answer: C Explanation: A) Denial and suppression indicate the client is experiencing the outward adjustment phase of rape trauma syndrome. B) Denial and suppression indicate the client is experiencing the outward adjustment phase of rape trauma syndrome. C) During reorganization, a victim adjusts her self-concept to include the rape. D) Sexual relationships often develop dysfunction after rape.

1) The student nurse is to perform Leopold maneuvers on a laboring client. Which assessment requires intervention by the staff nurse? A) The client is assisted into supine position, and the position of the fetus is assessed. B) The upper portion of the uterus is palpated, then the middle section. C) After determining where the back is located, the cervix is assessed. D) Following voiding, the client's abdomen is palpated from top to bottom.

Answer: C Explanation: A) Determination of fetal position and station is the point of Leopold maneuvers. The client is supine to facilitate uterine palpation. B) This is the correct order of the first and second Leopold maneuvers. C) The cervical exam is not part of Leopold maneuvers. Abdominal palpation is the only technique used for Leopold maneuvers. D) The client is instructed to void prior to beginning Leopold maneuvers to enhance comfort. Leopold maneuvers are essentially palpation of the uterus through the abdomen, beginning at the fundus and ending near the cervix.

1) A client who is 11 weeks pregnant presents to the emergency department with complaints of dizziness, lower abdominal pain, and right shoulder pain. Laboratory tests reveal a beta-hCG at a lower-than-expected level for this gestational age. An adnexal mass is palpable. Ultrasound confirms no intrauterine gestation. The client is crying and asks what is happening. The nurse knows that the most likely diagnosis is an ectopic pregnancy. Which statement should the nurse include? A) "You're feeling dizzy because the pregnancy is compressing your vena cava." B) "The pain is due to the baby putting pressure on nerves internally." C) "The baby is in the fallopian tube; the tube has ruptured and is causing bleeding." D) "This is a minor problem. The doctor will be right back to explain it to you."

Answer: C Explanation: A) Dizziness from vena cava compression occurs in the third trimester when women are supine. B) The fetus is too small to be putting pressure on the nerves. C) The woman who experiences one-sided lower abdominal pain or diffused lower abdominal pain, vasomotor disturbances such as fainting or dizziness, and referred right shoulder pain from blood irritating the subdiaphragmatic phrenic nerve is experiencing an ectopic pregnancy. D) Therapeutic communication requires giving the client an answer rather than referring the client to someone else.

1) The nurse determines the fundus of a postpartum client to be boggy. Initially, what should the nurse do? A) Document the findings. B) Catheterize the client. C) Massage the uterine fundus until it is firm. D) Call the physician immediately.

Answer: C Explanation: A) Documenting the findings would come after massage, reassessment, and evaluation. B) Catheterizing the client might be indicated if assessment reveals a full bladder and inability to void, but not as an initial intervention. C) The nurse would massage the uterine fundus until it is firm by keeping one hand in position and stabilizing the lower portion of the uterus. With one hand used to massage the fundus, the nurse would put steady pressure on the top of the now-firm fundus to see if she was able to express any clots. D) Calling the physician immediately is not necessary until more data are obtained.

1) Approximately what percentage of the newborn's body weight is water? A) 5% to 10% B) 90% to 95% C) 70% to 75% D) 50% to 60%

Answer: C Explanation: A) During the initial newborn period, term newborns have a physiologic weight loss of about 5% to 10% because fluid shifts. B) Approximately 70% to 75% of the newborn's body weight is water. C) Approximately 70% to 75% of the newborn's body weight is water. D) Approximately 70% to 75% of the newborn's body weight is water.

1) The pregnant 16-year-old is seeing the nurse during a prenatal visit. Based on the client's developmental level, which statement would the nurse expect the client to make? A) "My friends and I all wear totally different styles of clothing." B) "Having a baby will change my college plans." C) "I drink alcohol at parties most weekends." D) "My mom is my best friend."

Answer: C Explanation: A) Early and middle adolescents conform to group standards by wearing the same types of clothing that their friends wear. B) Late adolescence (18-19) is when the ability to think about the future develops, including understanding the impact that a baby will have on acquiring education. C) Middle adolescence (15-17) is a time of experimentation, including drinking alcohol, using other drugs, and sex. D) From early adolescence on, friends have an increasingly important role in the search for independence, which includes friends' replacing parents as the primary source of support.

1) Persistent early decelerations are noted. What would the nurse's first action be? A) Turn the mother on her left side and give oxygen. B) Check for prolapsed cord. C) Do nothing. This is a benign pattern. D) Prepare for immediate forceps or cesarean delivery.

Answer: C Explanation: A) Early decelerations do not require any intervention. B) Early decelerations do not indicate a prolapsed cord. C) Early decelerations are considered benign, and do not require any intervention. D) Early decelerations do not warrant an immediate delivery.

1) The nurse knows that a lecithin/sphingomyelin (L/S) ratio finding of 2:1 in amniotic fluid means which of the following? A) Fetal lungs are still immature. B) The fetus has a congenital anomaly. C) Fetal lungs are mature. D) The fetus is small for gestational age.

Answer: C Explanation: A) Early in pregnancy the lecithin concentration in amniotic fluid is less than that of sphingomyelin (0.5:1 at 20 weeks), resulting in a low lecithin/sphingomyelin (L/S) ratio, which is not the case in this instance. B) The L/S ratio isn't a measurement for congenital anomalies or size of the fetus. C) The concentration of lecithin begins to exceed that of sphingomyelin, and at 35 weeks the L/S ratio is 2:1. When at least two times as much lecithin as sphingomyelin is found in the amniotic fluid, R D S is very unlikely. D) The L/S ratio isn't a measurement for congenital anomalies or size of the fetus.

1) A client dilated to 5 c m has just received an epidural for pain. She complains of feeling lightheaded and dizzy within 10 minutes after the procedure. Her blood pressure was 120/80 before the procedure and is now 80/52. In addition to the bolus of fluids she has been given, which medication is preferred to increase her B P? A) Epinephrine B) Terbutaline C) Ephedrine D) Epifoam

Answer: C Explanation: A) Epinephrine is used to relieve bronchospasm, or during anaphylactic reactions. B) Terbutaline is used as a tocolytic. C) Ephedrine is the medication of choice to increase maternal blood pressure. D) Epifoam is a topical anesthetic.

1) A 49-year-old client comes to the clinic with complaints of severe perimenopausal symptoms including hot flashes, night sweats, urinary urgency, and vaginal dryness. The physician has prescribed a combination hormone replacement therapy of estrogen and progestin. When the client asks the nurse why she must take both hormones, what is the nurse's best reply? A) "Hot flashes respond better when replacement includes both hormones." B) "You are having very severe symptoms, so you need more hormones replaced." C) "There is an increased risk of tissue abnormality inside the uterus if only one is given." D) "Your blood pressure can become elevated if only one hormone is used."

Answer: C Explanation: A) Estrogen, not progestin, improves hot flashes and most other perimenopausal symptoms. B) The severity of symptoms will be considered by the physician in determining the appropriate dose for the client. C) Estrogen alone, in a woman with a uterus (unopposed estrogen), increases the risk of endometrial (the lining of the uterus) cancer by eightfold and, therefore, is never given without progesterone in these women. D) Estrogen therapy does not cause hypertension.

1) The transcultural nursing theory was developed in 1961 by Dr. Madeleine Leininger. Its foundation is in which of the following? A) The framework categorizes a family's progression over time B) The family life cycle of a traditional nuclear family C) Anthropology and nursing D) Holistic health beliefs

Answer: C Explanation: A) Family development theories use a framework to categorize a family's progression over time according to specific, typical stages in family life. B) Duvall's eight stages is the foundation of the family life cycle of a traditional nuclear family. C) Transcultural nursing theory is rooted in caring that embraces the beliefs and practices of individuals or groups of similar or different cultures. An example of a holistic health belief is the hot and cold theory of disease

1) After nalbuphine hydrochloride (Nubain) is administered, labor progresses rapidly, and the baby is born less than 1 hour later. The baby shows signs of respiratory depression. Which medication should the nurse be prepared to administer to the newborn? A) Fentanyl (Sublimaze) B) Butorphanol tartrate (Stadol) C) Naloxone (Narcan) D) Pentobarbital (Nembutal)

Answer: C Explanation: A) Fentanyl would not reverse the effects of the Nubain. B) Stadol would not reverse the effects of the Nubain. C) Narcan is useful for respiratory depression caused by nalbuphine (Nubain). Respiratory depression in the mother or fetus/newborn can be improved by the administration of naloxone (Narcan), which is a specific antagonist for this agent. D) Nembutal would not reverse the effects of the Nubain.

1) The nurse assesses the gestational age of a newborn and informs the parents that the newborn is premature. Which of the following assessment findings is not congruent with prematurity? A) Cry is weak and feeble B) Clitoris and labia minora are prominent C) Strong sucking reflex D) Lanugo is plentiful

Answer: C Explanation: A) Findings that indicate prematurity include a weak cry. B) Findings that indicate prematurity include a prominent clitoris and labia minora. C) Poor suck, gag, and swallow reflexes are characteristic of a preterm newborn. D) Findings that indicate prematurity include lanugo that is plentiful and widely distributed.

1) Women with pyelonephritis during pregnancy are at significantly increased risk for which condition? A) Foul-smelling discharge B) Ectopic pregnancy C) Preterm labor D) A colicky large intestine

Answer: C Explanation: A) Foul-smelling discharge is not a symptom of pyelonephritis. B) Ectopic pregnancy is not a symptom of pyelonephritis. C) Women with pyelonephritis during pregnancy are at significantly increased risk of preterm labor, preterm birth, development of adult respiratory distress syndrome, and septicemia. A colicky large intestine is an incorrect response

1) Which statement by a pregnant client to the nurse would indicate that the client understood the nurse's teaching? A) "Because of their birth relationship, fraternal twins are more similar to each other than if they had been born singly." B) "Identical twins can be the same or different sex." C) "Congenital abnormalities are more prevalent in identical twins." D) "Identical twins occur more frequently than fraternal twins."

Answer: C Explanation: A) Fraternal twins are not more similar to each other than if they had been born singly. B) Identical, or monozygotic twins, have identical chromosomal structures, and, therefore, are always the same sex. C) Monozygotic twinning is considered a random event and occurs in approximately 3 to 4 per 1000 live births. Congenital anomalies are more prevalent and both twins may have the same malformation. D) Dizygotic, or fraternal, twins occur more frequently than do monozygotic twins.

1) The postpartum client is about to go home. The nurse includes which subject in the teaching plan? A) Replacement of fluids B) Striae C) Diastasis of the recti muscles D) R E E D A scale

Answer: C Explanation: A) Frequently, the woman is quite thirsty following birth and will drink large amounts of fluid. Drinking fluids helps replace fluid lost during labor, in the urine, and through perspiration. This is not something that is taught to the patient at discharge. B) Striae (stretch marks) are a normal skin change in the pregnant woman. C) Diastasis recti abdominis and abdominal muscle tone can be improved with exercise. These exercises can be taught during the postpartum period prior to discharge. To evaluate the state of healing, the nurse inspects the Cesarean wound for redness, edema, ecchymosis, discharge, and approximation (REEDA scale). This is not something that is taught to the patient at discharge

1) The nurse is using the New Ballard Score to assess the gestational age of a newborn delivered 4 hours ago. The infant's gestational age is 33 weeks based on early ultrasound and last menstrual period. The nurse expects the infant to exhibit which of the following? A) Full sole creases, nails extending beyond the fingertips, scarf sign showing the elbow beyond the midline B) Testes located in the upper scrotum, rugae covering the scrotum, vernix covering the entire body C) Ear cartilage folded over, lanugo present over much of the body, slow recoil time D) 1 c m breast bud, peeling skin and veins not visible, rapid recoil of legs and arms to extension

Answer: C Explanation: A) Full sole creases and nails beyond the fingertips are seen in term infants; a scarf sign beyond the midline is characteristic of a preterm infant. B) Testes in the upper scrotum and rugae-covered scrotum are seen in term infants. Vernix covering the body is an indication of a preterm infant. C) Ear cartilage folded over, lanugo present over much of the body, and slow recoil time are all characteristics of a preterm infant. D) 1 c m breast bud, peeling skin, the presence of adipose tissue so that veins are not visible, and rapid recoil of the legs and arms are all indications of term or post-term infants.

1) The nurse is providing preoperative teaching to a client for whom a cesarean birth under general anesthesia is scheduled for the next day. Which statement by the client indicates that she requires additional information? A) "General anesthesia can be accomplished with inhaled gases." B) "General anesthesia usually involves administering medication into my I V." C) "General anesthesia will provide good pain relief after the birth." D) "General anesthesia takes effect faster than an epidural."

Answer: C Explanation: A) General anesthesia can be accomplished via inhalation, intravenous injection, or a combination of the two. B) General anesthesia can be accomplished via inhalation, intravenous injection, or a combination of the two. C) General anesthesia provides no pain relief after birth, as regional anesthesia does. D) General anesthesia takes effect very quickly.

1) The nurse is providing care to a client who would like to become pregnant within the next year. The client has a history of folic acid deficiency anemia and asks, "What can I do to decrease the risk of neural tube defects?" Which response by the nurse is most appropriate? A) "You will need to have genetic testing done to identify your risk." B) "Do you have a family history of these disorders?" C) "Daily consumption of orange juice decreases your risk." D) "You will be prescribed an iron supplement, which decreases your risk."

Answer: C Explanation: A) Genetic testing does not identify the risk for neural tube defects. This response is not appropriate by the nurse. B) Asking the client if there is a family history of neural tube defects will not decrease the client's risk. This response is not appropriate by the nurse. C) Daily consumption of foods rich in folic acid, such as orange juice, provides the client with the necessary amount of folic acid to decrease the risk of neural tube defects. This response is appropriate by the nurse. D) Folic acid supplements, not iron supplements, are prescribed to decrease the risk for neural tube defects. This response is not appropriate by the nurse.

1) A client is admitted to the labor and delivery unit with a history of ruptured membranes for 2 hours. This is her sixth delivery; she is 40 years old, and smells of alcohol and cigarettes. What is this client at risk for? A) Gestational diabetes B) Placenta previa C) Abruptio placentae D) Placenta accreta

Answer: C Explanation: A) Gestational diabetes is not an issue with this client. B) Placenta previa is not an issue with this client. C) Abruptio placentae is more frequent in pregnancies complicated by smoking, premature rupture of membranes, multiple gestation, advanced maternal age, cocaine use, chorioamnionitis, and hypertension. D) Placenta accreta is not an issue with this client.

1) The nurse is teaching an early pregnancy class for clients in the first trimester of pregnancy. Which statement by a client requires immediate intervention by the nurse? A) "When my nausea is bad, I will drink some ginger tea." B) "The fatigue I am experiencing will improve in the second trimester." C) "It is normal for my vaginal discharge to be green." D) "I will urinate less often during the middle of my pregnancy."

Answer: C Explanation: A) Ginger helps nausea, and is safe for use during pregnancy. B) First-trimester fatigue is common; fatigue usually improves during the second trimester. C) Increased whitish vaginal discharge, called leukorrhea, is common in pregnancy. Green discharge is not a normal finding, and indicates a vaginal infection. D) Urinary frequency, a common discomfort of pregnancy, occurs early in pregnancy and again during the third trimester because of the pressure of the enlarging uterus on the bladder.

1) Which of the following is the primary carbohydrate in the breastfeeding newborn? A) Glucose B) Fructose C) Lactose D) Maltose

Answer: C Explanation: A) Glucose is not the primary carbohydrate in the breastfeeding newborn. B) Fructose is not the primary carbohydrate in the breastfeeding newborn. C) Lactose is the primary carbohydrate in the breastfeeding newborn and is generally easily digested and well absorbed. D) Newborns have trouble digesting starches (changing more complex carbohydrates into maltose), so they should not eat them until after the first 6 months of life.

1) A nurse explains to new parents that their newborn has developed respiratory distress syndrome (R D S). Which of the following signs and symptoms would not be characteristic of R D S? A) Grunting respirations B) Nasal flaring C) Respiratory rate of 40 during sleep Chest retractions

Answer: C Explanation: A) Grunting with respirations is a characteristic of R D S. B) Nasal flaring is a characteristic of R D S. C) A respiratory rate of 40 during sleep is normal. D) Significant chest retractions are characteristic of R D S.

The client reports using an alternative therapy that involves the manipulation of soft tissues. This therapy has reduced the client's stress, diminished pain, and increased circulation. Which therapy has this client most likely received? A) Guided imagery B) Homeopathy C) Massage therapy D) Reflexology

Answer: C Explanation: A) Guided imagery involves picturing a desired outcome. B) Homeopathy uses the concept of like curing like. C) Massage therapy involves the manipulation of soft tissues. D) Reflexology is the application of pressure to designated points or reflexes on the client's feet, hands, or ears using the thumb and fingers.

1) A client from Mexico has just delivered a son, and the nurse offers to assist in putting the baby to breast. Although the client indicated before the birth that she wanted to breastfeed, she is very hesitant, and says she would like to bottle-feed for the first few days. After talking to her, the nurse understands that her primary reason for wanting to delay breastfeeding is based on what cultural belief? A) Breast milk causes skin rashes. B) It is harmful to breastfeed immediately. C) Colostrum is bad for the baby. Thin milk causes diarrhea

Answer: C Explanation: A) Haitian mothers may believe that "strong emotions" spoil breast milk and that thick breast milk causes skin rashes. B) Believing it is harmful to breastfeed immediately is not a cultural belief of Hispanics. C) Some Hispanics may delay breastfeeding because they believe colostrum is "bad." D) Haitian mothers may believe that "strong emotions" spoil breast milk and that thin milk results in diarrhea.

1) The nurse is listening to the fetal heart tones of a client at 37 weeks' gestation while the client is in a supine position. The client states, "I'm getting lightheaded and dizzy." What is the nurse's best action? A) Assist the client to sit up. B) Remind the client that she needs to lie still to hear the baby. C) Help the client turn onto her left side. Check the client's blood pressure

Answer: C Explanation: A) Having the client sit up will not offer the best and fastest relief. B) Having the client lie still will not improve the situation, and is not therapeutic. C) During pregnancy the enlarging uterus may put pressure on the vena cava when the woman is supine, resulting in supine hypotensive syndrome. This pressure interferes with returning blood flow and produces a marked decrease in blood pressure with accompanying dizziness, pallor, and clamminess, which can be corrected by having the woman lie on her left side. D) The client is hypotensive because she is at the end of pregnancy and lying supine. Checking her blood pressure will not relieve the situation.

1) The nurse is caring for an infant born at 37 weeks that weighs 1750 g (3 pounds 10 ounces). The head circumference and length are in the 25th percentile. What statement would the nurse expect to find in the chart? A) Preterm appropriate for gestational age, symmetrical I U G R B) Term small for gestational age, symmetrical I U G R C) Preterm small for gestational age, asymmetrical I U G R D) Preterm appropriate for gestational age, asymmetrical I U G R

Answer: C Explanation: A) Head circumference and length between the 10th and 90th percentiles indicate asymmetrical I U G R. B) Head circumference and length between the 10th and 90th percentiles indicate asymmetrical I U G R. C) The infant is preterm at 37 weeks. Because the weight is below the 10th percentile, the infant is small for gestational age. Head circumference and length between the 10th and 90th percentiles indicate asymmetrical I U G R. D) The infant is preterm at 37 weeks. Because the weight is below the 10th percentile, the infant is considered small for gestational age.

1) The community nurse is working with a client at 32 weeks' gestation who has been diagnosed with preeclampsia. Which statement by the client would indicate that additional information is needed? A) "I should call the doctor if I develop a headache or blurred vision." B) "Lying on my left side as much as possible is good for the baby." C) "My urine could become darker and smaller in amount each day." D) "Pain in the top of my abdomen is a sign my condition is worsening."

Answer: C Explanation: A) Headache and blurred vision or other visual disturbances are an indication of worsening preeclampsia and should be reported to the physician. B) The left lateral position maximizes uterine and renal blood flow, and therefore is the optimal position for a client with preeclampsia. C) Oliguria is a complication of preeclampsia. Specific gravity of urine readings over 1.040 correlate with oliguria and proteinuria and should be reported to the physician. D) Liver distention causes epigastric pain and may ultimately result in rupture. It is a symptom of worsening preeclampsia, and should be reported to the physician.

1) A client who is experiencing her first pregnancy has just completed the initial prenatal examination with a certified nurse-midwife. Which statement indicates that the client needs additional information? A) "Because we heard the baby's heartbeat, I am undoubtedly pregnant." B) "Because I have had a positive pregnancy test, I am undoubtedly pregnant." C) "My last period was 2 months ago, which means I'm 2 months along." D) "The increased size of my uterus means that I am finally pregnant."

Answer: C Explanation: A) Hearing the fetal heart rate is a positive, or diagnostic, change of pregnancy, so this statement would not indicate the need for further teaching. B) A positive pregnancy test is a positive, or diagnostic, indication of pregnancy. This statement would not indicate the need for further teaching. C) Amenorrhea is a subjective, or presumptive, change of pregnancy, and is not a reliable indicator of pregnancy in the early months. This statement requires additional teaching. D) Increased uterine size is an objective, or probable, change of pregnancy.

1) Which statement, if made by a pregnant client, would indicate that she understands health promotion during pregnancy? A) "I lie down after eating to relieve heartburn." B) "I try to limit my fluid intake to 3 or 4 glasses each day." C) "I elevate my legs while sitting at my desk." D) "I am avoiding exercise to stay well rested."

Answer: C Explanation: A) Heartburn is gastroesophageal reflux, and will be exacerbated by lying down. B) At least 8-10 glasses of fluids should be consumed each day to maintain the increased blood volume of pregnancy. C) Elevating the legs can help decrease lower leg edema. D) Regular mild to moderate exercise has many benefits for pregnant women.

1) What is the increased vascularization causing the softening of the cervix known as? A) Hegar sign B) Chadwick sign C) Goodell sign D) McDonald sign

Answer: C Explanation: A) Hegar sign is a softening of the isthmus of the uterus. B) Increased vascularization causes blue-purple discoloration of the cervix known as Chadwick sign. C) Increased vascularization causes the softening of the cervix known as Goodell sign. D) McDonald sign is an ease in flexing the body of the uterus against the cervix.

1) Specific cellular immunity is mediated by T lymphocytes, which enhance the efficiency of the phagocytic response. What do cytotoxic activated T cells do? A) Enable T or B cells to respond to antigens B) Repress responses to specific B or T lymphocytes to antigens C) Kill foreign or virus-infected cells D) Remove pathogens and cell debris

Answer: C Explanation: A) Helper activated T cells enable T or B cells to respond to antigens. B) Suppressor activated T cells repress responses to specific B or T lymphocytes to antigens. C) Cytotoxic activated T cells kill foreign or virus-infected cells. D) Phagocytosis is a major mechanism to remove pathogens and cell debris.

1) The nurse has instructed a new mother on quieting activities for her newborn. The nurse knows that the mother understands when she overhears the mother telling the father to do what? A) Hold the newborn in an upright position. B) Massage the hands and feet. C) Swaddle the newborn in a blanket. Make eye contact while talking to the newborn

Answer: C Explanation: A) Holding the newborn upright is a waking activity. B) Increasing skin contact and gently rubbing hands and feet is a waking activity. C) Swaddling or bundling the baby increases the sense of security and is a quieting activity. D) Talking to the newborn while making eye contact is a waking activity.

1) The nurse is caring for a client diagnosed with endometriosis. Which statement by the client would require a need for perhaps another treatment option? A) "I am having many hot flashes since I had the Lupron injection." B) "The pain I experience with intercourse is becoming more severe." C) "I have vaginal dryness, reduced libido, and my clitoris has become larger since taking danazol. Is this normal?" D) "I've noticed I have not had my period on a regular basis since being on the G n R H analogs."

Answer: C Explanation: A) Hot flashes are expected, and not a complication. B) Dyspareunia is a common symptom of endometriosis, and therefore is not a complication. C) Danazol is a testosterone derivative that suppresses ovulation and causes amenorrhea. It is intended for short-term therapy. Because of adverse effects, many clinicians have moved away from danazol to other treatment options. GnRH analogs suppress the menstrual cycle through estrogen antagonism

1) Which of the following may lead to the development of disseminated intravascular coagulation (D I C), also called consumption coagulopathy, in the mother? A) Hypertensive disorders B) Abruptio placentae C) Prolonged retention of the dead fetus Heritable thrombophilias

Answer: C Explanation: A) Hypertensive disorders may contribute to the incidence of stillbirth. B) Abruptio placentae contributes to the incidence of stillbirth. C) Prolonged retention of the dead fetus may lead to the development of disseminated intravascular coagulation (D I C), also called consumption coagulopathy, in the mother. Heritable thrombophilias may contribute to the incidence of stillbirth

1) If the woman is R h negative and not sensitized, she is given R h immune globulin to prevent what? A) The potential for hemorrhage B) Hyperhomocysteinemia C) Antibody formation D) Tubal pregnancy

Answer: C Explanation: A) If the woman is R h negative and not sensitized, she is not given R h immune globulin to prevent the potential for hemorrhage. B) Hyperhomocysteinemia (HHcy) is a risk factor for vascular disease. C) If the woman is R h negative and not sensitized, she is given R h immune globulin to prevent antibody formation. D) If the woman is R h negative and not sensitized, she is not given R h immune globulin to prevent a tubal pregnancy.

1) A woman with polycystic ovarian syndrome (P C O S) is prescribed clomiphene citrate for the treatment of infertility. Which statement does the nurse understand is true? A) The woman has abnormal ovaries B) The woman has low prolactin levels C) The woman's pituitary gland is intact D) The woman's thyroid gland is normal

Answer: C Explanation: A) In order to qualify for treatment with clomiphene citrate, the woman must have normal ovaries. B) In order to qualify for treatment with clomiphene citrate, the woman must have normal prolactin levels. C) In order to qualify for treatment with clomiphene citrate, the woman must have an intact pituitary gland. A normal-functioning thyroid gland is not essential for the client who takes clomiphene citrate; therefore, this statement may not be true

1. A client is admitted to the labor and delivery unit with contractions that are 2 minutes apart, lasting 60 seconds. She reports that she had bloody show earlier that morning. A vaginal exam reveals that her cervix is 100 percent effaced and 8 c m dilated. The nurse knows that the client is in which phase of labor? A) Active B) Latent C) Transition D) Fourth

Answer: C Explanation: A) In the active phase, the woman dilates from 4 to 7 centimeters. B) The latent phase is the beginning of labor contractions and the cervix may be dilated 0 to 3 centimeters. C) The transition phase begins with 8 c m to 10 c m of dilation, and contractions become more frequent, are longer in duration, and increase in intensity. D) There is no fourth phase. The fourth stage occurs after delivery of the placenta.

1) The postpartum nurse is caring for a client who gave birth to full-term twins earlier today. The nurse will know to assess for symptoms of which of the following? A) Increased blood pressure B) Hypoglycemia C) Postpartum hemorrhage D) Postpartum infection

Answer: C Explanation: A) Increased blood pressure would cause vasoconstriction, and is not identified in this client. B) Hypoglycemia would not be a usual assessment for the mother unless she was diabetic, and this is not identified in this client. C) The nurse will assess for postpartum hemorrhage. Overstretching of uterine muscles with conditions such as multiple gestation, polyhydramnios, or a very large baby may set the stage for slower uterine involution. D) Postpartum infection would be assessed through lochia odor. The odor of the lochia is nonoffensive and never foul. If foul odor is present, so is an infection. Infection would not be expected this soon after delivery.

1) During her first months of pregnancy, a client tells the nurse, "It seems like I have to go to the bathroom every 5 minutes." The nurse explains to the client that this is because of which of the following? A) The client probably has a urinary tract infection. B) Bladder capacity increases throughout pregnancy. C) The growing uterus puts pressure on the bladder. D) Some women are very sensitive to body function changes.

Answer: C Explanation: A) Increased frequency of urination in the first trimester of pregnancy does not indicate a urinary tract infection. B) Bladder capacity does not increase throughout pregnancy. C) During the first trimester, the growing uterus puts pressure on the bladder, producing urinary frequency until the second trimester, when the uterus becomes an abdominal organ. Near term, when the presenting part engages in the pelvis, pressure is again exerted on the bladder. D) Sensitivity is not the cause of an increased frequency of urination in the first trimester.

1. The labor and delivery nurse is preparing a prenatal class about facilitating the progress of labor. Which of the following frequent responses to pain should the nurse indicate is most likely to impede progress in labor? A) Increased pulse B) Elevated blood pressure C) Muscle tension D) Increased respirations

Answer: C Explanation: A) Increased pulse is a manifestation of pain, but does not impede labor. B) Elevated blood pressure is a manifestation of pain, but does not impede labor. C) It is important for the woman to relax each part of her body. Be alert for signs of muscle tension and tightening. Dissociative relaxation, controlled muscle relaxation, and specified breathing patterns are used to promote birth as a normal process. D) Increased respiration is a manifestation of pain, but does not impede labor.

1) During discharge planning for a drug-dependent newborn, the nurse explains to the mother how to do which of the following? A) Place the newborn in a prone position. B) Limit feedings to three a day to decrease diarrhea. C) Place the infant supine and operate a home apnea-monitoring system. D) Wean the newborn off the pacifier.

Answer: C Explanation: A) Infants with neonatal abstinence syndrome are at a significantly higher risk for sudden infant death syndrome (S I D S) when the mother used heroin, cocaine, or opiates. The infant should sleep in a supine position, and home apnea monitoring should be implemented. B) Small, frequent feedings are recommended. C) Infants with neonatal abstinence syndrome are at a significantly higher risk for sudden infant death syndrome (S I D S) when the mother used heroin, cocaine, or opiates. The infant should sleep in a supine position, and home apnea monitoring should be implemented. D) A pacifier may be offered to provide nonnutritive sucking.

6) An expectant father has been at the bedside of his laboring partner for more than 12 hours. An appropriate nursing intervention would be which of the following? A) Insist that he leave the room for at least the next hour. B) Tell him he is not being as effective as he was, and that he needs to let someone else take over. C) Offer to remain with his partner while he takes a break. D) Suggest that the client's mother might be of more help.

Answer: C Explanation: A) Insisting that the father leave does not reassure him about the care the woman will receive in his absence. B) Telling him that he is ineffective does not reassure him about the care the woman will receive in his absence. C) Support persons frequently are reluctant to leave the laboring woman to take care of their own needs. The laboring woman often fears being alone during labor. Even though there is a support person available, the woman's anxiety may be decreased when the nurse remains with her while he takes a break. D) Suggesting that the client's mother take his place does not reassure him about the care the woman will receive in his absence.

1) The nurse is teaching a client who has been diagnosed with vulvitis. Which statement by the client indicates that the nurse's instruction has not been effective? A) "I should stop having sexual intercourse." B) "Non-deodorized tampons could make this condition recur." C) "Wearing pantyhose daily will improve the problem." D) "A different brand of soap might eliminate the irritation."

Answer: C Explanation: A) Intercourse can occur, but with adequate lubrication. B) Use of deodorized and heavily scented products that come in contact with the vulva (toilet paper, soap, bubble bath, pads, tampons, etc.) can cause the inflammation. C) Vulvitis is inflammation of the vulva. Tight clothing, especially if made of synthetic fibers, can predispose women to the condition. Pantyhose should not be worn. D) Use of deodorized and heavily scented products that come in contact with the vulva (toilet paper, soap, bubble bath, pads, tampons, etc.) can cause the inflammation.

A female client is disappointed to learn that intrauterine contraception is not an option. For what reason is this form of contraception contraindicated for this client? A) Diabetes B) Breast cancer C) Endometriosis D) Uterine surgery

Answer: C Explanation: A) Intrauterine contraception is contraindicated in endometriosis. Intrauterine contraception is an excellent contraceptive option for women with diabetes; it may also be used in women with a history of breast cancer or uterine surgery. B) Intrauterine contraception is contraindicated in endometriosis. Intrauterine contraception is an excellent contraceptive option for women with diabetes; it may also be used in women with a history of breast cancer or uterine surgery. C) Intrauterine contraception is contraindicated in endometriosis. Intrauterine contraception is an excellent contraceptive option for women with diabetes; it may also be used in women with a history of breast cancer or uterine surgery. Intrauterine contraception is contraindicated in endometriosis. Intrauterine contraception is an excellent contraceptive option for women with diabetes; it may also be used in women with a history of breast cancer or uterine surgery

1) During a health interview focused on sexual history, a female patient makes a statement about douching and intercourse. What should the nurse do in response to this statement? A) Recommend the frequency of douching B) Explain the proper procedure to douche C) Take the time now to educate the patient about the practice D) Document that the patient has misunderstandings about the use of douches

Answer: C Explanation: A) It is essential that the nurse listen and use teachable moments to educate women about their bodies. Since the patient mentioned douching and intercourse, the time to review information about that practice with the patient is now. The nurse needs to do more than recommend the frequency of douching or explain the proper douching procedure. The nurse also needs to do more than document that the patient has misunderstandings about the use of douches. B) It is essential that the nurse listen and use teachable moments to educate women about their bodies. Since the patient mentioned douching and intercourse, the time to review information about that practice with the patient is now. The nurse needs to do more than recommend the frequency of douching or explain the proper douching procedure. The nurse also needs to do more than document that the patient has misunderstandings about the use of douches. C) It is essential that the nurse listen and use teachable moments to educate women about their bodies. Since the patient mentioned douching and intercourse, the time to review information about that practice with the patient is now. The nurse needs to do more than recommend the frequency of douching or explain the proper douching procedure. The nurse also needs to do more than document that the patient has misunderstandings about the use of douches. It is essential that the nurse listen and use teachable moments to educate women about their bodies. Since the patient mentioned douching and intercourse, the time to review information about that practice with the patient is now. The nurse needs to do more than recommend the frequency of douching or explain the proper douching procedure. The nurse also needs to do more than document that the patient has misunderstandings about the use of douches

1) Several adolescent female students are waiting to be seen by the school sexual health clinic nurse. Which student should the nurse see first? A) 14-year-old whose 17-year-old sister is pregnant B) 15-year-old who reports using condoms regularly C) 16-year-old who had chlamydia treated 2 weeks ago D) 17-year-old with a history of child abuse

Answer: C Explanation: A) It is not known whether this client is sexually active. This student is not the top priority. B) Condom use will decrease the risk of becoming pregnant. This client is a low priority. C) This client is the top priority. Teenagers 15 to 19 years old have a high incidence of S T Is. The incidence of chlamydial infection is increased in this age group. The presence of such infections during a pregnancy greatly increases the risk to the fetus. D) Although adolescents with a history of abuse are more likely to become pregnant than are their peers who have not experienced abuse, too little information is given about this client to determine risk for pregnancy. This client is not the top priority.

1) As the couple and their families begin to confront the pain of their loss, many normal manifestations of grief may be present. Which of the following would indicate an emotional response to the loss? A) Lack of meaning or direction B) Preoccupation C) Flat affect D) Dreams of the deceased

Answer: C Explanation: A) Lack of meaning or direction would be a spiritual response to loss. B) Preoccupation would be a cognitive response to loss. C) Flat affect would be an emotional response to loss. Dreams of the deceased would be a behavioral response to loss

1) The nurse is aware that a fetus that is not in any stress would respond to a fetal scalp stimulation test by showing which change on the monitor strip? A) Late decelerations B) Early decelerations C) Accelerations D) Fetal dysrhythmia

Answer: C Explanation: A) Late decelerations indicate uteroplacental insufficiency. B) Early decelerations are indicative of head compression. C) A fetus that is not experiencing stress responds to scalp stimulation with an acceleration of the F H R. D) Fetal dysrhythmia is associated with complete heart block in the fetus.

The nurse is speaking to students about changes in maternal-newborn care. One change is that self-care has gained wide acceptance with clients and the healthcare community due to research findings that suggest that it has which effect? A) Shortens newborn length of stay B) Decreases use of home health agencies C) Decreases healthcare costs D) Decreases the number of emergency department visits

Answer: C Explanation: A) Length of stay is often determined by third-party payer (insurance company) policies as well as the physiologic stability of the mother and newborn. Home healthcare agencies often are involved in client care to decrease hospital stay time. B) Home healthcare agencies often are involved in client care to decrease hospital stay time. C) Research indicates that self-care significantly decreases healthcare costs. D) Acute emergencies are addressed by emergency departments, and are not delayed by those practicing self-care.

1) Which of the following systems provides a uniform format and classification of terminology based on current understanding of cervical disease? A) Levonorgestrel intrauterine B) P A L M-C O E I N C) Bethesda D) B S E

Answer: C Explanation: A) Levonorgestrel intrauterine system is for contraception and control of excessive menstrual bleeding by suppression of endometrial growth. B) P A L M-C O E I N is a classification system developed for the causes of A U B. C) The Bethesda System for classifying Pap smears is a standardized method of reporting cytologic Pap smear findings and is the most widely used method in the United States. D) B S E is the acronym for breast self-examination and is not considered a system.

1) A client who wants to use the vaginal sponge method of contraception shows that she understands the appropriate usage when she makes which statement? A) "I need to use a lubricant prior to insertion." B) "I need to add spermicidal cream prior to intercourse." C) "I need to moisten it with water prior to use." D) "I need to leave it in no longer than 6 hours."

Answer: C Explanation: A) Lubricant and spermicidal cream are not needed with the vaginal sponge. B) Lubricant and spermicidal cream are not needed with the vaginal sponge. C) To activate the spermicide in the vaginal sponge, it must be moistened thoroughly with water. D) The sponge can remain in place for 24 hours.

1) Major perineal trauma (extension to or through the anal sphincter) is more likely to occur if what type of episiotomy is performed? A) Mediolateral B) Episiorrhaphy C) Midline D) Medical

Answer: C Explanation: A) Major perineal trauma is more likely to occur if a midline episiotomy is performed. B) Episiorrhaphy is the repair of the episiotomy. C) Major perineal trauma is more likely to occur if a midline episiotomy is performed. The major disadvantage is that a tear of the midline incision may extend through the anal sphincter and rectum. D) Medical is not a type of episiotomy.

1) The nurse has returned from working as a maternal-child nurse volunteer for a non-governmental organization. After completing a community presentation about this experience, the nurse knows that learning has occurred when a participant states which of the following? A) "Malaria is a chronic disease, and rarely causes fetal loss." B) "Escherichia coli bacteria can cause diarrhea, but not stillbirth." C) "Group B streptococci can cause infection and the death of the fetus." D) "Viral infections don't cause fetal death in developing nations."

Answer: C Explanation: A) Malaria is associated with a high stillbirth rate when contracted for the first time by the mother during the pregnancy, and it carries a much higher morbidity and mortality rate in developing countries. B) E. coli can cause ascending infections prior to or after rupture of membranes. C) Group B streptococci can cause ascending infections prior to or after rupture of membranes. D) Viral infections play a significant role in fetal death in developing countries.

A client who was raped is extremely upset when a pregnancy test confirms that she is pregnant, and requests information regarding pregnancy termination. Which statement is best for the nurse to make? A) "Abortion is morally wrong, and should not be undertaken." B) "Hypertension is a risk with any abortion." C) "Surgical abortion in the first trimester is technically easier and safer than abortion in the second trimester." D) "The most accurate method to determine gestational age are the results of a pregnancy test."

Answer: C Explanation: A) Many nurses are strongly opposed to abortion for religious, ethical, cultural, or personal reasons. In order to be effective in a therapeutic relationship, the nurse must avoid being judgmental. B) Endometritis is a risk with any abortion. C) Second-trimester abortion (greater than 13 weeks' gestation up to 24 weeks or per state law) may be done medically or surgically. D) The most accurate method to determine gestational age is by sonographic determination.

1) A laboring client's obstetrician has suggested amniotomy as a method for inducing labor. Which assessment(s) must be made just before the amniotomy is performed? A) Maternal temperature, B P, and pulse B) Estimation of fetal birth weight C) Fetal presentation, position, station, and heart rate D) Biparietal diameter

Answer: C Explanation: A) Maternal vital signs do not affect the decision to perform an amniotomy. B) Fetal birth weight does not affect the decision to perform an amniotomy. C) Before an amniotomy is performed, the fetus is assessed for presentation, position, station, and F H R. D) Biparietal diameter does not affect the decision to perform an amniotomy.

1) The nurse is teaching a class on infant care to new parents. Which statement by a parent indicates that additional teaching is needed? A) "The white spots on my baby's nose are called milia, and are harmless." B) "The whitish cheeselike substance in the creases is vernix, and will be absorbed." C) "The red spots with a white center on my baby are abnormal acne." D) "Jaundice is a yellowish discoloration of skin that if noticed on the 1st day of life should be reported to the physician."

Answer: C Explanation: A) Milia are exposed sebaceous glands, and appear as white spots, often across the nose. B) Disappearance of the protective vernix caseosa promotes skin desquamation. C) Red spots with white or yellow centers are erythema toxicum. The peak incidence is at 24 to 48 hours of life. The condition rarely presents at birth or after 5 days of life. The cause is unknown, and no treatment is necessary. D) Jaundice is a yellowish discoloration of skin and mucous membranes. Any jaundice noted before 24 hours of age should be reported to the physician or nurse practitioner.

1) The community nurse is working with a client whose only child is 8 months old. Which statement does the nurse expect the mother to make? A) "I have a lot more time to myself than I thought I would have." B) "My confidence level in my parenting is higher than I anticipated." C) "I am constantly tired. I feel like I could sleep for a week." D) "My baby likes everyone, and never fusses when she's held by a stranger."

Answer: C Explanation: A) Most new mothers have difficulty finding time for themselves. B) Feelings of incompetence at parenting are also common. C) Physical fatigue often affects adjustments and functions of the new mother. The nurse can also provide information about the fatigue that a new mother experiences, strategies to promote rest and sleep at home, and the impact fatigue can have on a woman's emotions and sense of control. D) At 8 months, some infants develop stranger anxiety, and they will cry when held by anyone other than the parents.

1) The nurse is talking with the parents of a pregnant 15-year-old. Which statement by one of the parents is expected? A) "We're very happy for her. It will be easier to focus on education with a new baby." B) "I'm not going to get involved. She understands how her health insurance works." C) "Her father told her to stop dating that boy. Now look at the trouble she's gotten into." D) "An abortion is the best choice for her. She can deal with our Catholic priest later."

Answer: C Explanation: A) Most parents accept the pregnancy but are not excited when their 15-year-old daughter is pregnant. Education is harder when childcare is involved. Teens who give birth are less likely to complete their education. B) The parent of a pregnant teen is usually the support person, and helps the teen understand how to access prenatal care. It is unlikely that a 15-year-old would understand health insurance. C) This statement indicates anger, which is to be expected when a parent finds out about a teen daughter's pregnancy. This is not necessarily expected from a parent. It indicates that the parent might be forcing the teen into abortion despite their being a part of a religious tradition that disapproves of abortion

1) The nurse is caring for a client at 30 weeks' gestation who is experiencing preterm premature rupture of membranes (P P R O M). Which statement indicates that the client needs additional teaching? A) "If I were having a singleton pregnancy instead of twins, my membranes would probably not have ruptured." B) "If I develop a urinary tract infection in my next pregnancy, I might rupture membranes early again." C) "If I want to become pregnant again, I will have to plan on being on bed rest for the whole pregnancy." D) "If I have amniocentesis, I might rupture the membranes again."

Answer: C Explanation: A) Multifetal gestation increases the risk for P P R O M. B) A urinary tract infection (U T I) increases the risk for P P R O M. C) There is no evidence that bed rest in a subsequent pregnancy decreases the risk for P P R O M. D) Amniocentesis increases the risk for P P R O M.

1) The nurse in a prenatal clinic finds that four clients have called with complaints related to their pregnancies. Which call should the nurse return first? A) Pregnant woman at 7 weeks' gestation reporting nasal stuffiness B) Pregnant woman at 38 weeks' gestation experiencing rectal itching and hemorrhoids C) Pregnant woman at 15 weeks' gestation with nausea and vomiting and a 15-pound weight loss D) Pregnant woman at 32 weeks' gestation treating constipation with prune juice

Answer: C Explanation: A) Nasal stuffiness is common in the first trimester as a result of increased estrogen. B) Hemorrhoids are common during pregnancy and often cause itching. C) The nurse should return this call first because this patient is the highest priority. A 15-pound weight loss is not an expected finding. Although some nausea is common, the woman who suffers from extreme nausea coupled with vomiting requires further assessment. D) Constipation during the third trimester is a common finding. Increased fluid and fiber from food sources are most effective in relieving constipation.

1) During an assessment of a 12-hour-old newborn, the nurse notices pale pink spots on the nape of the neck. The nurse documents this finding as which of the following? A) Nevus vasculosus B) Nevus flammeus C) Telangiectatic nevi D) A Mongolian spot

Answer: C Explanation: A) Nevus vasculosus is a strawberry hemangioma. B) Nevus flammeus is a port-wine stain hemangioma. C) Telangiectatic nevi (stork bites) appear as pale pink or red spots and are frequently found on the eyelids, nose, lower occipital bone, and nape of the neck. D) Congenital dermal melanocytosis (Mongolian blue spots) are macular areas of bluish black or gray-blue pigmentation commonly found on the dorsal area and the buttocks but may be anywhere on the body.

1) During a nonstress test, the nurse notes that the fetal heart rate decelerates about 15 beats during a period of fetal movement. The decelerations occur twice during the test, and last 20 seconds each. The nurse realizes these results will be interpreted as which of the following? A) A negative test B) A reactive test C) A nonreactive test D) An equivocal test

Answer: C Explanation: A) Nonstress tests are scored as either reactive or nonreactive. B) A reactive N S T has two or more fetal heart accelerations within a 20-minute period. C) The F H R acceleration must be at least 15 beats per minute above baseline for at least 15 seconds from baseline to baseline. A nonreactive N S T is one that lacks sufficient F H R accelerations over a 40-minute period. D) Nonstress tests are scored as either reactive or nonreactive.

1) The nurse is analyzing several fetal heart rate patterns. The pattern that would be of most concern to the nurse would be which of the following? A) Moderate variability B) Early decelerations C) Late decelerations D) Accelerations

Answer: C Explanation: A) Normal F H R variability is in the moderate range. B) Early deceleration is usually considered benign. C) Late decelerations are caused by uteroplacental insufficiency. The late deceleration pattern is considered a nonreassuring sign. D) Accelerations are thought to be a sign of fetal well-being.

1) The nurse is preparing a client in her second trimester for a three-dimensional ultrasound examination. Which statement indicates that teaching has been effective? A) "If the ultrasound is normal, it means my baby has no abnormalities." B) "The nuchal translucency measurement will diagnose Down syndrome." C) "I might be able to see who the baby looks like with the ultrasound." D) "Measuring the length of my cervix will determine whether I will deliver early."

Answer: C Explanation: A) Not all fetal anomalies are detectable by ultrasound. B) Nuchal translucency measurements are for detection, not diagnosis, of trisomies 13, 18, and 21. C) Three-dimensional ultrasound uses algorithms to vary opacity, transparency, and depth to project an image. This allows curved structures such as the fetal face to be viewed. D) Transvaginal ultrasound can most accurately identify shortened cervical length indicating cervical insufficiency or risk of preterm labor. However, a cervix of normal length does not preclude preterm birth.

1) A 14-year-old girl is brought to the clinic by her mother. The nurse determines that the teen is about 28 weeks pregnant. The mother states, "We knew she was gaining weight, but we can't tell anyone she is pregnant." The nurse understands that the client's mother's behavior exemplifies which of the following? A) Low self-esteem B) Anger C) Shame D) Ignorance

Answer: C Explanation: A) Nothing in the scenario indicates that the client's mother has low self-esteem. B) Anger will often manifest as loud or negative speech. Nothing in the scenario indicates that this is true. C) In families who foster educational and career goals for their children, adolescent pregnancy is often a shock. Anger, shame, and sorrow are common reactions. D) The parents might be ignorant of their child's sexual activity, but when faced with weight gain, they acknowledged the pregnancy.

1) The community health nurse manager is reviewing the charts of female elderly clients. Which issue are these clients most likely to experience? A) Adequate financial resources to purchase medications B) Senior services that provide transportation to healthcare appointments C) Multiple medications prescribed by different physicians D) Medicare that covers healthcare costs so no out-of-pocket expenses occur

Answer: C Explanation: A) Older women, particularly those who are poor, face multiple barriers in obtaining adequate healthcare services, including excessive medical costs not covered by Medicare. B) Not all elderly have adequate access to transportation for healthcare. C) Polypharmacy, which means multiple medicines, is a common problem in the elderly population. Elderly women (as well as elderly men) often have multiple healthcare providers with different specialties who provide pharmacologic intervention. Lack of private health insurance coverage and excessive medical costs not covered by Medicare can be barrier in obtaining adequate healthcare services

1) During the initial prenatal visit, the pregnant teenager states that she does not know how she got pregnant. The nurse can help to educate her regarding anatomy by doing what? A) Allowing her to witness a pelvic exam on another teenager B) Encouraging her to ask her mother about the physiology of pregnancy C) Including anatomic models and drawings in the teaching session D) Discussing the process of fetal development with the client

Answer: C Explanation: A) One client is not allowed to be present during another client's visit or exam. B) The client's mother might not have accurate information, or might be uncomfortable discussing these issues with her teen daughter. C) To teach the client about anatomy and physiology, use simple explanations and lots of audiovisuals. D) While fetal development is important to understand, discussing it now does not address the client's stated concern.

1) A client is consulting a certified nurse-midwife because she is hoping for a vaginal birth after cesarean (V B A C) with this pregnancy. Which statement indicates that the client requires more information about V B A C? A) "I can try a vaginal birth because my uterine incision is a low segment transverse incision." B) "The vertical scar on my skin doesn't mean that the scar on my uterus goes in the same direction." C) "There is about a 90% chance of giving birth vaginally after a cesarean." D) "Because my hospital has a surgery staff on call 24 hours a day, I can try a V B A C there."

Answer: C Explanation: A) Only low segment transverse uterine incisions are recommended for attempting a V B A C. B) Abdominal skin incisions and uterine incisions are not always the same. C) Women whose previous cesarean was performed because of nonrecurring indications have been reported to have approximately a 60% to 80% chance of success with V B A C. VBAC should be attempted only in facilities that have in-house anesthesia personnel available for emergency cesarean births if warranted

1) The nurse attempts to elicit the Moro reflex on a newborn, and assesses movement of the right arm only. Based on this finding, the nurse immediately assesses for which of the following? A) Ortolani maneuver B) Palmar grasping reflex C) Clavicle D) Tonic neck reflex

Answer: C Explanation: A) Ortolani maneuver is an assessment technique that rules out the possibility of hip dislocation or hip instability. B) Palmar grasping reflex is elicited by stimulating the newborn's palm with a finger or object; the newborn grasps and holds the object or finger firmly enough to be lifted momentarily from the crib. C) When the Moro reflex is elicited, the newborn straightens arms and hands outward while the knees flex. Slowly the arms return to the chest, as in an embrace. If this response is not elicited, the nurse assesses the clavicle for a possible fracture. D) Tonic neck reflex (fencer position) is elicited when the newborn is supine and the head is turned to one side. In response, the extremities on the same side straighten, whereas on the opposite side they flex.

1) The nurse is creating a handout on reproduction for teen clients. Which piece of information should the nurse include in this handout? A) The fertilized ovum is called a gamete. B) Prior to fertilization, the sperm are zygotes. C) Ova survive 12-24 hours in the fallopian tube if not fertilized. Sperm survive in the female reproductive tract up to a week

Answer: C Explanation: A) Ova and sperm are gametes; a fertilized ovum is a zygote. B) Sperm are gametes (as are ova); a zygote is a fertilized ovum. C) Ova are considered fertile for about 12 to 24 hours after ovulation. Sperm can survive in the female reproductive tract for 48 to 72 hours but are believed to be healthy and highly fertile for only about the first 24 hours

1) A mother states that her breasts leak between feedings. Which of the following can contribute to the letdown reflex in breastfeeding mothers? A) Pain with breastfeeding B) Number of hours passed since last feeding C) The newborn's cry D) Maternal fluid intake

Answer: C Explanation: A) Pain with breastfeeding is associated with improper positioning, and does not stimulate the letdown reflex. B) Allowing too many hours between feedings can affect the milk supply. It does not stimulate the letdown reflex. C) Some women will leak milk when their breasts are full and it is nearly time to breastfeed again or whenever they experience letdown, which can be triggered by hearing, seeing, or even thinking of their baby. D) Maternal fluid intake can affect milk supply. It does not stimulate the letdown reflex.

1) The client with an abnormal quadruple screen is scheduled for an ultrasound. Which statement indicates that the client understands the need for this additional antepartal fetal surveillance? A) "After the ultrasound, my partner and I will decide how to decorate the nursery." B) "During the ultrasound we will see which of us the baby looks like most." C) "The ultrasound will show whether there are abnormalities with the baby's spine." D) "The blood test wasn't run correctly, and now we need to have the sonogram."

Answer: C Explanation: A) Parents often try to identify the baby's sex and which parent the fetus resembles more during an ultrasound. B) Parents often try to identify the baby's sex and which parent the fetus resembles more during an ultrasound. C) Ultrasound is used to detect neural tube defects. An abnormal serum quadruple screen is not the result of a lab error, and can indicate either an open neural tube defect or trisomy 18 or 21. D) Ultrasound is used to detect neural tube defects.

15) A young adolescent is transferred to the labor and delivery unit from the emergency department. The client is in active labor, but did not know she was pregnant. What is the most important nursing action? A) Determine who might be the father of the baby for paternity testing. B) Ask the client what kind of birthing experience she would like to have. C) Assess blood pressure and check for proteinuria. D) Obtain a Social Services referral to discuss adoption.

Answer: C Explanation: A) Paternity testing is a lower priority than the physiologic well-being of the client and fetus. B) A client with a previously undiagnosed pregnancy is unlikely to have given any thought to childbearing preferences. C) Preeclampsia is more common among adolescents than in young adults, and is potentially life-threatening to both mother and fetus. This assessment is the highest priority. D) It would be inappropriate to discuss adoption or parenting during labor, especially with an adolescent who did not know she was pregnant prior to the onset of labor. The nurse should wait until after the birth to have this discussion.

1. The client delivered her second child 1 day ago. The client's temperature is 101.4° F, her pulse is 100, and her blood pressure is 110/70. Her lochia is moderate, serosanguinous, and malodorous. She is started on I V antibiotics. The nurse provides education for the client and her partner. Which statement indicates that teaching has been effective? A) "This condition is called parametritis." B) "Gonorrhea is the most common organism that causes this type of infection." C) "My positive Beta-strep culture might have contributed to this problem." D) "If I had walked more yesterday, this probably wouldn't have happened."

Answer: C Explanation: A) Pelvic cellulitis (parametritis) is an infection that has ascended to involve the connective tissue of the broad ligament or, in more severe forms, the connective tissue of all the pelvic structures. B) Gonorrhea is not a common cause of endometritis. C) Clinical findings of endometritis in the initial 24 to 36 hours postpartum tend to be related to group B streptococcus (G B S). D) Walking would prevent deep vein thrombophlebitis, not endometritis.

1) The nurse is planning care for a newborn. Which nursing intervention would best protect the newborn from the most common form of heat loss? A) Placing the newborn away from air currents B) Pre-warming the examination table C) Drying the newborn thoroughly D) Removing wet linens from the isolette

Answer: C Explanation: A) Placing the newborn away from air currents reduces heat loss by convection, which is not the most common form of heat loss. B) Pre-warming the examination table reduces heat loss by conduction, which is not the most common form of heat loss. C) The most common form of heat loss is evaporation. The newborn is particularly prone to heat loss by evaporation immediately after birth (when the baby is wet with amniotic fluid) and during baths; thus drying the newborn is critical. Removing wet linens from the isolette that are not in direct contact with the newborn reduces heat loss by radiation, which is not the most common form of heat loss

1) A nurse is caring for several postpartum clients. Which client is demonstrating a problem attaching to her newborn? A) The client who is discussing how the baby looks like her father B) The client who is singing softly to her baby C) The client who continues to touch her baby with only her fingertips D) The client who picks her baby up when the baby cries

Answer: C Explanation: A) Pointing out family traits or characteristics in the newborn is a positive sign of developing early attachment. B) Speaking or singing to the baby frequently and affectionately is a positive sign of developing early attachment. C) In a progression of touching activities, the mother proceeds from fingertip exploration of the newborn's extremities toward palmar contact with larger body areas and finally to enfolding the infant with the whole hand and arms. If the client continues to touch with only her fingertips, she might not be developing adequate early attachment. D) Being sensitive to the newborn's needs is a positive sign of developing early attachment.

1. The postpartum client states that she doesn't understand why she can't enjoy being with her baby. What would the nurse be concerned about? A) Postpartum psychosis B) Postpartum infection C) Postpartum depression D) Postpartum blues

Answer: C Explanation: A) Postpartum psychosis is more severe, and includes hallucinations and irrationality, which are not represented in this situation. B) Postpartum infection is not related to this situation. C) Postpartum depression can impair maternal-infant bonding and can cause developmental and cognitive delays in the child. D) Postpartum blues episodes occur frequently in the week after birth, are associated with hormonal fluctuations, and are typically transient.

1) The nurse is providing care to a pregnant client who is diagnosed with iron deficiency anemia. Which teaching statement from the nurse to the client is appropriate? A) "You will need to begin iron supplements at 30 m g/day." B) "You will need to increase your intake of bananas." C) "You will need to increase your intake of green leafy vegetables." D) "You will need to begin iron supplements at 150 m g/day."

Answer: C Explanation: A) Pregnant clients are prescribed 30 m g of iron daily to prevent anemia. Once anemia is diagnosed, the prescribed dosage is increased. This teaching statement is not appropriate. B) Pregnant clients diagnosed with anemia should increase their consumption of foods that are rich in iron. Bananas are not rich in iron. This teaching statement is not appropriate. C) Pregnant clients diagnosed with anemia should increase their consumption of foods that are rich in iron. Green, leafy vegetables are rich in iron. This teaching statement is appropriate. D) Pregnant clients diagnosed with iron deficiency anemia will require an increased dose of supplemental iron, typically 60 m g to 120 m g/day.

1) When assisting with a transabdominal ultrasound sampling, which of the following would the nurse do? A) Obtain preliminary urinary samples. B) Have the woman empty her bladder before the test begins. C) Assist the woman into a supine position on the examining table. D) Instruct the woman to eat a fat-free meal 2 hours before the scheduled test time.

Answer: C Explanation: A) Preliminary blood work may be obtained, not urinary samples. B) The morning of the procedure, the woman is asked to drink fluids to fill her bladder because displacement of an anteverted uterus by a full bladder may aid in positioning the uterus for catheter insertion. C) Clients are placed in a supine position on the table. D) Dietary intake is not relevant to this procedure.

1) The client has delivered her first child at 37 weeks. The nurse would describe this to the client as what type of delivery? A) Preterm B) Postterm C) Early term D) Near term

Answer: C Explanation: A) Preterm births are those that occur between 20 weeks and 37 completed weeks. B) Postterm births are those that occur at 42 weeks and beyond. C) Early term births extend from 37 to 38 weeks' gestation. D) Near term is not terminology used to describe birth.

1) The nurse should explain to new parents that their infant's position should be changed periodically during the early months of life to prevent which of the following? A) Muscle contractures B) Respiratory distress C) Permanently flattened areas of the skull D) Esophageal reflux

Answer: C Explanation: A) Preventing muscle contractures is not the rationale for periodically changing the infant's position. B) Respiratory distress would indicate complications, and would not be affected by periodic position changes. C) The nurse can describe plagiocephaly as a flattened area on the head and can recommend that parents alternate their infant's head position between the right and the left side when placing the infant supine for sleep. Placing the infant's head at alternate ends of the crib every few days is helpful as well. D) Esophageal reflux would indicate complications, and would not be affected by periodic position changes.

1) The client is recovering from a delivery that included a midline episiotomy. Her perineum is swollen and sore. Ten minutes after an ice pack is applied, the client asks for another. What is the best response from the nurse? A) "I'll get you one right away." B) "You only need to use one ice pack." C) "You need to leave it off for at least 20 minutes and then reapply." D) "I'll bring you an extra so that you can change it when you are ready."

Answer: C Explanation: A) Providing an additional ice pack before 20 minutes have passed would increase the perineal edema. B) More than one ice pack must be used in order to apply ice for 20 minutes on followed by 20 minutes off. C) For optimal effect, the ice pack should be applied for 20 to 30 minutes and removed for at least 20 minutes before being reapplied. D) An ice pack that is provided now for use in 20 minutes would melt before being used.

1) The nurse manager is preparing an educational in-service for staff nurses about elder abuse. The nurse manager develops a hypothetical situation: A wheelchair-bound client who lives with her daughter has experienced hunger because she cannot reach the cupboards to make lunch. Which category of elder abuse does this example describe? A) Psychologic abuse B) Physical abuse C) Neglect D) Financial abuse

Answer: C Explanation: A) Psychologic abuse is usually verbal. B) Physical abuse involves some degree of pain and injury. C) Neglect is a failure on the part of a caregiver, or any person having custody of an elder, to provide reasonable care, which is the degree of care that a reasonable person would provide. Financial abuse involves money

1) The nurse is supervising a student nurse who is working with a 14-year-old client who delivered her first child yesterday. Which statement indicates that the nursing student understands the particular needs of an adolescent client? A) "This client will need less teaching, because she will have gotten the right information in school." B) "Because of her age, this client will require less frequent fundal checks to assess for postpartal hemorrhage." C) "Because of her age, this client will probably need extra teaching about the terminology for her anatomy." D) "This client will need to have her grandmother provide day care and help raise the baby."

Answer: C Explanation: A) Public or private education likely does not cover the extent of the information that the adolescent needs to know about pregnancy and delivery. The nurse has many opportunities for teaching adolescent parents about their newborn in the postpartum unit and serves as a role model for new parents when responding to and caring for the newborn. B) Adolescent mothers have the same basic physical care needs as older mothers. C) Some adolescents may not have a working knowledge of their own anatomy and physiology or the related terminology, and they may require special assistance with postpartum hygiene and care. D) Although the client will require day care to continue with her education, the assistance does not have to come from her grandmother.

1) The client with a normal pregnancy had an emergency cesarean birth under general anesthesia 2 hours ago. The client now has a respiratory rate of 30, pale blue nail beds, a pulse rate of 110, and a temperature of 102.6°F, and is complaining of chest pain. The nurse understands that the client most likely is experiencing which of the following? A) Pulmonary embolus B) Pneumococcal pneumonia C) Pneumonitis D) Gastroesophageal reflux disease

Answer: C Explanation: A) Pulmonary embolus does not cause fever. B) General anesthesia does not cause pneumococcal pneumonia. C) Even when food and fluids have been withheld, the gastric juice produced during fasting is highly acidic and can produce chemical pneumonitis if aspirated. This pneumonitis is known as Mendelson syndrome. The signs and symptoms are chest pain, respiratory distress, cyanosis, fever, and tachycardia. Women undergoing emergency cesarean births appear to be at considerable risk for adverse events. D) Gastroesophageal reflux disease does not cause a fever or cyanosis.

1) The nurse is assessing a pregnant client who reports nasal stuffiness and congestion. Which term will the nurse use to document this data in the medical record? A) Rales B) Epistaxis C) Rhinitis of pregnancy D) Pregnancy-induced asthma

Answer: C Explanation: A) Rales is not the term the nurse uses to document nasal stuffiness and congestion that occurs during pregnancy. B) Epistaxis is not the term the nurse uses to document nasal stuffiness and congestion that occurs during pregnancy. C) Rhinitis of pregnancy is the term that the nurse will use when documenting nasal stuffiness and congestion that often occurs during pregnancy. D) Pregnancy-induced asthma is not the term the nurse uses to document nasal stuffiness and congestion that occurs during pregnancy.

1) A patient receiving chemotherapy for breast cancer writes in a journal during the treatments and reads devotional material. Which phase of psychologic adjustment should the nurse identify that this patient is experiencing? A) Shock B) Denial C) Reaction D) Recovery

Answer: C Explanation: A) Reaction occurs in conjunction with the initiation of treatment. Coping mechanisms become evident during this phase, and may include things like journaling and reading devotional material. Shock generally extends from the discovery of the lump through the process of diagnosis. Denial of the reality of the illness is a common response by the woman during the periods of diagnosis and treatment. Recovery begins during convalescence following the completion of medical treatment. B) Reaction occurs in conjunction with the initiation of treatment. Coping mechanisms become evident during this phase, and may include things like journaling and reading devotional material. Shock generally extends from the discovery of the lump through the process of diagnosis. Denial of the reality of the illness is a common response by the woman during the periods of diagnosis and treatment. Recovery begins during convalescence following the completion of medical treatment. C) Reaction occurs in conjunction with the initiation of treatment. Coping mechanisms become evident during this phase, and may include things like journaling and reading devotional material. Shock generally extends from the discovery of the lump through the process of diagnosis. Denial of the reality of the illness is a common response by the woman during the periods of diagnosis and treatment. Recovery begins during convalescence following the completion of medical treatment. Reaction occurs in conjunction with the initiation of treatment. Coping mechanisms become evident during this phase, and may include things like journaling and reading devotional material. Shock generally extends from the discovery of the lump through the process of diagnosis. Denial of the reality of the illness is a common response by the woman during the periods of diagnosis and treatment. Recovery begins during convalescence following the completion of medical treatment

1) Two hours after an epidural infusion has begun, a client complains of itching on her face and neck. What should the nurse do? A) Remove the epidural catheter and apply a Band-Aid to the injection site. B) Offer the client a cool cloth and let her know the itching is temporary. C) Recognize that this is a common side effect, and follow protocol for administration of Benadryl. D) Call the anesthesia care provider to re-dose the epidural catheter.

Answer: C Explanation: A) Removing the epidural catheter does not address the side effects of the medication. B) Using a cool cloth does not address the side effects of the medication. C) Itching is a side effect of the medication used for an epidural infusion. Benadryl, an antihistamine, can be administered to manage pruritus. D) The anesthesia care provider would not re-dose, as that would continue or worsen the side effects of the medication.

The nurse is developing a teaching plan for a client undergoing a tubal ligation. What information should be included in the plan? A) The surgical procedure is easily reversible. B) Laparotomy is performed following a vaginal birth. C) Minilaparotomy is performed in the postpartum period soon after a vaginal birth. D) Tubal ligation can be done at any time the woman is either pregnant or not pregnant.

Answer: C Explanation: A) Reversal of a tubal ligation depends on the type of procedure performed. Although theoretically reversible, clients are advised that the method should be considered irreversible. B) Laparotomy is performed following a cesarean birth or other abdominal surgery. C) A tubal ligation minilaparotomy is performed in the postpartum period soon after a vaginal birth. D) Tubal ligation can be done at any time the woman is not pregnant.

1. The nurse is aware that labor and birth will most likely proceed normally when the fetus is in what position? A) Right-acromion-dorsal-anterior B) Right-sacrum-transverse C) Occiput anterior D) Posterior position

Answer: C Explanation: A) Right-acromion-dorsal-anterior denotes a fetal position in a shoulder presentation, which would be a difficult delivery. B) Right-sacrum-transverse indicates a breech delivery, which would be a difficult delivery. C) The most common fetal position is occiput anterior. When this position occurs, labor and birth are likely to proceed normally. D) The fetal head presents a larger diameter in a posterior position than in an anterior position. A posterior position increases the pressure on the maternal sacral nerves, causing the laboring woman to experience backache and pelvic pressure.

1) Screening for gestational diabetes mellitus (G D M) is typically completed between which of the following weeks of gestation? A) 36 and 40 weeks B) Before 20 weeks C) 24 and 28 weeks D) 30 and 34 weeks

Answer: C Explanation: A) Screening for gestational diabetes mellitus (G D M) is not completed between 36 and 40 weeks' gestation. B) Screening for gestational diabetes mellitus (G D M) is not completed before 20 weeks' gestation. C) Screening for gestational diabetes mellitus (G D M) is typically completed between 24 and 28 weeks' gestation. D) Screening for gestational diabetes mellitus (G D M) is not completed between 30 and 34 weeks' gestation.

1) Before a newborn and mother are discharged from the hospital, the nurse informs the parents about routine screening tests for newborns. What is a good reason for having the screening tests done? A) The tests prevent infants from developing phenylketonuria. B) The tests detect such disorders as hypertension and diabetes. C) The tests detect disorders that cause physical, intellectual, and developmental complications or death if left undiscovered. D) The tests prevent sickle-cell anemia, galactosemia, and homocystinuria.

Answer: C Explanation: A) Screening tests do not prevent any disorders. B) Screening tests do not detect hypertension or diabetes. C) Newborn screening tests use a few drops of the newborn's blood to detect disorders that are often asymptomatic at birth but cause irreversible harm if not detected early. Profound physical, intellectual, and developmental complications and even death can result from many of the conditions for which newborns are screened prior to discharge. D) Screening tests do not prevent any disorders.

For prenatal care, the client is attending a clinic held in a church basement. The client's care is provided by registered nurses and a certified nurse-midwife. What is this type of prenatal care? A) Secondary care B) Tertiary care C) Community care D) Unnecessarily costly care

Answer: C Explanation: A) Secondary care is specialized care; an example is checking the hemoglobin A1C of a diabetic client at an endocrine clinic. B) Tertiary care is very specialized, and includes trauma units and neonatal intensive care units. C) Prenatal care is primary care. Community care is often provided at clinics in neighborhoods to facilitate clients' access to primary care, including prenatal care and prevention of illness. D) Community care decreases costs while improving client outcomes, and is not unnecessarily expensive.

1) The nurse who is counseling a group of middle-school girls on pregnancy avoidance should include which statement? A) "Although sexuality is common in the media, peer pressure to have sex is not an important factor." B) "It has become far less acceptable to give birth during your teenage years than it used to be." C) "Although condom use is growing, there is still an increasing rate of S T Is among teens." D) "You have learned enough from your friends and families to understand how pregnancy occurs."

Answer: C Explanation: A) Sexual innuendo permeates every aspect of the popular media. Peer pressure to have sex is also common, and is a strong influence on when a teen becomes sexually active. B) Premarital sexual activity is commonplace, and teenage pregnancy is more socially acceptable today than it was in the past. C) Condom use is increasing, but the rate of S T I infections, including H I V, is also rising. Comprehensive approaches to sex education do not increase initiation of sexual activity at an earlier age. In fact, it helps teens withstand the pressure to have sex too soon

1) The prenatal client in her third trimester tells the clinic nurse that she works 8 hours a day as a cashier and stands when at work. What response by the nurse is best? A) "No problem. Your baby will be fine." B) "Do you get regular breaks for eating?" C) "Your risk of poor pregnancy outcomes may be higher." D) "Standing might increase ankle swelling."

Answer: C Explanation: A) Standing more than 3 hours a day increases the risk of preterm labor. To be therapeutic in communication, avoid false reassurance. B) Although breaks for eating are important for pregnant employees, it is more important to tell the client about the increased risks associated with standing more than 3 hours a day. C) Pregnant women who are employed in jobs that require prolonged standing (more than 3 hours) may be at risk for poor pregnancy outcomes. D) Although this is true, it is less important than teaching the client about the risks of preterm labor from standing more than 3 hours a day.

1) The parents of a preterm newborn wish to visit their baby in the N I C U. A statement by the nurse that would not support the parents as they visit their newborn is which of the following? A) "Your newborn likes to be touched." B) "Stroking the newborn will help with stimulation." C) "Visits must be scheduled between feedings." D) "Your baby loves her pink blanket."

Answer: C Explanation: A) Statements that encourage the parents to touch the newborn will help them bond with their child. B) Statements that encourage the parents to stroke the newborn will help them bond with their child and provide stimulation. C) The nurse should always encourage parents to visit and get to know their newborn, even in the N I C U. Nurses foster the development of a safe, trusting environment by viewing the parents as essential caregivers, not as visitors or nuisances in the unit. Comments that personalize the baby will tell the parents their baby is unique and special

A client asks the nurse, "Can you explain to us how to use the basal body temperature method to detect ovulation and prevent pregnancy?" What is the nurse's best response? A) "Take your temperature every evening at the same time and keep a record for a period of several weeks. A noticeable drop in temperature indicates that ovulation has occurred." B) "Take your temperature every day at the same time and keep a record of the findings. A noticeable rise in temperature indicates ovulation." C) "Take your temperature each day, immediately upon awakening, and keep a record of each finding. A noticeable rise in temperature indicates that ovulation is about to occur." D) "This is an unscientific and unproven method of determining ovulation, and is not recognized as a means of birth control."

Answer: C Explanation: A) Taking the temperature every evening at the same time would not provide information about when ovulation occurs. B) Taking the temperature every day at the same time would not necessarily provide accurate information about when ovulation occurs. C) The basal body temperature method is used to detect ovulation by an increase in the basal temperature during the menstrual cycle. It requires that the woman take her temperature every morning upon awakening (before any activity) and record the findings on a temperature graph, and is based on the fact that the temperature almost always rises and remains elevated after ovulation because of the production of progesterone, a thermogenic (heat-producing) hormone. D) Using basal body temperature to determine the timing of ovulation is a proven scientific method, and is recognized as an effective means of birth control.

15) A client who wishes to have an unmedicated birth is in the transition stage. She is very uncomfortable and turns frequently in the bed. Her partner has stepped out momentarily. How can the nurse be most helpful? A) Talk to the client the entire time. B) Turn on the television to distract the client. C) Stand next to the bed with hands on the railing next to the client. D) Sit silently in the room away from the bed.

Answer: C Explanation: A) Talking might irritate the client. B) Turning on the television might irritate the client. C) Standing next to the bed is supportive without being irritating. The laboring woman fears being alone during labor. The woman's anxiety may be decreased when the nurse remains with her. D) Sitting silently away from the client can lead to her feeling abandoned.

1) The student nurse encounters a 15-year-old girl who reports that she has no pubic or axillary hair and has not yet experienced growth of her breasts. The student asks the nurse about the physiology of this occurrence. The nurse explains that the client probably lacks which hormone? A) Testosterone B) Progesterone C) Estrogen D) Prolactin

Answer: C Explanation: A) Testosterone is responsible for the development of secondary sex characteristics in males. B) Progesterone and prolactin do not accomplish this change. C) Estrogens influence the development of secondary sex characteristics in females. Progesterone and prolactin do not accomplish this change

1) The nurse is planning an educational presentation on hyperbilirubinemia for nursery nurses. Which statement is most important to include in the presentation? A) Conjugated bilirubin is eliminated in the conjugated state. B) Unconjugated bilirubin is neurotoxic, and cannot cross the placenta. C) Total bilirubin is the sum of the direct and indirect levels. D) Hyperbilirubinemia is a decreased total serum bilirubin level.

Answer: C Explanation: A) The (direct) conjugated bilirubin progresses down the intestines, where bacteria transform it into urobilinogen (urine bilirubin). Even after the bilirubin has been conjugated and bound, it can be changed back to unconjugated bilirubin via the enterohepatic circulation. B) Fetal unconjugated bilirubin crosses the placenta to be excreted, so the fetus does not need to conjugate bilirubin. C) Total serum bilirubin is the sum of conjugated (direct) and unconjugated (indirect) bilirubin. D) Hyperbilirubinemia is an elevated total serum bilirubin level.

15) At 1 minute after birth, the infant has a heart rate of 100 beats per minute, and is crying vigorously. The limbs are flexed, the trunk is pink, and the feet and hands are cyanotic. The infant cries easily when the soles of the feet are stimulated. How would the nurse document this infant's Apgar score? A) 7 B) 8 C) 9 D) 10

Answer: C Explanation: A) The Apgar score would be higher than 7. B) The Apgar score would be higher than 8. C) Two points each are scored in each of the categories of heart rate, respiratory effort, muscle tone, and reflex irritability. One point is scored in the category of skin color. The total Apgar would be 9. D) The infant has cyanotic limbs, so the Apgar score cannot be perfect.

1) The nurse is about to tell a client that her Pap smear result was abnormal. Which statement should the nurse include? A) "The Pap smear is used to diagnose cervical cancer." B) "A loop electrosurgical excision procedure (L E E P) is needed." C) "Colposcopy to further examine your cervix is the next step." D) "Your cervix needs to be treated with cryotherapy."

Answer: C Explanation: A) The Pap smear is a screening tool for cervical abnormalities; it is not diagnostic. B) Although L E E P (the removal of the surface tissue of the cervix) might be performed to treat cervical dysplasia or carcinoma in situ, this client has not had a diagnostic examination yet. C) Colposcopy is an examination of the cervix through a magnifying device. D) Cryotherapy, or freezing of the cervix, is one treatment option for precancerous cervical lesions.

1) The nurse is instructing a new mother on circumcision care with a Plastibell. The nurse knows the mother understands when she states that the Plastibell should fall off within how long? A) 2 days B) 10 days C) 8 days D) 14 days

Answer: C Explanation: A) The Plastibell does not fall off in 2 days. B) The Plastibell should fall off before 10 days. C) The Plastibell should fall off within 8 days. If it remains on after 8 days, the parents should consult with the newborn's physician. D) The Plastibell should fall off before 14 days.

1) Which of the following federal departments actively investigates and prosecutes individuals who cross state lines to avoid paying child support, and now intercepts delinquent parents' income tax refunds? A) U.S. Department of Health and Human Services B) U.S. Department of Labor C) U.S. Department of Justice D) U.S. Equal Employment Opportunity Commission

Answer: C Explanation: A) The U.S. Department of Health and Human Services does not investigate and prosecute individuals who cross state lines to avoid paying child support. B) The U.S. Department of Labor keeps track of labor statistics. C) The U.S. Department of Justice actively investigates and prosecutes individuals who cross state lines to avoid paying child support, and now intercepts delinquent parents' income tax refunds. D) The U.S. Equal Employment Opportunity Commission receives complaints from pregnant women who were turned down for jobs, denied transfers to positions for which they were qualified, or fired unjustly.

1) A 2-day-old newborn is asleep, and the nurse assesses the apical pulse to be 88 beats/min. What would be the most appropriate nursing action based on this assessment finding? A) Call the physician. B) Administer oxygen. C) Document the finding. D) Place the newborn under the radiant warmer.

Answer: C Explanation: A) The apical pulse rate is within normal range. There is no need to call the physician. B) There is no need to administer oxygen at this time. C) An apical pulse rate of 88 beats/min is within the normal range of a sleeping full-term newborn. The average resting heart rate in the first week of life is 110 to 160 beats/min in a healthy full-term newborn but may vary significantly during deep sleep or active awake states. In full-term newborns, the heart rate may drop to a low of 80 to 100 beats/min during deep sleep. D) There is no need to place the infant in a radiant warmer.

1) The postpartum nurse is performing a home care visit to a first-time mother on her third day after delivery. She reports that her nipples are becoming sore. What statement indicates that further teaching is needed? A) "I can apply lanolin cream to help with the nipple pain." B) "Watching how much areola is visible will help me see whether my baby has a good mouthful of breast or not." C) "My nipples will heal if I switch to bottle feeding for about 3 days while I pump my breasts." D) "Rotating breastfeeding positions will allow the sore areas of my nipples to have less friction."

Answer: C Explanation: A) The application of highly purified anhydrous (H P A) lanolin and/or breast milk accompanied by air-drying at the end of feedings has been shown to be safe and effective in treating nipple pain. B) When latched, the baby's lips should be flanged out, and the newborn should cover as much of the areola as possible. C) For severe cases in which the mother is unable to tolerate breastfeeding, the mother will need to pump or hand-express the breast milk until the nipple condition improves. Bottle-feeding may not be necessary. D) Encouraging the mother to rotate positions when feeding the infant may decrease nipple soreness. Changing positions alters the focus of greatest stress and promotes more complete breast emptying.

1) The nurse is providing care to a pregnant client who is experiencing an increase in white, thick, and "cottage-cheese-like" vaginal discharge. Based on this data, which diagnosis does the nurse anticipate for this client? A) Syphilis B) Gonorrhea C) Moniliasis D) Chlamydia

Answer: C Explanation: A) The assessment data does not support the diagnosis of syphilis. B) The assessment data does not support the diagnosis of gonorrhea. C) Vaginal secretions during pregnancy are often thick, white and acidic which increase the client's risk for moniliasis, a common yeast infection during pregnancy. D) The assessment data does not support the diagnosis of chlamydia.

1) What is the term for when children alternate between two homes, spending varying amounts of time with each parent in a situation called co-parenting and usually involving joint custody? A) Blended or reconstituted nuclear family B) Extended kin network family C) Binuclear family D) Extended family

Answer: C Explanation: A) The blended or reconstituted nuclear family includes two parents with biologic children from a previous marriage or relationship who marry or cohabitate. B) An extended kin network family is a specific form of an extended family in which two nuclear families of primary or unmarried kin live in proximity to each other. C) A binuclear family is a post-divorce family in which the biologic children are members of two nuclear households, with parenting by both the father and the mother. D) In an extended family, a couple shares household and childrearing responsibilities with parents, siblings, or other relatives.

The nurse is preparing educational materials at a family planning clinic. The client who is an appropriate candidate for using emergency contraception would be one who reports which of the following? A) Forgetting to start her pill pack yesterday B) Unprotected intercourse during her menses C) That a condom broke yesterday in the middle of her cycle D) Increased dysmenorrhea since I U C insertion

Answer: C Explanation: A) The client who forgot to start a new pill pack on time might not have had intercourse. B) Intercourse during menses does not lead to pregnancy. C) Research indicates that oral hormonal E C taken as soon as possible within 72 hours, but up to 5 days, can reduce the risk of pregnancy after a single act of unprotected intercourse by at least 74%. D) An I U C in place prevents conception, so emergency contraception is not indicated.

1) The nurse has just palpated contractions and compares the consistency to that of the forehead in order to estimate the firmness of the fundus. What would the intensity of these contractions be identified as? A) Mild B) Moderate C) Strong D) Weak

Answer: C Explanation: A) The consistency of mild contractions is similar to that of the nose. B) The consistency of moderate contractions is similar to that of the chin. C) The consistency of strong contractions is similar to that of the forehead. D) Weak contractions are not identified.

1) Which of the following tests has become a widely accepted method of evaluating fetal status? A) Contraction stress test (C S T) B) M S A F P test C) Nonstress test (N S T) D) Nuchal translucency test

Answer: C Explanation: A) The contraction stress test (C S T) is a means of evaluating the respiratory function (oxygen and carbon dioxide exchange) of the placenta. B) The maternal serum alpha-fetoprotein (M S A F P) test is a component of the screening test, the "quadruple check" that utilizes multiple markers to screen pregnancies for N T D, trisomy 21 (Down syndrome), and trisomy 18. C) The nonstress test (N S T) has become a widely accepted method of evaluating fetal status. This test involves using an external electronic fetal monitor to obtain a tracing of the fetal heart rate (F H R) and observation of acceleration of the F H R with fetal movement. The nuchal translucency test

The nurse in the clinic instructs a client who is using the natural method of contraception to begin counting the first day of her cycle as which day? A) The day her menstrual period ceases B) The first day after her menstrual period ceases C) The first day of her menstrual period D) The day of ovulation

Answer: C Explanation: A) The day her menstrual period ceases is not an indicator of the first day of the cycle. B) The first day after her menstrual period ceases is not an indicator of the first day of the cycle. C) The first day of menstruation is the first day of the cycle. D) The day of ovulation is not an indicator of the first day of the cycle.

When the nurse is teaching a woman about the use of a diaphragm, it is important to instruct her that the diaphragm should be rechecked for correct size how often? A) Every five years routinely B) When weight gain or loss beyond five pounds has occurred C) After each birth D) Only after significant weight loss

Answer: C Explanation: A) The diaphragm should be rechecked for correct size after each childbirth and whenever a woman has gained or lost 10 pounds or more. B) The diaphragm should be rechecked for correct size after each childbirth and whenever a woman has gained or lost 10 pounds or more. C) The diaphragm should be rechecked for correct size after each childbirth and whenever a woman has gained or lost 10 pounds or more. D) The diaphragm should be rechecked for correct size after each childbirth and whenever a woman has gained or lost 10 pounds or more.

1) The emergency department nurse is admitting a client who has been sexually assaulted. The nurse is explaining how the physical evidence will be collected. Which statement by the client indicates that teaching has been effective? A) "All the evidence will be kept in a locked cupboard until the police arrive." B) "You collect urine samples to make sure the rapist did not get me pregnant." C) "The evidence you collect might be able to identify the rapist." D) "Blood samples are taken to help identify whether the rapist had H I V."

Answer: C Explanation: A) The evidence must remain in the hands of the nurse until handed directly to the police. B) Urine should be collected in cases in which a drug-facilitated sexual assault is suspected. C) D N A can be obtained from collected evidence to identify the rapist. D) Blood is drawn to test for syphilis and to determine the woman's blood type, and additional blood may be drawn for a pregnancy test.

1) The laboring client with meconium-stained amniotic fluid asks the nurse why the fetal monitor is necessary, as she finds the belt uncomfortable. Which response by the nurse is most important? A) "The monitor is necessary so we can see how your labor is progressing." B) "The monitor will prevent complications from the meconium in your fluid." C) "The monitor helps us to see how the baby is tolerating labor." D) "The monitor can be removed, and oxygen given instead."

Answer: C Explanation: A) The fetal monitor does not help visualize labor progress. B) The fetal monitor does not prevent complications such as meconium aspiration syndrome. C) Electronic fetal monitoring (E F M) provides a continuous tracing of the fetal heart rate (F H R), allowing characteristics of the F H R to be observed and evaluated. D) Oxygen is an appropriate intervention for late decelerations, but no information is given about the fetal heart rate.

1) New parents decide not to have their newborn circumcised. What should the nurse teach regarding care for the uncircumcised infant? A) The foreskin will be retractable at 2 months. B) Retract the foreskin and clean thoroughly. C) Avoid retracting the foreskin. D) Use soap and Betadine to cleanse the penis daily.

Answer: C Explanation: A) The foreskin is not fully retractable at 2 months. B) The foreskin is not fully retractable in a newborn, and should not be forced back over the penis. C) The foreskin will retract normally over time, and may take 3 to 5 years. D) If retraction has occurred, daily gentle washing of the glans with soap and water is sufficient to maintain adequate cleanliness.

1) The nurse is working with a student nurse during assessment of a 2-hour-old newborn. Which action indicates that the student nurse understands neonatal assessment? A) The student nurse listens to bowel sounds, then assesses the head for skull consistency and size and tension of fontanelles. B) The student nurse checks for Ortolani's sign, then palpates the femoral pulse, then assesses respiratory rate. C) The student nurse determines skin color, then describes the shape of the chest and looks at structures and flexion of the feet. D) The student nurse counts the number of cord vessels, then assesses genitals, then sclera color and eyelids.

Answer: C Explanation: A) The head should be assessed before the bowel sounds. B) The respiratory rate should be assessed first, when the infant is at rest and undisturbed. C) Neonatal assessment proceeds in a head-to-toe fashion. D) The sclera and eye assessment should be done prior to assessing genitals.

1) When a woman who has been raped is admitted to the emergency department, the nurse caring for the woman knows that which of the following is the priority nursing intervention? A) Explaining exactly what will need to be done to preserve legal evidence B) Assuring the woman that everything will be all right C) Creating a safe, secure environment for her D) Contacting family members

Answer: C Explanation: A) The legal interventions would not take priority over safety at this time. B) Assuring the woman that everything will be all right is not the first priority nursing intervention in caring for a survivor of a sexual assault. C) The first priority in caring for a survivor of a sexual assault is to create a safe, secure environment. Contacting family members is important, but is not the priority nursing intervention

1) The nurse is teaching a class about domestic violence to enhance education within the community. Which statement regarding the cycle of violence should the nurse include in the presentation? A) "The tension-building phase lasts a few hours." B) "The batterer often feels remorse during the tension-building phase." C) "The acute battery incident is often triggered by an external event, such as the loss of a job." D) "The acute battery incident often finds the victim hoping the relationship will change for the better."

Answer: C Explanation: A) The length of the tension-building phase of the cycle of violence varies considerably across individual cases and can range from weeks to years. It is often the acute battery incident that lasts a few hours. B) The batterer often feels remorse during the tranquil phase or honeymoon period, not the tension-building phase. C) An acute battery incident is often triggered by an external event for the abuser, such as the loss of a job. D) The victim of abuse often hopes the relationship will improve in the tension-building phase, not during the acute battery incident.

28) A client's labor has progressed so rapidly that a precipitous birth is occurring. What should the nurse do? A) Go to the nurse's station and immediately call the physician. B) Run to the delivery room for an emergency birth pack. C) Stay with the client and ask auxiliary personnel for assistance. D) Hold back the infant's head forcibly until the physician arrives for the delivery.

Answer: C Explanation: A) The nurse can direct auxiliary personnel to contact the physician. B) The nurse can direct auxiliary personnel to retrieve the emergency birth pack. C) If birth is imminent, the nurse must not leave the client alone. D) With one hand, the nurse should apply gentle pressure against the fetal head to maintain flexion and prevent it from popping out rapidly. The nurse does not hold the head back forcibly.

1) The nurse is planning a home care visit to a mother who just recently delivered. The neighborhood is known to have a significant crime rate. What should the nurse do when planning this visit to facilitate personal safety? A) Be friendly to all pets encountered on the visit to build client rapport. B) Wait to find the exact location until arrival in the neighborhood. C) Put personal possessions in the trunk when leaving the office. D) Wear flashy jewelry to garner respect.

Answer: C Explanation: A) The nurse should ask clients to keep threatening pets secured in a different area of the home during the visit. B) The nurse should confirm the address, ask for directions during the previsit contact, and trace out the route to the client's home on a map or the Internet before leaving for the visit and take the map along; the nurse should use a G P S device if available. C) The nurse should lock personal belongings in the trunk of the car, out of sight, before starting out or before arriving at the home. D) The nurse should avoid wearing expensive or flashy jewelry.

1) Which nursing action is appropriate prior to presenting the stillborn newborn to the parents for holding? A) Staying with the family while they hold and say goodbye to their newborn B) Telling the family that they should not hold their baby because it will be too painful C) Preparing the family by telling them what they will see, feel, and smell D) Telling the family that they can have up to thirty minutes with their baby

Answer: C Explanation: A) The nurse should offer the family time alone with the baby, but check back frequently within previously specified time frames. B) The nurse should not impose his or her beliefs on the couple when providing care. C) The nurse should prepare the family for holding the baby by telling them what they will see, feel, and smell. D) The nurse should allow the family to have as much time as they need to hold their baby and say goodbye.

1) The nurse is conducting an admission assessment for a pregnant client who is in labor. Which finding would cause the nurse to expect that the client has been abusing an illegal substance? A) The client is asking for an epidural and rates pain as an 8 on a 0 to 10 numeric scale. B) The client is diaphoretic during the transition stage of labor. C) The client has constricted pupils and inflamed nasal mucosa. D) The client has a history of precipitous labor.

Answer: C Explanation: A) The nurse would not suspect that this client has been abusing an illegal substance based on asking for an epidural and rating pain as an 8 on a 0 to 10 numeric scale. B) The nurse would not suspect that this client has been abusing an illegal substance based on experiencing diaphoresis during the transition stage of labor. This is an expected finding. C) A client who has constricted pupils and inflamed nasal mucosa would be suspected of abusing illegal substances. D) A client who has a history of precipitous labor would not cause the nurse to expect substance abuse.

1) The nurse is caring for a postpartum client who is experiencing a sickle cell crisis. Which is the priority assessment by the nurse? A) Blood pressure B) Lochia C) Temperature D) Fundal height

Answer: C Explanation: A) The postpartum client experiencing a sickle cell crisis is at risk for infection. While monitoring blood pressure is important, this is not the priority assessment. B) The postpartum client experiencing a sickle cell crisis is at risk for infection. While monitoring lochia is important, this is not the priority assessment. C) The postpartum client experiencing a sickle cell crisis is at risk for infection. The priority assessment for this client is to closely monitor temperature. D) The postpartum client experiencing a sickle cell crisis is at risk for infection. While monitoring fundal height is important, this is not the priority assessment.

1) It is 1 week before a pregnant client's due date. The nurse notes on the chart that the client's pulse rate was 74-80 before pregnancy. Today, the client's pulse rate at rest is 90. What action should the nurse should take? A) Chart the findings. B) Notify the physician of tachycardia. C) Prepare the client for an electrocardiogram (E K G). Prepare the client for transport to the hospital

Answer: C Explanation: A) The pulse rate frequently increases during pregnancy, although the amount varies from almost no increase to an increase of 10 to 15 beats per minute. This is a normal response, and does not indicate a need for emergency measures or treatment. B) This pulse rate in a near-term client is not considered to be tachycardia. C) This pulse rate in a near-term client does not indicate a need for emergency measures or treatment. D) This client does not need to go to the hospital.

1) A client with a normal prepregnancy weight asks why she has been told to gain 25-35 pounds during her pregnancy while her underweight friend was told to gain more weight. What should the nurse tell the client the recommended weight gain is during pregnancy? A) 25-35 pounds, regardless of a client's prepregnant weight B) More than 25-35 pounds for an overweight woman C) Up to 40 pounds for an underweight woman D) The same for a normal weight woman as for an overweight woman

Answer: C Explanation: A) The recommended total weight gain during pregnancy for a woman of normal weight before pregnancy is 25 to 35 pounds. B) For women who were overweight before becoming pregnant, the recommended gain is 15 to 25 pounds. C) Prepregnant weight determines the recommended weight gain during pregnancy. Underweight women are advised to gain 28-40 pounds. D) Women of normal weight should gain 25-35 pounds during pregnancy, whereas overweight women should limit their weight gain to 15-25 pounds during pregnancy.

1) The nurse is planning a community education presentation on battering. Which statement about battering should the nurse include? A) Battering occurs in a small percentage of the population. B) Battering is mainly a lower-class, blue-collar problem. C) Battered women are at greatest risk for severe violence when they leave the batterer. D) If the batterer stops drinking, the violence usually stops.

Answer: C Explanation: A) The statistics on reported cases underrepresent the true incidence. As many as one in three women may be the victim of assault by her partner in her lifetime; however, it is a widely underreported crime. B) Domestic violence occurs among all sectors of society. It happens to women of all socioeconomic statuses, races, ethnicities, and religious faiths. C) Battered women are at greatest risk for injury or domestic homicide when they leave the abuser. D) Battered women sometimes think that the abuse will stop if their partners stop drinking or using drugs. Unfortunately, this usually does not happen.

1) A telephone triage nurse gets a call from a postpartum client who is concerned about jaundice. The client's newborn is 37 hours old. What data point should the nurse gather first? A) Stool characteristics B) Fluid intake C) Skin color D) Bilirubin level

Answer: C Explanation: A) The stool characteristic of green coloration indicates excretion of bilirubin. B) Breastfeeding is implicated in jaundice in some newborns. Breast milk jaundice occurs in approximately 2% to 4% of term infants with an onset of 4 to 7 days of life. C) Jaundice (icterus) is the yellowish coloration of the skin and sclera caused by the presence of bilirubin in elevated concentrations. Inspection of the skin would be the first step in assessing for jaundice. D) Bilirubin is primarily the metabolic end product of erythrocyte (R B C) breakdown. Conjugation, or the changing of bilirubin into an excretable form, is the conversion of the yellow lipid-soluble pigment (unconjugated, indirect) into water-soluble pigment (excretable, direct).

1. The client in early labor asks the nurse what the contractions are like as labor progresses. What would the nurse respond? A) "In normal labor, as the uterine contractions become stronger, they usually also become less frequent." B) "In normal labor, as the uterine contractions become stronger, they usually also become less painful." C) "In normal labor, as the uterine contractions become stronger, they usually also become longer in duration." D) "In normal labor, as the uterine contractions become stronger, they usually also become shorter in duration."

Answer: C Explanation: A) The uterine contractions of labor become more frequent as labor progresses. B) The uterine contractions of labor become more painful over time. C) During the active and transition phases, contractions become more frequent, are longer in duration, and increase in intensity. D) The uterine contractions of labor do not become shorter in duration as labor progresses.

1) The nurse is providing discharge teaching to a woman who delivered her first child 2 days ago. The nurse understands that additional information is needed if the client makes which statement? A) "I should expect a lighter flow next week." B) "The flow will increase if I am too active." C) "My bleeding will remain red for about a month." D) "I will be able to use a pantiliner in a day or two."

Answer: C Explanation: A) The uterus rids itself of the debris remaining after birth through a discharge called lochia. Lochia serosa is a pinkish color. It follows from about the 3rd to the 10th day toward a lighter amount of flow and a lighter color of discharge. When the lochia stops, the cervix is considered closed. B) The amount of lochia may be increased by exertion or breastfeeding. C) Lochia rubra is dark red in color. It is present for the first 2 to 3 days postpartum. Lochia serosa is a pinkish color and it follows from about the 3rd to the 10th day. D) By day three, the lochia shed is a lighter amount of flow and a lighter color of discharge. A pantiliner would be appropriate for use at this point.

1) If the physician indicates a shoulder dystocia during the delivery of a macrosomic fetus, how would the nurse assist? A) Call a second physician to assist. B) Prepare for an immediate cesarean delivery. C) Assist the woman into McRoberts maneuver. D) Utilize fundal pressure to push the fetus out.

Answer: C Explanation: A) The vaginal delivery of a macrosomic fetus does not require a second physician. B) Although a cesarean might be necessary, it would not be an immediate need. C) The McRoberts maneuver is thought to change the maternal pelvic angle and therefore reduce the force needed to extract the shoulders, thereby decreasing the incidence of brachial plexus stretching and clavicular fracture. D) Fundal pressure should not be performed, because this can further wedge the shoulder against the suprapubic bone.

1) The nurse is preparing a presentation on the menstrual cycle for a group of high school students. Which statement should the nurse include in this presentation? A) "The menstrual cycle has five distinct phases that occur during the month." B) "One hormone controls the phases of the menstrual cycle." C) "The secretory phase occurs when a woman is most fertile." D) "Menstrual cycle phases vary in order from one woman to another."

Answer: C Explanation: A) There are four phases of the menstrual cycle. B) Four hormones control ovulation and, therefore, the menstrual cycle. C) During the secretory phase, the vascularity of the entire uterus increases greatly, providing a nourishing bed for implantation. Although the length of the menstrual cycle might vary, the phases of the menstrual cycle always occur in the same order

1) Which statement made by the pregnant adolescent would indicate to the nurse that she understood her increased risk of physiologic complications during pregnancy? A) "It's no big deal that I started prenatal care in my seventh month." B) "My anemia and eating mostly fast food are not important." C) "I need to take good care of myself so my baby doesn't come early." D) "Smoking and using crack cocaine won't harm my baby."

Answer: C Explanation: A) There is an increased incidence of maternal complications, premature birth, and low-birth-weight babies among adolescent mothers. This statement indicates a lack of understanding of the client's increased risk of physiologic complications during pregnancy. B) There is an increased incidence of maternal complications, premature birth, and low-birth-weight babies among adolescent mothers. This statement indicates a lack of understanding of the client's increased risk of physiologic complications during pregnancy. C) In the adolescent age group, prenatal care is the critical factor that most influences pregnancy outcome. D) Adolescents are at high risk for experimentation with drugs and alcohol. This statement indicates a lack of understanding of the client's increased risk of physiologic complications during pregnancy.

1. The client at 39 weeks' gestation calls the clinic and reports increased bladder pressure but easier breathing and irregular, mild contractions. She also states that she just cleaned the entire house. Which statement should the nurse make? A) "You shouldn't work so much at this point in pregnancy." B) "What you are describing is not commonly experienced in the last weeks." C) "Your body may be telling you it is going into labor soon." D) "If the bladder pressure continues, come in to the clinic tomorrow."

Answer: C Explanation: A) There is no indication that the client should decrease her work schedule. B) Lightening is a common and expected finding. C) One of the premonitory signs of labor is lightening: The fetus begins to settle into the pelvic inlet (engagement). With fetal descent, the uterus moves downward, and the fundus no longer presses on the diaphragm, which eases breathing. D) Lightening does not indicate pathology, and therefore there is no need for the client to come to the clinic if the symptoms continue.

1. A postpartum client reports sharp, shooting pains in her nipple during breastfeeding and flaky, itchy skin on her breasts. Which of the following does the nurse suspect? A) Nipple soreness B) Engorgement C) Mastitis D) Letdown reflex

Answer: C Explanation: A) These are not symptoms of nipple soreness. B) These are not symptoms of engorgement. C) Signs of mastitis include late-onset nipple pain, followed by shooting pain between feedings, often radiating to the chest wall. Eventually, the skin of the affected breast may become pink, flaking, and pruritic. D) These are not symptoms of the letdown reflex.

1) The breastfeeding client asks the nurse about appropriate contraception. What is the nurse's best response? A) "Breastfeeding has many effects on sexual intercourse." B) "I U Ds are easy to use and easy to insert prior to sexual intercourse." C) "It's possible to get pregnant before your menstrual period returns. Let's talk about some different options for contraception." D) "Breastfeeding hampers ovulation, so no contraception is needed."

Answer: C Explanation: A) This answer does not address the client's question about contraception. B) I U Ds can only be placed by a healthcare provider in a clinic situation. C) The nurse should discuss the importance of contraception during the early postpartum period and provide information on the advantages and disadvantages of different methods, including special considerations for breastfeeding mothers. The woman's body needs adequate time to heal and recover from the stress of pregnancy and childbirth. D) Breastfeeding hampers ovulation, but to be safe, breastfeeding women should use a contraceptive. The nurse should discuss the importance of contraception during the early postpartum period and provide information on the advantages and disadvantages of different methods, including special considerations for breastfeeding mothers.

1) Which maternal-child client should the nurse see first? A) Blood type O, R h-negative B) Indirect Coombs' test negative C) Direct Coombs' test positive D) Blood type B, R h-positive

Answer: C Explanation: A) This client is R h-negative, but there is no indication that the alloimmunization has occurred. B) If the mother's indirect Coombs' test is negative and the infant's direct Coombs' test is negative (confirming that sensitization has not occurred), the mother is given R h immune globulin within 72 hours of birth. C) A direct Coombs' test is done on the infant's blood to detect antibody-coated R h-positive RBCs. If the mother's indirect Coombs' test is positive and her R h-positive infant has a positive direct Coombs' test, R h immune globulin is not given; in this case, the infant is carefully monitored for hemolytic disease. D) This client's blood type creates no problems.

1) The nurse walks in to find the client crying after the physician informed her of her diagnosis of human papilloma virus (H P V). Which statement by the nurse conveys an attitude of acceptance toward the client with a sexually transmitted infection? A) "Don't worry about it. In a few weeks, with treatment, the lesions will disappear." B) "You seem upset. I'll get the doctor." C) "You seem upset. Can I help answer any questions?" D) "I think you need to see a therapist."

Answer: C Explanation: A) This client needs a caring person to listen to her and convey acceptance of her. This statement does not convey this. B) The client does not need the doctor at this time. C) The nurse's attitude of acceptance and matter-of-factness conveys to the client that she is still an acceptable person who happens to have an infection. It is not up to the nurse to determine whether this client needs a therapist

1) The client demonstrates understanding of the implications for future pregnancies secondary to her classic uterine incision when she states which of the following? A) "The next time I have a baby, I can try to deliver vaginally." B) "The risk of rupturing my uterus is too high for me to have any more babies." C) "Every time I have a baby, I will have to have a cesarean delivery." D) "I can only have one more baby."

Answer: C Explanation: A) This client will not be able to deliver vaginally in future pregnancies. B) There could be a risk of uterine rupture if the client were allowed to labor with the next pregnancy. C) A classic uterine incision is made in the upper uterine segment and is associated with an increased risk of rupture in subsequent pregnancy, labor, and birth. Therefore, subsequent deliveries will be done by cesarean. D) The number of subsequent pregnancies is not limited to one.

1) The nurse is providing education to the new family. Which question by the nurse is best? A) "Do you know how to give the baby a bath?" B) "You have diapers and supplies at home, right?" C) "How have your breastfeedings been going?" D) "How much formal education do you have?"

Answer: C Explanation: A) This is a closed question; closed questions should be avoided. Also, bathing is a lower-priority need than is feeding. B) This is a leading question. Leading questions should be avoided. C) This is an open-ended question about an important physiologic issue. A discussion that includes both partners can facilitate an open dialog between them and can provide an opportunity for questions and answers. D) Although the family members' level of educational attainment helps when choosing written materials and words, it is rude to ask outright what education they have had.

1) A client is admitted to the labor and delivery unit with contractions that are regular, are 2 minutes apart, and last 60 seconds. She reports that her labor began about 6 hours ago, and she had bloody show earlier that morning. A vaginal exam reveals a vertex presenting, with the cervix 100% effaced and 8 c m dilated. The client asks what part of labor she is in. The nurse should inform the client that she is in what phase of labor? A) Latent phase B) Active phase C) Transition phase D) Fourth stage

Answer: C Explanation: A) This is not the latent phase; in the latent phase of labor, contractions are every 10-20 minutes, 15-20 seconds' duration progressing to every 5-7 minutes, and 30-40 seconds' duration. Dilatation is 1-3 c m. B) This is not the active phase; in the active phase of labor, contractions are every 2-3 minutes with a dilatation of 4-7 c m. C) The transition phase begins with 8 c m of dilatation, and is characterized by contractions that are closer and more intense. D) The fourth stage occurs after delivery of the placenta.

1) The nurse is completing a newborn care class. The nurse knows that teaching has been effective if a new parent states which of the following? A) "My baby might open her arms wide and pull her legs up to her tummy if she is passing gas." B) "When I hold my baby upright with one of his feet on the floor, his feet will automatically remain still." C) "When I put my finger in the palm of my daughter's hand, she will curl her fingers and hold on." D) "I can get my baby to turn his head toward the right if I lift his right arm over his head."

Answer: C Explanation: A) This is the Moro or startle reflex, and occurs when the infant is startled by a sudden movement or a loud noise. B) Stepping reflex happens when a newborn who is held upright with one foot touching a flat surface puts one foot in front of the other and "walks." C) This is the Palmar grasp reflex and is elicited by stimulating the newborn's palm with a finger or object. D) This is the tonic neck reflex and is elicited when the newborn is supine and the head is turned to one side. In response, the extremities on the same side straighten, whereas on the opposite side they flex.

1) A woman is experiencing preterm labor. The client asks why she is on betamethasone. Which is the nurse's best response? A) "This medication will halt the labor process until the baby is more mature." B) "This medication will relax the smooth muscles in the infant's lungs so the baby can breathe." C) "This medication is effective in stimulating lung development in the preterm infant." D) "This medication is an antibiotic that will treat your urinary tract infection, which caused preterm labor."

Answer: C Explanation: A) This medication has no effect on the labor process or on the smooth muscles in the lungs. B) This medication has no effect on the labor process or on the smooth muscles in the lungs. C) Betamethasone or dexamethasone is often administered to the woman whose fetus has an immature lung profile to promote fetal lung maturation. D) This medication is not an antibiotic, and therefore will not help resolve a urinary tract infection.

1) Which statement by a new mother 1 week postpartum indicates maternal role attainment? A) "I don't think I'll ever know what I'm doing." B) "This baby feels like a real stranger to me." C) "It works better for me to undress the baby and to nurse in the chair rather than the bed." D) "My sister took to mothering in no time. Why can't I?"

Answer: C Explanation: A) This statement indicates that the mother has not yet achieved a level of comfort or confidence in her new role as a mother. B) This statement indicates that "binding in" to the newborn has not yet occurred. C) This statement indicates a stage of maternal role attainment in which the new mother feels comfortable enough to make her own decisions about parenting. D) This statement indicates that the mother has not yet achieved a level of comfort or confidence in her new role.

1) The nurse is creating a poster for pregnant mothers. Which description of fetal development should the nurse include? A) Four primary germ layers form from the blastocyst. B) After fertilization, the cells only become larger for several weeks. C) Most organs are formed by 8 weeks after fertilization. The embryonic stage is from fertilization until 5 months

Answer: C Explanation: A) Three primary germ layers form from the blastocyst: ectoderm, mesoderm, and endoderm. B) After fertilization, the cells reproduce by mitosis, resulting in more cells, not larger cells. C) Most organs are formed during the embryonic stage, which lasts from the 15th day after fertilization until the end of the 8th week after conception. D) The embryonic stage ends before the fifth month.

1) The nurse is planning care for a preterm newborn. Which nursing diagnosis has the highest priority? A) Tissue Integrity, Impaired B) Infection, Risk for C) Gas Exchange, Impaired D) Family Processes, Dysfunctional

Answer: C Explanation: A) Tissue Integrity, Impaired is related to fragile capillary network in the germinal matrix, but is not the highest priority. B) Infection, Risk for is related to lack of passive immunity and immature immune defenses due to preterm birth, but is not the highest priority. C) Gas Exchange, Impaired is related to immature pulmonary vasculature and inadequate surfactant production, and has the highest priority. D) Family Processes, Dysfunctional is related to anger or guilt at having given birth to a premature baby and is a psychosocial need, and is therefore a lower priority than are physiologic needs.

1) The nurse is presenting a class of important "dos and don'ts" during pregnancy, including travel considerations. What method of travel does the nurse recommend as most appropriate for a client in her 25th week of pregnancy? A) Automobile B) Airplane C) Train D) None; this client should not travel

Answer: C Explanation: A) Travel by automobile can be especially fatiguing, aggravating many of the discomforts of pregnancy. The pregnant woman needs frequent opportunities to get out of the car and walk. B) Occasional flying is considered safe in the absence of any obstetric or medical complications. However, those women who have medical or obstetric complications, such as poorly controlled diabetes, sickle cell disease, or preeclampsia, and those women with placental abnormalities or who are at risk for preterm birth are advised to avoid flying during pregnancy. Before flying, the pregnant woman should check with her particular airline to see if it has any travel restrictions. C) As pregnancy progresses, travel by train is generally recommended for long distances. D) If medical or pregnancy complications are not present, there are no restrictions on travel. Pregnant women should avoid travel if there is a history of bleeding or preeclampsia, or if multiple births are anticipated.

1) The client is at 6 weeks' gestation, and is spotting. The client had an ectopic pregnancy 1 year ago, so the nurse anticipates that the physician will order which intervention? A) A urine pregnancy test B) The client to be seen next week for a full examination C) An antiserum pregnancy test D) An ultrasound

Answer: C Explanation: A) Urine pregnancy tests are not quantifiable. B) It is not appropriate to wait until next week to see the client. C) A β-Subunit radioimmunoassay (R I A) uses an antiserum with specificity for the β-subunit of h C G in blood plasma. This test may not only detect pregnancy but also detect an ectopic pregnancy or trophoblastic disease. D) An ultrasound may be used to diagnose an ectopic pregnancy, but would not be needed now.

1) The nurse assessing a pregnant African American woman in the first trimester understands that a cultural practice is which of the following? A) Use of herbs like dandelion during pregnancy to increase lactation B) Drinking ginseng tea for faintness C) Eating clay to supply dietary minerals D) Consulting a spiritual advisor to ensure a healthy pregnancy and birth

Answer: C Explanation: A) Use of dandelion is a practice of American Indians. B) Asian women who are pregnant often drink ginseng tea. C) African American pregnant women may be guided by their extended family into common practices such as geophagia, the ingestion of dirt or clay, which is believed to alleviate mineral deficiencies. D) Consulting a spiritual advisor is common among Navajo and many other Native American cultures.

1) After noting meconium-stained amniotic fluid and fetal heart rate decelerations, the physician diagnoses a depressed fetus. The appropriate nursing action at this time would be to do what? A) Increase the mother's oxygen rate. B) Turn the mother to the left lateral position. C) Prepare the mother for a higher-risk delivery. D) Increase the intravenous infusion rate.

Answer: C Explanation: A) Use of oxygen is not a nursing action that would change the status of the depressed fetus. B) Position change to the left side is not a nursing action that would change the status of the depressed fetus. C) Meconium-stained fluid and heart rate decelerations are indications that delivery is considered higher-risk. D) Increasing the I V rate is not a nursing action that would change the status of the depressed fetus.

1) The client has been pushing for 2 hours and is exhausted. The physician is performing a vacuum extraction to assist the birth. Which finding is expected and normal? A) The head is delivered after eight "pop-offs" during contractions. B) A cephalohematoma is present on the fetal scalp. C) The location of the vacuum is apparent on the fetal scalp after birth. Positive pressure is applied by the vacuum extraction during contractions

Answer: C Explanation: A) Use of the vacuum extraction for eight contractions is too many. If more than three "pop-offs" occur, the procedure should be discontinued. B) A cephalohematoma is a complication of vacuum extraction birth and is not an expected finding. C) The parents need to be informed that the caput (chignon) on the baby's head will disappear within 2 to 3 days. D) Negative pressure is suction, which is needed to facilitate the birth.

1) The client at 30 weeks' gestation is admitted with painless late vaginal bleeding. The nurse understands that expectant management includes which of the following? A) Limiting vaginal exams to only one per 24-hour period. B) Evaluating the fetal heart rate with an internal monitor. C) Monitoring for blood loss, pain, and uterine contractibility. D) Assessing blood pressure every 2 hours.

Answer: C Explanation: A) Vaginal exams are contraindicated because the exam can stimulate bleeding. B) Fetal heart rate monitoring will be done with an external, not internal, fetal monitor. C) Blood loss, pain, and uterine contractibility need to be assessed for client comfort and safety. D) Blood pressure measurements every 2 hours are unnecessary.

1) If a woman has the pre-existing condition of diabetes, the nurse knows that she would be prone to what high-risk factor when pregnant? A) Vasospasm B) Postpartum hemorrhage C) Episodes of hypoglycemia and hyperglycemia D) Cerebrovascular accident (C V A)

Answer: C Explanation: A) Vasospasm would be a high-risk factor for a client with pre-existing cardiac disease. B) Postpartum hemorrhage would be a high-risk factor for a client with pre-existing hyperthyroidism. C) Episodes of hypoglycemia and hyperglycemia would be a high-risk factor for a client with pre-existing diabetes. D) Cerebrovascular accident (C V A) would be a high-risk factor for a client with pre-existing hypertension.

1) The nurse is observing a new graduate perform a postpartum assessment. Which action requires intervention by the nurse? A) Asking the client to void and donning clean gloves B) Listening to bowel sounds and then asking when her last bowel movement occurred C) Offering the patient pre-medication 2 hours before the assessment D) Completing the assessment and explaining the results to the client

Answer: C Explanation: A) Voiding prior to the assessment helps ensure comfort; clean gloves prevent exposure to body fluids. B) It is appropriate to ask about a body part when assessing that part. C) The patient should be offered premedication 30-45 minutes before assessing the fundus, especially if the patient has had a cesarean section. D) The nurse should provide an explanation of the purposes of regular assessment to the woman.

1) A woman asks her nurse what she can do before she begins trying to get pregnant to help her baby, as she is prone to anemia. What would the nurse correctly advise her to do? A) Get pregnant, then start iron supplementation. B) Add more carbohydrates to her diet. C) Begin taking folic acid supplements daily. D) Have a hemoglobin baseline done now so her progress can be followed.

Answer: C Explanation: A) Waiting until one is pregnant to add iron or evaluate a diet is not being proactive. B) Adding carbohydrates to her diet will not help prevent anemia. C) The common anemias of pregnancy are due to insufficient hemoglobin production related to nutritional deficiency in either iron or folic acid during pregnancy. Folic acid deficiency during pregnancy is prevented by a daily supplement of 0.4 m g (400 micrograms) of folate. D) Having a hemoglobin baseline done will not help the patient prevent anemia in pregnancy.

2) The pregnant client cannot tolerate milk or meat. What would the nurse recommend to the client to assist in meeting protein needs? A) Wheat bread and pasta B) Ice cream and peanut butter C) Eggs and tofu D) Beans and potatoes

Answer: C Explanation: A) Wheat bread and pasta are not sources of complete protein. B) Ice cream is a milk by-product, and would not be tolerated by this client. C) The best food choices that are nondairy and complete proteins alone are eggs and tofu. D) Beans and potatoes would not provide the client with adequate protein.

1) The nurse teaches the parents of an infant who was recently circumcised to observe for bleeding. What should the parents be taught to do if bleeding does occur? A) Wrap the diaper tightly. B) Clean with warm water with each diaper change. C) Apply gentle pressure to the site with gauze. D) Apply a new petroleum ointment gauze dressing.

Answer: C Explanation: A) When diapering, ensure that the diaper is neither too loose, which can cause rubbing with movement, nor too tight, which can cause pain. B) Cleaning the newborn with warm water with each diaper change is part of the care plan, but it does not prevent bleeding. C) If bleeding does occur, apply light pressure with a sterile gauze pad to stop the bleeding within a short time. If this is not effective, contact the physician immediately or take the newborn to the healthcare provider. D) Continued application of a petroleum ointment can help protect the granulation tissue that forms as the glans heals, but does not stop any bleeding.

1) The nurse is assessing an obese pregnant client during a routine prenatal visit. Which is the priority assessment for this client? A) Complete blood count (C B C) B) Basic metabolic panel (B M P) C) Blood pressure D) Fetal heart rate

Answer: C Explanation: A) While it is important to monitor the client's C B C to assess for anemia, this is not the priority assessment for this client. B) The B M P is not commonly monitored during pregnancy. A blood glucose level may be monitored, as the client's weight places her at risk for gestational diabetes mellitus. C) The blood pressure would be monitored closely at each prenatal visit due to the client's weight. Obese clients have a greater risk for gestational hypertension. This is the priority assessment for this client. D) While it is important to monitor the fetal heart rate during every prenatal visit, this is not the priority for this client.

1) What is the advantage of a client using a patient-controlled analgesia (P C A) following a cesarean birth? A) The client receives a bolus of the analgesia when pressing the button. B) The client experiences pain relief within 30 minutes. C) The client feels a greater sense of control, and is less dependent on the nursing staff. D) The client can deliver as many doses of the medication as needed.

Answer: C Explanation: A) With this approach, the woman is given a bolus of analgesia, often morphine, at the beginning of therapy and is not repeated. B) I V pain medications provide rapid pain relief. C) Using a special intravenous (I V) pump system, the woman presses a button to self-administer small doses of the medication as needed. For safety, the pump is preset with a time lockout so that the pump cannot deliver another dose until a specified time has elapsed. Women using P C A feel less anxious and have a greater sense of control with less dependence on the nursing staff. D) For safety, the pump is preset with a time lockout so that the pump cannot deliver another dose until a specified time has elapsed.

1) Which of the following drugs and drug categories can cause multiple fetal central nervous system (C N S), facial, and cardiovascular anomalies? A) Category C: Zidovudine B) Category B: Penicillin C) Category X: Isotretinoin D) Category A: Vitamin C

Answer: C Explanation: A) Zidovudine does not cause multiple fetal central nervous system (C N S), facial, and cardiovascular anomalies. B) Penicillin does not cause multiple fetal central nervous system (C N S), facial, and cardiovascular anomalies. C) Isotretinoin (Accutane), the acne medication, can cause multiple central nervous system (C N S), facial, and cardiovascular anomalies. D) Vitamin C does not cause multiple fetal central nervous system (C N S), facial, and cardiovascular anomalies.

A patient with osteoporosis wants a medication that does not need to be taken every day. What should the nurse expect to be prescribed for this patient? A) Teriparatide (Forteo) B) Alendronate (Fosamax) C) Zoledronic acid (Zometa®) D) Salmon calcitonin (Miacalcin®)

Answer: C Explanation: A) Zoledronic acid (Zometa®) is administered via Ⅳ once a year. Teriparatide (Forteo) necessitates a daily subcutaneous injection. Alendronate (Fosamax) is a daily oral medication. Salmon calcitonin (Miacalcin®) is to be taken daily as a nasal spray. B) Zoledronic acid (Zometa®) is administered via Ⅳ once a year. Teriparatide (Forteo) necessitates a daily subcutaneous injection. Alendronate (Fosamax) is a daily oral medication. Salmon calcitonin (Miacalcin®) is to be taken daily as a nasal spray. C) Zoledronic acid (Zometa®) is administered via Ⅳ once a year. Teriparatide (Forteo) necessitates a daily subcutaneous injection. Alendronate (Fosamax) is a daily oral medication. Salmon calcitonin (Miacalcin®) is to be taken daily as a nasal spray. D) Zoledronic acid (Zometa®) is administered via Ⅳ once a year. Teriparatide (Forteo) necessitates a daily subcutaneous injection. Alendronate (Fosamax) is a daily oral medication. Salmon calcitonin (Miacalcin®) is to be taken daily as a nasal spray.

1) A female patient seeks medical attention for vaginal discharge that at times is bright red but is mostly dark brown in color. When preparing materials to share with this patient, why should the nurse include this diagram? A) It shows the size of the uterus. B) It explains the location of the cervix. C) It portrays the vessels within the uterus. D) It demonstrates the location of the uterus.

Answer: C Explanation: C) A common sign of a hydatidiform mole is vaginal bleeding, often brownish (the characteristic "prune juice" appearance), but sometimes bright red. In this figure, some of the hydropic vessels are being passed. This occurrence is diagnostic for hydatidiform mole. This diagram is not being used because it shows the size of the uterus, the location of the cervix, or the location of the uterus.

1. The fetus of a patient in labor is determined to be in the brow presentation. Which diagram should the nurse provide to the patient to explain this position?

Answer: C Explanation: C) In the brow presentation, the fetal head is in partial (halfway) extension. The occipitomental diameter, which is the largest diameter of the fetal head, presents to the pelvis. Choice 1 is the vertex presentation. Complete flexion of the head allows the suboccipitobregmatic diameter to present to the pelvis. Choice 2 is the Sinciput (median vertex) prese ntation (also called military presentation) with no flexion or extension. The occipitofrontal diameter presents to the pelvis. Choice 4 is the face presentation. The fetal head is in complete extension, and the submentobregmatic diameter presents to the pelvis.

1. The cervix of a laboring patient is measured as being 50% effaced. Which diagram should the nurse use to explain this finding to the patient?

Answer: C Explanation: C) The cervix is about one half (50%) effaced and slightly dilated. The increasing amount of amniotic fluid below the fetal head exerts hydrostatic pressure on the cervix. Choice 1 is the beginning of labor, where there is no cervical effacement or dilation. The fetal head is cushioned by amniotic fluid. Choice 2 is the beginning of cervical effacement. As the cervix begins to efface, more amniotic fluid collects below the fetal head. Choice 4 is complete effacement and dilation.

1) The nurse is instructing a new mother on basic infant care. For which activities should the nurse suggest that the mother use the football hold? A) Burping B) Bottle feeding C) Shampooing hair D) Changing a diaper

Answer: C Explanation: C) The football hold frees one of the caregiver's hands and permits eye contact. This hold is ideal for shampooing, carrying, or breastfeeding. The cradle hold is frequently used during feeding. The upright position is ideal for burping. The baby should be supine for changing a diaper.

1) A 24-year-old patient with an intellectual disability at 30 weeks' gestation asks when it is safe to have an abortion. How should the nurse respond to this patient? A) "Have you been hiding your pregnancy?" B) "The safe time to end a pregnancy has passed." C) "Is it safe for me to assume that you don't want to have this baby?" D) "I guess you didn't realize that an abortion should have occurred months ago."

Answer: C Explanation: C) The nurse should explain about the safe time to end a pregnancy in a more appropriate way. The patient has an intellectual disability and may not understand when an abortion can be performed. Asking if the patient is hiding the pregnancy is inappropriate. Asking if the patient does not want to have the baby is appropriate if the patient is asking for information about an abortion. Saying that the patient didn't realize that an abortion should have occurred months ago would be inappropriate because the patient has an intellectual disability.

A patient in the 2nd trimester of pregnancy is diagnosed with bacterial vaginosis. Which medication regimen should the nurse expect to be prescribed for this patient? A) Metronidazole 500 m g orally one dose B) Metronidazole 250 m g orally once a day for 7 days C) Metronidazole 500 m g orally twice a day for 7 days D) Metronidazole 250 m g orally twice a day for 14 days

Answer: C Explanation: C) The recommended treatment of bacterial vaginosis during pregnancy is Metronidazole 500 m g orally twice a day for 7 days. One dose of metronidazole is not sufficient. Metronidazole 250 m g should be taken 3 times for 7 days to be effective. Metronidazole 250 m g does not need to be taken for 14 days.

1) An older female patient with a known intellectual disability is newly diagnosed with osteoporosis and admitted with a fractured hip after falling in the home. What should the nurse realize could have contributed to this patient's health problem? A) Importance of resting during the day B) Need to reduce the amount of physical activity C) Understanding home environmental safety needs D) Reducing the oral intake of protein and carbohydrates

Answer: C Explanation: C) Understanding home environmental safety needs could have been an issue with the patient with an intellectual disability. Resting, reducing physical activity, and altering the intake of protein and carbohydrates would not be beneficial for the patient with osteoporosis.

A patient is being instructed on adverse effects of gonorrhea. For which reason should the nurse instruct the patient to contact the healthcare provider? A) Dysuria B) Urinary frequency C) Sharp abdominal pain D) Purulent, greenish-yellow vaginal discharge

Answer: C Explanation: C) Women should be informed of signs that the infection is worsening, such as sharp abdominal pain, and be encouraged to seek further care. Dysuria, urinary frequency, and purulent, greenish-yellow vaginal discharge are manifestations of gonorrhea and do not indicate that the infection is getting worse.

1) A 38-year-old female is scheduled for a laparoscopic-assisted vaginal hysterectomy (L A V H) for severe endometriosis with the removal of both ovaries. What should the nurse expect to be prescribed for this patient postoperatively? A) Corticosteroid therapy B) Mineralocorticoid therapy C) Estrogen replacement therapy D) Progesterone replacement therapy

Answer: C Explanation: Supplemental estrogen replacement therapy is recommended in a premenopausal woman having both fallopian tubes and ovaries removed. Corticosteroid and mineralocorticoid therapies are indicated for adrenal gland dysfunction. Progesterone replacement therapy is not indicated for this patient's health problem

1) The nurse is preparing an educational seminar about the frequency of intimate partner violence against females. Using the chart below, which age group should the nurse identify as experiencing the most intimate partner violence in 2010? A) 12-17 B) 18-24 C) 25-34 D) 35-49

Answer: C Explanation: The group experiencing the most intimate partner violence against women in 2010 is the solid green line which represents the 25-34 age group. The solid red line is the 12-17 age groups. The dotted red line is the 18-24 age group. The dotted blue line is the 35-49 age group. And the solid orange line is the 50 or older age group.

6) The labor and birth nurse is admitting a client. The nurse's assessment includes asking the client whom she would like to have present for the labor and birth, and what the client would prefer to wear. The client's partner asks the nurse the reason for these questions. What would the nurse's best response be? Note: Credit will be given only if all correct choices and no incorrect choices are selected. Select all that apply. A) "These questions are asked of all women. It's no big deal." B) "I'd prefer that your partner ask me all the questions, not you." C) "A client's preferences for her birth are important for me to understand." D) "Many women have beliefs about childbearing that affect these choices." E) "I'm gathering information that the nurses will use after the birth."

Answer: C, D Explanation: A) Although this information is asked of all clients, it is purposefully gathered. B) It is not therapeutic communication to tell the partner not to ask questions. C) The nurse incorporates the family's expectations into the plan of care to be culturally appropriate and to facilitate the birth. D) The nurse incorporates the family's expectations into the plan of care to be culturally appropriate and to facilitate the birth. The information gathered will be used during the labor and birth, not after delivery

1) The nurse is preparing a class for mothers who have just recently delivered and their partners. One topic of the class is infant attachment. Which statement by a participant indicates an understanding of this concept? Note: Credit will be given only if all correct choices and no incorrect choices are selected. Select all that apply. A) "We should avoid holding the baby too much." B) "Looking directly into the baby's eyes might frighten him." C) "Talking to the baby is good because he'll recognize our voices." D) "Holding the baby so we have direct face-to-face contact is good." E) "We should only touch the baby with our fingertips for the first month."

Answer: C, D Explanation: A) Attachment behaviors include holding the baby in the en face position. B) Attachment behaviors include making eye contact. C) Attachment behaviors include cuddling, soothing, and calling the baby by name. D) Attachment behaviors include holding the baby in the en face position. E) In a progression of touching activities, the mother proceeds from fingertip exploration of the newborn's extremities toward palmar contact with larger body areas and finally to enfolding the infant with the whole hand and arms. The time taken to accomplish these steps varies from minutes to days.

1) A nurse is providing a client with instructions regarding breast self-examination (B S E). Which of the following statements by the client would indicate that the teaching has been successful? Note: Credit will be given only if all correct choices and no incorrect choices are selected. Select all that apply. A) "I should perform B S E 1 week prior to the start of my period." B) "When I reach menopause, I will perform B S E every 2 months." C) "Knowing the density of my breast tissue is important." D) "I should inspect my breasts while standing with my arms down at my sides." E) "I should inspect my breasts while in a supine position with my arms at my sides."

Answer: C, D Explanation: A) B S E should be performed 1 week after the start of each menstrual period. B) B S E should be performed monthly, on the same day each month, during menopause. C) The effectiveness of B S E is determined by the woman's ability to perform the procedure correctly, by her knowledge of her own breast tissue, and by the density of her breast tissue. D) The breasts should be inspected while standing with arms at sides. E) Supine is not a correct position for B S E.

1) Clinical risk factors for severe hyperbilirubinemia include which of the following? Note: Credit will be given only if all correct choices and no incorrect choices are selected. Select all that apply. A) African American ethnicity B) Female gender C) Cephalohematoma D) Bruising E) Assisted delivery with vacuum or forceps

Answer: C, D, E Explanation: A) A clinical risk factor for severe hyperbilirubinemia includes Asian ethnicity. B) A clinical risk factor for severe hyperbilirubinemia includes male gender. C) A clinical risk factor for severe hyperbilirubinemia includes cephalohematoma. D) A clinical risk factor for severe hyperbilirubinemia includes bruising. E) A clinical risk factor for severe hyperbilirubinemia includes assisted delivery with vacuum or forceps.

1) A standard ultrasound examination is performed during the second or third trimester and includes an evaluation of which of the following? Note: Credit will be given only if all correct choices and no incorrect choices are selected. Select all that apply. A) Confirm fetal heart activity. B) Evaluate the cervix. C) Determine fetal presentation. D) Assess amniotic fluid volume. E) Determine fetal number.

Answer: C, D, E Explanation: A) A limited ultrasound may be used to address a specific question or determine specific information including confirming fetal heart activity. B) A limited ultrasound may be used to address a specific question or determine specific information including evaluating the cervix. C) A standard ultrasound examination is performed during the second or third trimester and includes an evaluation to determine fetal presentation. D) A standard ultrasound examination is performed during the second or third trimester and includes an evaluation of amniotic fluid volume. E) A standard ultrasound examination is performed during the second or third trimester and includes an evaluation of fetal number.

1) True postterm pregnancies are frequently associated with placental changes that cause a decrease in uterine-placental-fetal circulation. Complications related to alterations in placenta functioning include which of the following? Note: Credit will be given only if all correct choices and no incorrect choices are selected. Select all that apply. A) Increased fetal oxygenation B) Increased placental blood supply C) Reduced nutritional supply D) Macrosomia E) Risk of shoulder dystocia

Answer: C, D, E Explanation: A) Decreased, not increased, fetal oxygenation is a complication related to alternations in placenta functioning. B) Reduced, not increased, placental blood supply is a complication related to alternations in placenta functioning. C) Reduced nutritional supply is a complication related to alternations in placenta functioning. D) Macrosomia is a complication related to alternations in placenta functioning. E) Risk of shoulder dystocia is a complication related to alternations in placenta functioning.

1. The nurse is caring for laboring clients. Which women are experiencing problems related to a critical factor of labor? Note: Credit will be given only if all correct choices and no incorrect choices are selected. Select all that apply. A) Woman at 7 c m, fetus in general flexion B) Woman at 3 c m, fetus in longitudinal lie C) Woman at 4 c m, fetus with transverse lie D) Woman at 6 c m, fetus at -2 station, mild contractions E) Woman at 5 c m, fetal presenting part is right shoulder

Answer: C, D, E Explanation: A) Fetal attitude refers to the relation of the fetal body parts to one another and describes the posture the fetus assumes as it conforms to the shape of the uterine cavity. The normal attitude of the fetus is termed general flexion, where the head is flexed so that the chin is on the chest with the arms crossed over the chest, and the legs are flexed at the knees with the thighs on the abdomen. B) Fetal lie refers to the relationship of the long, or cephalocaudal, axis (spinal column) of the fetus to the long, or cephalocaudal, axis of the mother. The fetus may assume either a longitudinal (vertical) or a transverse (horizontal) lie; a longitudinal lie is normal. C) A transverse lie occurs when the cephalocaudal axis of the fetal spine is at a right angle to the woman's spine and is associated with a shoulder presentation and can lead to complications in the later stages of labor. D) Station refers to the relationship of the presenting part to an imaginary line drawn between the ischial spines of the maternal pelvis. If the presenting part is higher than the ischial spines, a negative number is assigned, noting centimeters above zero station. A -2 station is high in the pelvis. Contractions should be strong to cause fetal descent. Mild contractions will not move the baby down or open the cervix. This client is experiencing a problem between the maternal pelvis and the presenting part. E) When the fetal shoulder is the presenting part, the fetus is in a transverse lie and the acromion process of the scapula is the landmark. This type of presentation occurs less than 1% of the time. This client is experiencing a problem between the maternal pelvis and the presenting part.

1) The nurse is working with parents who have just experienced the birth of their first child at 34 weeks. Which statement(s) by the parents indicate that additional teaching is needed? Note: Credit will be given only if all correct choices and no incorrect choices are selected. Select all that apply. A) "Our baby will be in an incubator to keep him warm." B) "Breathing might be harder for our baby because he is early." C) "The growth of our baby will be faster than if he were term." D) "Tube feedings will be required because his stomach is small." E) "Because he came early, he will not produce urine for 2 days."

Answer: C, D, E Explanation: A) Preterm infants have little subcutaneous fat, and have difficulty maintaining their body temperature. An incubator or warmer is used to keep the baby warm. B) Surfactant production might not be complete at 34 weeks, which leads to respiratory distress syndrome. The infant may become hypoxic, pulmonary blood flow may be inefficient, and the preterm newborn's available energy is depleted. C) Preterm infants grow more slowly than do term infants because of difficulty in meeting high caloric and fluid needs for growth due to small gastric capacity. D) Although tube feedings might be required, it would be because preterm babies have a marked danger of aspiration and its associated complications due to the infant's poorly developed gag reflex, incompetent esophageal cardiac sphincter, and inadequate suck/swallow/breathe reflex. E) Although preterm babies have diminished kidney function due to incomplete development of the glomeruli, they can produce urine. Preterm infants usually have some urine output during the first 24 hours of life.

1) The nurse has given a community presentation on adolescent pregnancy. Which statements indicate that further teaching is necessary? Note: Credit will be given only if all correct choices and no incorrect choices are selected. Select all that apply. A) "Low educational achievement is major risk factor for adolescent pregnancy." B) "Teens who are on a low economic trajectory are more likely to become pregnant." C) "When a teen plans to attend college, abortion is less likely." D) "In the United States, the adolescent birth rate is higher among white teens." E) "Teens from low-income households are less likely to become pregnant."

Answer: C, D, E Explanation: A) Teenage pregnancy is the leading reason why adolescent women drop out of school. An estimated 30% to 40% of female teenage dropouts are mothers. B) Teens who are on a low economic trajectory are more likely to become pregnant because of the lack of economic opportunity and the social marginalization that comes with poverty. C) Teens with future goals (i.e., college or job) tend to use birth control more consistently compared with other teens; if they become pregnant, they are also more likely to have abortions. D) In the United States, the adolescent birth rate is higher among African American and Hispanic teens than among white teens. E) Poverty is a major risk factor for adolescent pregnancy. As many as 85% of births to unmarried teens occur to those from low-income families.

1) The nurse is planning discharge teaching for a postpartum woman. What recommendations should the woman receive before being discharged? Note: Credit will be given only if all correct choices and no incorrect choices are selected. Select all that apply. A) To abstain from sexual intercourse for 6 months B) To avoid showers for 4 weeks C) To avoid overexertion D) To practice postpartum exercises E) To obtain adequate rest

Answer: C, D, E Explanation: A) The client should abstain from sexual intercourse until lochia has ceased. B) The client may take a shower and may continue sitz baths at home if she desires. C) The client should avoid overexertion. D) The client should receive information and instruction on postpartum exercises. E) The client should receive information on the need for adequate rest.

1) The nurse is meeting with a new mother for the first time during a home visit. The client delivered her first child 3 days ago. She had a normal pregnancy and a vaginal delivery. The infant is breastfeeding. Which statements by the mother indicate that she needs more information about the home visit? Note: Credit will be given only if all correct choices and no incorrect choices are selected. Select all that apply. A) "You are going to check my baby's weight." B) "You are going to watch me nurse the baby and give me tips." C) "You are going to teach my mother about the baby." D) "You are checking for safety issues when my son starts crawling." E) "You are going to take blood samples from me and my son."

Answer: C, D, E Explanation: A) The components of a postpartum home visit typically include weighing the infant. B) The components of a postpartum home visit typically include assessing a feeding, if possible. C) Teaching of family members might occur, but the main purpose of the visit is to assess the infant's physiologic stability. D) Safety when the infant crawls should be assessed later. E) Not all home visits require blood samples. If there were no pregnancy or birth complications, there may not be the need to draw blood from either the mother or the child.

1) What should the healthcare provider consider when prescribing a medication to a woman who is breastfeeding? Note: Credit will be given only if all correct choices and no incorrect choices are selected. Select all that apply. A) Drug's potential effect on hormone production B) Amount of drug excreted into the mother's blood C) Drug's potential adverse effects to the infant D) Infant's age and health E) Mother's need for the medication

Answer: C, D, E Explanation: A) The healthcare provider should consider the drug's potential effect on milk production. B) The healthcare provider should consider the amount of drug excreted into the milk. C) The healthcare provider should consider the drug's potential adverse effects to the infant. D) The healthcare provider should consider the infant's age and health. E) The healthcare provider should consider the mother's need for the medication.

1) Lacerations of the cervi × or vagina may be present when bright red vaginal bleeding persists in the presence of a well-contracted uterus. The incidence of lacerations is higher among which of the following childbearing women? Note: Credit will be given only if all correct choices and no incorrect choices are selected. Select all that apply. A) Over the age of 35 B) Have not had epidural block C) Have had an episiotomy D) Have had a forceps-assisted or vacuum-assisted birth E) Nulliparous

Answer: C, D, E Explanation: A) The incidence of lacerations is higher among childbearing women who are younger. B) The incidence of lacerations is higher among childbearing women who have had an epidural block. C) The incidence of lacerations is higher among childbearing women who undergo an episiotomy. D) The incidence of lacerations is higher among childbearing women who undergo forceps-assisted or vacuum-assisted birth. E) The incidence of lacerations is higher among childbearing women who are nulliparous.

A patient experiencing symptoms of menopause asks if there are any vitamin supplements she should take at this time. Which vitamins should the nurse suggest to this patient? Note: Credit will be given only if all correct choices and no incorrect choices are selected. Select all that apply. A) Vitamin A B) Vitamin C C) Vitamin D D) Vitamin E E) Vitamin B complex

Answer: C, D, E Explanation: A) Therapeutic modalities proposed as treatment or prevention measures for the discomforts and ailments of the perimenopausal and postmenopausal years include vitamins E, D, and B complex. Vitamins A and C have not been identified as being beneficial to reduce the symptoms associated with menopause. B) Therapeutic modalities proposed as treatment or prevention measures for the discomforts and ailments of the perimenopausal and postmenopausal years include vitamins E, D, and B complex. Vitamins A and C have not been identified as being beneficial to reduce the symptoms associated with menopause. C) Therapeutic modalities proposed as treatment or prevention measures for the discomforts and ailments of the perimenopausal and postmenopausal years include vitamins E, D, and B complex. Vitamins A and C have not been identified as being beneficial to reduce the symptoms associated with menopause. D) Therapeutic modalities proposed as treatment or prevention measures for the discomforts and ailments of the perimenopausal and postmenopausal years include vitamins E, D, and B complex. Vitamins A and C have not been identified as being beneficial to reduce the symptoms associated with menopause. E) Therapeutic modalities proposed as treatment or prevention measures for the discomforts and ailments of the perimenopausal and postmenopausal years include vitamins E, D, and B complex. Vitamins A and C have not been identified as being beneficial to reduce the symptoms associated with menopause.

28) An abbreviated systematic physical assessment of the newborn is performed by the nurse in the birthing area to detect any abnormalities. Normal findings would include which of the following? Note: Credit will be given only if all correct choices and no incorrect choices are selected. Select all that apply. A) Skin color: Body blue with pinkish extremities B) Umbilical cord: two veins and one artery C) Respiration rate of 30-60 irregular D) Temperature of above 36.5°C (97.8°F) E) Sole creases that involve the heel

Answer: C, D, E Explanation: A) This is not correct. Skin color would be body pink with bluish extremities. B) This is not correct. The umbilical cord would have two arteries and one vein. C) Normal findings would include a respiration rate of 30-60 irregular. D) Normal findings would include temperature of above 36.5°C (97.8°F). E) Normal findings would include sole creases that involve the heel.

1) Which couples may benefit from prenatal diagnosis? Note: Credit will be given only if all correct choices and no incorrect choices are selected. Select all that apply. A) Couples including women under the age of 35 B) Couples with an unbalanced translocation C) Couples with a family history of a known or suspected single-gene disorder D) Couples including women with a teratogenic risk secondary to an exposure or maternal health condition E) Family history of birth defects and/or intellectual disability

Answer: C, D, E Explanation: A) Women age 35 or over at time of birth may benefit from prenatal diagnosis. B) Couples with a balanced translocation (chromosomal abnormality) may benefit from prenatal diagnosis. C) Couples with a family history of known or suspected single-gene disorder (e.g., cystic fibrosis, hemophilia A or B, Duchenne muscular dystrophy) may benefit from prenatal diagnosis. D) Women with a teratogenic risk secondary to an exposure or maternal health condition (e.g., diabetes, seizure disorder) may benefit from prenatal diagnosis. E) Family history of birth defects and/or intellectual disability (mental retardation) (e.g., neural tube defects, congenital heart disease, cleft lip and/or palate) may benefit from prenatal diagnosis.

1) The nurse takes a telephone call from a women's health clinic patient. What information should cause the nurse to suspect that the patient is experiencing a cystocele? Note: Credit will be given only if all correct choices and no incorrect choices are selected. Select all that apply. A) Bloody urine B) Low back pain C) Onset of stress incontinence D) Feeling of fullness in the perineum E) Feels like something "fell out" of the vagina

Answer: C, D, E Explanation: Manifestations of a cystocele include symptoms of stress urinary incontinence (SUI), pelvic pressure and the perception of something "falling out" of the vagina. Hematuria and low back pain are not manifestations of a cystocele

1) The nurse is assessing the newborn for symptoms of anemia. If the blood loss is acute, the baby may exhibit which of the following signs of shock? Note: Credit will be given only if all correct choices and no incorrect choices are selected. Select all that apply. A) Increased pulse B) High blood pressure C) Tachycardia D) Bradycardia Capillary filling time greater than 3 seconds

Answer: C, E Explanation: A) Decreased pulse would be a sign of shock. B) Low blood pressure would be a sign of shock. C) Tachycardia would be a sign of shock. D) Tachycardia, not bradycardia, would be a sign of shock. Capillary filling time greater than 3 seconds would be a sign of shock

1) The nurse is explaining the difference between meiosis and mitosis. Which statements would be best? Note: Credit will be given only if all correct choices and no incorrect choices are selected. Select all that apply. A) Meiosis is the division of a cell into two exact copies of the original cell. B) Mitosis is splitting one cell into two, each with half the chromosomes of the original cell. C) Meiosis is a type of cell division by which gametes, or the sperm and ova, reproduce. D) Mitosis occurs in only a few cells of the body. Meiotic division leads to cells that halve the original genetic material

Answer: C, E Explanation: A) Meiosis creates two cells that have half of the chromosomes of the original cell. B) Mitosis creates two cells that are exact copies of the original cell. C) Meiosis is a special type of cell division by which diploid cells give rise to gametes (sperm and ova). D) Mitosis makes growth and development possible. In mature individuals it is the process by which our body cells continue to divide and replace themselves. E) Meiosis creates two cells that contain half the genetic material of the parent cell.

1) The client is at 42 weeks' gestation. Which order should the nurse question? A) Obtain biophysical profile today. B) Begin nonstress test now. C) Schedule labor induction for tomorrow. D) Have the client return to the clinic in 1 week.

Answer: D Explanation: A) A biophysical profile is a commonly used assessment for the postterm fetus. B) The nonstress test is a commonly used assessment for the postterm fetus. C) Most practitioners consider induction at 41 gestational weeks to reduce maternal and fetal-newborn risks associated with postterm pregnancy. D) Many practitioners use twice-weekly testing, providing the amniotic fluid level is normal. One week is too long a period between assessments.

1) The nurse is completing discharge teaching for a client who delivered 2 days ago. Which statement by the client indicates that further information is required? A) "Because I have a midline episiotomy, I should keep my perineum clean." B) "I can use an ice pack to relieve some the pain from the episiotomy." C) "I can take ibuprofen (Motrin) when my perineum starts to hurt." D) "The tear I have through my rectum is unrelated to my episiotomy."

Answer: D Explanation: A) A complication associated with an episiotomy is infection. Perineal hygiene is important when a client has an episiotomy to prevent infection and facilitate healing. B) Pain relief measures may begin immediately after birth with application of an ice pack to the perineum. C) Healing episiotomies can be very painful, and pain medication should be provided for clients experiencing pain. D) This statement is incorrect. The major disadvantage is that a tear of the midline incision may extend through the anal sphincter and rectum.

1) The postpartum home care nurse is assessing a new mother, and finds her temperature to be 101.6°F. What is the most important nursing action? A) Ask the mother how often and how well the baby is nursing. B) Determine the frequency of the mother's voiding and stooling. C) Verify how many hours of sleep she is getting per day. D) Assess the odor and color of the lochia and perineum.

Answer: D Explanation: A) A fever might indicate mastitis. Palpation of the breasts for warm or hardened areas is much better than asking about feedings because mothers of good feeders can develop mastitis. B) If she is voiding frequently, she might have a urinary tract infection (U T I), but the frequency of bowel movements is not related to a U T I. C) Although it is common for new mothers to be fatigued, fatigue does not cause fever. D) If the lochia is malodorous, or if the perineum is reddened or malodorous, an infection is present that could be causing the fever.

1) During a postpartum examination of a client who delivered an 8-pound newborn 6 hours ago, the following assessment findings are noted: fundus firm and at the umbilicus, and moderate lochia rubra with a steady trickle of blood from the vagina. What is the assessment finding that would necessitate follow-up? A) Firm fundus B) Fundus at the umbilical level C) Moderate lochia rubra D) Steady trickle of blood

Answer: D Explanation: A) A firm fundus is desirable, and considered normal. B) Six hours after birth, the fundus at the umbilicus would not be a concern. C) Lochia rubra would be considered normal. D) The continuous seepage of blood is more consistent with cervical or vaginal lacerations. Lacerations should be suspected if the uterus is firm and of expected size, and no clots can be expressed. This finding would indicate a follow-up.

1) The client has delivered a 4200 g fetus. The physician performed a midline episiotomy, which extended into a third-degree laceration. The client asks the nurse where she tore. Which response is best? A) "The episiotomy extended and tore through your rectal mucosa." B) "The episiotomy extended and tore up near your vaginal mucous membrane." C) "The episiotomy extended and tore into the muscle layer." D) "The episiotomy extended and tore through your anal sphincter."

Answer: D Explanation: A) A fourth degree laceration is through the rectal mucosa. B) A first degree laceration is through the vaginal mucous membrane. C) A second degree laceration involves skin and muscle. D) A third degree laceration includes the anal sphincter.

1) The nurse assesses the newborn and notes the following behaviors: nasal flaring, facial grimacing, and excessive mucus. What is the nurse most concerned about? A) Neonatal jaundice B) Neonatal hypothermia C) Neonatal hyperthermia D) Respiratory distress

Answer: D Explanation: A) A high bilirubin level would be an indication of jaundice. B) Temperature instability would indicate either hyperthermia or hypothermia. C) A high temperature would indicate hyperthermia. D) Nasal flaring and facial grimacing are signs of respiratory distress.

1) A couple asks the nurse what is the safest method of sterilization. What should the nurse reply? A) "Laparotomy tubal ligation." B) "Laparoscopy tubal ligation." C) "Minilaparotomy." D) "Vasectomy."

Answer: D Explanation: A) A laparotomy tubal ligation is a female sterilization procedure that involves more risks. B) A laparoscopy tubal ligation is a female sterilization procedure that involves more risks. C) Minilaparotomy is a female sterilization procedure that involves more risks. D) Vasectomy (male sterilization) is a relatively minor procedure.

1. During the fourth stage of labor, the client's assessment includes a B P of 110/60, pulse 90, and the fundus is firm midline and halfway between the symphysis pubis and the umbilicus. What is the priority action of the nurse? A) Turn the client onto her left side. B) Place the bed in Trendelenburg position. C) Massage the fundus. D) Continue to monitor.

Answer: D Explanation: A) A left lateral position is not necessary with a B P of 110/60 and a pulse of 90. B) The Trendelenburg position is not necessary with a B P of 110/60 and a pulse of 90. C) The uterus should be midline and firm; massage is not necessary. D) The client's assessment data are normal for the fourth stage of labor, so monitoring is the only action necessary. During the fourth stage of labor, the mother experiences a moderate drop in both systolic and diastolic blood pressure, increased pulse pressure, and moderate tachycardia.

1) The clinic nurse is compiling data for a yearly report. Which client would be classified as a primigravida? A) A client at 18 weeks' gestation who had a spontaneous loss at 12 weeks B) A client at 13 weeks' gestation who had an ectopic pregnancy at 8 weeks C) A client at 14 weeks' gestation who has a 3-year-old daughter at home D) A client at 15 weeks' gestation who has never been pregnant before

Answer: D Explanation: A) A pregnant woman who has been pregnant before is called a multigravida. B) A pregnant woman who has been pregnant before is called a multigravida. C) A pregnant woman who has been pregnant before is called a multigravida. D) Primigravida means a woman who is pregnant for the first time.

The registered nurse who has completed a master's degree program and passed a national certification exam has clinic appointments with clients who are pregnant or seeking well-woman care. What is the role of this nurse considered to be? A) Professional nurse B) Certified registered nurse (R N C) C) Clinical nurse specialist D) Nurse practitioner

Answer: D Explanation: A) A professional nurse is one who has completed an accredited basic educational program and has passed the N C L E X-R N® exam. B) A certified registered nurse (R N C) has shown expertise in the field and has taken a national certification exam. C) A clinical nurse specialist has completed a master's degree program, has specialized knowledge and competence in a specific clinical area, and often is employed in hospitals on specialized units. D) A nurse practitioner has completed either a master's or doctoral degree in nursing and passed a certification exam, and functions as an advanced practice nurse. Ambulatory care settings and the community are common sites for nurse practitioners to provide client care.

1) Which of the following is a sign of dehydration in the newborn? A) Slow, weak pulse B) Soft, loose stools C) Light colored, concentrated urine D) Depressed fontanelles

Answer: D Explanation: A) A rapid, weak pulse is a sign of dehydration in the newborn. B) Dry, hard stools are a sign of dehydration in the newborn. C) Dark, concentrated urine is a sign of dehydration in the newborn. D) Depressed fontanelles are a sign of dehydration in the newborn.

A nurse is examining different nursing roles. Which example best illustrates an advanced practice nursing role? A) A registered nurse who is the manager of a large obstetrical unit B) A registered nurse who is the circulating nurse during surgical deliveries (cesarean sections) C) A clinical nurse specialist working as a staff nurse on a mother-baby unit D) A clinical nurse specialist with whom other nurses consult for her expertise in caring for high-risk infants

Answer: D Explanation: A) A registered nurse who is the manager of a large obstetrical unit is a professional nurse who has graduated from an accredited program in nursing and completed the licensure examination. B) A registered nurse who is a circulating nurse at surgical deliveries (cesarean sections) is a professional nurse who has graduated from an accredited program in nursing and completed the licensure examination. C) A clinical nurse specialist working as a staff nurse on a mother-baby unit might have the qualifications for an advanced practice nursing staff member but is not working in that capacity. A clinical nurse specialist with whom other nurses consult for expertise in caring for high-risk infants is working in an advanced practice nursing role. This nurse has specialized knowledge and competence in a specific clinical area, and is master's prepared

1) Which of the following is common in many non-Western cultures and is on the increase in the United States? A) Ceremonial rituals and rites B) Cultural assessment C) Cultural values D) Co-sleeping

Answer: D Explanation: A) A universal tendency exists to create ceremonial rituals and rites around important life events. B) Healthcare professionals are becoming increasingly aware of the importance of addressing cultural, physiologic, and psychologic needs in the prenatal assessment in order to provide culture-specific healthcare during pregnancy. C) Identification of cultural values is useful in planning and providing culturally sensitive care. D) Some parents advocate cosleeping or bed sharing (one or both parents sleeping with their baby or young child). Cosleeping, which is common in many non-Western cultures, is on the increase in the United States.

The nurse manager is planning a presentation on ethical issues in caring for childbearing families. Which example should the nurse manager include to illustrate maternal-fetal conflict? A) A client chooses an abortion after her fetus is diagnosed with a genetic anomaly. B) A 39-year-old nulliparous client undergoes therapeutic insemination. C) A family of a child with leukemia requests cord-blood banking at a sibling's birth. D) A cesarean delivery of a breech fetus is court ordered after the client refuses

Answer: D Explanation: A) Abortion is a different type of ethical situation. B) Achieving pregnancy through the use of therapeutic insemination is a form of reproductive assistance, and is not considered a maternal-fetal conflict. C) Cord-blood banking is a different type of ethical situation. D) Maternal-fetal conflict is a special ethical situation where the rights of the fetus and the rights of the mother are considered separately. Forced cesarean birth, coercion of mothers who practice high-risk behaviors, and, perhaps most controversial, mandating experimental in utero therapy or surgery in an attempt to correct a specific birth defect are interventions that infringe on the mother's autonomy.

1) A patient is demonstrating manifestations of acute cervicitis. Which laboratory test should the nurse expect to be completed for this patient? A) Sedimentation rate B) Blood test for V D R L C) White blood cell count D) Vaginal smear for S T Is

Answer: D Explanation: A) Acute inflammation of the cervix is usually the result of infection from Neisseria gonorrhoeae or Chlamydia trachomatis. A vaginal smear for S T Is will most likely be done. Sedimentation rate, blood test for V D R L, and white blood cell count are not indicated in the diagnosis of acute cervicitis. B) Acute inflammation of the cervix is usually the result of infection from Neisseria gonorrhoeae or Chlamydia trachomatis. A vaginal smear for S T Is will most likely be done. Sedimentation rate, blood test for V D R L, and white blood cell count are not indicated in the diagnosis of acute cervicitis. C) Acute inflammation of the cervix is usually the result of infection from Neisseria gonorrhoeae or Chlamydia trachomatis. A vaginal smear for S T Is will most likely be done. Sedimentation rate, blood test for V D R L, and white blood cell count are not indicated in the diagnosis of acute cervicitis. Acute inflammation of the cervix is usually the result of infection from Neisseria gonorrhoeae or Chlamydia trachomatis. A vaginal smear for STIs will most likely be done. Sedimentation rate, blood test for VDRL, and white blood cell count are not indicated in the diagnosis of acute cervicitis

1) The nurse is providing care to several pregnant clients at an O B-G Y N clinic. Which client might benefit from prenatal diagnostic testing? A) Paternal age of 35 years B) Maternal age of 30 years C) Family history of anxiety D) Family history of cystic fibrosis

Answer: D Explanation: A) Advanced paternal age is not a reason for prenatal diagnostic testing. B) Advanced maternal age is a reason for prenatal diagnostic testing; however, advanced maternal age is considered at the age of 35 years, not 30 years. C) A family history of anxiety is not a reason for prenatal diagnostic testing. D) Cystic fibrosis is an example of a single-gene disorder. This client would benefit from prenatal diagnostic testing.

1) A clinic nurse is planning when to administer R h immune globulin (R h o G A M) to an R h-negative pregnant client. When should the first dose of R h o G A M be administered? A) After the birth of the infant B) 1 month postpartum C) During labor D) At 28 weeks' gestation

Answer: D Explanation: A) After birth would be too late for the first dose of R h o G A M if transplacental hemorrhage, which is possible during pregnancy, has occurred. B) One month postpartum would be too late for the first dose of R h o G A M if transplacental hemorrhage, which is possible during pregnancy, has occurred. C) During labor would be too late for the first dose of R h o G A M if transplacental hemorrhage, which is possible during pregnancy, has occurred. D) When the woman is R h negative and not sensitized and the father is R h positive or unknown, R h immune globulin is given prophylactically at 28 weeks' gestation.

1) A 26-year-old client is 26 weeks pregnant. Her previous births include two large-for-gestational-age babies and one unexplained stillbirth. Which tests would the nurse anticipate as being most definitive in diagnosing gestational diabetes? A) A 50g, 1-hour glucose screening test B) A single fasting glucose level C) A 100g, 1-hour glucose tolerance test D) A 100g, 3-hour glucose tolerance test

Answer: D Explanation: A) All women get the initial 50 g of glucose and a 1-hour screening. B) A single fasting glucose level is not an adequate indicator of the glucose level in relation to food. C) The 100g, 1-hour tolerance test is not an adequate indicator of the glucose level in relation to food. D) Gestational diabetes is diagnosed if two or more of the following values are met or exceeded after taking the 100 g, 3-hour O G T T: Fasting: 95 m g/d L; 1 hour: 180 m g/d L; 2 hours: 155 m g/d L; 3 hours: 140 m g/d L.

1) The nurse is performing a postpartum assessment on a newly delivered client. When checking the fundus, there is a gush of blood. The client asks why that is happening. What is the nurse's best response? A) "We see this from time to time. It's not a big deal." B) "The gush is an indication that your fundus isn't contracting." C) "Don't worry. I'll make sure everything is fine." D) "Blood pooled in the vagina while you were in bed."

Answer: D Explanation: A) Although a gush of blood during fundus assessment is fairly common, this response is not therapeutic, because it does not answer the client's question. B) The fundus might be contracting well. The gush is from pooled lochia in the vagina. C) This response is not therapeutic because it focuses on the nurse and has a "don't worry" aspect that most clients find demeaning. D) A gush of blood when a fundal massage is undertaken may occur because of normal pooling of blood in vagina when the woman lies down to rest or sleep.

1) The nurse is caring for a client in active labor. The membranes spontaneously rupture, with a large amount of clear amniotic fluid. Which nursing action is most important to undertake at this time? A) Assess the odor of the amniotic fluid. B) Perform Leopold maneuvers. C) Obtain an order for pain medication. D) Complete a sterile vaginal exam.

Answer: D Explanation: A) Although it is important to assess amniotic fluid for odors, checking the cervi × to assess for cord prolapse is a higher priority. B) This assessment is not called for at this time. C) Pain medication is a low priority at this time. D) Checking the cervi × will determine whether the cord prolapsed when the membranes ruptured. The nurse would assess for prolapsed cord via vaginal examination.

1) The nurse is working with an adolescent mother who tells the nurse, "I'm really scared that I won't take care of my baby correctly. My mother says I'll probably hurt the baby because I'm too young to be a mother." What is the best response by the nurse? A) "You are very young, and parenting will be a challenge for you." B) "Your mother was probably right. Be very careful with your baby." C) "Mothers have instincts that kick in when they get their babies home." D) "We can give the baby a bath together. I'll help you learn how to do it."

Answer: D Explanation: A) Although this statement is true, it does not teach the client anything, or increase her confidence in being able to care for her infant. B) This statement is very judgmental, and does not teach the client anything, or increase her confidence in being able to care for her infant. C) Maternal instincts might indeed exist, but this client has expressed a specific fear about being a safe mother. It is best to work with her to teach her skills and increase her confidence. D) This response is best because bathing the newborn offers an excellent opportunity for teaching and welcoming parent involvement in the care of their baby.

1. A client had a cesarean birth 3 days ago. She has tenderness, localized heat, and redness of the left leg. She is afebrile. As a result of these symptoms, what would the nurse anticipate would be the next course of action? A) That the client would be encouraged to ambulate freely B) That the client would be given aspirin 650 m g by mouth C) That the client would be given Methergine I M D) That the client would be placed on bed rest

Answer: D Explanation: A) Ambulation would increase the inflammation. B) Aspirin 650 m g by mouth has anticoagulant properties, but usually is not necessary unless complications occur. C) Methergine is given only for postpartum hemorrhage, and would only cause vasoconstriction of an already inflamed vessel. D) These symptoms indicate the presence of superficial thrombophlebitis. The treatment involves bed rest, elevation of the affected limb, analgesics, and use of elastic support hose.

1) The nurse is caring for a new breastfeeding mother who is from Pakistan. The nurse plans her care so that the newborn is offered the breast on which of the following? A) Day of birth B) First day after birth C) Second day after birth D) Third to fourth day after birth

Answer: D Explanation: A) Among some traditional cultures around the world, it is believed that colostrum is "unclean" or even harmful to a newborn. Because of this ancient belief, mothers living the Middle East and parts of Asia even today discard their colostrum or wait 2 to 4 days to begin breastfeeding, when their "true milk" arrives. This mother would not start breastfeeding the day of the birth. B) Among some traditional cultures around the world, it is believed that colostrum is "unclean" or even harmful to a newborn. Because of this ancient belief, mothers living the Middle East and parts of Asia even today discard their colostrum or wait 2 to 4 days to begin breastfeeding, when their "true milk" arrives. This mother would not start breastfeeding the first day after the birth. C) Among some traditional cultures around the world, it is believed that colostrum is "unclean" or even harmful to a newborn. Because of this ancient belief, mothers living the Middle East and parts of Asia even today discard their colostrum or wait 2 to 4 days to begin breastfeeding, when their "true milk" arrives. This mother would not start breastfeeding the second day after the birth. D) Among some traditional cultures around the world, it is believed that colostrum is "unclean" or even harmful to a newborn. Because of this ancient belief, mothers living the Middle East and parts of Asia even today discard their colostrum or wait 2 to 4 days to begin breastfeeding, when their "true milk" arrives. This mother would begin breastfeeding on the third or fourth day after the birth.

1) The nurse is analyzing assessment findings on four newborns. Which finding might suggest a congenital heart defect? A) Apical heart rate of 140 beats per minute B) Respiratory rate of 40 C) Temperature of 36.5°C D) Visible, blue discoloration of the skin

Answer: D Explanation: A) An apical heart rate of 140 is a normal assessment finding for newborns. B) A respiratory rate of 40 is a normal assessment finding for newborns. C) Temperature of 36.5°C is a normal assessment finding for newborns. D) Central cyanosis is defined as a visible, blue discoloration of the skin caused by decreased oxygen saturation levels and is a common manifestation of a cardiac defect.

1) After inserting prostaglandin gel for cervical ripening, what should the nurse do? A) Apply an internal fetal monitor. B) Insert an indwelling catheter. C) Withhold oral intake and start intravenous fluids. D) Place the client in a supine position with a right hip wedge.

Answer: D Explanation: A) An internal fetal monitor cannot be applied until adequate cervical dilatation has occurred. B) The client should void on her own and not need a catheter. C) Until labor begins, there is no rationale for withholding all intake. D) After the gel, intravaginal insert, or tablet is inserted, the woman is instructed to remain lying down with a rolled blanket or hip wedge under her right hip to tip the uterus slightly to the left for the first 30 to 60 minutes to maintain the cervical ripening agent in place.

1) The nurse is assessing a newly pregnant client. Which finding does the nurse note as a normal psychosocial adjustment in this client's first trimester? A) An unlisted telephone number B) Reluctance to tell the partner of the pregnancy C) Parental disapproval of the woman's partner D) Ambivalence about the pregnancy

Answer: D Explanation: A) An unlisted telephone number does not indicate psychosocial adjustment. B) Reluctance to tell the partner about the pregnancy might indicate that the client anticipates disapproval, and is not a normal psychosocial adjustment. C) Parental disapproval of the client's partner does not indicate psychosocial adjustment. D) Ambivalence toward a pregnancy is a common psychosocial adjustment in early pregnancy.

1) The multiparous client at term has arrived to the labor and delivery unit in active labor with intact membranes. Leopold maneuvers indicate the fetus is in a transverse lie with a shoulder presentation. Which physician order is most important? A) Artificially rupture membranes. B) Apply internal fetal scalp electrode. C) Monitor maternal blood pressure every 15 minutes. D) Alert surgical team of urgent cesarean.

Answer: D Explanation: A) Artificial rupture of the membranes is contraindicated with a transverse lie because of the high risk for prolapsed cord. B) An internal fetal scalp electrode cannot be applied until membranes have ruptured. C) The fetus is at risk for hypoxia secondary to prolapsed cord if the membranes rupture. The maternal blood pressure is less important than getting the cesarean under way. D) This is the highest priority because vaginal birth is impossible with a transverse lie. Labor should not be allowed to continue, and a cesarean birth should be done quickly.

1) The nurse is preparing to assess the sexual history of a 35-year-old female patient. Which approach should the nurse first use to facilitate this data collection? A) Ask if the patient is sexually active B) Review the present method of birth control C) Determine the patient's number of children D) Talk about the patient's medical-surgical history

Answer: D Explanation: A) Asking if the patient is sexually active, reviewing the present method of birth control, and determining the patient's number of children are all intimate areas that should not be used to start a sexual history. B) Asking if the patient is sexually active, reviewing the present method of birth control, and determining the patient's number of children are all intimate areas that should not be used to start a sexual history. C) Asking if the patient is sexually active, reviewing the present method of birth control, and determining the patient's number of children are all intimate areas that should not be used to start a sexual history. D) When taking a history, the interview should start with less intimate areas, such as medical and surgical history, and then proceed to the sexual history toward the end of the history-taking session. This approach helps the woman develop a comfort level with the nurse before disclosing personal information.

1) The client with limited English language skills has a black eye and bruises across her face and arms. The client's husband has been acting as an interpreter for her, and answers all of the questions the nurse asks, often without talking to his wife first. The nurse suspects the client has been a victim of domestic abuse. What should the nurse do next? A) Ask the husband whether he has beaten his wife. B) Ask the husband to have a female friend come in with his wife. C) Provide written materials in English for the client to read at home. D) Ask the husband to step out of the room, and obtain an interpreter.

Answer: D Explanation: A) Asking the abuser whether he has abused his spouse is useless, as most abusers see their behavior as appropriate. B) Asking the husband to have a female friend come with his wife is not the best action for the nurse to take next. C) Written proficiency develops after verbal fluency; therefore, written materials in English are inappropriate for this client. D) Screening for women experiencing domestic violence must be done privately. An interpreter should also be provided as necessary.

When teaching a culturally diverse group of childbearing families about hospital birthing options, the culturally competent nurse does which of the following? A) Understands that the families have the same values as the nurse B) Teaches the families how childbearing takes place in the United States C) Insists that the clients answer questions instead of their husbands D) Incorporates the specific beliefs of the cultural groups that are attending the class

Answer: D Explanation: A) Assuming that the families have the same values as the nurse is ethnocentrism. B) Although it is important to explain health care during pregnancy and childbearing, this is not the top priority. C) The husband's answering questions might be a cultural norm, and insisting that the client answer could decrease the family's trust in the healthcare system. D) Providing culturally competent care involves recognizing the importance of the childbearing family's value system, acknowledging that differences occur among people, and respecting and responding to ethnic diversity in a way that leads to mutually desirable outcomes.

1) The primigravida at 22 weeks' gestation has a fundal height palpated slightly below the umbilicus. Which of the following statements would best describe to the client why she needs to be seen by a physician today? A) "Your baby is growing too much and getting too big." B) "Your uterus might have an abnormal shape." C) "The position of your baby can't be felt." D) "Your baby might not be growing enough."

Answer: D Explanation: A) At 22 weeks' gestation, the fundal height should be at about 22 c m. B) Uterine shape can be assessed only with diagnostic imaging techniques such as ultrasound or C T scan. C) The position of the baby is not noted until 36 weeks' gestation. D) The fundal height at 20-22 weeks should be about even with the umbilicus. At 22 weeks' gestation, a fundal height below the umbilicus and a uterine size that is inconsistent with length of gestation could indicate fetal demise.

1) The labor and delivery nurse is caring for a laboring client who has asked for a priest to visit her during labor. The client's mother died during childbirth, and although there were no complications during her pregnancy, the client is fearful of her own death during labor. What would be the best way for the nurse to respond? A) "Nothing is going to happen to you. We'll take very good care of you during your birth." B) "Would you like to have an epidural so that you won't feel the pain of the contractions?" C) "The priest won't be able to prevent complications, and might get in the way of your providers." D) "Would you like me to contact someone from your parish or our hospital chaplain to come see you?"

Answer: D Explanation: A) Avoid statements of reassurance, as there are no guarantees of outcome during health care. Using such statements shuts down effective communication because the client's concern is downplayed. B) The client's expressed concern is not about pain; it is a fear of death and a desire to see a priest. The nurse should address the client's concern directly. C) Although this statement is true, it is not therapeutic. It downplays the client's concerns, and will shut down effective communication. The nurse should address the concerns the client expresses. Providing spiritually sensitive care involves determining the current spiritual and religious beliefs and practices that will affect the mother and baby and accommodating these practices where possible

1) Nurses should educate parents about which of the following A A P recommendations to promote a safe sleep environment and decrease the risk of S I D S and S U I D in infants less than 12 months of age? A) Babies should not be offered a pacifier while falling asleep. B) Babies should be bottle-fed unless contraindicated. C) Babies should be under many covers when sleeping to keep them warm. D) Babies should have "tummy time" when they are awake.

Answer: D Explanation: A) Babies should be offered a pacifier while falling asleep. B) Babies should be breastfed unless contraindicated. Exclusive breastfeeding for at least 6 months is ideal, but any amount is better than none. C) Babies should not be overheated by the use of too many covers during sleep. D) Babies should have "tummy time" when they are awake and observed by an adult to prevent positional plagiocephaly and to promote motor development.

1) A 21-year-old woman is at 12 weeks' gestation with her first baby. She has cardiac disease, class Ⅲ, as a result of having had childhood rheumatic fever. Which planned activity would indicate to the nurse that the client needs further teaching? A) "I will be sure to take a rest period every afternoon." B) "I would like to take childbirth education classes in my last trimester." C) "I will have to cancel our trip to Disney World." D) "I am going to start my classes in water aerobics next week."

Answer: D Explanation: A) Because the heart disease is class Ⅲ, this client is encouraged to get adequate rest. B) Childbirth classes would be helpful for the client as long as she were careful not to overexert herself. C) Travel during the pregnancy would be based upon the tolerance of the client. However, a trip to Disney World would involve a large amount of activity, which would stress the pregnancy. D) With the slightest exertion, the client's heart rate will rise, and she will become symptomatic. Therefore, she should not establish a new exercise program.

1. The midwife performs a vaginal exam and determines that the fetal head is at a -2 station. What does this indicate to the nurse about the birth? A) The birth is imminent. B) The birth is likely to occur in 1-2 hours. C) The birth will occur later in the shift. D) The birth is difficult to predict.

Answer: D Explanation: A) Birth is not imminent at this time. B) Birth will not likely occur in 1-2 hours. C) Birth cannot really be predicted at this time. D) A -2 station means that the fetus is 2 c m above the ischial spines. The ischial spines as a landmark have been designated as zero station. If the presenting part is higher than the ischial spines, a negative number is assigned, noting centimeters above zero station. With the fetus's head that high in the pelvis, it is difficult to predict when birth will occur.

The nurse is identifying complementary and alternative therapies for a patient with a history of liver disorders who is experiencing symptoms of menopause. Which herbal supplement should this patient be counseled to avoid? A) Ginger B) Ginseng C) Red clover D) Black cohosh

Answer: D Explanation: A) Black cohosh has been associated with liver inflammation and disease. Ginger is useful to control nausea and vomiting. Ginseng helps with mood symptoms and sleep disorders. Red clover helps with hot flashes. Ginger, ginseng, and red clover are not associated with liver disease. B) Black cohosh has been associated with liver inflammation and disease. Ginger is useful to control nausea and vomiting. Ginseng helps with mood symptoms and sleep disorders. Red clover helps with hot flashes. Ginger, ginseng, and red clover are not associated with liver disease. C) Black cohosh has been associated with liver inflammation and disease. Ginger is useful to control nausea and vomiting. Ginseng helps with mood symptoms and sleep disorders. Red clover helps with hot flashes. Ginger, ginseng, and red clover are not associated with liver disease. D) Black cohosh has been associated with liver inflammation and disease. Ginger is useful to control nausea and vomiting. Ginseng helps with mood symptoms and sleep disorders. Red clover helps with hot flashes. Ginger, ginseng, and red clover are not associated with liver disease.

1) A 7 pound 14 ounce girl was born to an insulin-dependent type Ⅱ diabetic mother 2 hours ago. The infant's blood sugar is 47 m g/d L. What is the best nursing action? A) To recheck the blood sugar in 6 hours B) To begin an I V of 10% dextrose C) To feed the baby 1 ounce of formula D) To document the findings in the chart

Answer: D Explanation: A) Blood glucose determinations should be performed on blood by heel stick hourly during the first 4 hours after birth, and subsequently at 4-hour intervals. B) A blood sugar reading of 47 m g/d L is considered normal for a neonate. No I V is needed. C) Feeding would be appropriate if the infant's blood sugar was below 45 m g/d L, but this infant's reading is 47. D) A blood sugar level of 47 m g/d L is a normal finding; documentation is an appropriate action.

1) After several hours of labor, the electronic fetal monitor (E F M) shows repetitive variable decelerations in the fetal heart rate. The nurse would interpret the decelerations to be consistent with which of the following? A) Breech presentation B) Uteroplacental insufficiency C) Compression of the fetal head D) Umbilical cord compression

Answer: D Explanation: A) Breech presentations by themselves do not cause decelerations. B) Uteroplacental insufficiency causes late decelerations. C) Early decelerations occur with fetal head compression. D) Variable decelerations occur when there is umbilical cord compression.

1. The client delivered her second child yesterday, and is preparing to be discharged. She expresses concern to the nurse because she developed an upper urinary tract infection (U T I) after the birth of her first child. Which statement indicates that the client needs additional teaching about this issue? A) "If I start to have burning with urination, I need to call the doctor." B) "Drinking 8 glasses of water each day will help prevent another U T I." C) "I will remember to wipe from front to back after I move my bowels." D) "Voiding 2 or 3 times per day will help prevent a recurrence."

Answer: D Explanation: A) Burning with urination is a common symptom of a U T I. B) Drinking 8 to 10 8-oz glasses of water daily will help to prevent the development of a U T I. C) Wiping from front to back after bowel movements will help to prevent the development of a U T I. D) Voiding only 2 or 3 times per day is not sufficient to prevent recurrence of a urinary tract infection (U T I). The woman needs to empty her bladder whenever she feels the urge to void, or at least every 2 to 4 hours while awake.

A patient who is postmenopausal is encouraged to take calcium 1500 mg every day. How should the nurse instruct the patient to take this supplement? A) Take calcium 750 m g with breakfast and dinner B) Take the complete dose first thing in the morning C) Take the complete dose prior to bedtime every day D) Take calcium 500 m g three times a day with meals

Answer: D Explanation: A) Calcium supplementation is most effective when single doses do not exceed 500 m g and when taken with a meal. Taking calcium 750 m g twice a day is less effective. Taking calcium 1500 m g in the morning or at night is not recommended since the mineral will not have peak absorption. B) Calcium supplementation is most effective when single doses do not exceed 500 m g and when taken with a meal. Taking calcium 750 m g twice a day is less effective. Taking calcium 1500 m g in the morning or at night is not recommended since the mineral will not have peak absorption. C) Calcium supplementation is most effective when single doses do not exceed 500 m g and when taken with a meal. Taking calcium 750 m g twice a day is less effective. Taking calcium 1500 m g in the morning or at night is not recommended since the mineral will not have peak absorption. D) Calcium supplementation is most effective when single doses do not exceed 500 m g and when taken with a meal. Taking calcium 750 m g twice a day is less effective. Taking calcium 1500 m g in the morning or at night is not recommended since the mineral will not have peak absorption.

1) The parents of a newborn comment to the nurse that their infant seems to enjoy being held, and that holding the baby helps him calm down after crying. They ask the nurse why this happens. After explaining newborn behavior, the nurse assesses the parents' learning. Which statement indicates that teaching was effective? A) "Some babies are easier to deal with than others." B) "We are lucky to have a baby with a calm disposition." C) "Our baby spends more time in the active alert phase." D) "Cuddliness is a social behavior that some babies have."

Answer: D Explanation: A) Calling the baby easy or difficult is a judgment that does not explain the baby's behavior. B) Describing the baby as calm is a judgment that does not explain the baby's behavior. C) The active alert phase of the sleep-awake cycle is characterized by motor activity. It does not explain the baby's behavior D) According to Brazelton Neonatal Behavioral Assessment Scale, cuddliness can be an indicator of personality.

20) e nurse is working with a pregnant 14-year-old. Which statement indicates that additional education is required? A) "Because I am still growing, I need more calories than a pregnant adult." B) "I need to eat fruit and vegetables every day to get enough vitamins." C) "My favorite food is pizza, and I eat it once a week." D) "Because I don't eat breakfast, I'll have to eat more at supper."

Answer: D Explanation: A) Caloric needs of pregnant adolescents vary widely. Figures as high as 50 kcal/kg have been suggested for young, growing teens who are very active physically. B) Eating a variety of fruits and vegetables helps ensure adequate intake of vitamins and some minerals. C) Pizza is not contraindicated during pregnancy. Eating a food once per week will not lead to nutritional imbalance. D) Pregnant young adolescents should eat breakfast to ensure that adequate calorie and protein intake is achieved. In assessing the diet of the pregnant adolescent, the nurse should consider the eating pattern over time, not simply a single day's intake.

1) The nurse admits into the labor area a client who is in preterm labor. What assessment finding would constitute a diagnosis of preterm labor? A) Cervical effacement of 30% or more B) Cervical change of 0.5 c m per hour C) 2 contractions in 30 minutes D) 8 contractions in 1 hour

Answer: D Explanation: A) Cervical effacement of 80% or more would define preterm labor. B) A cervical change of at least 1 c m per hour would define preterm labor. C) Uterine contractions every 5 minutes for 20 minutes would define preterm labor. D) 8 contractions in a 60-minute period does define a diagnosis of preterm labor.

1) A woman is in labor. The fetus is in vertex position. When the client's membranes rupture, the nurse sees that the amniotic fluid is meconium-stained. What should the nurse do immediately? A) Change the client's position in bed. B) Notify the physician that birth is imminent. C) Administer oxygen at 2 liters per minute. D) Begin continuous fetal heart rate monitoring.

Answer: D Explanation: A) Changing the client's position is not indicated. B) Meconium-stained amniotic fluid does not indicate that birth is imminent. C) Oxygen administration is not indicated. D) Meconium-stained amniotic fluid is an abnormal fetal finding, and is an indication for continuous fetal monitoring.

1) The nurse is preparing to receive a newly delivered client. The client is a young single mother who is relinquishing custody of her newborn through an open adoption. What action is most important? A) Assign the client a room on the G Y N surgical floor instead of on the postpartum floor. B) Prepare to complete teaching in time for discharge at 24 hours post-delivery. C) Make an effort not to bring up the topic of the baby, and discuss the mother's health instead. D) Ask the client how much contact she would like with the baby, and whether she wants to feed it.

Answer: D Explanation: A) Clients relinquishing their newborns should be given options concerning contact with the infant, including where they would feel most comfortable if they opt for contact at all. B) Not all clients who relinquish their infants want early discharge. C) It is up to the client to decide how much she wants to talk about her birth, her newborn, or her decision to relinquish the child. D) Assessing the birth mother's preferences by respectfully asking questions and making no assumptions facilitates a more positive experience.

27) As compared with admission considerations for an adult woman in labor, the nurse's priority for an adolescent in labor would be which of the following? A) Cultural background B) Plans for keeping the infant C) Support persons D) Developmental level

Answer: D Explanation: A) Cultural background is important to planning anyone's care. B) It is important to first determine the client's developmental level when planning nursing care for the mother who is keeping her infant. C) Support persons are important to planning anyone's care. D) Because her cognitive development is incomplete, the younger adolescent may have fewer problem-solving capabilities. The very young woman needs someone to rely on at all times during labor. She may be more childlike and dependent than older teens.

1) A couple who experienced a fetal demise would like to determine the specific cause. Which method of fetal testing will the nurse prepare for based on this request? A) Diabetes testing B) Urine toxicology testing C) Infectious disease studies D) Chromosomal studies

Answer: D Explanation: A) Diabetes testing is a method of maternal testing, not fetal testing, to determine the cause of fetal demise. B) Urine toxicology testing is a method of maternal testing, not fetal testing, to determine the cause of fetal demise. C) Infectious disease studies are a method of maternal testing, not fetal testing, to determine the cause of fetal demise. D) Chromosomal studies are a method of fetal testing that is used to determine the cause of fetal demise.

1) The nurse is interviewing a client who has admitted to being a victim of domestic violence. What is the most typical description of how the domestic violence developed in a relationship? A) "He changed overnight. Everything was fine, and all of a sudden he flipped out and beat me up; he nearly killed me." B) "It was severe from the beginning. As soon as we got married, he began hitting me and threatening to kill me." C) "We've both always dated other people. I thought that was understood. He was as emotionally abusive in the beginning as he is now." D) "I don't know when it started, really. It was gradual. First, just yelling, blaming, and shoving. Then the beatings started; and now they're more frequent."

Answer: D Explanation: A) Domestic violence does not begin suddenly, and will always escalate. B) Typically, these forms of abuse begin slowly and subtly after some form of commitment, such as engagement, onset of a sexual relationship, or marriage. C) Typically, these forms of abuse begin slowly and subtly after some form of commitment, such as engagement, onset of a sexual relationship, or marriage. D) Typically, these forms of abuse begin slowly and subtly after some form of commitment, such as engagement, onset of a sexual relationship, marriage, pregnancy, or first childbirth.

1. A postpartum client with endometritis is being discharged home on antibiotic therapy. The new mother plans to breastfeed her baby. What should the nurse's discharge instruction include? A) The client can douche every other day. B) Sexual intercourse can be resumed when the client feels up to it. C) Light housework will provide needed exercise. D) The baby's mouth should be examined for thrush.

Answer: D Explanation: A) Douching is contraindicated for this client. B) Pelvic rest is necessary for this client, and sexual activity should be resumed only when the physician says it is safe. C) The woman with a puerperal infection needs assistance when she is discharged from the hospital. If the family cannot provide this home assistance, a referral to home care services is needed. D) A breastfeeding mother on antibiotics should check her baby's mouth for signs of thrush, which should be reported to the physician.

1) The client is having fetal heart rate decelerations. An amnioinfusion has been ordered to alleviate the decelerations. The nurse understands that the type of decelerations that will be alleviated by amnioinfusion is which of the following? A) Early decelerations B) Moderate decelerations C) Late decelerations D) Variable decelerations

Answer: D Explanation: A) Early decelerations require no intervention. B) Moderate is not a descriptor used to identify decelerations. C) Late decelerations are consistent with head compression. Amnioinfusion does not relieve head compression. D) When cord compression is suspected, amnioinfusion (A I) may be considered. A I helps to prevent the possibility of variable decelerations by increasing the volume of amniotic fluid.

1) The nurse working with a client who is seeking a family and medical leave knows that the employee must meet which eligibility requirement of the Family and Medical Leave Act (F M L A) of 1993? A) Work at least 40 hours per week B) Have been employed for at least 1 month C) Work for a company with fewer than 50 employees D) Parental leave for childbirth or adoption by her employer

Answer: D Explanation: A) Employees must work at least 25 hours per week to be eligible. B) Employees must have been in their position for at least 1 year. C) F M L A applies only to companies with 50 or more employees. D) The Family and Medical Leave Act mandates parental leave for childbirth or adoption but applies only to companies with 50 or more employees.

1) The nurse has presented a session on pain relief options to a prenatal class. Which statement indicates that additional teaching is needed? A) "An epidural can be continuous or can be given in one dose." B) "A spinal is usually used for a cesarean birth." C) "Pudendal blocks are effective when a vacuum is needed." D) "Local anesthetics provide good labor pain relief."

Answer: D Explanation: A) Epidurals can be given either as a bolus or as a continuous infusion. B) Spinals are anesthesia, and are commonly used for cesarean birth. C) The advantages of pudendal block are ease of administration and absence of maternal hypotension. It also allows the use of low forceps or vacuum extraction for birth. D) Local anesthetics are not used for labor pain relief. They are used prior to episiotomy and for laceration repair.

1) A 20-year-old woman is at 28 weeks' gestation. Her prenatal history reveals past drug abuse, and urine screening indicates that she has recently used heroin. The nurse should recognize that the woman is at increased risk for which condition? A) Erythroblastosis fetalis B) Diabetes mellitus C) Abruptio placentae D) Pregnancy-induced hypertension

Answer: D Explanation: A) Erythroblastosis fetalis is secondary to physiological blood disorders such as R h incompatibility. B) Diabetes is an endocrine disorder that is unrelated to drug use and abuse. C) Abruptio placentae is seen more commonly with cocaine or crack use. D) Women who use heroin are at risk for poor nutrition, anemia, and pregnancy-induced hypertension (or preeclampsia-eclampsia).

1) The nurse is presenting a community education session on female hormones. Which statement from a participant indicates the need for further information? A) "Estrogen is what causes females to look female." B) "The presence of some hormones causes other to be secreted." C) "Progesterone is present at the end of the menstrual cycle." D) "Prostaglandin is responsible for achieving conception."

Answer: D Explanation: A) Estrogens are associated with characteristics contributing to femaleness, including breast alveolar lobule growth and duct development. B) It is true that the presence of some hormones causes other to be secreted. .X X C C) It is true that the proportions of progesterone and estrogen control the events of both ovarian and menstrual cycles. D) Prostaglandin is not related to conception. Prostaglandin production increases during follicular maturation and has basic regulatory functions in cells.

1) The nurse is discussing parent-infant attachment with a prenatal class. Which statement indicates that teaching was successful? A) "I should avoid looking directly into the baby's eyes to prevent frightening the baby." B) "My baby will be very sleepy immediately after birth and should go to the nursery." C) "Newborns cannot focus their eyes, so it doesn't matter how I hold my new baby." D) "Giving my baby his first bath can really give me a chance to get to know him."

Answer: D Explanation: A) Eye-to-eye contact between parents and their newborn is extremely important during the early hours after birth, when the newborn is in the first period of reactivity. B) Newborns are usually alert and responsive in the first few hours after birth. Interacting with the newborn during this first period of reactivity facilitates parent-infant attachment. C) Newborns can have direct eye contact with human faces, with an optimal range for visual acuity of 7 to 8 inches. Eye contact is an important component of the emerging parent-baby bond. D) Another situation that can facilitate attachment is the interactive bath. While bathing their newborn for the first time, parents attend closely to their baby's behavior and the nurse can observe and point out behaviors.

1) The community health nurse is teaching a class about causes of traumatic injury leading to pregnancy complications. What statement should the nurse include in the teaching? A) "Although falls are an uncommon cause of trauma, it is important to know what to do in the case of these incidents." B) "In early pregnancy, the woman is at a greater risk for injury due to decreased balance and coordination." C) "The pregnant woman should be given the same care as any person suffering from trauma." D) "Violence, including domestic violence, is the most common cause of injury for pregnant women, after motor vehicle accidents."

Answer: D Explanation: A) Falls are a common cause of trauma in the client who is pregnant. B) Late pregnancy, not early pregnancy, causes a decrease in balance in the client who is pregnant. C) The physiologic changes that occur with pregnancy have clinical implications for victims of trauma; the client who is pregnant does not receive the same care. D) Violence, including domestic violence, is the next most common cause of injury for clients who are pregnant after motor vehicle accidents. This is the statement the nurse should include in the teaching.

1) The client is undergoing lab work and ultrasound for a possible diagnosis of polycystic ovarian syndrome (P C O S). Which problem does the nurse expect to find in the client's history? A) Multiple first-trimester fetal losses B) Dyspareunia C) Vulvitis D) Oligomenorrhea

Answer: D Explanation: A) First-trimester fetal loss is not associated with polycystic ovarian syndrome (P C O S). B) Dyspareunia is not associated with polycystic ovarian syndrome (P C O S). C) Vulvitis is not associated with polycystic ovarian syndrome (P C O S). Irregular menses, ranging from total absence of periods (amenorrhea) to intermittent or infrequent periods (oligomenorrhea) are the hallmarks of PCOS

1) A client at 20 weeks' gestation has not decided on a feeding method for her infant. She asks the nurse for advice. The nurse presents information about the advantages and disadvantages of formula-feeding and breastfeeding. Which statements by the client indicate that the teaching was successful? A) "Formula-feeding gives the baby protection from infections." B) "Breast milk cannot be stored; it has to be thrown away after pumping." C) "Breastfeeding is more expensive than formula-feeding." D) "My baby will have a lower risk of food allergies if I breastfeed."

Answer: D Explanation: A) Formula does not provide the baby with protection from infections; breast milk does. B) Breast milk can be refrigerated or frozen after pumping. C) Formula must be purchased, and therefore is more expensive. D) Secretory I g A, an immunoglobulin present in colostrum and mature breast milk, has antiviral, antibacterial, and antigenic-inhibiting properties and plays a role in decreasing the permeability of the small intestine to help prevent large protein molecules from triggering an allergic response.

1) A 15-year-old client has delivered a 22-week stillborn fetus. What does the nurse understand? A) Grieving a fetal loss manifests with very similar behaviors regardless of the age of the client. B) Teens tend to withhold emotions and need older adults with the same type of loss to help process the experience. C) Most teens have had a great deal of contact with death and loss and have an established method of coping. Assisting the client might be difficult because of her mistrust of authority figures

Answer: D Explanation: A) Grieving behaviors are very different in adolescents than in older clients. Adolescents rely heavily on peer support. B) Adolescents rely more heavily on peer support than on adults for emotional support. C) For many adolescents, a pregnancy loss may be the first time they are experiencing the psychologic impact of grief. D) Adolescents rely heavily on peer support and have a natural mistrust of authority figures, which can make assisting them more difficult.

1) A pregnant woman is married to an intravenous drug user. She had a negative H I V screening test just after missing her first menstrual period. What would indicate that the client needs to be retested for H I V? A) Hemoglobin of 11 g/d L and a rapid weight gain B) Elevated blood pressure and ankle edema C) Shortness of breath and frequent urination D) Persistent candidiasis

Answer: D Explanation: A) Hemoglobin of 11 g/d L and a rapid weight gain do not indicate a need to be retested for H I V. B) Elevated blood pressure and ankle edema do not indicate a need to be retested for H I V. C) Shortness of breath and frequent urination do not indicate a need to retest for H I V. D) Signs and symptoms of infections include fever, weight loss, fatigue, persistent candidiasis, diarrhea, cough, and skin lesions (Kaposi's sarcoma and hairy leukoplakia in the mouth).

1) The nurse is preparing a handout on the ovarian cycle to a group of middle school girls. Which information should the nurse include? A) The hormone human chorionic gonadotropin stimulates ovulation. B) Irregular menstrual cycles have varying lengths of the luteal phase. C) The ovum leaves its follicle during the follicular phase. D) There are two phases of the ovarian cycle: luteal and follicular.

Answer: D Explanation: A) Human chorionic gonadotropin (h C G) is secreted by a fertilized ovum, and does not stimulate ovulation. B) In women whose menstrual cycles vary, usually it is only the length of the follicular phase that varies, while the luteal phase is of fixed length. C) The luteal phase begins when the ovum leaves its follicle. D) The ovarian cycle has two phases: the follicular phase (days 1 to 14) and the luteal phase (days 15 to 28 in a 28-day cycle).

1) A patient with amenorrhea has an elevated serum prolactin level. Which diagnostic test should the nurse expect will be prescribed for this patient? A) Laparoscopy B) Abdominal ultrasound C) C T scan of the abdomen D) Magnetic resonance imaging (M R I)

Answer: D Explanation: A) If serum prolactin levels are elevated, magnetic resonance imaging (M R I) will be ordered to rule out a pituitary tumor. A laparoscopy, abdominal ultrasound, or C T scan of the abdomen is not indicated for a patient with amenorrhea and an elevated serum prolactin level. B) If serum prolactin levels are elevated, magnetic resonance imaging (M R I) will be ordered to rule out a pituitary tumor. A laparoscopy, abdominal ultrasound, or C T scan of the abdomen is not indicated for a patient with amenorrhea and an elevated serum prolactin level. C) If serum prolactin levels are elevated, magnetic resonance imaging (M R I) will be ordered to rule out a pituitary tumor. A laparoscopy, abdominal ultrasound, or C T scan of the abdomen is not indicated for a patient with amenorrhea and an elevated serum prolactin level. D) If serum prolactin levels are elevated, magnetic resonance imaging (M R I) will be ordered to rule out a pituitary tumor. A laparoscopy, abdominal ultrasound, or C T scan of the abdomen is not indicated for a patient with amenorrhea and an elevated serum prolactin level.

1) The neonatal special care unit nurse is overseeing the care provided by a nurse new to the unit. Which action requires immediate intervention? A) The new nurse holds the infant after giving a gavage feeding. B) The new nurse provides skin-to-skin care. C) The new nurse provides care when the baby is awake. D) The new nurse gives the feeding with room-temperature formula.

Answer: D Explanation: A) If the infant cannot be held during a feeding, she should be held after feedings for comfort. B) Skin-to-skin (kangaroo) care has become the norm in N I C Us across the United States and is defined as the practice of holding infants skin to skin next to their parents. C) Preterm babies spend more time in sleep cycles; it is best to not interrupt sleep when possible. D) Preterm babies have little subcutaneous fat, and do not maintain their body temperature well. Formula should be warmed prior to feedings to help the baby maintain its temperature.

1) A 21-year-old at 12 weeks' gestation with her first baby has known cardiac disease, class Ⅲ, as a result of childhood rheumatic fever. During a prenatal visit, the nurse reviews the signs of cardiac decompensation with her. The nurse will know that the client understands these signs and symptoms if she states that she would notify her doctor if she had which symptom? A) "A pulse rate increase of 10 beats per minute" B) "Breast tenderness" C) "Mild ankle edema" D) "A frequent cough"

Answer: D Explanation: A) In most pregnant clients, the heart rate will increase. B) Most pregnant clients will develop breast tenderness. C) The client with rheumatic heart disease who develops congestive heart failure would have severe ankle edema. D) The heart's signal of its decreased ability to meet the demands of pregnancy includes frequent cough (with or without hemoptysis).

1) The nurse examines the client's placenta and finds that the umbilical cord is inserted at the placental margin. The client comments that the placenta and cord look different than they did for her first two births. The nurse should explain that this variation in placenta and cord is called what? A) Placenta accreta B) Circumvallate placenta C) Succenturiate placenta D) Battledore placenta

Answer: D Explanation: A) In placenta accreta, the chorionic villi attach directly to the myometrium of the uterus. B) A circumvallate placenta has a double fold of chorion and amnion that form a ring around the umbilical cord, on the fetal side of the placenta. C) In succenturiate placenta, one or more accessory lobes of fetal villi will develop on the placenta. D) In battledore placenta, the umbilical cord is inserted at or near the placental margin.

1) The nurse teaching the phases of the menstrual cycle should include that the corpus luteum begins to degenerate, estrogen and progesterone levels fall, and extensive vascular changes occur in which phase? A) Menstrual phase B) Proliferative phase C) Secretory phase D) Ischemic phase

Answer: D Explanation: A) In the menstrual phase, estrogen levels are low, cervical mucus is scant, viscous, and opaque, and endometrium is shed. B) In the proliferative phase, endometrium and myometrium thickness increases and estrogen peaks just before ovulation. C) In the secretory phase, estrogen drops sharply, and progesterone dominates; vascularity of the entire uterus increases; and tissue glycogen increases, making the uterus ready for implantation. D) In the ischemic phase, the corpus luteum begins to degenerate, and as a result, both estrogen and progesterone levels fall. Small blood vessels rupture, and the spiral arteries constrict and retract, causing a deficiency of blood in the endometrium, which becomes pale.

1) A nurse is caring for a newborn on a ventilator who has respiratory distress syndrome (R D S). The nurse informs the parents that the newborn is improving. Which data support the nurse's assessment? A) Decreased urine output B) Pulmonary vascular resistance increases C) Increased P C O2 Increased urination

Answer: D Explanation: A) Increased urination, not decreased urine output, could be an indication that the newborn's condition is improving. B) Pulmonary vascular resistance increases with hypoxia. C) Increased P C O2 results from alveolar hypoventilation. In babies with respiratory distress syndrome (RDS) who are on ventilators, increased urination/diuresis may be an early clue that the baby's condition is improving

1) Intercourse is contraindicated if the pregnancy is vulnerable because of which diagnosis? A) Gestational diabetes B) Cervical insufficiency (cerclage) C) Abruptio placentae D) Placenta previa

Answer: D Explanation: A) Intercourse is not contraindicated if the pregnancy is vulnerable because of the diagnosis of gestational diabetes. B) Intercourse is not contraindicated if the pregnancy is vulnerable because of the diagnosis of cervical insufficiency (cerclage). C) Intercourse is not contraindicated if the pregnancy is vulnerable because of the diagnosis of abruptio placentae. D) Intercourse is contraindicated if the pregnancy is vulnerable because of the diagnosis of threatened spontaneous abortion, placenta previa, or the risk of preterm labor.

1) The client with thalassemia intermedia has a hemoglobin level of 9.0. The nurse is preparing an education session for the client. Which statement should the nurse include? A) "You need to increase your intake of meat and other iron-rich foods." B) "Your low hemoglobin could put you into preterm labor." C) "Increasing your vitamin C intake will help your hemoglobin level." D) "You should not take iron supplements."

Answer: D Explanation: A) Iron-rich foods and supplementation should be avoided. B) Because the client's iron levels are normal, increasing dietary iron will not affect the hemoglobin. C) Vitamin C increases iron absorption, but a client with thalassemia does not need additional iron. D) Folic acid supplements are indicated for women with thalassemia, but iron supplements are not.

1) A client has just been admitted for labor and delivery. She is having mild contractions lasting 30 seconds every 15 minutes. The client wants to have a medication-free birth. When discussing medication alternatives, the nurse should be sure the client understands which of the following? A) In order to respect her wishes, no medication will be given. B) Pain relief will allow a more enjoyable birth experience. C) The use of medications allows the client to rest and be less fatigued. D) Maternal pain and stress can have a more adverse effect on the fetus than would a small amount of analgesia.

Answer: D Explanation: A) It is important to respect the client's wishes when possible. Once the effects are explained, it is still the client's choice whether to receive medication. B) That pain relief can lead to a more enjoyable experience might be the view of the nurse, but not of the mother. C) While pain relief can allow the mother to be less fatigued, that might not be the mother's first priority. D) The nurse can explain to the client that, although pharmacologic agents do affect the fetus, so does the pain and stress experienced by the laboring mother. If the woman's pain and anxiety are more than she can cope with, the adverse physiologic effects on the fetus may be as great as would occur with the administration of a small amount of an analgesic agent. Once the effects are explained, however, it is still the client's choice whether to receive medication.

1) A newborn has been diagnosed with a disorder that occurs through an autosomal recessive inheritance pattern. The parents ask the nurse, "Which of us passed on the gene that caused the disorder?" What should the nurse tell them? A) The female B) The male C) Neither D) Both

Answer: D Explanation: A) It is not a sex-linked disorder or an abnormal chromosome disorder. B) It is not a sex-linked abnormality. C) In an autosomal recessive inherited disorder, both parents are carriers of the abnormal gene. D) An affected individual can have clinically normal parents, but both parents are generally carriers of the abnormal gene.

1) A postpartum client calls the nursery to report that her 3-day-old newborn has passed a green stool. What is the nurse's best response? A) "Take your newborn to the pediatrician." B) "There might be a possible food allergy." C) "Your newborn has diarrhea" D) "Your newborn's stools change from meconium to transitional stools"

Answer: D Explanation: A) It is not necessary for the client to take her newborn to the pediatrician. B) The green color of stool is not due to food allergies. C) The green color of stool is not due to diarrhea. D) The newborn's stools change from meconium (thick, tarry, black) to transitional stools (thinner, brown to green).

1) A new parent reports to the nurse that the baby looks cross-eyed several times a day. The nurse teaches the parents that this finding should resolve in how long? A) 2 months B) 2 weeks C) 1 year D) 4 months

Answer: D Explanation: A) It usually takes a little longer than 2 months for transient strabismus to disappear. B) Transient strabismus lasts longer than 2 weeks. C) Transient strabismus generally does not last 1 year. D) Transient strabismus is caused by poor neuromuscular control of the eye muscles and gradually regresses in 3 to 4 months.

1) The client has just been diagnosed as diabetic. The nurse knows teaching was effective when the client makes which statement? A) "Ketones in my urine mean that my body is using the glucose appropriately." B) "I should be urinating frequently and in large amounts to get rid of the extra sugar." C) "My pancreas is making enough insulin, but my body isn't using it correctly." D) "I might be hungry frequently because the sugar isn't getting into the tissues the way it should."

Answer: D Explanation: A) Ketones are produced when fat is being utilized for glucose, and this is not a desirable response. B) Frequent urination is an indication of glucose above the renal threshold, and is not a good indicator of diabetic stability. C) Diabetes is a result of lack of insulin production. If the insulin is being produced, the body will utilize it. D) The client who understands the disease process is aware that if the body is not getting the glucose it needs, the message of hunger will be sent to the brain.

1) A 38-week newborn is found to be small for gestational age (S G A). Which nursing intervention should be included in the care of this newborn? A) Monitor for feeding difficulties. B) Assess for facial paralysis. C) Monitor for signs of hyperglycemia. D) Maintain a warm environment.

Answer: D Explanation: A) L G A, not S G A, newborns are more difficult to arouse to a quiet alert state, and can have feeding difficulties. B) L G A, not S G A, newborns often are prone to birth trauma such as facial paralysis, due to cephalopelvic disproportion. C) S G A newborns are more prone to hypoglycemia. Hypothermia is a common complication in the SGA newborn; therefore, the newborn's environment must remain warm, to decrease heat loss

1) To assess the healing of the uterus at the placental site, what does the nurse assess? A) Lab values B) Blood pressure C) Uterine size D) Type, amount, and consistency of lochia

Answer: D Explanation: A) Lab values is an incomplete response because it does not indicate which lab values are referred to. B) Blood pressure varies slightly in normal postpartal women, and would not affect the placental site. C) Uterine size alone is not enough to assess the placental site. D) The type, amount, and consistency of lochia determine the state of healing of the placental site, and a progressive color change from bright red at birth to dark red to pink to white or clear should be observed.

1) A client in her third trimester of pregnancy reports frequent leg cramps. What strategy would be most appropriate for the nurse to suggest? A) Point the toes of the affected leg B) Increase intake of protein-rich foods C) Limit activity for several days D) Flex the foot to stretch the calf

Answer: D Explanation: A) Leg cramps are exacerbated by pointing the toes. B) Leg cramps often result from an imbalance in the calcium-phosphorus ratio, not from a lack of protein-rich foods. C) Leg cramps are not caused by excess activity. D) The nurse should advise the client to practice dorsiflexion of her feet to stretch the affected muscle.

1) The client at 20 weeks' gestation has had an ultrasound that revealed a neural tube defect in her fetus. The client's hemoglobin level is 8.5. The nurse should include which statement when discussing these findings with the client? A) "Your low iron intake has caused anemia, which leads to the neural tube defect." B) "You should increase your vitamin C intake to improve your anemia." C) "You are too picky about food. Your poor diet caused your baby's defect." D) "You haven't had enough folic acid in your diet. You should take a supplement."

Answer: D Explanation: A) Low hemoglobin does not cause neural tube defects. B) Vitamin C will increase iron absorption, but this client is deficient in folic acid. C) Therapeutic communication requires that the nurse avoid making judgmental statements. D) An inadequate intake of folic acid has been associated with neural tube defects (N T Ds) (e.g., spina bifida, anencephaly, meningomyelocele) in the fetus or newborn.

1) The nurse developing a care plan for a pregnant teen knows that what plan is best suited to pregnant teens? A) Assess menstrual history and the presence of any food allergies. B) Inquire about family relationships and location of the home. C) Ask what her body image is, then correct her misconceptions. D) Determine whether there are substance abuse issues.

Answer: D Explanation: A) Menstrual history is important for determining gestational age, and food allergies can impact nutritional intake. However, assessment of these factors is not necessarily better suited to pregnant teens than to pregnant women of other ages. B) Family relationships are important to understand, specifically to find out what support systems are available to the teen. However, the location of the home is not important. C) Body image must be addressed more indirectly, not by asking a client what she thinks about her body. A client's body image is her belief, and cannot be corrected by the nurse. D) Substance abuse issues are important to assess during pregnancy of teens.

1) The nurse is preparing a handout for female adolescents on the menstrual cycle. What phase of the cycle occurs if fertilization does not take place? A) Menstrual B) Proliferative C) Secretory D) Ischemic

Answer: D Explanation: A) Menstruation occurs during the menstrual phase. Some endometrial areas are shed, whereas others remain. B) The proliferative phase begins when the endometrial glands enlarge, the blood vessels become prominent and dilated, and the endometrium increases in thickness. C) The secretory phase follows ovulation. D) The ischemic phase occurs if fertilization does not occur.

1) A home care nurse is looking over the charts of four elderly female clients. The nurse knows that which client has the highest risk for developing diabetes and heart disease? A) A woman who is 55 and white B) A woman who is 60 and from a middle-class background C) All women over 55 D) A woman over 65 who is African American

Answer: D Explanation: A) Minority and low-income women 65 years old and older are more likely than white, higher income women to have serious health problems. B) Minority and low-income women 65 years old and older are more likely than white, higher income women to have serious health problems. C) By age 65, not age 55, half of all women have developed two or more chronic diseases. D) Minority and low-income women 65 years old and older are more likely than white, higher income women to have serious health problems.

1) Which of the following would be a newborn care procedure that will decrease the probability of high bilirubin levels? A) Monitor urine for amount and characteristics. B) Encourage late feedings to promote intestinal elimination. C) All infants should be routinely monitored for iron intake. D) Maintain the newborn's skin temperature at 36.5°C (97.8°F) or above.

Answer: D Explanation: A) Monitor stool for amount and characteristics. Bilirubin is eliminated in the feces. B) Encourage early feedings to promote intestinal elimination and bacterial colonization, and to provide the caloric intake necessary for hepatic binding proteins to form. C) All infants should be routinely monitored for the development of jaundice. D) Maintain the newborn's skin temperature at 36.5°C (97.8°F) or above; cold stress results in acidosis.

1) The client is carrying monochorionic-monoamniotic twins. The nurse teaches the client what this is, and the implications of this finding. The nurse knows that teaching is successful when the client states which of the following? A) "My babies came from two eggs." B) "About two thirds of twins have this amniotic sac formation." C) "My use of a fertility drug led to this issue." D) "My babies have a lower chance of surviving to term than fraternal twins do."

Answer: D Explanation: A) Monochorionic-monoamniotic twins lie in the same amniotic sac. B) About 2% of twins are of this type. C) The majority of twins conceived through in vitro fertilization are fraternal (dizygotic) because multiple fertilized ova are inserted into the uterus, and are not monochorionic-monoamniotic twins. D) Monochorionic-monoamniotic twins are both in one amniotic sac. There is an increased risk of umbilical cords becoming tangled or knotted and a higher incidence of fetal demise.

1. Which of the following is a risk factor for urinary retention after childbirth? A) Multiparity B) Precipitous labor C) Unassisted childbirth D) Not sufficiently recovering from the effects of anesthesia

Answer: D Explanation: A) Nulliparity is a risk factor for urinary retention after childbirth. B) Prolonged labor is risk factor for urinary retention after childbirth. C) Instrumental childbirth is a risk factor for urinary retention after childbirth. D) Women who have not sufficiently recovered from the effects of anesthesia and cannot void spontaneously are at risk for urinary retention after childbirth.

1) The nurse is caring for a newborn in the special care nursery. The infant has hydrocephalus, and is positioned in a prone position. The nurse is especially careful to cleanse all stool after bowel movements. This care is most appropriate for an infant born with which of the following? A) Omphalocele B) Gastroschisis C) Diaphragmatic hernia D) Myelomeningocele

Answer: D Explanation: A) Omphalocele is a herniation of abdominal contents into the base of the umbilical cord. Hydrocephalus is not associated with an omphalocele. B) Gastroschisis is a full-thickness defect of the abdominal wall that results in the abdominal organs being located on the outside of the body. Hydrocephalus is not associated with a gastroschisis. C) Diaphragmatic hernia is a portion of the intestines in the thoracic cavity due to an abnormal opening in diaphragm, occurring commonly on the left side. Hydrocephalus is not associated with a diaphragmatic hernia. D) Myelomeningocele is a saclike cyst containing meninges, spinal cord, and nerve roots in the thoracic and/or lumbar area. Meticulous cleaning of the buttocks and genitals helps prevent infection. The infant is positioned on his or her abdomen or side and restrained to prevent pressure and trauma to the sac. Hydrocephalus is often present with this condition.

15) Why is it important for the nurse to assess the bladder regularly and encourage the laboring client to void frequently? A) A full bladder impedes oxygen flow to the fetus. B) Frequent voiding prevents bruising of the bladder. C) Frequent voiding encourages sphincter control. D) A full bladder can impede fetal descent.

Answer: D Explanation: A) Oxygen flow to the fetus is not impacted by a full bladder. B) Frequent voiding has nothing to do with bruising of the bladder. C) Frequent voiding has nothing to do with sphincter control. D) The woman should be encouraged to void because a full bladder can interfere with fetal descent. If the woman is unable to void, catheterization may be necessary.

1) The nurse is teaching a pregnant client the clinical manifestations associated with preterm labor. Which client statement indicates the need for further education? A) "Menstrual-like cramps are a sign of preterm labor." B) "A dull low backache is a sign of preterm labor." C) "Diarrhea is a sign of preterm labor." D) "Vomiting is a sign of preterm labor."

Answer: D Explanation: A) Painful menstrual-like cramps are a sign of preterm labor. This statement indicates appropriate understanding of the information presented. B) A dull low backache is a sign of preterm labor. This statement indicates appropriate understanding of the information presented. C) Diarrhea is a sign of preterm labor. This statement indicates appropriate understanding of the information presented. D) Vomiting is not a clinical manifestation associated with preterm labor. This statement indicates the need for further education.

1) Recommendations for parents to help their teens avoid pregnancy include which of the following? A) Parents need to encourage frequent and steady dating. B) Parents need to let their children set their own goals for the future. C) Parents need to have their children taught about sex by school programs and community resources. D) Parents should be clear about their own sexual attitudes and values.

Answer: D Explanation: A) Parents need to clearly discourage early dating as well as frequent and steady dating. B) Parents need to help children set goals for their future and have options that are more attractive than early pregnancy and childrearing. C) Parents need to talk with their children about sex early and often and be specific in the discussions. D) This is true. Parents should be clear about their own sexual attitudes and values in order to communicate clearly with children.

A nonpregnant client is diagnosed with bacterial vaginosis (B V). What does the nurse expect to administer? A) Penicillin G 2 million units I M one time B) Zithromax 1 m g P O bid for 2 weeks C) Doxycycline 100 m g P O bid for a week D) Metronidazole 500 m g P O bid for a week

Answer: D Explanation: A) Penicillin is not used to treat bacterial vaginosis. B) Zithromax is not used to treat bacterial vaginosis. C) Doxycycline is not used to treat bacterial vaginosis. D) The nonpregnant woman who is diagnosed with bacterial vaginosis (B V) is treated with metronidazole 500 m g orally twice a day for 7 days.

1) The nurse explains to a preconception class that if only a small volume of sperm is discharged into the vagina, an insufficient quantity of enzymes might be released when they encounter the ovum. In that case, pregnancy would probably not result, because of which of the following? A) Peristalsis of the fallopian tube would decrease, making it difficult for the ovum to enter the uterus. B) The block to polyspermy (cortical reaction) would not occur. C) The fertilized ovum would be unable to implant in the uterus. Sperm would be unable to penetrate the zona pellucida of the ovum

Answer: D Explanation: A) Peristalsis of the fallopian tube is not a factor in this stage of fertilization. B) A block to polyspermy would indicate that the ovum had already been penetrated by a fertilizing sperm, which would occur later in the fertilization process. C) The ovum has not yet been fertilized in this example. D) About a thousand acrosomes must rupture to clear enough hyaluronic acid for even a single sperm to penetrate the ovum's zona pellucida successfully. If only a small amount of sperm were released, there most likely would be an insufficient quantity of acrosomes to penetrate the zona pellucida of the ovum and allow fertilization.

1) A newborn is receiving phototherapy. Which intervention by the nurse would be most important? A) Measurement of head circumference B) Encouraging the mother to stop breastfeeding C) Stool blood testing D) Assessment of hydration status

Answer: D Explanation: A) Phototherapy does not affect head circumference. B) Breastfeeding most likely can be continued. C) The stools do not need to be tested for blood. D) Infants undergoing phototherapy treatment have increased water loss and loose stools as a result of bilirubin excretion. This increases their risk of dehydration.

1) Before the newborn and mother are discharged from the birthing unit, the nurse teaches the parents about newborn screening tests that includes which of the following? A) Preeclampsia screening B) Congenital kidney disease screening C) Visual screening D) Hearing screening

Answer: D Explanation: A) Preeclampsia is a maternal condition, and not part of the newborn screening tests. B) Congenital heart disease screening, not kidney disease screening, is part of the newborn screening tests. C) Visual screening is not part of newborn screening tests. D) Newborn screening tests include hearing screening tests.

1) The nurse is working at a clinic for pregnant teens. What issues related to development will the nurse expect to encounter in most of the pregnant clients? A) Peer pressure to stop using alcohol once pregnancy is diagnosed B) Contraception failure that resulted in this pregnancy C) The baby's father being emotionally supportive to the client D) Feelings of not living up to parents' expectations

Answer: D Explanation: A) Pregnant teens face risk factors based on the developmental tasks of adolescence. One of these is peer pressure to engage in substance use. B) Pregnant teens face risk factors based on the developmental tasks of adolescence. These include sexual activity without contraception due to lack of a future orientation. C) Pregnant teens face risk factors based on the developmental tasks of adolescence. One of these is sexual activity as experimentation without emotional intimacy. D) Pregnant teens face risk factors based on the developmental tasks of adolescence. One of these is developing an identity. If the adolescent feels she has not lived up to parental expectations by becoming pregnant, she could adopt a negative identity.

1) Intervention to reduce preterm birth can be divided into primary prevention and secondary prevention. What does secondary prevention include? A) Diagnosis and treatment of infections B) Cervical cerclage C) Progesterone administration D) Antibiotic treatment and tocolysis

Answer: D Explanation: A) Primary prevention includes diagnosis and treatment of infections, cervical cerclage, and progesterone administration. B) Primary prevention includes diagnosis and treatment of infections, cervical cerclage, and progesterone administration. C) Primary prevention includes diagnosis and treatment of infections, cervical cerclage, and progesterone administration. D) Secondary prevention strategies are antibiotic treatment and tocolysis.

1) To actively involve the postpartal client during discharge teaching, the postpartum nurse applies which learning principle? A) Reprints of magazine articles B) Classroom lectures C) Audiotapes D) Interactive nurse-patient relationships

Answer: D Explanation: A) Providing magazine articles does not actively involve the client in learning. B) Classroom lectures do not actively involve the client in learning. C) Listening to audiotapes does not actively involve the client in learning. D) Effective parent learning requires precise timing of teaching, as well as choice of a teaching method that is effective for the family, such as D V Ds and return demonstration. Content on self-care, infant care, and anticipatory guidance is important.

1) The nurse caring for a postterm newborn would not perform what intervention? A) Providing warmth B) Frequently monitoring blood glucose C) Observing respiratory status D) Restricting breastfeeding

Answer: D Explanation: A) Provision of warmth is an important intervention for postterm newborns. B) Frequent monitoring of blood glucose is an important intervention for postterm newborns. C) Observation of respiratory status is an important intervention for postterm newborns. D) Breastfeeding is an appropriate means of feeding for the postterm newborn.

1) The nurse is explaining "quickening" to a client who is pregnant for the first time. Which client indicates the need for further education on this topic? A) "It will feel like butterflies in my stomach." B) "It might feel like I have gas." C) "It should occur during the second trimester of my pregnancy." D) "It is an indication that I am experiencing preterm labor."

Answer: D Explanation: A) Quickening is often described as if there are butterflies in the stomach. This statement indicates appropriate understanding of the information presented. B) Quickening is often mistaken for gas. This statement indicates appropriate understanding of the information presented. C) Quickening often occurs during the second trimester of pregnancy, between 16 and 22 weeks' gestation. This statement indicates appropriate understanding of the information presented. Quickening is not an indication of preterm labor, but an expected finding during pregnancy. This statement indicates the need for further education

1) The nurse is providing care to a female client who presents in the emergency department (E D) with multiple bruises and lacerations. The nurse suspects the client is the victim of domestic violence. Which action by the nurse is appropriate? A) Reporting the incident to the police to enhance safety B) Documenting domestic violence in the medical record C) Avoiding photographs of the injuries to prevent embarrassment D) Communicating the level of confidentiality that can be expected

Answer: D Explanation: A) Reporting domestic violence may be mandatory in some states. However, it is important to note that reporting domestic violence may increase the client's risk for further abuse, not enhance the client's safety. B) The nurse would document the client's injuries in the medical record and use the term "probable battering." To protect the client's confidentiality and safety, it is critical that the nurse not refer to domestic violence or abuse on any discharge papers. C) Photographs of the client's injuries can be of great value along with documentation of the extent of the injuries and noting of the client's exact words in the medical record. D) It is important for the nurse to explain the assessment process to the client and communicate the level of confidentiality that can be expected.

1) A sexually active female asks why an H I V test is needed since she uses condoms with spermicidal agents when having intercourse. How should the nurse respond to this client? A) "Condoms do not protect against contracting H I V." B) "Spermicides only control bacteria and not viruses." C) "All sexually active people are at risk for contracting H I V." D) "The spermicide can make your vaginal cells more susceptible to H I V."

Answer: D Explanation: A) Research suggests that N-9 does not offer protection against the organisms that cause H I V/A I D S, and N-9 alone may actually increase a woman's risk of H I V infection because it has a negative effect on the integrity of vaginal cells, making them more susceptible to invasion by organisms such as H I V. Condoms do help reduce the risk of contracting H I V. Spermicides are prepared to render sperm inactive, not to kill bacteria or viruses. Although all sexually active individuals are at some risk for contracting H I V, this does not explain why the client should be tested for the virus. B) Research suggests that N-9 does not offer protection against the organisms that cause H I V/A I D S, and N-9 alone may actually increase a woman's risk of H I V infection because it has a negative effect on the integrity of vaginal cells, making them more susceptible to invasion by organisms such as H I V. Condoms do help reduce the risk of contracting H I V. Spermicides are prepared to render sperm inactive not to kill bacteria or viruses. Although all sexually active individuals are at some risk for contracting H I V, this does not explain why the client should be tested for the virus. C) Research suggests that N-9 does not offer protection against the organisms that cause H I V/A I D S, and N-9 alone may actually increase a woman's risk of H I V infection because it has a negative effect on the integrity of vaginal cells, making them more susceptible to invasion by organisms such as H I V. Condoms do help reduce the risk of contracting H I V. Spermicides are prepared to render sperm inactive not to kill bacteria or viruses. Although all sexually active individuals are at some risk for contracting H I V, this does not explain why the client should be tested for the virus. D) Research suggests that N-9 does not offer protection against the organisms that cause H I V/A I D S, and N-9 alone may actually increase a woman's risk of H I V infection because it has a negative effect on the integrity of vaginal cells, making them more susceptible to invasion by organisms such as H I V. Condoms do help reduce the risk of contracting H I V. Spermicides are prepared to render sperm inactive not to kill bacteria or viruses. Although all sexually active individuals are at some risk for contracting H I V, this does not explain why the client should be tested for the virus.

1) The nurse is caring for a client who recently emigrated from a Southeast Asian country. The mother has been resting since the birth, while her sister has changed the diapers and fed the infant. What is the most likely explanation for this behavior? A) The client is not attaching to her infant appropriately. B) The client is not going to be a good mother, and the baby is at risk. C) The client has no mother present to role-model behaviors. D) The client is exhibiting normal behavior for her culture.

Answer: D Explanation: A) Resting since the birth is expected behavior in certain cultures. B) The mother-infant couplet is not at risk. Resting since birth is expected behavior in certain cultures. C) Even though her mother is not present, her sister is role-modeling behaviors. D) Rest, seclusion, and dietary restraint practices in many traditional non-Western cultures (South Asian groups) are designed to assist the woman and her baby during postpartum vulnerable periods.

1) The hospital is developing a new maternity unit. What aspects should be included in the planning of the new unit to best promote family wellness? A) Normal newborn nursery centrally located to all client rooms B) A kitchen with a refrigerator stocked with juice and sandwiches C) Small, cozy rooms with a client bed and rocking chair D) A nursing care model based on providing couplet care

Answer: D Explanation: A) Rooming-in provides the childbearing family with opportunities to interact with their newborn during the first hours and days of life. B) Although having snacks can be good for postpartum clients, some cultures prohibit drinking cold liquids after birth. C) Small rooms can become overly crowded when siblings and grandparents come to visit. Larger rooms that facilitate family attachment are better. D) Couplet care, which is care of both the mother and her baby, is an important part of the family-centered care approach, in which the infant remains at the mother's bedside and both are cared for by the same nurse.

1) The nurse assesses the postpartum client who has not had a bowel movement by the third postpartum day. Which nursing intervention would be appropriate? A) Encourage the new mother, saying, "It will happen soon." B) Instruct the client to eat a low-fiber diet. C) Decrease fluid intake. D) Obtain an order for a stool softener.

Answer: D Explanation: A) Saying "It will happen soon" does not address the client's needs and could increase the chance for constipation. B) Eating a low-fiber diet would not increase bulk or moisture in the stool. C) Decreasing fluid intake would decrease moisture in the fecal material, encouraging constipation. D) Obtaining an order for a stool softener is the correct intervention by the third day. In resisting or delaying the bowel movement, the woman may cause increased constipation and more pain when elimination finally occurs.

1) The nurse is preparing a stillborn newborn to be held by the parents. Which action by the nurse is appropriate? A) Using baby lotion so that the newborn smells like a baby B) Placing cotton in the newborn's nostrils if seepage is noted C) Using a washcloth when bathing the newborn's skin to decrease tissue sloughing D) Placing the newborn in the provided outfit and blanket

Answer: D Explanation: A) Scented baby products should be avoided, as they will enhance skin maceration. B) Petroleum jelly, not cotton, should be placed in the newborn's nostrils if seepage is noted. C) A gloved hand, not a washcloth, should be used when bathing the newborn's skin to decrease tissue sloughing. D) The baby should be placed in the provided outfit and wrapped in a blanket in preparation for parents who wish to hold their baby after death.

1) The nurse is analyzing various strategies for teaching new mothers about newborn care. To enhance learning, which teaching method should the nurse implement? A) Select videos on various topics of newborn care. B) Organize a class that includes first-time mothers only. C) Have mothers return in 1 week, when they feel more rested. D) Schedule time for one-to-one teaching in the mother's room.

Answer: D Explanation: A) Selecting videos on various topics of newborn care would not ensure one-to-one teaching. B) Organizing a class that includes first-time mothers only would not ensure one-to-one teaching. C) It is not appropriate or realistic to expect new mothers to return in 1 week. D) One-to-one teaching while the nurse is in the mother's room is shown to be the most effective educational model. Individual instruction is helpful to answer specific questions.

1) The couple at 12 weeks' gestation has been told that their fetus has sickle cell disease. Which statement by the couple indicates that they are adequately coping? A) "We knew we were both carriers of sickle cell disease. We shouldn't have tried to have a baby." B) "If we had been healthier when we conceived, our baby wouldn't have this disease now." C) "Taking vitamins before we got pregnant would have prevented this from happening." D) "The doctor told us there was a 25% chance that our baby would have sickle cell disease."

Answer: D Explanation: A) Self-blame and judgment do not indicate adequate coping. B) Preconception health and nutrition do not affect transmission of an autosomal recessive trait. Self-blame and judgment do not indicate adequate coping. C) Preconception health and nutrition do not affect transmission of an autosomal recessive trait. D) A true statement indicates adequate coping. When both parents are carriers of an autosomal recessive disease, there is a 25% risk for each pregnancy that the fetus will be affected.

1) The nurse is returning phone calls from clients. Which client does the nurse anticipate would not require a serum beta h C G? A) A client with a risk of ectopic pregnancy B) A client with spotting during pregnancy C) A client with previous pelvic inflammatory disease D) A client with a previous history of twins

Answer: D Explanation: A) Serial quantitative beta h C G testing can be used to distinguish a normally developing fetus from an ectopic pregnancy. B) Serologic evaluation in the first trimester is indicated for women with vaginal bleeding. C) Serial quantitative beta h C G testing can be used to distinguish a normally developing fetus from a risk of ectopic pregnancy (intrauterine device in place, previous pelvic inflammatory disease, or reversal of a tubal sterilization). D) A previous history of twins is not a risk factor for ectopic pregnancy. Beta h C G testing is not indicated for this client.

1) A 38-year-old patient is concerned that a month after becoming a widow, her menstrual cycles stopped. What should the nurse suspect as being the cause for this patient's secondary amenorrhea? A) Ovarian failure B) Pituitary dysfunction C) Anatomical abnormality D) Hypothalamic dysfunction

Answer: D Explanation: A) Severe or prolonged stress such as that which occurs with an unexpected death can lead to hypothalamic dysfunction. Ovarian failure is related to exposure to radiation, chemotherapy, viral infection, and surgical removal of the ovary. Pituitary dysfunction is related to pituitary tumors or disease, use of antipsychotic medication, low prolactin levels, head trauma, and cancer. With an anatomic abnormality the patient would not have ever had a menstrual cycle. B) Severe or prolonged stress such as that which occurs with an unexpected death can lead to hypothalamic dysfunction. Ovarian failure is related to exposure to radiation, chemotherapy, viral infection, and surgical removal of the ovary. Pituitary dysfunction is related to pituitary tumors or disease, use of antipsychotic medication, low prolactin levels, head trauma, and cancer. With an anatomic abnormality the patient would not have had a menstrual cycle. C) Severe or prolonged stress such as that which occurs with an unexpected death can lead to hypothalamic dysfunction. Ovarian failure is related to exposure to radiation, chemotherapy, viral infection, and surgical removal of the ovary. Pituitary dysfunction is related to pituitary tumors or disease, use of antipsychotic medication, low prolactin levels, head trauma, and cancer. With an anatomic abnormality the patient would not have had a menstrual cycle. D) Severe or prolonged stress such as that which occurs with an unexpected death can lead to hypothalamic dysfunction. Ovarian failure is related to exposure to radiation, chemotherapy, viral infection, and surgical removal of the ovary. Pituitary dysfunction is related to pituitary tumors or disease, use of antipsychotic medication, low prolactin levels, head trauma, and cancer. With an anatomic abnormality the patient would not have had a menstrual cycle.

1) The nurse is preparing a female client for a scheduled pelvic examination. During the health history interview, the client states, "My husband constantly criticizes me and calls me stupid. I am afraid that he will begin to hit me one of these days." Which type of intimate partner violence is the client experiencing based on the assessment data? A) Sexual abuse B) Physical abuse C) Economic abuse D) Emotional abuse

Answer: D Explanation: A) Sexual abuse is forced sex, including vaginal, oral, or anal intercourse. This type of abuse also includes sexually demeaning treatment, forced use of objects, or forcing a woman to have sex with someone else against her will. B) Physical abuse may include acts such as pushing, shoving, slapping, hitting with a fist or object, kicking, choking, threatening with a gun or knife, or using a gun or knife against a woman. This type of abuse can also include forcing alcohol or drug use or denying a partner medical care. C) Economic abuse would include preventing a spouse or significant other from getting or keeping a job; making a spouse or significant other ask for money; controlling a spouse or significant other's money; destruction of property; or making all financial decisions for the spouse or significant other. D) Emotional abuse includes constant criticism, name calling, and unreasonable demands from a spouse or significant other. This type of abuse also includes damaging a spouse or significant other's relationship with a child and others who matter to him or her.

1) The nurse is teaching a group of new parents about newborn behavior. Which statement made by a parent would indicate a need for additional information? A) "Sleep and alert states cycle throughout the day." B) "We can best bond with our child during an alert state." C) "About half of the baby's sleep time is in active sleep." D) "Babies sleep during the night right from birth."

Answer: D Explanation: A) Sleep and alert states are noticeable behaviors in infants, beginning immediately after birth with the first period of alert activity. B) Bonding between infant and parents takes place with interaction during alert states. C) About 45% to 50% of the newborn's total sleep is active sleep, 35% to 45% is quiet sleep, and 10% is transitional between these two periods. D) Over time, the newborn's sleep-wake patterns become diurnal, that is, the newborn sleeps at night and stays awake during the day. Page Ref: 654

6) The client has stated that she wants to avoid an epidural and would like an unmedicated birth. Which nursing action is most important for this client? A) Encourage the client to vocalize during contractions. B) Perform vaginal exams only between contractions. C) Provide a C D of soft music with sounds of nature. D) Offer to teach the partner how to massage tense muscles.

Answer: D Explanation: A) Some clients want to vocalize during labor, and some vocalize only when they perceive that they are losing control. The client should determine whether vocalization is desirable for her. B) Vaginal exams are performed between contractions for all laboring clients in order to decrease discomfort. C) The nurse should ask the client what type of music she would like to listen to instead of making assumptions. D) Massage is helpful for many clients, especially during latent and active labor. Massage can increase relaxation and therefore decrease tension and pain.

1) Duvall's eight stages in the family life cycle of a traditional nuclear family have been used as the foundation for contemporary models that describe the developmental processes and role expectations for different family types. Which of the following is an example of Stage Ⅳ of this family life cycle? A) Families launching young adults (all children leave home) B) Families with preschool-age children (oldest child is between 2.5 and 6 years of age) C) Middle-aged parents (empty nest through retirement) D) Families with schoolchildren (oldest child is between 6 and 13 years of age)

Answer: D Explanation: A) Stage Ⅵ is families launching young adults (all children leave home). B) Stage Ⅲ is families with preschool-age children (oldest child is between 2.5 and 6 years of age). C) Stage Ⅶ is middle-aged parents (empty nest through retirement). D) Stage Ⅳ is families with schoolchildren (oldest child is between 6 and 13 years of age).

1. The charge nurse has received the shift change report. Which client requires immediate intervention? A) Woman at 6 c m undergoing induction of labor, strong contractions every 3 minutes B) Woman at 4 c m whose fetus is in a longitudinal lie with a cephalic presentation C) Woman at 10 c m and fetus at +2 station experiencing a strong expulsion urge D) Woman at 3 c m screaming in fear because her mother died during childbirth

Answer: D Explanation: A) Strong contractions every 3 minutes constitute an adequate labor pattern during induction of labor. This client is experiencing no complications. B) Longitudinal lie with cephalic presentation is a head-down position. This is expected. C) 10 c m is fully dilated; a +2 station is low in the pelvis. A strong expulsion urge is the urge to push, which will facilitate the birth of the child. These are expected. D) This client is most likely fearful that she will die during labor because her mother died during childbirth. This client requires education and a great deal of support, and is therefore the top priority.

1) The nurse working in an adolescent prenatal clinic knows which of the following about the clients who are 15-17 years old? A) They are more at ease with their individuality. B) They see authority as resting with parents. C) They are able to solve problems and make decisions. D) They seek independence and identify with their peer group.

Answer: D Explanation: A) Teens in late adolescence are more at ease with their individuality. B) Teens in early adolescence see authority as resting with parents. C) Teens in late adolescence are able to solve problems and make decisions. D) Teens who are in middle adolescence (15-17 years old) seek independence and identify with their peer group.

1) The nurse is providing care to a couple who have experienced a stillbirth. Which is a therapeutic statement from the nurse when providing care to the client? A) "You are young; you can try again." B) "It's a blessing in disguise." C) "God needed another angel in heaven." D) "I am here to listen if you would like to talk about it."

Answer: D Explanation: A) Telling the couple that they are young and can try again diminishes their current pain. B) Telling the couple that it was a blessing in disguise often unfortunately occurs when there are fetal deformities. There are no adequate explanations as to why the couple is experiencing this tragedy, and none should be offered. C) It is not appropriate for the nurse to impose his or her belief system on the family. This statement reassures the couple that the nurse cares and is a therapeutic statement from the nurse

1) The nurse is caring for a premature infant in the N I C U, and is going to attempt a bottle feeding with thawed breast milk. How long can thawed breast milk be stored in the refrigerator before the nurse must discard it? A) 4 hours B) 8 hours C) 12 hours D) 24 hours

Answer: D Explanation: A) Thawed breast milk can stay refrigerated longer than 4 hours before it should be discarded. B) Thawed breast milk can stay refrigerated longer than 8 hours before it should be discarded. C) Thawed breast milk can stay refrigerated longer than 12 hours before it should be discarded. D) Previously frozen thawed breast milk is good in the refrigerator for 24 hours only.

21) When preparing nutritional instruction, which pregnant client would the nurse consider the highest priority? A) 40-year-old gravida 2 B) 22-year-old primigravida C) 35-year-old gravida 4 D) 15-year-old nulligravida

Answer: D Explanation: A) The 40-year-old woman has completed her growth cycle, and her body can focus on diverting nutrition to the fetus. B) The 22-year-old woman has completed her growth cycle, and her body can focus on diverting nutrition to the fetus. C) The 35-year-old woman has completed her growth cycle, and her body can focus on diverting nutrition to the fetus. D) An expectant adolescent must meet the nutritional needs for her own growth in addition to the nutritional needs of pregnancy.

A client scheduled to have a Mirena levonorgestrel intrauterine system (L N g-I U C) inserted asks how this device stops conception. What should the nurse say in response to this client? A) "It stops ovulation." B) "It slows sperm motility." C) "It shortens the menstrual cycle." D) "It causes the lining of the uterus to waste away."

Answer: D Explanation: A) The Mirena L N g-I U Cs causes the lining of the uterus (endometrium) to become waste away. This device does not stop ovulation, slow sperm motility, or shorten the menstrual cycle. B) The Mirena L N g-I U Cs causes the lining of the uterus (endometrium) to become waste away. This device does not stop ovulation, slow sperm motility, or shorten the menstrual cycle. C) The Mirena L N g-I U Cs causes the lining of the uterus (endometrium) to become waste away. This device does not stop ovulation, slow sperm motility, or shorten the menstrual cycle. D) The Mirena L N g-I U Cs causes the lining of the uterus (endometrium) to become waste away. This device does not stop ovulation, slow sperm motility, or shorten the menstrual cycle.

1) The nurse is speaking to a community group about the controversy regarding the length of the hospital stay for postpartum clients. Which statement indicates that a participant needs additional information? A) "As of 1998, there's a law that requires insurance to pay for a 48-hour stay after an uncomplicated birth." B) "The length of stay was shortened by insurance companies to decrease healthcare costs." C) "Early discharge became more popular in the 1980s as an alternative to having a home birth." D) "With current length-of-stay laws, newborns have no problems at home, and get recommended follow-up care."

Answer: D Explanation: A) The Newborns' and Mothers' Health Protection Act, which took effect in January 1998, provides for a guaranteed minimum stay of up to 48 hours following an uncomplicated vaginal birth and 96 hours following an uncomplicated cesarean birth at the discretion of the new mother and her healthcare provider. B) The average L O S decreased during the early 1990s in an effort to contain healthcare costs, with third-party payers reimbursing only a 24-hour stay following an uncomplicated vaginal delivery. C) Early discharge became more popular in the 1980s as an alternative to home delivery. D) Even with the current length-of-stay laws, many newborns do not always receive the recommended follow-up care when they go home early. The health and stability of the mother and baby, the mother's ability and confidence regarding self and newborn care, support systems available, and access to follow-up care should form the basis of the decision.

The nurse is teaching a client who is having the Skyla L N g-I U C device inserted for contraception. What should the nurse emphasize to the client about this device? A) This device will provide protection for 5 years B) This device will provide protection for 10 years C) This device should not be used with a copper allergy D) This device has a silver ring and could interfere with an M R I

Answer: D Explanation: A) The Skyla L N g-I U C device has a radiopaque silver ring at the top of the "T." The technician must be informed of this device if an M R I is required at any time. The Mirena levonorgestrel intrauterine system (L N g-I U C) provides protection for 5 years. The Copper I U C (ParaGard T 380A) provides protection for 10 years. The Skyla L N g-I U C device does not contain copper. B) The Skyla L N g-I U C device has a radiopaque silver ring at the top of the "T." The technician must be informed of this device if an M R I is required at any time. The Mirena levonorgestrel intrauterine system (L N g-I U C) provides protection for 5 years. The Copper I U C (ParaGard T 380A) provides protection for 10 years. The Skyla L N g-I U C device does not contain copper. C) The Skyla L N g-I U C device has a radiopaque silver ring at the top of the "T." The technician must be informed of this device if an M R I is required at any time. The Mirena levonorgestrel intrauterine system (L N g-I U C) provides protection for 5 years. The Copper I U C (ParaGard T 380A) provides protection for 10 years. The Skyla L N g-I U C device does not contain copper. D) The Skyla L N g-I U C device has a radiopaque silver ring at the top of the "T." The technician must be informed of this device if an M R I is required at any time. The Mirena levonorgestrel intrauterine system (L N g-I U C) provides protection for 5 years. The Copper I U C (ParaGard T 380A) provides protection for 10 years. The Skyla L N g-I U C device does not contain copper.

1. When comparing the anterior and posterior fontanelles of a newborn, the nurse knows that both are what? A) Both are approximately the same size. B) Both close within 12 months of birth. C) Both are used in labor to identify station. D) Both allow for assessing the status of the newborn after birth.

Answer: D Explanation: A) The anterior fontanelle measures approximately 2-3 c m. The posterior fontanelle is much smaller. B) The anterior fontanelle closes around the 18th month. The posterior fontanelle closes between 8 and 12 weeks after birth. C) In labor, the presenting part, not the fontanelles, is used to identify station. D) The anterior and posterior fontanelles are clinically useful in identifying the position of the fetal head in the pelvis and in assessing the status of the newborn after birth.

1) The nurse is teaching a pregnant 14-year-old client who is at 10 weeks' gestation about the expected body changes that will occur during pregnancy. Which client statement indicates that additional information is needed? A) "My breasts are going to get even bigger than they've gotten over the past couple of years." B) "My belly will gradually get more round, especially from the middle of pregnancy on." C) "I will feel my baby move in about 16 weeks." D) "My diet can stay the same even though I'm pregnant."

Answer: D Explanation: A) The breasts will enlarge. B) The abdomen will enlarge. C) Fetal movement begins in the second trimester. D) Because the adolescent is at risk for anemia, she will need education regarding the importance of iron in her diet.

1) A client is having contractions that last 20-30 seconds and that are occurring every 8-20 minutes. The client is requesting something to help relieve the discomfort of contractions. What should the nurse suggest? A) That a mild analgesic be administered B) An epidural C) A local anesthetic block D) Nonpharmacologic methods of pain relief

Answer: D Explanation: A) The client does not have an established labor pattern, and analgesics given for pain relief could prolong labor or stop the process. B) The client does not have an established labor pattern, and an epidural given for pain relief could prolong labor or stop the process. C) The pudendal block technique provides perineal anesthesia for the second stage of labor, birth, and episiotomy repair. D) For this pattern of labor, nonpharmacologic methods of pain relief should be suggested. These can include back rubs, providing encouragement, and clean linens.

1. The nurse is caring for a laboring client. A cervical exam indicates 8 c m dilation. The client is restless, frequently changing position in an attempt to get comfortable. Which nursing action is most important? A) Leave the client alone so she can rest. B) Ask the family to take a coffee-and-snack break. C) Encourage the client to have an epidural for pain. D) Reassure the client that she will not be left alone.

Answer: D Explanation: A) The client is in the transitional phase of the first stage of labor, and will not want to be alone. B) The client is in the transitional phase of the first stage of labor. The family members might want to take a break, but the client will not want to be alone. C) The client is in the transitional phase of the first stage of labor. There is no indication that the client wants pain relief. D) Because the client is in the transitional phase of the first stage of labor, she will not want to be left alone; staying with the client and reassuring her that she will not be alone are the highest priorities at this time.

1) Each of the following pregnant women is scheduled for a 14-week antepartal visit. In planning care, the nurse would give priority teaching on amniotic fluid alpha-fetoprotein (A F P) screening to which client? A) 28-year-old with history of rheumatic heart disease B) 18-year-old with exposure to H I V C) 20-year-old with a history of preterm labor D) 35-year-old with a child with spina bifida

Answer: D Explanation: A) The client with rheumatic heart disease would need to be monitored for pregnancy-related physiological stress. B) The client with H I V exposure needs H I V testing and protection education. C) The client with a history of preterm labor needs education on prevention and on signs and symptoms of preterm labor. D) Alpha-fetoprotein (A F P) is a fetal protein that is excreted from the fetal yolk sac during the first 6 weeks of pregnancy. A F P levels can be high or low, with each having different implications for the fetus. If the fetus has a neural tube defect (N T D), the A F P levels will be elevated. N T Ds can range from anencephaly to spina bifida. With a past history of a child with spina bifida, this client would be strongly encouraged to have the A F P screening.

1) The nurse is working in an outpatient clinic. Which client's indications most warrant fetal monitoring in the third trimester? A) Gravida 4, para 3, 39 weeks, with a history of one spontaneous abortion at 8 weeks B) Gravida 1, para 0, 40 weeks, with a history of endometriosis and a prior appendectomy C) Gravida 3, para 2, with a history of gestational diabetes controlled by diet D) Gravida 2, para 1, 36 weeks, with a history of preterm labor or cervical insufficiency

Answer: D Explanation: A) The client with the spontaneous abortion would have needed to be monitored in the first trimester. B) The client with endometriosis and appendectomy would have been a concern with conception. C) The client with a history of gestational diabetes controlled by diet would need maternal monitoring and fetal monitoring if she developed gestational diabetes again. D) The client with a history of preterm labor or cervical insufficiency needs close monitoring for preterm labor onset.

1) The nurse is assessing the emotional state of a client following the delivery of her newborn. Which response by the client requires further follow up by the nurse? A) Excitability B) Crying C) Quiet D) Withdrawn

Answer: D Explanation: A) The emotional response to birth varies, and excitability is considered a normal finding. B) The emotional response to birth varies, and crying is considered a normal finding. C) The emotional response to birth varies, and being quiet is considered a normal finding. D) Being withdrawn is not considered a normal emotional response to delivery of a newborn, and requires further follow up by the nurse.

1) The client at 30 weeks' gestation with her first child is upset. She tells the prenatal clinic nurse that she is excited to become a mother, and has been thinking about what kind of parent she will be. But her mother has told her that she doesn't want to be a grandmother because she doesn't feel old enough, while her husband has said that the pregnancy doesn't feel real to him yet, and he will become excited when the baby is actually here. What is the most likely explanation for what is happening within this family? A) Her husband will not attach with this child and will not be a good father. B) Her mother is rejecting the role of grandparent, and will not help out. C) The client is not progressing through the developmental tasks of pregnancy. D) The family members are adjusting to the role change at their own paces.

Answer: D Explanation: A) The expectant father must first deal with the reality of the pregnancy and then struggle to gain recognition as a parent from his partner, family, friends, coworkers, society-and from his baby as well. B) Younger grandparents leading active lives may not demonstrate as much interest as the young couple would like. C) This is a false statement. The client is at the stage of seeking acceptance of this child by others, which first will be her partner and other family members. D) This is a true statement. With each pregnancy, routines and family dynamics are altered, requiring readjustment and realignment.

1) A woman calls the clinic and tells a nurse that she thinks she might be pregnant. She wants to use a home pregnancy test before going to the clinic, and asks the nurse how to use it correctly. What information should the nurse give? A) The false-positive rate of these tests is quite high. B) If the results are negative, the woman should repeat the test in 2 weeks if she has not started her menstrual period. C) A negative result merely indicates growing trophoblastic tissue and not necessarily a uterine pregnancy. D) The client should follow up with a healthcare provider after taking the home pregnancy test.

Answer: D Explanation: A) The false-positive rate of these tests is quite low. B) If the results are negative, the woman should repeat the test in 1 week if she has not started her menstrual period. C) A positive result merely indicates growing trophoblastic tissue and not necessarily a uterine pregnancy. D) It is important that clients remember that the tests are not always accurate and they should follow up with a healthcare provider.

1) What is the function of the scrotum? A) Produce testosterone, the primary male sex hormone B) Deposit sperm in the female vagina during sexual intercourse so that fertilization of the ovum can occur C) Provide a reservoir where spermatozoa can survive for a long period D) Protect the testes and the sperm by maintaining a temperature lower than that of the body

Answer: D Explanation: A) The interstitial cells produce testosterone, the primary male sex hormone. B) The primary reproductive function of the penis is to deposit sperm in the female vagina during sexual intercourse so that fertilization of the ovum can occur. C) The epididymis provides a reservoir where spermatozoa can survive for a long period. D) The function of the scrotum is to protect the testes and the sperm by maintaining a temperature lower than that of the body.

1) The client delivered her first child vaginally 7 hours ago. She has not voided since delivery. She has an I V of lactated Ringer's solution running at 100 m L/h r. Her fundus is firm and to the right of midline. What is the best nursing action? A) To massage the fundus vigorously B) To assess the client's pain level C) To increase the rate of the I V D) To assist the client to the bathroom

Answer: D Explanation: A) The location of the fundus and the time since delivery indicate that the bladder is distended. Emptying the bladder is the top priority. B) To assess the client's pain level is not important at this time. Emptying the bladder is the top priority. C) Emptying the bladder is the top priority. A distended bladder is an indication that hydration is adequate. D) Emptying the bladder is the top priority.

1) The nurse is preparing to assess the development of a family new to the clinic. The nurse understands that which of the following is the primary use of a family assessment tool? A) Obtain a comprehensive medical history of family members. B) Determine to which clinic the client should be referred. C) Predict how a family will likely change with the addition of children. D) Understand the physical, emotional, and spiritual needs of members.

Answer: D Explanation: A) The medical history is one area that is explored using a family assessment tool, but it is not the primary use of the family assessment. B) Although referrals might take place as a result of the family assessment findings, this is not the primary purpose of the assessment. C) Family development models help predict how a family will likely change with the addition of children. Understanding the physical, emotional, and spiritual needs of members is the main reason for using a family assessment tool

1) A woman at 28 weeks' gestation is asked to keep a fetal activity record and to bring the results with her to her next clinic visit. One week later, she calls the clinic and anxiously tells the nurse that she has not felt the baby move for more than 30 minutes. Which of the following would be the nurse's most appropriate initial comment? A) "You need to come to the clinic right away for further evaluation." B) "Have you been smoking?" C) "When did you eat last?" D) "Your baby might be asleep."

Answer: D Explanation: A) The mother would need to come to the clinic only if there had been no fetal activity for several hours. B) Certain substances such as tobacco, drugs, alcohol, and caffeine have been shown to affect fetal movements and can increase fetal movement. C) After meals, an infant typically has increased movement. D) Lack of fetal activity for 30 minutes typically is insignificant. Movement varies considerably, but most women feel fetal movement at least 10 times in 3 hours.

1. While caring for a client in labor, the nurse notices during a vaginal exam that the fetus's head has rotated internally. What would the nurse expect the next set of cardinal movements for a fetus in a vertex presentation to be? A) Flexion, extension, restitution, external rotation, and expulsion B) Expulsion, external rotation, and restitution C) Restitution, flexion, external rotation, and expulsion D) Extension, restitution, external rotation, and expulsion

Answer: D Explanation: A) The next set of cardinal movements would not begin with flexion. B) This is not the correct order of fetal position changes. C) This is not the correct order of fetal position changes. D) The fetus changes position in the following order: descent, flexion, internal rotation, extension, restitution, external rotation, and expulsion.

1) The nurse has assessed four newborns' respiratory rates immediately following birth. Which respiratory rate would require further assessment by the nurse? A) 60 breaths per minute B) 70 breaths per minute C) 64 breaths per minute D) 20 breaths per minute

Answer: D Explanation: A) The normal range for respirations of a newborn within 2 hours after birth is 60 to 70 breaths per minute. B) The normal range for respirations of a newborn within 2 hours after birth is 60 to 70 breaths per minute. C) The normal range for respirations of a newborn within 2 hours after birth is 60 to 70 breaths per minute. D) If respirations drop below 20 when the baby is at rest the primary care provider should be notified.

1) The nurse is working with new parents who have recently immigrated to the United States. The nurse is not familiar with the family's cultural background. Which approach is most appropriate when discussing the newborn? A) "You appear to be Muslim. Do you want your son circumcised?" B) "Let me explain newborn care here in the United States." C) "Your baby is a United States citizen. You must be very happy about that." D) "Could you explain your preferences regarding childrearing?"

Answer: D Explanation: A) The nurse should avoid making assumptions about clients based on appearance. B) The nurse should not assume the family doesn't understand the United States healthcare system. C) The nurse should avoid making assumptions regarding family beliefs and values. D) The nurse must be sensitive to the cultural beliefs and values of the family and be aware of cultural variations in newborn care.

The nurse reviewing charts for quality improvement notes that a client experienced a complication during labor. The nurse is uncertain whether the labor nurse took the appropriate action during the situation. What is the best way for the nurse to determine what the appropriate action should have been? A) Call the nurse manager of the labor and delivery unit and ask what the nurse should have done. B) Ask the departmental chair of the obstetrical physicians what the best nursing action would have been. C) Examine other charts to find cases of the same complication, and determine how it was handled in those situations. D) Look in the policy and procedure book, and examine the practice guidelines published by a professional nursing organization.

Answer: D Explanation: A) The nurse should find the standards, and not rely on another person to determine appropriateness of care. B) Physician care and nursing care are very different; physicians might not be up to date on nursing standards of care or nursing policies and procedures. C) What nursing action was undertaken in a different situation might not be based on the policies and procedures or other standards of care. The quality improvement nurse will obtain the most accurate information by examining the policies, procedures, and standards of care. Agency policies, procedures, and protocols contain guidelines for nursing action in specific situations. Professional organizations such as the Association of Women's Health, Obstetrical, and Neonatal Nurses (AWHONN) also publish standards of practice that should guide nursing care

1) The nurse is providing care to a client who is entering the second trimester of pregnancy. Which client statement does the nurse anticipate when assessing this client? A) "We picked out a name for a boy and for a girl." B) "We bought the baby's crib and car seat this past weekend." C) "I am so uncomfortable all the time and I can't seem to sleep at night." D) "I am angry with my husband for not showing more interest in my pregnancy."

Answer: D Explanation: A) The nurse would expect this client statement during the third, not second, trimester of pregnancy. B) The nurse would expect this client statement during the third, not second, trimester of pregnancy. C) The nurse would expect this client statement during the third, not second, trimester of pregnancy. The nurse would expect this statement during the second trimester of pregnancy

1) The nurse is discharging a 15-year-old first-time mother. Which statement should the nurse include in the discharge teaching? A) "Call your pediatrician if the baby's temperature is below 98.6°F axillary." B) "Your baby's stools will change to a greenish color when your milk comes in." C) "You can wipe away any eye drainage that might form." D) "Your infant should wet a diaper at least 6 times per day."

Answer: D Explanation: A) The pediatrician should be called if the temperature is lower than 97.8°F axillary. B) Stool color for a breastfed infant is a yellow gold, soft or mushy stools. C) Eye drainage is abnormal, and should be reported to the baby's provider. D) A minimum of 6 to 10 wet diapers per day indicates adequate fluid intake.

1) The nurse is assessing a newborn at 1 hour of age. Which finding requires an immediate intervention? A) Respiratory rate 60 and irregular in depth and rhythm B) Pulse rate 145, cardiac murmur heard C) Mean blood pressure 55 m m H g D) Pauses in respiration lasting 30 seconds

Answer: D Explanation: A) The respiratory rate is normal. The normal newborn respiratory rate is 30 to 60 breaths per minute. Initial respirations may be largely diaphragmatic, shallow, and irregular in depth and rhythm. B) This pulse rate is normal. The average resting heart rate in the first week of life is 110 to 160 beats/min. Cardiac murmurs are often present in the initial newborn period as transition from fetal to neonatal circulation occurs. C) This is a normal finding in an infant 1 hour old. The average mean blood pressure is 31 to 61 m m H g in full-term resting newborns. D) Pauses in respirations greater than 20 seconds are considered episodes of apnea, and require further intervention.

1) The nurse works in a facility that cares for clients from a broad range of racial, ethnic, cultural, and religious backgrounds. Which statement should the nurse include in a presentation to recently hired nurses on the client population of the facility? A) "Our clients come from a broad range of backgrounds, but we have a good interpreter service." B) "Many of our clients come from backgrounds different from your own, but it doesn't cause problems for the nurses." C) "Because most of the doctors are bilingual, we don't have to deal with the differences in cultural backgrounds of our clients." D) "Understanding the common values and health practices of our diverse clients will facilitate better care and health outcomes."

Answer: D Explanation: A) The role of a foreign language interpreter is to facilitate communication. The interpreter might not be able to interpret the cultural practices of clients. An example is a Spanish interpreter: The interpreter might be from Spain, but interprets language for clients from Guatemala and Nicaragua, countries about which the interpreter might know virtually nothing. B) Racial, ethnic, cultural, and religious backgrounds of clients have significant implications for how the clients perceive health, illness, and health care. It is important for nurses to understand the backgrounds of the client population that attends that facility. C) Bilingual physicians, like all physicians, have very busy schedules, and often do not understand nursing care. It is the responsibility of the nurse to become familiar with the backgrounds of the client population. Because of the implications for care based on cultural background, it is important for nurses to understand the backgrounds of the client population that accesses the facility. Without cultural awareness, caregivers tend to project their own cultural responses onto foreign-born clients; clients from different socioeconomic, religious, or educational groups; or clients from different regions of the country

1) The nurse is instructing the parents of a newborn about car seat safety. Which statement indicates that the parents need additional information? A) "The baby should be in the back seat." B) "Newborns must be in rear-facing car seats." C) "We need instruction on how to use the car seat before installing it." D) "We can bring the baby home from the hospital without a car seat, as it is only a short drive."

Answer: D Explanation: A) The safest spot in any car is the middle of the back seat. The car seat should be positioned to face the rear of the car. B) The safest spot in any car is the middle of the back seat. The car seat should be positioned to face the rear of the car. C) Nurses need to ensure that all parents are knowledgeable about the benefits of child safety seat use and proper installation. Nurses can encourage parents to have their infant safety seats checked by local groups trained specifically for that purpose. D) Newborns must go home from the birthing unit in a car seat adapted to fit newborns.

The nurse at an elementary school is performing T B screenings on all of the students. Permission slips were returned for all but the children of one family. When the nurse phones to obtain permission, the parent states in clearly understandable English that permission cannot be given because the grandmother is out of town for 2 more weeks. Which cultural element is contributing to the dilemma that faces the nurse? A) Permissible physical contact with strangers B) Beliefs about the concepts of health and illness C) Religion and social beliefs D) Presence and influence of the extended family

Answer: D Explanation: A) The situation the nurse faces is not being caused by permissible contact with strangers. B) The situation the nurse faces is not caused by beliefs about the concepts of health and illness. C) The situation the nurse faces is not caused by religion and social beliefs. D) The presence and influence of the extended family is contributing to the situation the nurse faces. In many cultures, a family elder is the primary decision maker when it comes to health care. In this case, the parent cannot grant permission to the nurse until the parent consults the grandmother.

1) A client is planning to use condoms with a spermicidal cream as contraception. What should the nurse include when reviewing this method with the client? A) Coat the condom with spermicide before using B) Insert the spermicide 1 hour before having intercourse C) Insert the cream high into the vagina and remain supine D) Wait 15 minutes after inserting the spermicide into the vagina

Answer: D Explanation: A) The spermicidal preparation is inserted into the vagina before intercourse. The nurse needs to instruct the woman to insert any of these spermicidal preparations high in the vagina as close to the cervix as possible. Maintaining a supine position after application will help keep the preparation in the vagina after it dissolves. The spermicide is not applied to the condom. The spermicide does not need to be inserted 1 hour before intercourse. Waiting 15 minutes is appropriate if the spermicide is a suppository. B) The spermicidal preparation is inserted into the vagina before intercourse. The nurse needs to instruct the woman to insert any of these spermicidal preparations high in the vagina as close to the cervix as possible. Maintaining a supine position after application will help keep the preparation in the vagina after it dissolves. The spermicide is not applied to the condom. The spermicide does not need to be inserted 1 hour before intercourse. Waiting 15 minutes is appropriate if the spermicide is a suppository. C) The spermicidal preparation is inserted into the vagina before intercourse. The nurse needs to instruct the woman to insert any of these spermicidal preparations high in the vagina as close to the cervix as possible. Maintaining a supine position after application will help keep the preparation in the vagina after it dissolves. The spermicide is not applied to the condom. The spermicide does not need to be inserted 1 hour before intercourse. Waiting 15 minutes is appropriate if the spermicide is a suppository. D) The spermicidal preparation is inserted into the vagina before intercourse. The nurse needs to instruct the woman to insert any of these spermicidal preparations high in the vagina as close to the cervix as possible. Maintaining a supine position after application will help keep the preparation in the vagina after it dissolves. The spermicide is not applied to the condom. The spermicide does not need to be inserted 1 hour before intercourse. Waiting 15 minutes is appropriate if the spermicide is a suppository.

1) The community clinic nurse manager is working on a long-term budget. The manager understands that in the next few years, Medicaid is expected to pay for fewer births. This is, in part, because of which of the following? A) The U.S. economy is becoming stronger. B) More women are able to pay for private insurance. C) New public policies are providing other forms of payment. D) Rules for Medicaid have been changed.

Answer: D Explanation: A) The strength of the economy is not a factor in Medicaid regulations. B) Private insurance is expensive, and not affordable to poor women who would be eligible for Medicaid. C) Although some states are implementing affordable healthcare options, not all states have chosen to do so. D) Women receiving Temporary Assistance for Needy Families (T A N F) do not automatically receive Medicaid services when they become pregnant.

1) In the operating room, a client is being prepped for a cesarean delivery. The doctor is present. What is the last assessment the nurse should make just before the client is draped for surgery? A) Maternal temperature B) Maternal urine output C) Vaginal exam D) Fetal heart tones

Answer: D Explanation: A) The supine position would not cause an abnormality in maternal temperature. B) The supine position would not cause an abnormality in maternal urine output. C) There is no indication that a vaginal exam should be performed. D) Ascertain fetal heart rate (F H R) before surgery and during preparation, because fetal hypoxia can result from aortocaval compression.

1) The nurse is explaining the nutritional differences between breast milk and formula to an expectant couple. The mother-to-be asks whether breast milk is nutritionally superior to formula. What should the nurse reply? A) The vitamins and minerals in formula are more bioavailable to the infant. B) There is no cholesterol in breast milk. C) The only carbohydrate in breast milk is lactose. D) The ratio of whey to casein proteins in breast milk changes to meet the nutritional needs of the growing infant.

Answer: D Explanation: A) The vitamins and minerals in breast milk have a higher bioavailability. B) Approximately 98% of human milk fat is in the form of triglycerides, and a very small but clinically significant amount is from cholesterol. C) Lactose is the primary carbohydrate in breast milk. Human milk also contains trace amounts of other carbohydrates such as glucosamines and nitrogen-containing oligosaccharides. D) The ratio of whey to casein proteins in breast milk, unlike that in formula, is not static. It changes to meet the nutritional needs of the growing infant.

1) The nurse is preparing a community presentation on family development. Which statement should the nurse include? A) The youngest child determines the family's current stage. B) A family does not experience overlapping of stages. C) Family development ends when the youngest child leaves home. D) The stages describe the family's progression over time.

Answer: D Explanation: A) The youngest child is not a marker for which stage the family is in. B) Families with more than one child can experience multiple stages simultaneously. C) Families' development continues after the youngest child leaves home. D) Family development stages describe the changes and adaptations that a family goes through over time as children are added to the family.

1) A nurse counsels a couple on sex-linked disorders. Both the man and the woman are carriers of the disorder. They ask the nurse how this disorder will affect any children they might have. What is the nurse's best response? A) "If you have a daughter, she will not be affected." B) "Your son will be affected because the father has the disorder." C) "There is a 25% chance that your son will have the disorder because the mother has the disorder." D) "There is a 50% chance that your son will be a carrier only."

Answer: D Explanation: A) There is a 50% chance that a carrier mother will pass the abnormal gene to each of her daughters, who will become carriers. B) Fathers affected with a sex-linked disorder cannot pass the disorder to their sons, but all of their daughters become carriers of the disorder. C) Because it is sex-linked, there will be more than a 25% chance that a son would be a carrier only. D) There is a 50% chance that a carrier mother will pass the normal gene on to each of her sons, who will be unaffected.

1) Toward the end of the first stage of labor, a pudendal block is administered transvaginally. What will the nurse anticipate the client's care will include? A) Monitoring for hypotension every 15 minutes B) Monitoring F H R every 15 minutes C) Monitoring for bladder distention D) No additional assessments

Answer: D Explanation: A) There is no need for any additional monitoring of the mother. B) There is no need for any additional monitoring of the fetus. C) There is no need to monitor for bladder distension. D) Because a pudendal block is done using a local anesthetic, there is no need for additional monitoring of the mother or the fetus.

A female client who is 36 years old, weighs 200 pounds, is monogamous, and does not smoke desires birth control. The nurse understands that which contraceptive method is inappropriate for this client? A) Intrauterine device B) Vaginal sponge C) Combined oral contraceptives D) Transdermal hormonal contraception

Answer: D Explanation: A) This client may use an intrauterine device. B) This client may use the vaginal sponge. C) This client may use combined oral contraceptives. D) Transdermal hormonal contraception is contraindicated because of the client's weight.

1) The nurse is assessing a pregnant client during a scheduled prenatal visit who reports dizziness and clamminess when lying in bed each morning. Which statement by the nurse is appropriate based on this data? A) "The doctor may order an amniocentesis to determine if the fetus is healthy." B) "This information indicates that you are developing gestational hypertension." C) "Be sure to sit up slowly and stay sitting for several minutes prior to getting up." "Try lying on your left side to enhance blood flow, which will help your symptoms

Answer: D Explanation: A) This data does not warrant an amniocentesis. B) This data does not support the diagnosis of gestational hypertension. C) This statement is appropriate for a client who is experiencing orthostatic hypotension and is not appropriate for the data assessed. The data suggests that the client is experiencing supine hypotension, which is often corrected by having the client lie on her left side

1) The nurse is planning visits to the homes of new parents and their newborns. Which client should the nurse see first? A) 3-day-old male who received hepatitis B vaccine prior to discharge B) 4-day-old female whose parents are both hearing-impaired C) 5-day-old male with light, sticky, yellow drainage on the circumcision site D) 6-day-old female with greenish discharge from the umbilical cord site

Answer: D Explanation: A) This infant has no indications of unexpected findings. Immunization programs against the hepatitis B virus during the newborn period and infancy are in place in many states. B) This infant is not at risk, but the appointment should be scheduled when the sign language interpreter is available. C) This is normal healing and a light, sticky, yellow drainage may form over the head of the penis. Oozing greenish yellow material or reddened areas around the cord is not an expected finding. This family should be seen first because the child is experiencing a complication

1) The partner of a client at 16 weeks' gestation accompanies her to the clinic. The partner tells the nurse that the baby just doesn't seem real to him, and he is having a hard time relating to his partner's fatigue and food aversions. Which statement would be best for the nurse to make? A) "If you would concentrate harder, you'd be aware of the reality of this pregnancy." B) "My husband had no problem with this. What was your childhood like?" C) "You might need professional psychological counseling. Ask your physician." D) "Many men feel this way. Feeling the baby move in a few weeks will help make it real to you."

Answer: D Explanation: A) This is inappropriate for the nurse say. B) This is an inappropriate comment for the nurse to make. C) The partner's feelings are not indicative of psychological pathology. D) Initially, expectant fathers may have ambivalent feelings. The extent of ambivalence depends on many factors, including the father's relationship with his partner, his previous experience with pregnancy, his age, his economic stability, and whether the pregnancy was planned. The expectant father must first deal with the reality of the pregnancy and then struggle to gain recognition as a parent from his partner, family, friends, coworkers, society-and from his baby as well.

1) The nurse is beginning the postpartum teaching of a mother who has given birth to her first child. What aspect of teaching is most important? A) Describe the likely reaction of siblings to the new baby. B) Discuss adaptation to grandparenthood by her parents. C) Determine whether father-infant attachment is taking place. D) Assist the mother in identifying the baby's behavior cues.

Answer: D Explanation: A) This is not appropriate, because the baby has no siblings. B) Adaptation to grandparenthood is a task for her parents, and not a high priority for teaching to the new mother. C) Although father-infant attachment is important, the mother is the main client, and teaching her directly is a higher priority. D) Helping the mother to identify her baby's behavior cues facilitates the acquaintance phase of maternal-infant attachment.

1) How does the nurse assess for Homans' sign? A) Extending the foot and inquiring about calf pain. B) Extending the leg and inquiring about foot pain. C) Flexing the knee and inquiring about thigh pain. D) Dorsiflexing the foot and inquiring about calf pain.

Answer: D Explanation: A) This is not the correct procedure for assessing for Homans' sign. B) This is not the correct procedure for assessing for Homans' sign. C) This is not the correct procedure for assessing for Homans' sign. D) To assess for thrombophlebitis, the nurse should have the woman stretch her legs out, with the knees slightly flexed and the legs relaxed. The nurse then grasps the foot and dorsiflexes it sharply. If pain is elicited, the nurse notifies the physician/C N M that the woman has a positive Homans' sign. The pain is caused by inflammation of a vessel.

1) Which client would the nurse document as exhibiting signs and symptoms of primary dysmenorrhea? A) 17-year-old, has never had a menstrual cycle B) 16-year-old, had regular menses for 4 years, but has had no menses in 4 months C) 19-year-old, regular menses for 5 years that have suddenly become painful D) 14-year-old, irregular menses for 1 year, experiences cramping every cycle

Answer: D Explanation: A) This is primary amenorrhea, or the lack of menses. B) Secondary amenorrhea is the term used when a client has had regular cycles that cease. C) Secondary dysmenorrhea is the sudden onset of pain and discomfort with menses. D) Dysmenorrhea, or painful menstruation, occurs at, or a day before, the onset of menstruation and disappears by the end of menses. Primary dysmenorrhea is defined as cramps without underlying disease.

1) The nurse is preparing to assess a newborn's neurological status. Which finding would require an immediate intervention? A) At rest, the infant has partially flexed arms and the legs drawn up to the abdomen. B) When the corner of the mouth is touched, the infant turns the head that direction. C) The infant blinks when the exam light is turned on over the face and body. D) The right arm is flaccid while the infant brings the left arm and fist upward to the head.

Answer: D Explanation: A) This is the normal resting posture of the infant. B) This is the rooting reflex, a normal finding in a newborn. C) Blinking in response to bright lights is an expected finding. D) Asymmetrical movement is not an expected finding, and could indicate neurological abnormality. Muscle tone should be symmetric and diminished muscle tone and flaccidity requires further evaluation.

1) Prior to receiving lumbar epidural anesthesia, the nurse would anticipate placing the laboring client in which position? A) On her right side in the center of the bed with her back curved B) Lying prone with a pillow under her chest C) On her left side with the bottom leg straight and the top leg slightly flexed D) Sitting on the edge of the bed

Answer: D Explanation: A) This position is not consistent with access to the epidural spaces. B) This position is not consistent with access to the epidural spaces. C) This position is not consistent with access to the epidural spaces. D) The woman is positioned on her left or right side, at the edge of the bed with the assistance of the nurse, with her legs slightly flexed, or she is asked to sit on the edge of the bed.

1) The nurse is working with a client from Southeast Asia. The client tells the nurse that she should not put the baby to breast until her milk comes in and her breasts are warm, because "cold milk" (colostrum) is bad for the baby. After the nurse explains the benefits of colostrum, the client still insists that "cold milk" is bad. Which response by the nurse is best? A) "What kind of formula would you like to use?" B) "That idea is folklore. Colostrum is good for the baby." C) "Now that you are here, you need to feed your baby the right way." D) "Let's give the baby formula after you breastfeed."

Answer: D Explanation: A) This statement does not facilitate breastfeeding, and does not support the education provided to the client that colostrum is beneficial. B) Although colostrum is indeed good for newborns, this response is disrespectful. C) This answer is disrespectful and therefore not therapeutic. D) This response attempts to provide a compromise between acknowledging the client's desire to give formula and getting the baby to breast to get colostrum. Nurses should be aware that some immigrant mothers may have this misconception about their colostrum.

1) Parents have been told their child has fetal alcohol syndrome (F A S). Which statement by a parent indicates that additional teaching is required? A) "Our baby's heart murmur is from this syndrome." B) "He might be a fussy baby because of this." C) "His face looks like it does due to this problem." D) "Cuddling and rocking will help him stay calm."

Answer: D Explanation: A) Valvular and septal defects are common in babies with F A S. B) F A S babies can be irritable and hyperactive in childhood. C) Facial characteristics of the F A S child include short palpebral fissures, epicanthal folds, broad nasal bridge, flattened midface, short upturned or beaklike nose, micrognathia (abnormally small lower jaw) or hypoplastic maxilla, thin upper lip or vermilion border, and smooth philtrum (groove on upper lip). D) The F A S D baby is most comfortable in a quiet, minimally stimulating environment.

1. The nurse has just palpated a laboring woman's contractions. The uterus cannot be indented during a contraction. What would the intensity of these contractions best be characterized as? A) Weak B) Mild C) Moderate D) Strong

Answer: D Explanation: A) Weak contractions are not identified. B) If the uterine wall can be indented easily, the contraction is considered mild. C) Moderate intensity falls between these two ranges. When intensity is measured with an intrauterine catheter, the normal resting tonus (between contractions) is about 10 to 12 m m H g of pressure. During acme the intensity ranges from 25 to 40 m m H g in early labor, 50 to 70 m m H g in active labor, 80 to 100 m m H g during transition, and greater than 100 m m H g while the woman is pushing in the second stage. D) Strong intensity exists when the uterine wall cannot be indented.

1) The nurse is caring for a pregnant client. The client's husband has come to the prenatal visit. Which question is best for the nurse to use to assess the father's adaptation to the pregnancy? A) "What kind of work do you do?" B) "What furniture have you gotten for the baby?" C) "How moody has your wife been lately?" D) "How are you feeling about becoming a father?"

Answer: D Explanation: A) What kind of work the husband does is not an indicator of his adaptation to the pregnancy. B) What furniture has been obtained is not an indicator of the father's adaptation to the pregnancy. C) The husband's perceptions of his wife's moodiness are not an indicator of the father's adaptation to the pregnancy. D) A husband's adaptation to pregnancy includes his feelings about impending fatherhood.

1) The nurse will be bringing the parents of a neonate with sepsis to the neonatal intensive care nursery for the first time. Which statement is best? A) "I'll bring you to your baby and then leave so you can have some privacy." B) "Your baby is on a ventilator with 50% oxygen, and has an umbilical line." C) "I am so sorry this has all happened. I know how stressful this can be." D) "Your baby is working hard to breathe and lying quite still, and has an I V."

Answer: D Explanation: A) When bringing parents to see their ill newborn for the first time, it is important to prepare them for what they will see. Bringing parents without preparation is inappropriate. B) Although this statement describes the treatment the baby is receiving, it is worded in medical jargon that will not be understood by most parents. The nurse should describe the equipment being used for the at-risk newborn and its purpose before entering the unit. C) This response focuses on the nurse. Avoid saying "I know how you feel," because it is impossible for the nurse to actually know how clients feel. D) This answer is best because it explains what the parents will see in terminology that they will understand. A trusting relationship is essential for collaborative efforts in caring for the infant. The nurse should respond therapeutically to relate to the parents on a one-to-one basis.

1) The spouse of a laboring patient is concerned that the baby is taking too long to be delivered and he has overhead some discussion about using a vacuum. Which diagram should the nurse show the spouse to explain the procedure being discussed?

Answer: D Explanation: D) For vacuum extractor traction the cup is placed on the fetal occiput, creating suction. Traction is applied in a downward and outward direction. Choice 1 is a diagram of a forceps delivery. Choice 2 is a diagram showing locations of episiotomies. Choice 3 is a diagram showing the location of the incision for a classic cesarean section.

1) The community nurse is planning to visit a family. The grandparents are helping the adult parents with child-rearing activities. For which type of family should the nurse plan care? A) Nuclear B) Blended C) Binuclear D) Extended

Answer: D Explanation: D) In an extended family, a couple shares household and childrearing responsibilities with parents, siblings, or other relatives. Families may reside together to share housing expenses and child care. In many cases, a child may be residing with a grandparent and one parent because of issues associated with unemployment, parental separation, parental death, or parental substance abuse. Grandparents may raise children due to the inability of parents to care for their own children. In the nuclear family, children live in a household with both biologic parents and no other relatives or persons. The blended family includes two parents with biologic children from a previous marriage or relationship who marry or cohabitate. A binuclear family is a post-divorce family in which the biologic children are members of two nuclear households, with parenting by both the father and the mother.

1) A lesbian female is surprised to learn of contracting the human papillomavirus. What should the nurse explain to this patient? A) "Are you telling me everything about your sexual orientation?" B) "It is rare for this infection to occur in women such as yourself." C) "Is it possible that your partner has been having intercourse with a man?" D) "Exposure to vaginal secretions can increase the risk of sexually transmitted infections."

Answer: D Explanation: D) Lesbian sexual contact can transmit a number of S T Is and the virus that causes cervical cancer, because it involves exposure to vaginal secretions. Women's health services in particular have not shown acceptance of lesbian status, as demonstrated by asking if the patient has been truthful about sexual orientation, stating that the infection is rare in people like the patient, and suggesting that the patient's partner is having intercourse with a man.

A patient treated for a urinary tract infection a month ago is experiencing symptoms of the same infection. What should the nurse suspect is the reason for the reoccurrence of the infection? A) Using oral contraceptives B) Wearing cotton underwear C) Cleansing from front to back D) Stopped antibiotics after 3 days

Answer: D Explanation: D) Not completing a full course of prescribed antibiotics could cause remaining bacteria to grow, leading to another infection. Use of oral contraceptives is not a risk factor for the development of a urinary tract infection. Wearing cotton underwear and cleansing from front to back are actions that reduce the risk of developing urinary tract infections.

1) The elderly parent of an employee at an insurance company sustains a myocardial infarction and needs assistance for several weeks after returning home from the hospital. What option should the employee consider to cover her absence while caring for her parent? A) Sick days B) Personal days C) Vacation days D) Family medical leave

Answer: D Explanation: The Family and Medical Leave Act (FMLA) offers twelve weeks of unpaid leave to care for a parent who is in recovery from an illness. The employee does not need to take sick days, personal days, or vacation days to care for her parent

1) The client at 24 weeks' gestation is experiencing painless vaginal bleeding after intercourse. The physician has ordered a transvaginal ultrasound examination. Which statements by the client indicate an understanding of why this exam has been requested? Note: Credit will be given only if all correct choices and no incorrect choices are selected. Select all that apply. A) "This ultrasound will show the baby's gender." B) "This ultrasound might cause the miscarriage of my baby." C) "This ultrasound carries a risk of creating a uterine infection." D) "This ultrasound can determine the location of my placenta." E) "This ultrasound might detect whether the placenta is detaching prematurely."

Answer: D, E Explanation: A) Although gender can sometimes be detected with second-trimester ultrasound, that is never the primary reason for the procedure. B) Ultrasound is non-invasive, and does not increase the risk for either fetal loss or uterine infection. C) Ultrasound is non-invasive, and does not increase the risk for either fetal loss or uterine infection. D) Painless bleeding in the second and third trimesters can be a symptom of placenta previa. Transvaginal ultrasound will determine the placental location. Painless bleeding in the second and third trimesters can be a symptom of placenta previa. Transvaginal ultrasound will determine the placental location

1) The mother of a client who has experienced a term stillbirth arrives at the hospital and goes to the nurses' desk. The mother asks what she should say to her daughter in this difficult time. What is the nurse's best response? Note: Credit will be given only if all correct choices and no incorrect choices are selected. Select all that apply. A) "Use clichés; your daughter will find the repetition comforting." B) "Remind her that she is young and can have more children." C) "Keep talking about other things to keep her mind off the loss." D) "Express your sadness, and sit silently with her if she doesn't respond." E) "Encourage her to talk about the baby whenever she wants to."

Answer: D, E Explanation: A) Although grieving people often say that it is easier to forgive less-than-helpful remarks than to forgive those who do not reach out to them at all, the use of clichés can be harmful. B) Reminding the mother that she is young enough to have other children downplays the importance of this baby. C) Nonstop talking does not allow the client to express her emotions and feelings. Sitting in silence with a grieving family is often the most helpful form of intervention available. D) Silence is commonly what is needed most, and simply saying "I'm sorry for your loss" might help to facilitate communication. E) Talking is a way for the client experiencing grief and begin to come to terms with what has happened, and is important for resolution of grief. Intuitive grievers will need to talk about the event.

1) The nurse is teaching an in-service educational presentation about working with battered women. The nurse should explain that it is often frustrating for nurses to work with battered women for which reasons? Note: Credit will be given only if all correct choices and no incorrect choices are selected. Select all that apply. A) There is little the nurse can really do to help. B) Healthcare policies and practices are not supportive of abused women. C) Both husband and wife must agree to therapy. D) These women might return to the abusive situation. E) Women often believe that they are the cause of the abuse.

Answer: D, E Explanation: A) Healthcare providers can play a critical role in identifying and reducing violence, even in homicide prevention efforts. B) Since 1980, there have been a number of notable changes in healthcare policy and practices aimed at responding to violence against women. C) The abuser must seek behavior change therapy to accomplish permanent change. D) Women often believe that escape is futile, or escape and then return when the crisis is over. E) Women are often convinced by the abusers that it is their own behavior that causes the abuse.

Once ruptured of membranes (ROM) has occurred the nursing intervention should include which of the following? A. immediate assessment of the FHR B. Monitor the client's temperature every 4 hrs C. Keep the client positioned on her left side D. Perform perineal care

B. Monitor the client's temperature every 4 hrs

The client's fetal heart rate is 150 before a contraction begins. At the beginning of the contraction the FHR gradually starts to fall to 110 and returns to baseline at the end of the contraction. What is the priority nursing action in response to this finding? A. Place the client into a semi-fowler's position B. Administration oxygen by nasal cannula at 2 L per minute C. Document the findings and continue to monitor D. Place the client in left lateral position

C. Document the findings and continue to monitor

Which of these events usually increase the father's interest and involvement in the pregnancy? A. learning the results of the pregnancy test B. Attending childbirth education classes C. Hearing the fetal heartbeat D. Meeting the obstetrician or midwife

C. Hearing the fetal heartbeat

1) The nurse is caring for a client who experienced the birth of a stillborn son earlier in the day. The client is from a culture where a woman's status is dominated by themes of motherhood and childrearing. What behavior would the nurse expect in this client? A) Crying inconsolably B) Expressing feelings of failure as a woman C) Requesting family members to be present Showing little emotion

Explanation: A) Inconsolable crying is not a sign of a maternalistic culture, but is a universal sign of deep grief. B) Mothers will often blame themselves, whether by commission or omission, particularly in cultures where a woman's status is dominated by themes of motherhood and childrearing. C) Inconsolable crying is not a sign of a maternalistic culture, nor is the presence of multiple family members. Showing little emotion indicates a stoic culture, which might not be maternalistic

1) A patient's placenta it is identified as having a double fold of chorion and amnion that formed a ring around the umbilical cord on the fetal side of the placenta. What should the nurse expect when examining this placenta?

Explanation: B) A circumvallate placenta has a double fold of chorion and amnion that forms a ring around the umbilical cord, on the fetal side of the placenta. Choice 1: In a succenturiate placenta, one or more accessory lobes of fetal villi develops on the placenta. Choice 3: In a battledore placenta, the umbilical cord is inserted at or near the placental margin. Choice 4: In a velamentous insertion of the umbilical cord, the vessels of the umbilical cord divide some distance from the placenta in the placental membranes.

1) A laboring patient is attempting a vaginal birth. An episiotomy has already been performed and the healthcare provider is prepared to extend the episiotomy if necessary. What is this fetus's most likely presentation?

Explanation: C) In a brow presentation, the forehead of the fetus becomes the presenting part and the head is slightly extended instead of flexed, which results in the head entering the birth canal with the widest diameter of the head (occipitomental) foremost. If a vaginal birth is attempted, the woman will probably need an episiotomy and may require extension of the episiotomy at the moment of birth. In a normal cephalic presentation, the occiput is the presenting part, and the head is flexed with the chin on the chest. The military presentation is probably the least difficult for the woman and fetus. In most cases, as soon as the head reaches the pelvic floor, flexion occurs and a vaginal birth results. In a face presentation, the face of the fetus is the presenting part. The fetal neck is hyperextended.

1) The nurse is working with a mother who has just delivered her third child at 33 weeks' gestation. The mother says to the nurse, "This baby doesn't turn her head and suck like the older two children did. Why?" What is the best response by the nurse? A) "Every baby is different. This is just one variation of normal that we see on a regular basis." B) "This baby might not have a rooting or sucking reflex because she is premature." C) "When she is wide awake and alert, she will probably root and suck even if she is early." D) "She might be too tired from the birthing process and need a couple of days to recover."

nswer: B Explanation: A) Although each baby is unique and different from siblings, this response is not accurate. B) Preterm babies may have suppressed or absent root and suck reflexes. C) This statement is true of term infants, but this infant is preterm. D) Although birth is stressful even to term newborns, and some require a day or two of recovery to become fully alert, this infant is preterm.

1) The visiting nurse evaluates a 2-day-old breastfed newborn at home and notes that the baby appears jaundiced. When explaining jaundice to the parents, what would the nurse tell them? A) "Jaundice is uncommon in newborns." B) "Some newborns require phototherapy." C) "Jaundice is a medical emergency." D) "Jaundice is always a sign of liver disease."

nswer: B Explanation: A) Physiologic jaundice is a normal process that can occur after 24 hours of life and develops in more than 60% of term newborns and 80% of preterm neonates. B) Physiologic jaundice is a normal process that can occur after 24 hours of life in about half of healthy newborns. It is not a sign of liver disease. Physiologic jaundice might require phototherapy. C) Physiologic jaundice is a normal process that can occur after 24 hours of life in about half of healthy newborns. It is not a medical emergency. D) Physiologic jaundice is not a sign of liver disease.

1) The client in the prenatal clinic tells the nurse that she is sure she is pregnant because she has not had a menstrual cycle for 3 months, and her breasts are getting bigger. What response by the nurse is best? A) "Lack of menses and breast enlargement are presumptive signs of pregnancy." B) "The changes you are describing are definitely indicators that you are pregnant." C) "Lack of menses can be caused by many things. We need to do a pregnancy test." D) "You're probably not pregnant, but we can check it out if you like."

swer: C Explanation: A) Although a lack of menses and breast enlargement are presumptive signs of pregnancy, the nurse should not state this without explaining that these symptoms also can be caused by other conditions. B) This statement is false because amenorrhea and breast enlargement can be caused by other conditions. C) This is a true statement, and addresses that these changes could be caused by conditions other than pregnancy. D) While lack of menses and breast enlargement might not be caused by pregnancy, they likely are the result of pregnancy, and it is inappropriate for the nurse to suggest the client is not pregnant.

1) The client diagnosed with endometriosis asks the nurse whether there are any long-term health risks associated with this condition. The nurse should include which statement in the client teaching about endometriosis? A) "There are no other health risks associated with endometriosis." B) "Pain with intercourse rarely occurs as a long-term problem." C) "You are at increased risk for ovarian and breast cancer." D) "Most women with this condition develop severe migraines."

wer: C Explanation: A) There are long-term health risks associated with endometriosis. B) Dyspareunia is a common symptom of endometriosis. C) An increased risk for cancer of the ovary and breast is associated with endometriosis. Endometriosis is not associated with increased migraines

1) The nurse is preparing an educational program for new parents. In which order should the nurse provide the Purple Crying information? 1. Unexpected 2. Long lasting 3. Pain-like face 4. Peak of crying 5. Resists soothing 6. Evening and late afternoon

Answer: 4, 1, 5, 3, 2, 6 Explanation: The acronym PURPLE is derived from the character of the crying that some healthy infants experience between 2 and 4 months of age: P: Peak of crying; U: Unexpected; R: Resists soothing; P: Pain-like face; L: Long lasting; and E: Evening and late afternoon

1) A patient weighing 80 k g with a body mass index of 29.8 is 6 weeks pregnant. What should be this patient's maximum weight at the time of delivery?

Answer: 89 k g Explanation: Women who are obese are advised to limit weight gain to 5 to 9 k g (11 to 20 lb). Since the patient weighs 80 k g at 6 weeks pregnant, the maximum amount she should weigh would be 80 k g + 9 k g = 89 k g.

1) The nurse on the birthing unit is caring for a client who has an amputated cervix and is about to undergo a cerclage procedure. Which type of cerclage procedure should the nurse anticipate? A) Abdominal cerclage B) Rescue cerclage C) Emergency cerclage D) Elective cerclage

Answer: A Explanation: A) An abdominal cerclage approach may be required for women with an amputated cervix. B) A rescue cerclage is one that is placed for emergent reasons, when dilatation and effacement have already occurred. C) An emergency cerclage is one that is placed for emergent reasons, when dilatation and effacement have already occurred. D) An elective, or cervical, cerclage will not be performed for a client who has an amputated cervix.

A client comes to the clinic complaining of difficulty urinating, flu-like symptoms, genital tingling, and blister-like vesicles on the upper thigh and vagina. She denies having ever had these symptoms before. The medication the physician is most likely to order would be: A) Oral acyclovir B) Ceftriaxone I M C) Azithromycin P O D) Penicillin G I M

Answer: A Explanation: A) Malaise, dysuria, and tingling or painful vesicles are indicative of a primary herpes simplex outbreak. Acyclovir treats herpes. B) Ceftriaxone I M does not treat herpes. C) Azithromycin does not treat herpes. D) Penicillin does not treat herpes.

1) The nurse is working with a group of pregnant teens. Which statement indicates that teaching has been successful? A) "Pregnant teens are more likely to quit school prior to graduation." B) "Because I am young, I have a low risk for preeclampsia." C) "My baby could come late because I am a teenager." D) "I am more likely to use birth control after I have this baby."

Answer: A Explanation: A) Many teenage mothers drop out of school during their pregnancy and then are less likely to complete their schooling. B) Preeclampsia-eclampsia is the most prevalent medical complication of pregnant adolescents. C) A physical risk of adolescent pregnancies includes preterm births. D) Many factors contribute to the high teenage pregnancy rate, including lack of knowledge about conception and lack of easy access to contraception.

28) The nurse has completed an initial physical assessment for a client admitted to the birthing unit. Which action should the nurse take next? A) Obtain the client's social history B) Document the physical assessment findings C) Report findings to the physician D) Perform interventions for pain management

Answer: A Explanation: A) Once initial physical assessments are performed, the nurse can then take a detailed social history that provides a comprehensive view of both the woman's social habits and psychologic factors that may affect her birth experience. B) Documenting the physical assessment findings may be performed after the initial physical assessment is performed and social history is obtained. C) It may not be necessary to report findings to the physician in the absence of other complicating factors. D) Interventions for pain management may not be needed at this time. Additionally, effective pain management may depend on the client's social history.

1) A postpartum client calls the nursery to report that her newborn's umbilical cord stump is draining, and has a foul odor. What is the nurse's best response? A) "Take your newborn to the pediatrician." B) "Cover the cord stump with gauze." C) "Apply Betadine around the cord stump." D) "This is normal during healing."

Answer: A Explanation: A) Parents should check each day for any odor, oozing of greenish yellow material, or reddened areas around the cord. They should report to the healthcare provider any signs of infection. B) Parents should fold diapers below the umbilical cord to air-dry the cord. Contact with wet or soiled diapers slows the drying process and increases the possibility of infection. C) Betadine is not used on the cord stump. D) These symptoms are not normal.

1) The nurse is performing a vaginal exam on a client who was admitted to the birthing unit after her membranes ruptured, and discovers a cord prolapse. Which intervention is priority at this time? A) Pushing the presenting fetal part upward B) Administering oxygen C) Initiating intravenous fluid D) Inserting an indwelling bladder catheter

Answer: A Explanation: A) Pushing the presenting fetal part upward is a life-saving measure that relieves pressure on the umbilical cord and supports fetal gas exchange. B) Administering oxygen is performed, but at a later time. C) Initiating intravenous fluid is performed, but at a later time. Inserting an indwelling bladder catheter may be later used to fill the woman's bladder and relieve pressure on the umbilical cord, but this should not be done in place of pushing the presenting fetal part upward

When assessing a client asking about birth control, the nurse knows that the client would not be a good candidate for Depo-Provera (D M P A) if which of the following is true? A) She wishes to get pregnant within 3 months. B) She is a nursing mother. C) She has a vaginal prolapse. D) She weighs 200 pounds.

Answer: A Explanation: A) Return of fertility after use may be delayed for an average of 10 months. B) A nursing mother can use Depo-Provera. C) Obesity and vaginal prolapse do not contraindicate the use of Depo-Provera. D) Obesity and vaginal prolapse do not contraindicate the use of Depo-Provera.

1) The nurse is instructing parents of a newborn about voiding and stool characteristics. Which of the following would be considered an abnormal pattern? A) Large amounts of uric acid crystals in the first days of life B) At least 6 to 10 wet diapers a day after the first few days of life C) 1 to 2 stools a day for a formula-fed baby D) Urine that is straw to amber color without foul smell

Answer: A Explanation: A) Small, not large, amounts of uric acid crystals are normal in the first days of life. B) 6 to 10 wet diapers a day after the first few days of life is normal. C) 1 to 2 stools a day for a formula-fed baby is normal. D) Urine that is straw to amber color without foul smell is normal.

The nurse seeing a client just diagnosed with Chlamydia trachomatis knows that which client is at greatest risk for the infection? A) 16-year-old sexually active girl, using no contraceptive B) 22-year-old mother of two, developed dyspareunia C) 35-year-old woman on oral contraceptives D) 48-year-old woman with hot flashes and night sweats

Answer: A Explanation: A) Teens have the highest incidence of sexually transmitted infections, especially chlamydia. A client not using contraceptives is not using condoms, which decrease the risk of contracting a S T I. B) Dyspareunia sometimes develops with chlamydia infection, but dyspareunia is not a symptom specific to chlamydia. C) There is no correlation between oral contraceptive use and an increased rate of chlamydia infection. Additionally, chlamydia is more commonly seen in young women. This client is experiencing signs of menopause, not of chlamydia infection

1) The laboring client's fetal heart rate baseline is 120 beats per minute. Accelerations are present to 135 beats/min. During contractions, the fetal heart rate gradually slows to 110, and is at 120 by the end of the contraction. What nursing action is best? A) Document the fetal heart rate. B) Apply oxygen via mask at 10 liters. C) Prepare for imminent delivery. D) Assist the client into Fowler's position.

Answer: A Explanation: A) The described fetal heart rate has a normal baseline; the presence of accelerations indicates adequate fetal oxygenation, and early decelerations are normal. No intervention is necessary. B) No oxygen is necessary. C) There is no indication that delivery will be occurring soon. D) There is no need to put the client in Fowler's position.

1) A pregnant client who is at 14 weeks' gestation asks the nurse why the doctor used to call her baby an embryo, and now calls it a fetus. What is the best answer to this question? A) "Fetus is the term used from the ninth week of gestation onward." B) "We call a baby a fetus when it is larger than an embryo." C) "An embryo is a baby from conception until the eighth week." D) "The official term for a baby in utero is really zygote."

Answer: A Explanation: A) The fetal stage begins in the ninth week. B) The embryonic stage ends with the eighth week, regardless of size. C) The preembryonic stage is from conception until day 15. A zygote is a fertilized ovum

1) What is the most significant maternal risk factor for preterm birth? A) Previous preterm birth B) Smoking C) Stress D) Substance abuse

Answer: A Explanation: A) The most significant maternal risk factor for preterm birth is a previous preterm birth. B) Modifiable risk factors, such as smoking, substance abuse, stress, alcohol use, and other behavioral factors are not the most significant maternal risk factors for preterm birth. C) Modifiable risk factors, such as smoking, substance abuse, stress, alcohol use, and other behavioral factors are not the most significant maternal risk factors for preterm birth. D) Modifiable risk factors, such as smoking, substance abuse, stress, alcohol use, and other behavioral factors are not the most significant maternal risk factors for preterm birth.

1) The nurse is reviewing the F H R monitor for a client in labor. The rhythm strip yields the following result: How should the nurse interpret this pattern? A) Moderate variability B) Minimal variability C) Absent variability D) Marked variability

Answer: A Explanation: A) The pattern demonstrates Moderate F H R variability (normal) with an amplitude range of 6 to 25 beats/minute. B) Minimal variability refers to a F H R with an amplitude range detectable but 5 beats/minute or less. C) Absent variability refers to a pattern with no detectable amplitude range. D) Marked variability refers to a pattern with an amplitude range greater than 25 beats/minute.

Answer: B Explanation: B) The clamp is removed in the newborn nursery approximately 24 hours after birth if the cord has dried. The clamp does not stay intact for one week. The clamp does not eliminate the need to wash the infant's abdomen. A plastic device is used when removing the clamp. The tissue beneath the clamp is not cut.

28) A newborn has the following applied to the umbilical cord. What should the nurse instruct the new parents about regarding this device? A) It has to stay intact for at least one week B) It is removed in 24 hours if the cord has dried C) It eliminates the need to wash the infant's abdomen D) It is removed by cutting the tissue beneath the clamp

1) The nurse is reviewing the contents of breastmilk with a new mother. What amount of water should the nurse explain is in 8 ounces of breastmilk? Record your answer rounding to the nearest whole number.

Answer: 7 ounces Explanation: Breastmilk contains 87% water. The nurse should multiply 8 ounces by 87% to determine that 6.96 ounces out of 8 ounces of breast milk is water. With rounding, the volume of water in 8 ounces of breastmilk is 7 ounces.

15) The laboring client is complaining of tingling and numbness in her fingers and toes, dizziness, and spots before her eyes. The nurse recognizes that these are clinical manifestations of which of the following? A) Hyperventilation B) Seizure auras C) Imminent birth D) Anxiety

Answer: A Explanation: A) These symptoms all are consistent with hyperventilation. B) Seizure auras do not have these symptoms. C) Imminent birth does not have these symptoms. Anxiety does not have these symptoms

1. Clinical features of posttraumatic stress disorder (P T S D) include which of the following? A) Difficulty sleeping B) Acute awareness C) Flashbacks D) The need to be constantly around others E) Irritability

Answer: A, C, E Explanation: A) A clinical feature of P T S D is difficulty sleeping. B) Numbness, not acute awareness, is a clinical feature of P T S D. C) A clinical feature of P T S D is intrusive thoughts and flashbacks to the threatening event. D) Avoidance of others is a clinical feature of P T S D. E) A clinical feature of P T S D is irritability.

1) The newborn's cry should have which of the following characteristics? Note: Credit will be given only if all correct choices and no incorrect choices are selected. Select all that apply. A) Medium pitch B) Shrillness C) Strength D) High pitch E) Lusty

Answer: A, C, E Explanation: A) The newborn's cry should be strong, lusty, and of medium pitch. B) A high-pitched, shrill cry is abnormal and may indicate neurologic disorders or hypoglycemia. C) The newborn's cry should be strong, lusty, and of medium pitch. D) A high-pitched, shrill cry is abnormal and may indicate neurologic disorders or hypoglycemia. E) The newborn's cry should be strong, lusty, and of medium pitch.

1) A nurse examining a prenatal client recognizes that a lag in progression of measurements of fundal height from week to week and month to month could signal what condition? A) Twin pregnancy B) Intrauterine growth restriction C) Hydramnios D) Breech position

Answer: B Explanation: A) A sudden increase in fundal height could indicate twins. B) A lag in progression of measurements of fundal height from month to month could signal intrauterine growth restriction (I U G R). C) A sudden increase in fundal height could indicate hydramnios. D) A fetus in breech position would still have a normal fundal height measurement.

1) A postpartum mother is concerned that her newborn has not had a stool since birth. The newborn is 18 hours old. What is the nurse's best response? A) "I will call your pediatrician immediately." B) "Passage of the first stool within 48 hours is normal." C) "Your newborn might not have a stool until the third day." D) "Your newborn must be dehydrated."

Answer: B Explanation: A) This is not an emergency situation. B) The first voiding should occur within 24 hours and first passage of stool within 48 hours. C) The passage of the first stool should occur sooner. D) Decreased urinary output and depressed fontanelles indicate dehydration.

1) A woman is hospitalized with severe preeclampsia. The nurse is meal-planning with the client and encourages a diet that is high in what? A) Sodium B) Carbohydrates C) Protein D) Fruits

Answer: C Explanation A) It is important that the client limit her intake of sodium. B) While it is important that the client have an adequate intake of carbohydrates, another food group is more important. C) The client who experiences preeclampsia is losing protein. D) It is important that the client have adequate intake of fruits, but another food group is more important.

The nurse provides a couple with education about the consequences of not treating chlamydia, and knows they understand when they make which statement? A) "She could become pregnant." B) "She could have severe vaginal itching." C) "He could get an infection in the tube that carries the urine out." D) "It could cause us to develop a rash."

Answer: C Explanation: A) Chlamydia does not cause a woman to become pregnant. B) Chlamydia does not cause vaginal itching. C) Chlamydia is a major cause of nongonococcal urethritis (N G U) in men. D) Chlamydia does not cause a rash.

1) The labor and delivery nurse is assigned to four clients in early labor. Which electronic fetal monitoring finding would require immediate intervention? A) Early decelerations with each contraction B) Variable decelerations that recover to the baseline C) Late decelerations with minimal variability D) Accelerations

Answer: C Explanation: A) Early decelerations are usually benign. B) Variable decelerations indicate cord compression, but those that recover to the baseline indicate that the fetus is tolerating the decelerations. C) Late decelerations are considered a nonreassuring fetal heart rate (F H R) pattern, and therefore require immediate intervention. Accelerations of the fetal heart rate indicate good oxygen reserve

The current emphasis on federal healthcare reform has yielded what unexpected benefit? A) Assessment of the details of the family's income and expenditures B) Case management to limit costly, unnecessary duplication of services C) Many healthcare providers and consumers are becoming more aware of the vitally important role nurses play in providing excellent care to clients and families D) Education of the family about the need for keeping regular well-child visit appointments

Answer: C Explanation: A) Financial assessment is more commonly the function of a social worker. The social worker is part of the interdisciplinary team working with clients, and this professional's expertise is helping clients get into the appropriate programs. B) The case management activity mentioned will not provide a source of funding. C) Nurses must clearly articulate their role in the changing environment to define and differentiate practice roles and the educational preparation required for their new roles. D) The education of the family will not provide a source of funding.

During a newborn assessment, the notices that the newborn flexes its knees, straightens its arms and hands outward with the fingers spread forming a "c". In response to this finding, the nurse should complete which of the following actions first? A. Complete a Ballard score to ensure the newborn is not premature B. Notify the physician of the possibility of neonatal abstinence syndrome C. Document the findings as this a normal finding in a newborn D. Take the infant to feed immediately to prevent further hypoglycemia

C. Document the findings as this a normal finding in a newborn

A nurse infused an intravenous fluid to a client who is undergoing an epidural block. What is the purpose of this nursing action? A. To treat hypovolemia that may result from hemorrhage B. To increase urine output C. To prevent insensible fluid losses D. To treat hypotension due to sympathetic blockade

D. To treat hypotension due to sympathetic blockade

1) A newborn who has not voided by 48 hours after birth should be assessed for which of the following? Note: Credit will be given only if all correct choices and no incorrect choices are selected. Select all that apply. A) Restlessness B) Pain C) Kidney distention D) Adequacy of fluid intake E) Lethargy

nswer: A, B, D Explanation: A) A newborn who has not voided by 48 hours after birth should be assessed for restlessness. B) A newborn who has not voided by 48 hours after birth should be assessed for pain. C) A newborn who has not voided by 48 hours after birth should be assessed for bladder distention, not kidney distention. D) A newborn who has not voided by 48 hours after birth should be assessed for adequacy of fluid intake. E) A newborn who has not voided by 48 hours after birth should be assessed for restlessness, not lethargy.


संबंधित स्टडी सेट्स

Types of Understatement and Irony (Figures of speech)

View Set

NURS 482 - Final Exam Practice Questions (Module 5-7)

View Set

Chapter 17.3 and 17.5 Connect Conceptual, Comprehensive, and Concept Questions

View Set

Final COMLEX/USMLE STEP 1 Study Set

View Set

18FALL-SOC1051-001 - Introductory Sociology (ch. 11/12 homework/review)

View Set